You are on page 1of 301

T

h
.
s

M
i
n
h
T
u

n
Th.s MINH TUN
TI LIU LUYN THI I HC
MN TON
NAM NH, NM 2010
http: //www. aotrangtb. com
T
h
.
s

M
i
n
h
T
u

n
Li ni u
Trong nhng nm gn y, thi i hc tr nn c bn hn trc rt nhiu , khng cn
tnh nh cng nh bt hc sinh phi nh nhiu nhng mo rt lt vt. Mt s ti liu ging
dy rt hay ngy trc nh "Cc bi ging luyn thi mn Ton", "B thi tuyn sinh" ch cn
li mt t gi tr thc tin ca n. Cht lc nhng ti liu ny, bm st nhng thi tuyn sinh
nhng nm gn y (T nm 2002-2010) cng vi nhng kinh nghim trong thc tin ging dy
luyn thi ca mnh (c tham kho mt s bi ging nhng trang web dy hc) ti bin son
ti liu ny mc ch chnh mnh ging dy mt cch bi bn.
Ti ngh rng ti liu ny s c ch i vi nhng ngi dy ton, cng nh nhng bn ngp
ngh cng trng i hc.
Ti liu ny gm 13 chuyn (vn cn thiu)
1. Phng trnh i s.
2. Phng trnh lng gic.
3. Phng trnh cha cn v du gi tr tuyt i.
4. H phng trnh i s
5. Gii tch t hp
6. Hnh phng ta
7. Gii hn
8. Bt ng thc
9. Hm s v th
10. Hnh hc khng gian ta ( chnh sa, ch thiu phn ta ha hnh hc khng gian)
11. Tch phn v ng dng
12. S phc
13. Hnh hc khng gian c in.
V s lng cc chuyn ln nn khng th trnh khi nhng li nh my, li tnh ton sai,
... Mong cc bn lng th, mi gp xin gi v:
Th.s Minh Tun.
Trng CSP Nam nh, 813 ng Trng Chinh, TP Nam nh
Email: xuxutit@gmail.com
Mobile: 0982843882.

Chc cc bn thnh cng trong k thi i hc sp ti!


Nam nh, ngy 18 thng 12 nm 2010
Tc gi
Minh Tun
Ngi son : Th.s Minh Tun Trang 2 Khoa T nhin - Trng CSP Nam nh
http: //www. aotrangtb. com
T
h
.
s

M
i
n
h
T
u

n
Mc lc Mc lc
Mc lc
Li ni u 2
1 Phng trnh i s 9
1.1 L thuyt v a thc . . . . . . . . . . . . . . . . . . . . . . . . . . . . . . . . . 9
1.1.1 Phn tch a thc thnh nhn t . . . . . . . . . . . . . . . . . . . . . . 9
1.1.2 Tnh gi tr mt a thc, phn thc ti im l . . . . . . . . . . . . . . 10
1.2 Phng trnh bc nht . . . . . . . . . . . . . . . . . . . . . . . . . . . . . . . . 10
1.2.1 Phng php gii . . . . . . . . . . . . . . . . . . . . . . . . . . . . . . . 10
1.2.2 Cc v d . . . . . . . . . . . . . . . . . . . . . . . . . . . . . . . . . . . 10
1.3 Phng trnh bc hai . . . . . . . . . . . . . . . . . . . . . . . . . . . . . . . . . 11
1.3.1 Phng php gii . . . . . . . . . . . . . . . . . . . . . . . . . . . . . . . 11
1.4 Phng trnh bc 3 . . . . . . . . . . . . . . . . . . . . . . . . . . . . . . . . . . 15
1.4.1 Tnh cht ca a thc . . . . . . . . . . . . . . . . . . . . . . . . . . . . 15
1.4.2 a thc bc 3 . . . . . . . . . . . . . . . . . . . . . . . . . . . . . . . . . 15
1.4.3 Cc v d . . . . . . . . . . . . . . . . . . . . . . . . . . . . . . . . . . . 16
1.5 Phng trnh bc 4 . . . . . . . . . . . . . . . . . . . . . . . . . . . . . . . . . . 17
1.5.1 Dng tng qut . . . . . . . . . . . . . . . . . . . . . . . . . . . . . . . . 17
1.5.2 Cc dng ca phng trnh bc 4 . . . . . . . . . . . . . . . . . . . . . . 18
1.5.3 Cc v d . . . . . . . . . . . . . . . . . . . . . . . . . . . . . . . . . . . 19
1.6 Du ca a thc . . . . . . . . . . . . . . . . . . . . . . . . . . . . . . . . . . . 21
1.6.1 a thc bc 1 - bc 2 . . . . . . . . . . . . . . . . . . . . . . . . . . . . . 21
1.6.2 a thc - Phn thc tng qut . . . . . . . . . . . . . . . . . . . . . . . 24
1.6.3 Gii h bt phng trnh . . . . . . . . . . . . . . . . . . . . . . . . . . . 27
1.7 Bi tp . . . . . . . . . . . . . . . . . . . . . . . . . . . . . . . . . . . . . . . . . 27
2 Phng trnh lng gic 33
2.1 Cc kin thc c bn . . . . . . . . . . . . . . . . . . . . . . . . . . . . . . . . . 33
2.1.1 Cng thc lin h gia cc hm lng gic . . . . . . . . . . . . . . . . . 33
2.1.2 Cc cng thc ca cc gc lin h vi . . . . . . . . . . . . . . . . . . 33
2.1.3 Bng du ca cc hm lng gic . . . . . . . . . . . . . . . . . . . . . . 34
2.1.4 Bng cc gi tr lng gic . . . . . . . . . . . . . . . . . . . . . . . . . . 34
2.1.5 Cng thc lng gic ca tng, hiu . . . . . . . . . . . . . . . . . . . . 34
2.1.6 Cng thc cng lng gic . . . . . . . . . . . . . . . . . . . . . . . . . . 35
2.1.7 Cng thc bin i tch thnh tng . . . . . . . . . . . . . . . . . . . . . 35
2.1.8 Cng thc gc nhn i, nhn ba - Cng thc h bc . . . . . . . . . . . 35
2.1.9 Cng thc tnh sin 2x, cos 2x, tan2x, cot 2x theo t = tan x . . . . . . . . 36
2.1.10 Bi tp . . . . . . . . . . . . . . . . . . . . . . . . . . . . . . . . . . . . 36
2.2 Cc phng trnh lng gic c bn . . . . . . . . . . . . . . . . . . . . . . . . . 37
Ngi son : Th.s Minh Tun Trang 3 Khoa T nhin - Trng CSP Nam nh
http: //www. aotrangtb. com
T
h
.
s

M
i
n
h
T
u

n
Mc lc Mc lc
2.2.1 Phng trnh sin x = m . . . . . . . . . . . . . . . . . . . . . . . . . . . 37
2.2.2 Phng trnh cos x = m . . . . . . . . . . . . . . . . . . . . . . . . . . . 37
2.2.3 Phng trnh tanx = m, cot x = m . . . . . . . . . . . . . . . . . . . . . 37
2.2.4 Cc v d . . . . . . . . . . . . . . . . . . . . . . . . . . . . . . . . . . . 38
2.2.5 Bi tp . . . . . . . . . . . . . . . . . . . . . . . . . . . . . . . . . . . . 39
2.3 Cc phng trnh lng gic khc . . . . . . . . . . . . . . . . . . . . . . . . . . 40
2.3.1 Phng trnh a sin x +b cos x = c . . . . . . . . . . . . . . . . . . . . . . 40
2.3.2 Phng trnh ng cp cha sin v cos . . . . . . . . . . . . . . . . . . . 41
2.3.3 i s ha phng trnh lng gic . . . . . . . . . . . . . . . . . . . . . 42
2.3.4 Phng trnh i xng sin, cos . . . . . . . . . . . . . . . . . . . . . . . . 43
2.3.5 Phn tch thnh nhn t . . . . . . . . . . . . . . . . . . . . . . . . . . . 44
2.3.6 S dng bt ng thc . . . . . . . . . . . . . . . . . . . . . . . . . . . . 45
2.3.7 Loi nghim khng thch hp . . . . . . . . . . . . . . . . . . . . . . . . 45
2.3.8 Bi tp . . . . . . . . . . . . . . . . . . . . . . . . . . . . . . . . . . . . 46
3 Phng trnh cha cn v du gi tr tuyt i 51
3.1 Phng trnh cha du gi tr tuyt i . . . . . . . . . . . . . . . . . . . . . . . 51
3.1.1 Kin thc cn nh . . . . . . . . . . . . . . . . . . . . . . . . . . . . . . 51
3.1.2 Cc dng bi tp . . . . . . . . . . . . . . . . . . . . . . . . . . . . . . . 51
3.1.3 Cc v d . . . . . . . . . . . . . . . . . . . . . . . . . . . . . . . . . . . 52
3.2 Phng trnh cha cn thc . . . . . . . . . . . . . . . . . . . . . . . . . . . . . 53
3.2.1 Cc dng bi tp . . . . . . . . . . . . . . . . . . . . . . . . . . . . . . . 53
3.2.2 Cc v d . . . . . . . . . . . . . . . . . . . . . . . . . . . . . . . . . . . 54
3.3 Bt phng trnh cha du gi tr tuyt i . . . . . . . . . . . . . . . . . . . . 55
3.3.1 Dng c bn . . . . . . . . . . . . . . . . . . . . . . . . . . . . . . . . . . 55
3.3.2 Cc v d . . . . . . . . . . . . . . . . . . . . . . . . . . . . . . . . . . . 55
3.4 Bt phng trnh cha cn thc . . . . . . . . . . . . . . . . . . . . . . . . . . . 55
3.4.1 Dng c bn . . . . . . . . . . . . . . . . . . . . . . . . . . . . . . . . . . 55
3.4.2 Cc v d . . . . . . . . . . . . . . . . . . . . . . . . . . . . . . . . . . . 56
3.5 Bi tp . . . . . . . . . . . . . . . . . . . . . . . . . . . . . . . . . . . . . . . . . 57
4 H phng trnh i s 61
4.1 H phng trnh bc nht . . . . . . . . . . . . . . . . . . . . . . . . . . . . . . 61
4.1.1 H phng trnh bc nht hai n . . . . . . . . . . . . . . . . . . . . . . 61
4.1.2 H phng trnh bc nht ba n . . . . . . . . . . . . . . . . . . . . . . . 62
4.1.3 H phng trnh bc nht bn n . . . . . . . . . . . . . . . . . . . . . . 62
4.2 H phng trnh bc nht - bc hai: . . . . . . . . . . . . . . . . . . . . . . . . . 63
4.3 H phng trnh ng cp bc 2: . . . . . . . . . . . . . . . . . . . . . . . . . . 64
4.3.1 Phng trnh ng cp bc 2 . . . . . . . . . . . . . . . . . . . . . . . . 64
4.3.2 H phng trnh ng cp bc 2 . . . . . . . . . . . . . . . . . . . . . . . 65
4.4 H i xng . . . . . . . . . . . . . . . . . . . . . . . . . . . . . . . . . . . . . . 67
4.4.1 H i xng loi I: . . . . . . . . . . . . . . . . . . . . . . . . . . . . . . 67
4.4.2 H i xng loi II: . . . . . . . . . . . . . . . . . . . . . . . . . . . . . . 68
4.5 H phng trnh tng qut . . . . . . . . . . . . . . . . . . . . . . . . . . . . . . 71
4.6 Bi tp . . . . . . . . . . . . . . . . . . . . . . . . . . . . . . . . . . . . . . . . . 73
Ngi son : Th.s Minh Tun Trang 4 Khoa T nhin - Trng CSP Nam nh
T
h
.
s

M
i
n
h
T
u

n
Mc lc Mc lc
5 Gii tch t hp 79
5.1 Khi qut chung . . . . . . . . . . . . . . . . . . . . . . . . . . . . . . . . . . . 79
5.2 Kin thc c bn . . . . . . . . . . . . . . . . . . . . . . . . . . . . . . . . . . . 79
5.2.1 Quy tc cng - nhn . . . . . . . . . . . . . . . . . . . . . . . . . . . . . 79
5.2.2 T hp - chnh hp - hon v . . . . . . . . . . . . . . . . . . . . . . . . . 80
5.2.3 Cng thc nh thc Newton . . . . . . . . . . . . . . . . . . . . . . . . . 80
5.3 Cc v d . . . . . . . . . . . . . . . . . . . . . . . . . . . . . . . . . . . . . . . 81
5.4 Bi tp . . . . . . . . . . . . . . . . . . . . . . . . . . . . . . . . . . . . . . . . . 84
6 Hnh phng ta 87
6.1 Vc t, im, ng thng . . . . . . . . . . . . . . . . . . . . . . . . . . . . . . 87
6.1.1 Kin thc c bn . . . . . . . . . . . . . . . . . . . . . . . . . . . . . . . 87
6.1.2 Dng bi . . . . . . . . . . . . . . . . . . . . . . . . . . . . . . . . . . . . 88
6.2 ng trn . . . . . . . . . . . . . . . . . . . . . . . . . . . . . . . . . . . . . . 93
6.2.1 Kin thc c bn . . . . . . . . . . . . . . . . . . . . . . . . . . . . . . . 93
6.2.2 Cc dng bi . . . . . . . . . . . . . . . . . . . . . . . . . . . . . . . . . 96
6.3 Ba ng Conic . . . . . . . . . . . . . . . . . . . . . . . . . . . . . . . . . . . . 103
6.3.1 Kin thc chung v 3 ng Conic . . . . . . . . . . . . . . . . . . . . . 103
6.3.2 Elip . . . . . . . . . . . . . . . . . . . . . . . . . . . . . . . . . . . . . . 103
6.3.3 Hyperbol . . . . . . . . . . . . . . . . . . . . . . . . . . . . . . . . . . . 109
6.3.4 Parabol . . . . . . . . . . . . . . . . . . . . . . . . . . . . . . . . . . . . 114
6.4 Bi tp . . . . . . . . . . . . . . . . . . . . . . . . . . . . . . . . . . . . . . . . . 116
7 Gii hn 128
7.1 Gii hn dy s . . . . . . . . . . . . . . . . . . . . . . . . . . . . . . . . . . . . 128
7.1.1 Cc tnh cht c bn ca gii hn . . . . . . . . . . . . . . . . . . . . . . 128
7.1.2 Cc v d . . . . . . . . . . . . . . . . . . . . . . . . . . . . . . . . . . . 129
7.2 Gii hn hm s . . . . . . . . . . . . . . . . . . . . . . . . . . . . . . . . . . . . 130
7.2.1 Gii hn c bn . . . . . . . . . . . . . . . . . . . . . . . . . . . . . . . . 130
7.2.2 Phng php tnh gii hn . . . . . . . . . . . . . . . . . . . . . . . . . . 131
7.2.3 Cc v d . . . . . . . . . . . . . . . . . . . . . . . . . . . . . . . . . . . 131
7.3 Bi tp . . . . . . . . . . . . . . . . . . . . . . . . . . . . . . . . . . . . . . . . . 134
8 Bt ng thc 137
8.1 Cc bt ng thc c bn . . . . . . . . . . . . . . . . . . . . . . . . . . . . . . 137
8.2 Bt ng thc Cauchy . . . . . . . . . . . . . . . . . . . . . . . . . . . . . . . . 140
8.2.1 Tm min tng, max ca tch . . . . . . . . . . . . . . . . . . . . . . . . . 140
8.2.2 Bt ng thc i xng . . . . . . . . . . . . . . . . . . . . . . . . . . . 142
8.2.3 Cc tr c iu kin . . . . . . . . . . . . . . . . . . . . . . . . . . . . . . 146
8.3 Bt ng thc Bunhiacopxki . . . . . . . . . . . . . . . . . . . . . . . . . . . . . 149
8.4 Bt ng thc Cauchy - Schwarz . . . . . . . . . . . . . . . . . . . . . . . . . . 151
8.5 Bi tp . . . . . . . . . . . . . . . . . . . . . . . . . . . . . . . . . . . . . . . . . 152
9 Hm s v th 155
9.1 Kho st v v th hm s . . . . . . . . . . . . . . . . . . . . . . . . . . . . 155
9.1.1 Kin thc cn nh . . . . . . . . . . . . . . . . . . . . . . . . . . . . . . 155
9.1.2 Cc bc kho st hm s . . . . . . . . . . . . . . . . . . . . . . . . . . 158
9.1.3 Hm a thc . . . . . . . . . . . . . . . . . . . . . . . . . . . . . . . . . 158
9.1.4 Hm phn thc . . . . . . . . . . . . . . . . . . . . . . . . . . . . . . . . 160
Ngi son : Th.s Minh Tun Trang 5 Khoa T nhin - Trng CSP Nam nh
T
h
.
s

M
i
n
h
T
u

n
Mc lc Mc lc
9.2 Cc tr v tim cn ca hm s . . . . . . . . . . . . . . . . . . . . . . . . . . . 161
9.2.1 Quy tc tm cc i v cc tiu ca hm s . . . . . . . . . . . . . . . . 161
9.2.2 Cc tr hm s . . . . . . . . . . . . . . . . . . . . . . . . . . . . . . . . 162
9.2.3 Cc bi ton tim cn . . . . . . . . . . . . . . . . . . . . . . . . . . . . 165
9.2.4 Cng c kin thc . . . . . . . . . . . . . . . . . . . . . . . . . . . . . . 168
9.3 Gi tr ln nht, nh nht ca hm s . . . . . . . . . . . . . . . . . . . . . . . 170
9.3.1 Kin thc c bn . . . . . . . . . . . . . . . . . . . . . . . . . . . . . . . 170
9.3.2 Cc bi ton n thun . . . . . . . . . . . . . . . . . . . . . . . . . . . 171
9.3.3 Bi ton gi tr ln nht, nh nht cha tham s . . . . . . . . . . . . . 173
9.3.4 Phng php min gi tr ca hm s . . . . . . . . . . . . . . . . . . . . 175
9.3.5 Phng php chiu bin thin . . . . . . . . . . . . . . . . . . . . . . . . 177
9.3.6 Cng c kin thc . . . . . . . . . . . . . . . . . . . . . . . . . . . . . . 179
9.4 Vit phng trnh tip tuyn th . . . . . . . . . . . . . . . . . . . . . . . . . 180
9.4.1 Kin thc cn nh . . . . . . . . . . . . . . . . . . . . . . . . . . . . . . 180
9.4.2 Tip tuyn vi ng cong ti im M . . . . . . . . . . . . . . . . . . . 181
9.4.3 Tip tuyn vi ng cong i qua im M . . . . . . . . . . . . . . . . . 181
9.4.4 Lp cc bi ton v s tip xc rt a dng . . . . . . . . . . . . . . . . 182
9.4.5 Cng c kin thc . . . . . . . . . . . . . . . . . . . . . . . . . . . . . . 183
9.5 Xc nh im tha mn iu kin cho trc . . . . . . . . . . . . . . . . . . . . 184
9.5.1 Kin thc c bn . . . . . . . . . . . . . . . . . . . . . . . . . . . . . . . 184
9.5.2 Tm im khng thuc mi ng cong trong h y = f(x, m) . . . . . . . 186
9.6 S tng giao . . . . . . . . . . . . . . . . . . . . . . . . . . . . . . . . . . . . . 188
9.6.1 Kin thc c bn . . . . . . . . . . . . . . . . . . . . . . . . . . . . . . . 188
9.6.2 S tng giao ca hm a thc vi trc Ox . . . . . . . . . . . . . . . . 189
9.6.3 S tng giao ca hm phn thc . . . . . . . . . . . . . . . . . . . . . . 191
9.6.4 Cng c kin thc . . . . . . . . . . . . . . . . . . . . . . . . . . . . . . 193
9.7 S tip xc ca 2 ng cong . . . . . . . . . . . . . . . . . . . . . . . . . . . . 194
9.7.1 Kin thc c bn . . . . . . . . . . . . . . . . . . . . . . . . . . . . . . . 194
9.7.2 Cc v d . . . . . . . . . . . . . . . . . . . . . . . . . . . . . . . . . . . 195
9.7.3 Cng c . . . . . . . . . . . . . . . . . . . . . . . . . . . . . . . . . . . . 198
9.8 Bin lun s nghim bng th . . . . . . . . . . . . . . . . . . . . . . . . . . . 199
9.8.1 Kin thc c bn . . . . . . . . . . . . . . . . . . . . . . . . . . . . . . . 199
9.8.2 Cc v d . . . . . . . . . . . . . . . . . . . . . . . . . . . . . . . . . . . 199
9.9 Bi tp . . . . . . . . . . . . . . . . . . . . . . . . . . . . . . . . . . . . . . . . . 202
10 Hnh khng gian ta 210
10.1 H ta , vc t, im . . . . . . . . . . . . . . . . . . . . . . . . . . . . . . . . 210
10.1.1 H ta Cc . . . . . . . . . . . . . . . . . . . . . . . . . . . . . . . 210
10.2 Php ton trn vc t . . . . . . . . . . . . . . . . . . . . . . . . . . . . . . . . . 210
10.3 im . . . . . . . . . . . . . . . . . . . . . . . . . . . . . . . . . . . . . . . . . . 211
10.3.1 Tch c hng ca 2 vc t v ngha . . . . . . . . . . . . . . . . . . . 211
10.3.2 Bi tp . . . . . . . . . . . . . . . . . . . . . . . . . . . . . . . . . . . . 212
10.4 Phng trnh mt phng . . . . . . . . . . . . . . . . . . . . . . . . . . . . . . . 213
10.4.1 Phng trnh tng qut ca mt phng . . . . . . . . . . . . . . . . . . . 213
10.4.2 Phng php xc nh mt phng . . . . . . . . . . . . . . . . . . . . . . 213
10.4.3 V tr tng i ca 2 mt phng . . . . . . . . . . . . . . . . . . . . . . 214
10.4.4 Bi tp . . . . . . . . . . . . . . . . . . . . . . . . . . . . . . . . . . . . 214
10.5 Phng trnh ng thng . . . . . . . . . . . . . . . . . . . . . . . . . . . . . . 214
Ngi son : Th.s Minh Tun Trang 6 Khoa T nhin - Trng CSP Nam nh
T
h
.
s

M
i
n
h
T
u

n
Mc lc Mc lc
10.5.1 V tr tng i ca ng thng v mt phng . . . . . . . . . . . . . . 215
10.5.2 V tr tng i ca ng thng v ng thng . . . . . . . . . . . . . 216
10.5.3 Mt s dng ton v ng thng v mt phng . . . . . . . . . . . . . . 216
10.5.4 Bi tp . . . . . . . . . . . . . . . . . . . . . . . . . . . . . . . . . . . . 218
10.6 Gc . . . . . . . . . . . . . . . . . . . . . . . . . . . . . . . . . . . . . . . . . . 220
10.6.1 Kin thc c bn . . . . . . . . . . . . . . . . . . . . . . . . . . . . . . . 220
10.6.2 Cc v d . . . . . . . . . . . . . . . . . . . . . . . . . . . . . . . . . . . 220
10.6.3 Bi tp . . . . . . . . . . . . . . . . . . . . . . . . . . . . . . . . . . . . 221
10.7 Khong cch . . . . . . . . . . . . . . . . . . . . . . . . . . . . . . . . . . . . . . 221
10.7.1 Cc cng thc khong cch . . . . . . . . . . . . . . . . . . . . . . . . . 221
10.7.2 Cc v d . . . . . . . . . . . . . . . . . . . . . . . . . . . . . . . . . . . 222
10.7.3 Bi tp . . . . . . . . . . . . . . . . . . . . . . . . . . . . . . . . . . . . 223
10.8 Mt cu v ng trn . . . . . . . . . . . . . . . . . . . . . . . . . . . . . . . . 224
10.8.1 Mt cu . . . . . . . . . . . . . . . . . . . . . . . . . . . . . . . . . . . . 224
10.8.2 V tr tng i ca mt phng v mt cu . . . . . . . . . . . . . . . . . 224
10.8.3 ng trn . . . . . . . . . . . . . . . . . . . . . . . . . . . . . . . . . . 225
10.8.4 Bi tp . . . . . . . . . . . . . . . . . . . . . . . . . . . . . . . . . . . . 226
10.9 Ta ha hnh hc khng gian . . . . . . . . . . . . . . . . . . . . . . . . . . . 227
10.9.1 Hnh chp . . . . . . . . . . . . . . . . . . . . . . . . . . . . . . . . . . . 227
11 Tch phn 229
11.1 Vi phn . . . . . . . . . . . . . . . . . . . . . . . . . . . . . . . . . . . . . . . . 229
11.1.1 nh ngha . . . . . . . . . . . . . . . . . . . . . . . . . . . . . . . . . . 229
11.1.2 Cc tnh cht . . . . . . . . . . . . . . . . . . . . . . . . . . . . . . . . . 229
11.1.3 Bng o hm cc hm s s cp thng gp . . . . . . . . . . . . . . . 229
11.2 Nguyn hm v tch phn bt nh . . . . . . . . . . . . . . . . . . . . . . . . . 230
11.2.1 nh ngha . . . . . . . . . . . . . . . . . . . . . . . . . . . . . . . . . . 230
11.2.2 Cc tnh cht . . . . . . . . . . . . . . . . . . . . . . . . . . . . . . . . . 230
11.2.3 Bng nguyn hm cc hm s s cp thng gp . . . . . . . . . . . . . 231
11.3 Cc phng php tnh tch phn . . . . . . . . . . . . . . . . . . . . . . . . . . . 232
11.3.1 Php i bin s . . . . . . . . . . . . . . . . . . . . . . . . . . . . . . . 232
11.3.2 Tch phn tng phn . . . . . . . . . . . . . . . . . . . . . . . . . . . . . 234
11.3.3 Tch phn hm phn thc . . . . . . . . . . . . . . . . . . . . . . . . . . 235
11.4 Tch phn xc nh . . . . . . . . . . . . . . . . . . . . . . . . . . . . . . . . . . 237
11.5 ng dng ca tch phn . . . . . . . . . . . . . . . . . . . . . . . . . . . . . . . 238
11.5.1 Tnh din tch . . . . . . . . . . . . . . . . . . . . . . . . . . . . . . . . . 239
11.5.2 Tnh th tch vt th trn xoay . . . . . . . . . . . . . . . . . . . . . . . 239
11.6 Bi tp . . . . . . . . . . . . . . . . . . . . . . . . . . . . . . . . . . . . . . . . . 240
12 S phc 263
12.1 Kin thc c bn . . . . . . . . . . . . . . . . . . . . . . . . . . . . . . . . . . . 263
12.1.1 Cc kin thc chung . . . . . . . . . . . . . . . . . . . . . . . . . . . . . 263
12.1.2 Cc php ton trn s phc . . . . . . . . . . . . . . . . . . . . . . . . . 263
12.2 Cc dng bi tp . . . . . . . . . . . . . . . . . . . . . . . . . . . . . . . . . . . 264
12.2.1 Thc hin cc php ton . . . . . . . . . . . . . . . . . . . . . . . . . . . 264
12.2.2 Khai cn bc 2 . . . . . . . . . . . . . . . . . . . . . . . . . . . . . . . . 264
12.2.3 Gii phng trnh i s v cc vn lin quan . . . . . . . . . . . . . . 266
12.2.4 Biu din s phc trn mt phng . . . . . . . . . . . . . . . . . . . . . . 267
Ngi son : Th.s Minh Tun Trang 7 Khoa T nhin - Trng CSP Nam nh
T
h
.
s

M
i
n
h
T
u

n
Mc lc Mc lc
12.2.5 Chng minh ng thc t hp . . . . . . . . . . . . . . . . . . . . . . . . 267
12.3 Bi tp . . . . . . . . . . . . . . . . . . . . . . . . . . . . . . . . . . . . . . . . . 267
13 Hnh hc khng gian 269
13.1 M u v hnh hc khng gian . . . . . . . . . . . . . . . . . . . . . . . . . . . 269
13.1.1 i tng ca hnh hc khng gian . . . . . . . . . . . . . . . . . . . . . 269
13.1.2 Quan h . . . . . . . . . . . . . . . . . . . . . . . . . . . . . . . . . . . . 269
13.1.3 Hnh biu din trong hnh hc khng gian . . . . . . . . . . . . . . . . . 269
13.1.4 Mt s hnh thng dng . . . . . . . . . . . . . . . . . . . . . . . . . . . 270
13.1.5 Cc tin hnh hc khng gian . . . . . . . . . . . . . . . . . . . . . . . 272
13.1.6 Cc tin . . . . . . . . . . . . . . . . . . . . . . . . . . . . . . . . . . 272
13.1.7 nh l v giao tuyn . . . . . . . . . . . . . . . . . . . . . . . . . . . . . 272
13.2 V tr tng i . . . . . . . . . . . . . . . . . . . . . . . . . . . . . . . . . . . . 272
13.2.1 V tr tng i ca mt phng v mt phng . . . . . . . . . . . . . . . 272
13.2.2 V tr tng i ca ng thng v mt phng . . . . . . . . . . . . . . 272
13.2.3 V tr tng i ca ng thng v ng thng . . . . . . . . . . . . . 272
13.2.4 Cc phng php xc nh mt phng . . . . . . . . . . . . . . . . . . . 273
13.3 Cc dng ton . . . . . . . . . . . . . . . . . . . . . . . . . . . . . . . . . . . . . 273
13.3.1 S dng tin , v tr tng i . . . . . . . . . . . . . . . . . . . . . . . 273
13.3.2 Tm giao tuyn gia 2 mt phng (Cch 1) . . . . . . . . . . . . . . . . . 276
13.3.3 Vit phng trnh ng thng i qua mt im v ct c 2 ng thng 283
13.3.4 Tm giao im ca ng thng v mt phng . . . . . . . . . . . . . . . 285
13.3.5 Chng minh 3 im thng hng, 3 ng ng quy . . . . . . . . . . . . 288
13.3.6 Thit din . . . . . . . . . . . . . . . . . . . . . . . . . . . . . . . . . . . 292
13.3.7 Bi tp . . . . . . . . . . . . . . . . . . . . . . . . . . . . . . . . . . . . 300
Ngi son : Th.s Minh Tun Trang 8 Khoa T nhin - Trng CSP Nam nh
T
h
.
s

M
i
n
h
T
u

n
Chng 1. Phng trnh i s
Chng 1
Phng trnh i s
1.1 L thuyt v a thc
1.1.1 Phn tch a thc thnh nhn t
+) Nu P(x) l mt a thc bc 2 c 2 nghim x
1
, x
2
th P(x) = a.(x x
1
).(x
2
) (a l h s bc
cao nht ca P(x)).
+) Tng qut: Nu P(x) l mt a thc bc n c n nghim x
1
, x
2
, , x
n
th
P(x) = a(x x
1
)(x x
2
) (x x
n
)
+) Mt a thc P(x) bt k bao gi cng phn tch thnh tch nhng a thc bc nht v a
thc bc 2 (v nghim).
V d 1.1.1: Phn tch cc a thc sau thnh nhn t:
a) P(x) = 2x
2
5x + 2.
b) P(x) = 3x
2
+ 12x 12
c) P(x) = 4x
3
4x
2
7x 2.
d) P(x) = 6x
3
13x
2
+ 4x + 3
Gii: a) P(x) c a = 2, x
1
= 2, x
2
=
1
2
nn P(x) = 2(x 2)

x
1
2

= (x 2)(2x 1).
b) P(x) c nghim kp x = 2 nn P(x) = 3(x 2)
2
.
c) P(x) c a = 4 v 2 nghim x =
1
2
v x = 2???
Ch : P(x) l a thc bc 3 nhng li ch c 2 nghim. Nn s c mt nghim l nghim
kp. Tt nht trong trng hp ny ta dng lc Hoocne gii quyt.
Kt qu: P(x) = 4

x +
1
2

2
(x 2).
Ngi son : Th.s Minh Tun Trang 9 Khoa T nhin - Trng CSP Nam nh
http: //aotrangtb. com
T
h
.
s

M
i
n
h
T
u

n
1.2. Phng trnh bc nht Chng 1. Phng trnh i s
d) P(x) c a = 6 v 3 nghim x = 1, x =
1
3
, x =
3
2
P(x) = 6(x 1).

x +
1
3

x
3
2

= (x 1)(3x + 1)(2x 3).


1.1.2 Tnh gi tr mt a thc, phn thc ti im l
Cch lm: Nhp hm, s dng tnh nng CALC ca my 570ES.
V d 1.1.2: Tnh gi tr biu thc:
a) y = x
3
3x
2
x 1 ti x = 1

3 v x = 1 +

3
b) y =
x
2
x 1
2x + 3
ti x = 3 +

2 v x = 3

2
Gii: a) x = 1

3 y = 4 +

3
x = 1 +

3 y = 4

3
b) x = 3 +

2 y =
43 + 31

2
73
x = 3

2 y =
43 31

2
73
1.2 Phng trnh bc nht
1.2.1 Phng php gii
Dng ca phng trnh: ax +b = 0
Cch gii:
> Vi a = 0, b = 0: Phng trnh nghim ng x R
> Vi a = 0, b = 0: Phng trnh v nghim.
> Vi a = 0 Phng trnh c nghim duy nht x =
b
a
1.2.2 Cc v d
V d 1.2.1: Gii v bin lun phng trnh: (m
2
1)x +m1 = 0
Gii: - Nu m
2
1 = 0 m = 1.
+) Vi m = 1 phng trnh tr thnh: 0x + 0 = 0. Phng trnh nghim ng x R.
+) Vi m = 1 phng trnh tr thnh: 0x 2 = 0. Phng trnh v nghim.
- Nu m
2
1 = 0 m = 1.
Phng trnh c nghim duy nht: x =
1
m + 1
V d 1.2.2: Tm im c nh ca h ng thng:(d
m
) : y = (m2)x + 2m3
Ngi son : Th.s Minh Tun Trang 10 Khoa T nhin - Trng CSP Nam nh
http: //aotrangtb. com
T
h
.
s

M
i
n
h
T
u

n
1.3. Phng trnh bc hai Chng 1. Phng trnh i s
Gii: Gi (x
0
, y
0
) l im c nh ca (d
m
)
y
0
= (m2)x
0
+ 2m3 m
m(x
0
+ 2) 2x
0
3 y
0
= 0 m

x
0
+ 2 = 0
2x
0
3 y
0
= 0

x
0
= 20
y
0
= 1
Vy im c nh ca h (d
m
) l im A(2; 1)
1.3 Phng trnh bc hai
1.3.1 Phng php gii
Dng ca phng trnh: ax
2
+bx +c = 0.
Bin lun:
~ Nu a = 0: phng trnh bc nht
~ Nu a = 0: = b
2
4ac hoc

= b
2
ac.
+) Nu < 0: Phng trnh v nghim.
+) Nu = 0: Phng trnh c nghim kp x
1
= x
2
=
b
2a
=
b

a
+) Nu > 0: Phng trnh c 2 nghim phn bit
x
1,2
=
b

2a
=
b

a
Nhm nghim:
~ Nu a +b +c = 0 th phng trnh c 2 nghim: x
1
= 1, x
2
=
c
a
~ Nu a b +c = 0 th phng trnh c 2 nghim: x
1
= 1, x
2
=
c
a
Phn tch mt tam thc bc 2 thnh nhn t.
Gi s f(x) = ax
2
+bx +c c 2 nghim x
1
, x
2
th f(x) = a(x x
1
)(x x
2
).
V d: f(x) = 2x
2
5x + 2 c 2 nghim x
1
= 2, x
2
=
1
2
nn f(x) = 2(x 2)(x
1
2
) = (x 2)(2x 1).
nh l Vi-et: Gi s x
1
, x
2
l 2 nghim ca phng trnh th ta c:

x
1
+x
2
=
b
a
x
1
x
2
=
c
a
Ngi son : Th.s Minh Tun Trang 11 Khoa T nhin - Trng CSP Nam nh
T
h
.
s

M
i
n
h
T
u

n
1.3. Phng trnh bc hai Chng 1. Phng trnh i s
nh l Vi-et o:
Nu

x +y = S
x.y = P
, x, y l 2 nghim ca phng trnh:
X
2
S.X +P = 0
Du ca nghim:
~ Pt c 2 nghim phn bit dng

> 0
S > 0
P > 0
~ Pt c 2 nghim phn bit m

> 0
S < 0
P > 0
~ Pt c 2 nghim tri du: P < 0.
~ Pt c nghim dng tng ng vi phng trnh c 2 nghim dng hoc c 2
nghim tri du

max(x
1
, x
2
) > 0
0
max (x
1
, x
2
) =

b +

2a
Nu a > 0
b

2a
Nu a < 0
Hoc ta c th xt 2 trng hp:
- Phng trnh c 2 nghim dng (khng cn phn bit) hoc c mt nghim bng
khng, mt nghim dng

0
S > 0
P 0
- Phng trnh c 2 nghim tri du P < 0.
~ Phng trnh c nghim m ta lm tng t nh trn:

0
S < 0
P 0
P < 0

0
min (x
1
, x
2
) < 0
min (x
1
, x
2
) =

2a
Nu a > 0
b +

2a
Nu a < 0
So snh nghim vi mt s:
~ (x
1
, x
2
) a.f () < 0.
~ / [x
1
, x
2
]

0
a.f () > 0
~ x
1
< x
2
<

> 0
a.f () > 0
S/2 <
Ngi son : Th.s Minh Tun Trang 12 Khoa T nhin - Trng CSP Nam nh
http: //aotrangtb. com
T
h
.
s

M
i
n
h
T
u

n
1.3. Phng trnh bc hai Chng 1. Phng trnh i s
~ x
1
> x
2
>

> 0
a.f () > 0
S/2 >
V d 1.3.1: Gii cc phng trnh sau:
a) x
2
5x + 4 = 0
b) x
2
2x 3 = 0
Gii: a) a +b +c = 0 phng trnh c nghim x
1
= 1, x =
c
a
= 4
b) a b +c = 0 phng trnh c 2 nghim x
1
= 1, x =
c
a
= 3
V d 1.3.2: Gii v bin lun phng trnh sau: (m1) x
2
(2m+ 1) x +m5 = 0.
Gii: +) TH 1: Nu m1 = 0 m = 1 thay vo phng trnh ta c:
3x 4 = 0 x =
3
4
.
+) TH 2: Nu m1 = 0 m = 1.
= (2m+ 1)
2
4 (m1) (m5) = 28m19
- Nu > 0 m >
19
28
c phng trnh c 2 nghim phn bit:
x
1,2
=
2m+ 1

28m19
2 (m1)
- Nu = 0 m =
19
28
c nghim kp:
x
1
= x
2
=
2m+ 1
2 (m1)
=
2.
19
28
+ 1
2

19
28
1
=
11
3
- Nu < 0 m <
19
28
: Phng trnh v nghim.
V d 1.3.3: Cho phng trnh x
2
(m1) x + 2m5 = 0
a) Tm m phng trnh c 2 nghim phn bit x
1
, x
2
.
b) Tm h thc lin h gia x
1
, x
2
khng ph thuc vo m.
c) Lp phng trnh bc 2 nhn 2x
1
+x
2
l nghim.
Gii: a) = (m1)
2
4 (2m5) = m
2
2m+ 1 8m+ 20 = m
2
10m+ 21.
Phng trnh c 2 nghim phn bit > 0 m
2
10m+ 21 > 0

m > 7
m < 3
Ngi son : Th.s Minh Tun Trang 13 Khoa T nhin - Trng CSP Nam nh
T
h
.
s

M
i
n
h
T
u

n
1.3. Phng trnh bc hai Chng 1. Phng trnh i s
b) S = x
1
+x
2
= m1, P = x
1
.x
2
= 2m5. Do 2S P = 2(m1) (2m5) = 3
H thc lin h x
1
, x
2
khng ph thuc m l : 2(x
1
+x
2
) x
1
.x
2
= 3
c) t u = 2x
1
+x
2
, v = x
1
+ 2x
2
. Do :
u +v = 3(x
1
+x
2
) = 3.(m1) = 3m3,
u.v = (2x
1
+x
2
)(x
1
+ 2x
2
) = 2x
2
1
+ 5x
1
.x
2
+x
2
2
= 2(x
1
+x
2
)
2
+x
1
.x
2
= 2(m1)
2
+ 2m5 = 2m
2
2m3
Do u, v l 2 nghim ca phng trnh:
X
2
(3m3)X + 2m
2
2m3 = 0
V d 1.3.4: Cho phng trnh: x
2
(m + 1)x +m +
9
4
= 0
1. Tm m phng trnh c 2 nghim x
1
, x
2
.
2. Tm m phng trnh c 2 nghim dng.
3. Tm m phng trnh c nghim dng.
4. Tm m phng trnh c 2 nghim tha mn x
1
< 1 < x
2
.
5. Tm m phng trnh c 2 nghim tha x
1
x
2
< 2
Gii: a) = (m + 1)
2
4(m+
9
4
) = m
2
+ 2m+ 1 4m9 = m
2
2m8
Phng trnh c 2 nghim phn bit > 0 m
2
2m8 > 0

m > 4
m < 2
b) Phng trnh c 2 nghim dng

> 0
S = m + 1 > 0
P = m+ 9/4 > 0

m > 4
m < 2
m > 1
m >
9
4
m > 4
c) Phng trnh c nghim dng

0
max (x
1
, x
2
) > 0

m
2
2m8 0
m+ 1 +

m
2
2m8
2
> 0

m
2
2m8 > m1

m1 < 0
m
2
2m8 0

m1 0
m
2
2m8 > (m1)
2

m > 1

m 4
m 2

m 1
m < 9/4

m 4
m < 9/4
d) Phng trnh c nghim x
1
< 1 < x
2
a.f(1) < 0
1. (1 (m+ 1) +m+ 9/4 < 0) 9/4 < 0 (V nghim)
Ngi son : Th.s Minh Tun Trang 14 Khoa T nhin - Trng CSP Nam nh
T
h
.
s

M
i
n
h
T
u

n
1.4. Phng trnh bc 3 Chng 1. Phng trnh i s
e) Phng trnh c nghim tha mn x
1
x
2
< 2

0
a.f (2) > 0
S/2 < 2

m 4 m 2
m < 17/4
m < 3

4 m < 17/4
m 2
1.4 Phng trnh bc 3
1.4.1 Tnh cht ca a thc
0 nh l Berzout: Cho P(x) l mt a thc bt k. Khi vi mi x
0
, a thc P(x)
chia a thc x x
0
c s d l P(x
0
).
O H qu: Nu x
0
tha mn P(x
0
) = 0 th P(x)
.
.
. x x
0
.
O Lc Hoocne: Gi s P(x) = a
n
x
n
+a
n1
x
n1
+ +a
1
x +a
0
.
a
n
a
n1
a
1
a
0
x
0
b
n
b
n1
b
1
b
0
b
n
= a
n
, b
n1
= b
n
.x
0
+a
n1
, b
n2
= b
n1
.x
0
+a
n2
, , b
0
= b
1
.x
0
+a
0
.
P(x) = (x x
0
)(b
n
x
n1
+b
n1
x
n2
+ +b
2
x +b
1
) +b
0
.
Nu P(x)
.
.
. x x
0
th b
0
= 0 v P(x) = (x x
0
)(b
n
x
n1
+b
n1
x
n2
+ +b
2
x +b
1
).
1.4.2 a thc bc 3
Dng ax
3
+bx
2
+cx +d = 0 (1).
Cch gii :
~ Nhm nghim : S dng my tnh nhm mt nghim x
0
no .
~ Dng lc Hooc-ne phn tch a thc trn thnh nhn t :
P(x) = (x x
0
).Q(x). Q(x) l mt a thc bc 2.
nh l Viet: Gi s x
1
, x
2
, x
3
l 3 nghim ca phng trnh (1).

x
1
+x
2
+x
3
=
b
a
x
1
x
2
+x
2
x
3
+x
3
x
1
=
c
a
x
1
x
2
x
3
=
d
a
nh l Viet o: Gi s x, y, z l 3 s tha mn

x +y +z = m
xy +yz +zx = n
xyz = p
Ngi son : Th.s Minh Tun Trang 15 Khoa T nhin - Trng CSP Nam nh
T
h
.
s

M
i
n
h
T
u

n
1.4. Phng trnh bc 3 Chng 1. Phng trnh i s
Khi x, y, z l 3 nghim ca phng trnh : X
3
mX
2
+nX p = 0
1.4.3 Cc v d
V d 1.4.1: Gii cc phng trnh sau:
a) 2x
3
x
2
+x + 4 = 0.
b) x
3
4x + 3 = 0.
Gii: a) Dng my tnh ta thy c mt nghim l : x = 1.
Dng lc Hooc-ne ta c:
2 1 1 4
1 2 3 4 0
Phng trnh (x + 1) (2x
2
3x + 4) = 0

x = 1
2x
2
3x + 4 = 0Phng trnh v nghim
x = 1.
b) Dng my tnh ta nhm c nghim x = 1.
1 0 4 3
1 1 1 3 0
Phng trnh (x 1) (x
2
+x 3) = 0

x = 1
x
2
+x 3 = 0

x = 1
x =
1

13
2
V d 1.4.2: Cho phng trnh 2x
3
3x
2
5x + 5 = 0
a) Chng minh phng trnh c 3 nghim x
1
, x
2
, x
3
phn bit.
b) Tnh P = 3 (x
2
1
+x
2
2
+x
2
3
) 2 (x
3
1
+x
3
2
+x
3
3
).
Gii: a) t f(x) = 2x
3
3x
2
5x+5. Ta c f(2) = 13, f(1) = 5, f(1) = 1, f(3) = 17.
Ta c f(2).f(1) < 0 nn tn ti x
1
(2; 1) sao cho f(x
1
) = 0.
f(1).f(1) < 0 nn tn ti x
2
(1; 1) sao cho f(x
2
) = 0.
f(1).f(3) < 0 nn tn ti x
3
(1; 3) sao cho f(x
3
) = 0.
Do ta c f(x
1
) = f(x
2
) = f(x
3
) = 0 v x
1
< x
2
< x
3
nn phng trnh f(x) = 0 c 3
nghim phn bit.
b) Theo nh l Viet ta c:

x
1
+x
2
+x
3
=
3
2
x
1
x
2
+x
2
x
3
+x
3
x
1
=
5
2
x
1
x
2
x
3
=
5
2
Ngi son : Th.s Minh Tun Trang 16 Khoa T nhin - Trng CSP Nam nh
T
h
.
s

M
i
n
h
T
u

n
1.5. Phng trnh bc 4 Chng 1. Phng trnh i s
x
2
1
+x
2
2
+x
2
3
= (x
1
+x
2
+x
3
)
2
2 (x
1
x
2
+x
2
x
3
+x
3
x
1
) =
29
4
x
3
1
+x
3
2
+x
3
3
= (x
3
1
+x
3
2
+x
3
3
3x
1
x
2
x
3
) + 3x
1
x
2
x
3
= (x
1
+x
2
+x
3
) (x
2
1
+x
2
2
+x
2
3
(x
1
x
2
+x
2
x
3
+x
3
x
1
)) + 3x
1
x
2
x
3
=
3
2

29
4
+
5
2

+ 3.

5
2

=
57
8
Do ta c P = 3.
29
4
2.
57
8
=
15
2
.
V d 1.4.3: Gii h phng trnh:

x +y +z = 2
x
2
+y
2
+z
2
= 6
x
3
+y
3
+z
3
= 8
Gii: Phng trnh tng ng vi

x + y +z = 2
(x +y +z)
2
2 (xy +yz +zx) = 6
(x
3
+y
3
+z
3
3xyz) + 3xyz = 8

x +y +z = 2
2
2
2 (xy +yz +zx) = 6
(x +y +z) (x
2
+y
2
+z
2
xy yz zx) + 3xyz = 8

x +y +z = 2
xy +yz + zx = 1
2 (6 + 1) + 3xyz = 8

x +y +z = 2
xy +yz +zx = 1
xyz = 2
T ta c x, y, z l 3 nghim ca phng trnh:
X
3
2X
2
X + 2 = 0

X = 1
X = 1
X = 2
Vy h c 6 nghim phn bit
(1; 1; 2) , (1; 2; 1) , (1; 1; 2) , (1; 2; 1) , (2; 1; 1) , (2; 1; 1)
1.5 Phng trnh bc 4
1.5.1 Dng tng qut
ax
4
+bx
3
+cx
2
+dx +e = 0 (a = 0)
Hng gii:
~ Dng tnh nng SOLVE hoc TABLE ca my tnh fx-570ES, fx-500ES nhm
nghim ca phng trnh, sau dng lc Hooc-ne phn tch thnh phng
trnh bc 3 v gii tip nh trn.
~ Tuy nhin mt s trng hp cch gii trn tr nn v hiu hoc qu phc tp khng
cn thit, nhng trng hp c cch gii ring bit.
Ngi son : Th.s Minh Tun Trang 17 Khoa T nhin - Trng CSP Nam nh
T
h
.
s

M
i
n
h
T
u

n
1.5. Phng trnh bc 4 Chng 1. Phng trnh i s
1.5.2 Cc dng ca phng trnh bc 4
0 Phng trnh trng phng : ax
4
+bx
2
+c = 0.
Cch gii: t t = x
2
0. Phng trnh tr thnh : at
2
+bt +c = 0.
O Phn tch thnh nhn t:
Cch gii: Bit c mt nghim, hoc dng cch nhm, s dng hng ng thc phn
tch thnh nhn t, quy v phng trnh bc thp hn.
O Phng trnh i xng: ax
4
+bx
3
+cx
2
+dx +e = 0 tha mn

d
b

2
=
e
a
Cch gii: Xt x = 0 thay vo phng trnh xem c tha mn khng?
Vi x = 0. Chia c 2 v ca phng trnh cho x
2
ta c:
ax
2
+bx +c +
d
x
+
e
x
2
= 0 a

x
2
+
e
ax
2

+b

x +
d
bx

+c = 0
t t = x +
b
dx
()
t
2
= x
2
+
b
2
d
2
x
2
+
2d
b
= x
2
+
e
ax
2
+
2d
b
.
Phng trnh tr thnh: a

t
2

2d
b

+bt +c = 0
Gii phng trnh bc 2 n t. Sau thay vo () tm x.
O Phng trnh dng (x +a) (x +b) (x +c) (x +d) = e sao cho a +b = c +d.
Cch gii: Phng trnh (x
2
+ (a +b) x +ab) (x
2
+ (c +d) x +cd) = e
t t = x
2
+ (a +b) x = x
2
+ (c +d) x ()
Thay vo phng trnh ta c:
(t +ab) (t +cd) = e
Gii phng trnh bc 2 n t sau thay vo () tm x.
O Phng trnh dng (x +a) (x +b) (x +c) (x +d) = ex
2
sao cho ab = cd.
Cch gii: ging cch gii phng trnh i xng.
Nu x = 0: ta c abcd = 0.
Nu x = 0: Phng trnh (x
2
+ (a +b) x +ab) (x
2
+ (c +d) x +cd) = ex
2
.

x +
ab
x
+a +b

x +
cd
x
+c +d

= e
t t = x +
ab
x
= x +
cd
x
(). Phng trnh tr thnh:
(t +a +b) (t +c +d) = e
Gii phng trnh bc 2 ta tm c t. Thay vo () tm x.
Ngi son : Th.s Minh Tun Trang 18 Khoa T nhin - Trng CSP Nam nh
T
h
.
s

M
i
n
h
T
u

n
1.5. Phng trnh bc 4 Chng 1. Phng trnh i s
1.5.3 Cc v d
V d 1.5.1: Gii phng trnh 2x
4
x
2
3 = 0
Gii: t t = x
2
0. Phng trnh tr thnh :
2t
2
t 3 = 0

t = 1 (loi)
t =
3
2
t =
3
2
x
2
=
3
2
x =

6
2
V d 1.5.2: Gii cc phng trnh sau:
a) 8x
4
+ 16x
3
8x
2
91x 42 = 0.
b) x
4
4x
3
+ 4x
2
16 = 0.
c) x
4
4x 1 = 0.
Gii: a) Dng my tnh ta nhm c mt nghim l x = 2.
Dng lc Hooc - ne ta c:
8 16 8 91 42
2 8 32 56 21 0
Phng trnh (x 2) (8x
3
+ 32x
2
+ 56x + 21) = 0.
Tip tc ta nhm c 1 nghim l x =
1
2
. Theo lc Hooc - ne ta c:
8 32 56 21

1
2
8 28 42 0
Phng trnh (x 2)

x +
1
2

(8x
2
+ 28x + 42) = 0

x = 2
x =
1
2
8x
2
+ 28x + 42 = 0

V nghim

b) Phng trnh (x
2
2x)
2
4
2
= 0 (x
2
2x 4) (x
2
2x + 4) = 0

x
2
2x 4 = 0
x
2
2x + 4 = 0

V nghim

x = 1

5
c) Phng trnh x
4
+ 2x
2
+ 1 2 (x
2
+ 2x + 1) = 0
(x
2
+ 1)
2

2 (x + 1)

2
= 0

x
2
+ 1

2 (x + 1)

x
2
+ 1 +

2 (x + 1)

= 0

x
2

2x + 1

2

x
2
+

2x + 1 +

= 0

x
2

2x + 1

2 = 0
x
2
+

2x + 1 +

2 = 0

V nghim

x =

2 + 4

2
2
.
Ngi son : Th.s Minh Tun Trang 19 Khoa T nhin - Trng CSP Nam nh
T
h
.
s

M
i
n
h
T
u

n
1.5. Phng trnh bc 4 Chng 1. Phng trnh i s
V d 1.5.3: Gii cc phng trnh sau:
a) x
4
+ 4x
3
x
2
+ 8x + 4 = 0.
b) 2x
4
3x
3
3x
2
+ 3x + 2 = 0.
Gii: a) Vi x = 0, phng trnh tr thnh 2 = 0 (v l). Vy x = 0.
Chia c 2 v phng trnh cho x
2
ta c
x
2
+ 4x 1 +
8
x
+
4
x
2
= 0

x
2
+
4
x
2

+ 4

x +
2
x

1 = 0
t t = x +
2
x
t
2
= x
2
+
4
x
2
+ 4
Phng trnh tr thnh : t
2
4 + 4t 1 = 0
t
2
+ 4t 5 = 0

t = 1
t = 5
+) Vi t = 1: x +
2
x
= 1 x
2
x + 2 = 0

v nghim

+) Vi t = 5: x +
2
x
= 5 x
2
+ 5x + 2 = 0 x =
5

17
2
b) x = 0 khng l nghim ca phng trnh nn chia c 2 v ca phng trnh cho x
2
= 0 ta
c
2x
2
3x 3 +
3
x
+
2
x
2
= 0 2

x
2
+
1
x
2

x
1
x

3 = 0
t t = x
1
x
t
2
= x
2
+
1
x
2
2, thay vo phng trnh ta c:
2 (t
2
+ 2) 3t 3 = 0 2t
2
3t + 1 = 0

t = 1
t =
1
2
+) Vi t = 1: x
1
x
= 1 x
2
x 1 = 0 x =
1

5
2
+) Vi t =
1
2
: x
1
x
=
1
2
2x
2
x 2 = 0 x =
1

17
4
.
V d 1.5.4: Gii phng trnh sau : x(x + 1) (x 3) (x 2) = 2
Gii: Phng trnh (x
2
2x) (x
2
2x 3) = 2
t t = x
2
2x. Phng trnh tr thnh:
t (t 3) = 2 t
2
3t + 2 = 0

t = 1
t = 2
+) Vi t = 1: x
2
2x = 1 x
2
2x 1 = 0 x = 1

2.
+) Vi t = 2: x
2
2x = 2 x
2
2x 2 = 0 x = 1

3.
V d 1.5.5: Gii phng trnh (x 2) (x + 3) (x 1) (x + 6) = 21x
2
Ngi son : Th.s Minh Tun Trang 20 Khoa T nhin - Trng CSP Nam nh
T
h
.
s

M
i
n
h
T
u

n
1.6. Du ca a thc Chng 1. Phng trnh i s
Gii: Phng trnh (x
2
+x 6) (x
2
+ 5x 6) = 21x
2
Do x = 0 khng l nghim ca
phng trnh nn chia c 2 v ca phng trnh cho x
2
= 0 ta c:

x + 1
6
x

x + 5
6
x

= 21
t t = x
6
x
thay vo phng trnh ta c:
(t + 1) (t + 5) = 21 t
2
+ 6t + 5 = 21
t
2
+ 6t 16 = 0

t = 8
t = 2
+) Vi t = 8: x
6
x
= 8 x
2
+ 8x 6 = 0 x = 4

22
+) Vi t = 2: x
6
x
= 2 x
2
2x 6 = 0 x = 1

7
1.6 Du ca a thc
1.6.1 a thc bc 1 - bc 2
0 Dng: P(x) = ax +b (a = 0). Ta c bng xt du:
x
b
a
+
P(x) sign(a) 0 +sign(a)
sign(a) l du ca a.
O Dng P(x) = ax
2
+bx +c (a = 0).
= b
2
4ac. Ta c cc trng hp sau:
+) < 0: Du ca a thc l:
x +
P(x) +sign(a)
+) = 0: Du ca a thc l:
x
b
2a
+
P(x) +sign(a) 0 +sign(a)
+) > 0: P(x) c 2 nghim phn bit x
1
, x
2
. Du ca a thc l :
x
x
1
x
2 +
P(x) +sign(a) 0 sign(a) 0 +sign(a)
Ch : Nu P(x) l mt a thc bc 2 ta lun c:
Ngi son : Th.s Minh Tun Trang 21 Khoa T nhin - Trng CSP Nam nh
T
h
.
s

M
i
n
h
T
u

n
1.6. Du ca a thc Chng 1. Phng trnh i s
> P(x) > 0 x R

< 0
a > 0
> P(x) < 0 x R

< 0
a < 0
> P(x) 0 x R

0
a > 0
V d 1.6.1: Xt du ca cc biu thc sau:
a) P(x) = 2x + 3
b) P(x) = x
2
+ 4x 5
c) P(x) = 4x
2
12x + 9
d) P(x) = x
2
x 6
e) P(x) = 2x
2
+ 3x + 2
Gii: a) P(x) = 0 x =
3
2
, a = 2 < 0. Do du ca P(x) l:
x
3
2
+
P(x) + 0
b) = 4 < 0, a = 1 < 0, ta c du ca P(x) l:
x +
P(x) +
c) = 0, a = 4 > 0 v du ca P(x) l:
x
3
2
+
P(x) + 0 +
d) > 0, x
1
= 3, x
2
= 2, a = 1 > 0. Do du ca P(x) l:
x 2 3 +
P(x) + 0 0 +
e) > 0, x
1
=
1
2
, x
2
= 2, a = 2 < 0. Do du ca P(x) l:
x
1
2
2 +
P(x) 0 + 0
Ngi son : Th.s Minh Tun Trang 22 Khoa T nhin - Trng CSP Nam nh
T
h
.
s

M
i
n
h
T
u

n
1.6. Du ca a thc Chng 1. Phng trnh i s
Ch : Trong mt bi ton thng thng khng ai li hi trc tip du ca mt a thc
m thng hi cc cu hi v gii bt phng trnh. Chng ta cn xt du ca cc a thc
tng ng t tm thy c tp nghim ca bt phng trnh. Chng hn:
2x + 3 > 0 th tp nghim S =

;
3
2

.
2x + 3 0 th tp nghim S =

3
2
; +

x
2
+ 4x 5 > 0 th S =
x
2
+ 4x 5 0 th S =
x
2
+ 4x 5 < 0 th S = R
x
2
+ 4x 5 0 th S = R
4x
2
12x + 9 > 0 th S = R`

3
2

.
4x
2
12x + 9 0 th S = R
4x
2
12x + 9 < 0 th S =
4x
2
12x + 9 0 th S =

3
2

x
2
x 6 > 0 th S = (; 2) (3; +)
x
2
x 6 0 th S = (; 2] [3; +)
x
2
x 6 < 0 th S = (2; 3)
x
2
x 6 0 th S = [2; 3]
2x
2
+ 3x + 2 > 0 th S =

1
2
; 2

2x
2
+ 3x + 2 0 th S =

1
2
; 2

2x
2
+ 3x + 2 < 0 th S =

;
1
2

(2; +)
2x
2
+ 3x + 2 0 th S =

;
1
2

[2; +)
V d 1.6.2: Cho tam thc bc 2: P(x) = (3m1)x
2
2(m+ 1)x + 2
Ngi son : Th.s Minh Tun Trang 23 Khoa T nhin - Trng CSP Nam nh
T
h
.
s

M
i
n
h
T
u

n
1.6. Du ca a thc Chng 1. Phng trnh i s
a) Tm m P(x) = 0 c 2 nghim phn bit x
1
, x
2
.
b) Tm m f(x) =

P(x) xc nh trn R.
c) Tm m f(x) = ln P(x) xc nh trn R.
Gii: a) Ta c

= (m + 1)
2
2(3m1) = m
2
4m+ 3.
phng trnh P(x) = 0 c 2 nghim phn bit

> 0
a = 0

m
2
4m+ 3 > 0
3m1 = 0

m > 3
m < 1
m =
1
3

m > 3

m < 1
m =
1
3
b) f(x) =

P(x) xc nh trn R P (x) 0 x R.


+) Nu 3m1 = 0 m =
1
3
khi :
P(x) =
8
3
x + 2, r rng P(3) = 6 < 0 nn P(x) 0 khng ng vi mi x R.
+) Nu 3m1 = 0. Khi P(x) l mt a thc bc 2 do :
P(x) 0 x R

0
a > 0

m
2
4m+ 3 0
3m1 > 0

1 m 3
m >
1
3
1 m 3
Kt lun: 1 m 3 tha mn iu kin bi ton.
c) f(x) = ln P(x) xc nh trn R P (x) > 0 x R
+) Nu 3m1 = 0 m =
1
3
khi :
P(x) =
8
3
x + 2, r rng P(3) = 6 < 0 nn P(x) > 0 khng ng vi mi x R.
+)Nu 3m1 = 0. Khi P(x) l mt a thc bc 2 do :
P(x) > 0 x R

> 0
a > 0

m
2
4m+ 3 > 0
3m1 > 0

1 < m < 3
m >
1
3
1 < m < 3
Kt lun: 1 < m < 3 tha mn iu kin bi ton.
1.6.2 a thc - Phn thc tng qut
a thc bc n: P(x) = a
n
x
n
+a
n1
x
n1
+ +a
1
x +a
0
.
Phn thc hu t: f(x) =
P(x)
Q(x)
. Trong P(x), Q(x) l cc a thc.
Ngi son : Th.s Minh Tun Trang 24 Khoa T nhin - Trng CSP Nam nh
T
h
.
s

M
i
n
h
T
u

n
1.6. Du ca a thc Chng 1. Phng trnh i s
nh l 1 (nh l c bn i s). Cho P(x) l mt a thc bt k th P(x) s phn tch thnh
tch cc a thc bc nht v a thc bc 2. Hn th na cc a thc bc u c < 0
nh l 2. Cho f(x) =
P(x)
Q(x)
l mt phn thc hu t no . Khi trn khong gia 2 khng
im lin tip ca f(x), hm f(x) ch mang mt du.
Khng im l nhng gi tr ca x m P(x) = 0, hoc Q(x) = 0.
nh l 3. Cho f(x) =
P(x)
Q(x)
, khi bin x chy qua khng im bi chn th f(x) khng i du,
cn qua khng im bi l th f(x) i du
x
0
l khng im bi chn (t. l) ca f(x) nu n l mt khng im ca f(x) v f(x) cha
nhn t (x x
0
)
k
vi k Z v k l s chn (t. l).
Cch xt du phn thc hu t: xt du ca mt phn thc hu t ta phn tch
cc a thc ca t v mu thnh tch cc a thc bc 1, v bc 2. Cc a thc bc 2 nu
c 0 ta phn tch chng thnh tch cc a thc bc 1, cn nu < 0 ta thay th a
thc bi h s ca hng t bc 2. Cui cng ta c phn thc ch cn tch cc a thc
bc 1. Dng cc nh l trn xt du.
V d 1.6.3: Xt du ca biu thc sau:
a) f(x) =
(x + 1)
2
.(x 2)
3
. (2x 1)
(x
2
+ 2x + 2)
7
(2x + 1)
5
(1 4x) (x
2
+ 4x 5)
3
b) f(x) =

x
2
4x + 3. (2x
2
5x + 2)
Gii: a) Gii cc phng trnh:
+) x + 1 = 0 x = 1.
+) x 2 = 0 x = 2.
+) 2x 1 = 0 x =
1
2
+) x
2
+ 2x + 2 = 0, < 0, a = 1.
+) 2x + 1 = 0 x =
1
2
+) 1 4x = 0 x =
1
4
+) x
2
+ 4x 5 = 0, < 0, a = 1.
Do du ca f(x) l du ca g(x) vi:
g(x) =
(x + 1)
2
(x 2)
3
(2x 1)
1
7
(2x + 1)
5
(1 4x) (1)
3
=
(x + 1)
2
(x 2)
3
(2x 1)
(2x + 1)
5
(1 4x)
Cc khng im x = 1; 2;
1
2
;
1
2
;
1
4
. Ta c bng du:
Ngi son : Th.s Minh Tun Trang 25 Khoa T nhin - Trng CSP Nam nh
T
h
.
s

M
i
n
h
T
u

n
1.6. Du ca a thc Chng 1. Phng trnh i s
x 1
1
2
1
4
1
2
2 +
g(x) + 0 + + 0 0 +
f(x) + 0 + + 0 0 +
b) TX: x
2
4x + 3 0

x 3
x 1
Ta c bng xt du:
x
1
2
1 2 3 +

x
2
4x + 3
+ + 0 0 +
2x
2
5x + 2
+ 0 0 + +
f(x) + 0 0 0 +
Dng kt qu ca v d trn ta c th gii c cc bt phng trnh:
+)
(x + 1)
2
.(x 2)
3
. (2x 1)
(x
2
+ 2x + 2)
7
(2x + 1)
5
(1 4x) (x
2
+ 4x 5)
3
0
c tp nghim l: S =

;
1
2

1
4
;
1
2

[2; +)
+)
(x + 1)
2
.(x 2)
3
. (2x 1)
(x
2
+ 2x + 2)
7
(2x + 1)
5
(1 4x) (x
2
+ 4x 5)
3
> 0
c tp nghim l: S = (; 1)

1;
1
2

1
4
;
1
2

(2; +)
+)
(x + 1)
2
.(x 2)
3
. (2x 1)
(x
2
+ 2x + 2)
7
(2x + 1)
5
(1 4x) (x
2
+ 4x 5)
3
0
c tp nghim l: S =

1
2
;
1
4

1
2
; 2

1
+)
(x + 1)
2
.(x 2)
3
. (2x 1)
(x
2
+ 2x + 2)
7
(2x + 1)
5
(1 4x) (x
2
+ 4x 5)
3
< 0
c tp nghim l: S =

1
2
;
1
4

1
2
; 2

+)

x
2
4x + 3. (2x
2
5x + 2) 0 c tp nghim S =

;
1
2

[3; +) 1
+)

x
2
4x + 3. (2x
2
5x + 2) > 0 c tp nghim S =

;
1
2

(3; +)
+)

x
2
4x + 3. (2x
2
5x + 2) 0 c tp nghim S =

1
2
; 1

3
+)

x
2
4x + 3. (2x
2
5x + 2) < 0 c tp nghim S =

1
2
; 1

Ngi son : Th.s Minh Tun Trang 26 Khoa T nhin - Trng CSP Nam nh
T
h
.
s

M
i
n
h
T
u

n
1.7. Bi tp Chng 1. Phng trnh i s
1.6.3 Gii h bt phng trnh
Kin thc cn nh
gii c h cc bt phng trnh ta phi bit c cc thao tc ly giao v ly hp ca 2
tp hp. C th nh sau:
Ly giao: phi ng thi thuc 2 (hay nhiu) tp hp. Biu din tng tp hp mt trn
trc s, xa nhng phn khng thuc tp hp i. Phn cn trng (cha b gch) chnh
l tp hp cn tm.
Ly hp: ch cn thuc mt trong 2 (hay nhiu) tp hp. Biu din tng tp trn trc s:
xa nhng phn khng thuc tp . Hp ca n tp hp l nhng tp khng b xa khng
qu n 1 ln.
Hc sinh thng gp kh khn khi ly hp 2 tp hp, thng ch lm tt vi trng hp
ly giao.
V d 1.6.4: Tnh tp hp X trong cc trng hp sau:
a) X = A B C vi A = (2; 1] [2; +), B = [3; 0), C = (; 1].
b) X = A B vi A = (3; 3) v B = [1; 5].
Gii: a) X = (2; 1]
b) X = (3; 5]
V d 1.6.5: Gii h phng trnh sau:
a)

x
2
3x + 2 > 0
2x
2
3x 2
3x + 1
0
b)

x
2
+ 3x 4 > 0
2x + 1
2x
2
5x + 2
0
Gii: a) x

;
1
2

1
3
; 1

b) x

;
1
2

1
2
; +

1.7 Bi tp
Bi 1.1: Gii v bin lun phng trnh sau theo m:
a) (m
2
+m)x +m = 2x +m
2
.
b)
mx +m2x
x 2
= 0
Ngi son : Th.s Minh Tun Trang 27 Khoa T nhin - Trng CSP Nam nh
T
h
.
s

M
i
n
h
T
u

n
1.7. Bi tp Chng 1. Phng trnh i s
Bi 1.2: Cho phng trnh: (m
2
4)x
2
+ 2(m+ 2)x + 1 = 0
a) Tm m phng trnh c nghim.
b) Tm m phng trnh c nghim duy nht.
Bi 1.3: Bin lun s nghim ca phng trnh sau theo m:
x
3
m(x + 2) +x
2
+ 4 = 0
Bi 1.4: Gii v bin lun phng trnh sau theo m:
a) x
2
mx + 3m8 = 0.
b) x
2
mx +m
2
3 = 0.
c) (m2)x
2
2(m+ 1)x +m = 0.
Bi 1.5: Cho phng trnh 2x
2
+ 7x + 1 = 0. Gi x
1
, x
2
l 2 nghim ca phng trnh.
a) Tnh gi tr ca cc biu thc:
A = 2

x
2
1
+x
2
2

x
3
1
+x
3
2

B = x
2
1
x
3
2
+x
3
1
x
2
2
+ 2 [x
1
x
2
[
b) Tm phng trnh vi h s nguyn nhn x
1
+ 2x
2
, 2x
1
+x
2
l nghim.
c) Tm phng trnh vi h s nguyn nhn x
1
+
1
x
2
l nghim.
Bi 1.6: Tm m phng trnh 3x
2
+4(m1)x +m
2
4m+1 = 0 c 2 nghim phn bit x
1
,
x
2
tha mn
1
x
1
+
1
x
2
=
1
2
.(x
1
+x
2
)
Bi 1.7: Cho phng trnh x
2
mx + (m2)
2
= 0.
a) Tm m phng trnh c 2 nghim phn bit.
b) Tm gi tr ln nht, nh nht ca biu thc: F = x
1
x
2
+ 2x
1
+ 2x
2
.
Bi 1.8: Tm m phng trnh : x
2
(2m + 1)x + m
2
+ 1 = 0 c 2 nghim x
1
, x
2
tha mn
x
1
= 2x
2
.
Bi 1.9: Tm m phng trnh x
2
mx + 3m8 = 0.
a) C 2 nghim phn bit.
b) C 2 nghim dng.
c) C 2 nghim m.
d) C 2 nghim tri du.
e) C nghim dng.
Ngi son : Th.s Minh Tun Trang 28 Khoa T nhin - Trng CSP Nam nh
T
h
.
s

M
i
n
h
T
u

n
1.7. Bi tp Chng 1. Phng trnh i s
Bi 1.10: Bin lun theo m s nghim ca phng trnh :
a) x
4
2mx
2
+m + 12 = 0.
b) mx
4
(2m5)x
2
+m+ 1 = 0.
c) 4
x
(m+ 2)2
x+1
+m
2
4 = 0.
Bi 1.11: Gii cc phng trnh sau:
a) 2x
3
8x
2
+x + 14 = 0.
b) x
3
x
2
+ 2 = 0.
c) 3x
3
4x
2
5x + 6 = 0.
Hng dn. a) x = 2.
b) x = 1.
c) x = 1, x =
1

73
6
.
Bi 1.12: Gii cc phng trnh sau:
a) 2x
4
5x
2
7 = 0.
b) x
4
5x
2
+ 6 = 0.
c) x
4
5x
3
12x
2
+ 15x + 9 = 0
d) 2x
4
+ 3x
3
16x
2
17x + 12 = 0
e) x
4
5x
3
+ 8x
2
10x + 4 = 0.
f) x
4
2x
3
5x
2
+ 2x + 1 = 0.
g) (x
2
2x)(x
2
+ 4x + 3) = 7.
h) (x
2
3x + 2)(x
2
+ 9x + 18) = 12x
2
.
i) x
4
4x
3
+ 8x 12 = 0.
j) x
4
+ 2x
3
2x
2
9x 6 = 0.
k) x
4
3x
3
2x
2
+ 5x + 3 = 0.
l) x
4
5x
3
12x
2
+ 15x + 9 = 0.
m) 2x
4
+ 3x
3
16x
2
17x + 12 = 0.
Hng dn. a) x =

14
2
b) x = 2

2.
Ngi son : Th.s Minh Tun Trang 29 Khoa T nhin - Trng CSP Nam nh
T
h
.
s

M
i
n
h
T
u

n
1.7. Bi tp Chng 1. Phng trnh i s
c) x =
1

5
2
, x =
3

13
2
.
d) x =
1

29
2
.
e) x = 3, x = 2, x =
7

73
2
f) x = 1

7.
g) x = 2, x = 1.
Bi 1.13: Cho bt phng trnh: (m+ 1)x +m + 2 > 0.
a) Gii v bin lun bt phng trnh.
b) Tm m bt phng trnh nghim ng vi mi x 2.
Bi 1.14: Tm a h bt phng trnh sau v nghim:

x
2
+ 7x 8 < 0
a
2
x + 1 > 3 + (3a 2) x
Bi 1.15: Tm m cc bt phng trnh sau nghim ng vi mi x:
a) (m1)x
2
(2m+ 1)x +m + 3 > 0.
b) x
2
2mx +m+ 12 > 0.
c) x
2
2x +m[x 1[ +m
2
2 > 0.
d)
2
3

x
2
mx + 1
x
2
x + 1

3
2
.
Bi 1.16: Gii cc bt phng trnh sau:
a) 4x
2
+ 12x 9 < 0.
b) 2x
2
5x + 4 > 0.
c) x
2
4x + 5 0.
Hng dn. a) x =
3
2
.
b) x R.
c) x [5; 1].
Bi 1.17: Tm a biu thc

(a + 1) x
2
2 (a 1) x + 3a 3 c ngha vi mi x.
Bi 1.18: Gii v bin lun bt phng trnh sau theo tham s m:
a) x
2
mx + 2m3 0.
b) (3m8)x
2
+mx + 1 < 0.
Ngi son : Th.s Minh Tun Trang 30 Khoa T nhin - Trng CSP Nam nh
T
h
.
s

M
i
n
h
T
u

n
1.7. Bi tp Chng 1. Phng trnh i s
Bi 1.19: Gii cc bt phng trnh sau:
a) x
3
3x
2
+ 2 > 0.
b) x
4
+ 4x
3
+ 4x
2
25 > 0.
c) x
4
4x 1 < 0.
d) x(x + 1)(x + 2)(x + 3) > 24.
e) x
2
+x
3
x
2
+x 2
0
f)
1
x + 1
+
1
x + 2
+
1
x + 3
0
g) x
4
4x
3
+ 8x 5 < 0.
h) x
4
2x
3
+ 6x 9 < 0.
i)
3x + 7
x
2
x 2
5.
Hng dn. a) x

3; 1

1 +

3; +

b) x (; 1

6) (1 +

6; +)
c) x

2 + 4

2
2
;

2 +

2 + 4

2
2

d) x (; 4) (1; +).
e) x

13
2
; 2

1;
1 +

13
2

f) x

3; 2
1

2; 2 +
1

(1; +)
g) x

6; 1

1; 1 +

h) x (

3;

3)
i) x (; 1)

3
5
; 1

(2; +)
Bi 1.20: Gii cc bt phng trnh sau:
a)

x
2
2x 3.x
2
.(2x + 1)
3
(3x 8) .(x 2)
5
0
b)
(x 1)
4
. (x + 2)
(x + 3) . (2x 5)
< 0
Ngi son : Th.s Minh Tun Trang 31 Khoa T nhin - Trng CSP Nam nh
T
h
.
s

M
i
n
h
T
u

n
1.7. Bi tp Chng 1. Phng trnh i s
Hng dn. a) x [3; +) 1
b) x (; 3) (2; 1)

1;
5
2

Bi 1.21: Gii h bt phng trnh sau:


a)

(2x
2
+x 2) (x 2) > 0
x
2
2x 8
x + 2
0
b)

x
3
x
2
x 2 0
(x
2
+ 2x 3) .

x + 1 0
Hng dn. a) x

17
4
;
1 +

17
4

(2; 4]
b) x [1; +) 1
Bi 1.22: Tnh gi tr biu thc trong cc trng hp sau:
a) y = x
3
3x
2
2x 1 khi x = 1

2.
b) y =
x
2
x 4
2x + 1
khi x = 2 +

3.
Ngi son : Th.s Minh Tun Trang 32 Khoa T nhin - Trng CSP Nam nh
T
h
.
s

M
i
n
h
T
u

n
Chng 2. Phng trnh lng gic
Chng 2
Phng trnh lng gic
2.1 Cc kin thc c bn
2.1.1 Cng thc lin h gia cc hm lng gic
sin
2
+ cos
2
= 1
1
cos
2

= 1 + tan
2

1
sin
2

= 1 + cot
2

tancot = 1
Nhn xt:
> Nu bit mt trong cc gi tr lng gic th ta c th tnh c cc gi tr lng gic cn
li.
> sin , cos [1; 1]
2.1.2 Cc cng thc ca cc gc lin h vi
v :
cos( ) = cos sin( ) = sin
tan( ) = tan cot( ) = cot
v :
cos() = cos sin() = sin
tan() = tan cot() = cot
v +:
cos( +) = cos sin( +) = sin
tan( +) = tan cot( +) = cot
v

2
:
Ngi son : Th.s Minh Tun Trang 33 Khoa T nhin - Trng CSP Nam nh
T
h
.
s

M
i
n
h
T
u

n
2.1. Cc kin thc c bn Chng 2. Phng trnh lng gic
cos(

2
) = sin sin(

2
) = cos
tan(

2
) = cot cot(

2
) = tan
2.1.3 Bng du ca cc hm lng gic
I II III IV
cos + +
sin + +
tan + +
cot + +
2.1.4 Bng cc gi tr lng gic
0
0
30
0
45
0
60
0
90
0
120
0
135
0
150
0
180
0
0

2
2
3
3
4
5
6

cos 1

3
2

2
2
1
2
0
1
2

2
2

3
2
1
sin 0
1
2

2
2

3
2
1

3
2

2
2
1
2
0
tan 0
1

3
1

3 |

3 1
1

3
0
cot |

3 1
1

3
0
1

3
1

3 |
2.1.5 Cng thc lng gic ca tng, hiu
cos(a b) = cos a cos b sin a sin b
sin(a b) = sin a cos b cos a sin b
tan(a b) =
tana tan b
1 tana tanb
cot(a b) =
cot a cot b 1
cot b cot a
Ngi son : Th.s Minh Tun Trang 34 Khoa T nhin - Trng CSP Nam nh
T
h
.
s

M
i
n
h
T
u

n
2.1. Cc kin thc c bn Chng 2. Phng trnh lng gic
2.1.6 Cng thc cng lng gic
cos a + cos b = 2 cos
a +b
2
cos
a b
2
cos a cos b = 2 sin
a +b
2
sin
a b
2
sin a + sin b = 2 sin
a +b
2
cos
a b
2
sin a sin b = 2 cos
a +b
2
sin
a b
2
tana tan b =
sin(a b)
cos a cos b
cot a cot b =
sin(b a)
sin a sin b
H qu:
tana + cot a =
2
sin 2a
tana cot a = 2 cot 2a
sin a + cos a =

2 sin(x +

4
) =

2 cos(x

4
)
sin a cos a =

2 sin(x

4
) =

2 cos(x +

4
)
2.1.7 Cng thc bin i tch thnh tng
cos a cos b =
1
2
[cos(a +b) + cos(a b)]
sin a sin b =
1
2
[cos(a +b) cos(a b)]
sin a cos b =
1
2
[sin(a +b) + sin(a b)]
2.1.8 Cng thc gc nhn i, nhn ba - Cng thc h bc
cos 2x = cos
2
x sin
2
x = 2 cos
2
x 1 = 1 2 sin
2
x
sin 2x = 2 sinxcos x
tan2x =
2 tanx
1 tan
2
x
cot 2x =
cot
2
x 1
2 cot x
cos
2
x =
1 + cos 2x
2
Ngi son : Th.s Minh Tun Trang 35 Khoa T nhin - Trng CSP Nam nh
T
h
.
s

M
i
n
h
T
u

n
2.1. Cc kin thc c bn Chng 2. Phng trnh lng gic
sin
2
x =
1 cos 2x
2
cos 3x = 4 cos
3
x 3 cos x
sin 3x = 3 sinx 4 sin
3
x
tan3x =
3 tanx tan
3
x
1 3 tan
2
x
cot 3x =
3 cot x cot
3
x
1 3 cot
2
x
2.1.9 Cng thc tnh sin 2x, cos 2x, tan2x, cot 2x theo t = tanx
sin 2x =
2t
1 +t
2
, cos 2x =
1 t
2
1 +t
2
, tan2x =
2t
1 t
2
, cot 2x =
1 t
2
2t
2.1.10 Bi tp
Bi 2.1: Cho l gc sao cho sin =
1
3
, vi thuc gc phn t th III.
a) Tnh tan, cos , cot .
b) Tnh cos(
5
2
), tan(
7
2
).
Bi 2.2: Tnh gi tr biu thc sau:
a) A =
1
sin
2
20
0
+
1
cos
2
40
0
tan
2
70
0
cot
2
50
0
.
b) B = sin
2
1
0
+ sin
2
3
0
+ + sin
2
89
0
.
c) C = cos 2
0
+ cos 4
0
+ + cos 180
0
.
Bi 2.3: Tnh gi tr biu thc:
a) A = cos

8
+ cos
5
8
+ sin
9
8
+ sin
5
8
b) B = cos

8
+ tan

8
Rt gn biu thc:
a) A = cos xcos 2xcos 4xcos 8x bit x = k (k Z).
b) B = sin xsin(

3
x) sin(

3
+x).
Bi 2.4: Tnh gi tr biu thc:
a) A = cos
6
x + sin
6
x.
b) B = cos
4
x + sin
4
x.
c) C = cos 12x .
Bit rng cos 4x =
1
3
.
Ngi son : Th.s Minh Tun Trang 36 Khoa T nhin - Trng CSP Nam nh
T
h
.
s

M
i
n
h
T
u

n
2.2. Cc phng trnh lng gic c bn Chng 2. Phng trnh lng gic
2.2 Cc phng trnh lng gic c bn
2.2.1 Phng trnh sin x = m
iu kin c nghim: 1 m 1.
Nghim ca phng trnh l :

x = arcsin m +k2
x = arcsin m+ k2
(k Z)
Ch : cc gi tr c bit ca m l m = 0, 1,
1
2
,

2
2
,

3
2
Phng trnh c bit:
sin x = 1 x =

2
+k2
sin x = 1 x =

2
+k2
sin x = 0 x = k
(k Z)
Dng sin A = sin B

A = B +k2
A = B +k2
2.2.2 Phng trnh cos x = m
iu kin c nghim: 1 m 1.
Nghim ca phng trnh l :

x = arccos m +k2
x = arccos m +k2
(k Z)
Phng trnh c bit:
cos x = 1 x = +k2
cos x = 1 x = k2
cos x = 0 x =

2
+k
(k Z)
Dng cos A = cos B

A = B +k2
A = B +k2
2.2.3 Phng trnh tanx = m, cot x = m
iu kin c nghim m R
Nghim tanx = m x = arctanm +k
cot x = m x = arccot m +k
Dng tanA = tanB A = B +k
cot A = cot B A = B +k
Ch khi gii phng trnh cha tan, cot ta phi t iu kin cho bin.
Ngi son : Th.s Minh Tun Trang 37 Khoa T nhin - Trng CSP Nam nh
T
h
.
s

M
i
n
h
T
u

n
2.2. Cc phng trnh lng gic c bn Chng 2. Phng trnh lng gic
2.2.4 Cc v d
V d 2.2.1: Gii cc phng trnh sau:
a) sin x =
1
2
.
b) sin

3x

4

= cos

2x +

.
c) cos

5x

3

= cos 2x.
Gii: a)

x =

6
+k2
x =
7
6
+k2
(k Z)
b) pt sin

3x

4

= sin

2
2x

6

sin

3x

4

= sin

2x +

3x

4
= 2x +

3
+k2
3x

4
= 2x

3
+k2

x =
7
12
+k2
x =
11
60
+
k2
5
(k Z).
c) pt cos

5x

3

= cos ( 2x)

5x

3
= 2x +k2
5x

3
= 2x +k2

7x =
4
3
+k2
3x =
2
3
+k2

x =
4
21
+k
2
7
x =
2
9
+k
2
3
(k Z)
V d 2.2.2: Gii cc phng trnh:
a) tanx. cot 2x 2 tanx +

3 cot 2x = 2

3
b) tan3x = cot

x +
2
5

.
c) tanx +
2
cot x
= 3.
d) sin 2x cos x + 2 sin x 1 = 0.
e) sin 2x

3 cos x 2 sin x +

3 = 0.
Ngi son : Th.s Minh Tun Trang 38 Khoa T nhin - Trng CSP Nam nh
T
h
.
s

M
i
n
h
T
u

n
2.2. Cc phng trnh lng gic c bn Chng 2. Phng trnh lng gic
Gii: a) pt

tanx +

. (cot 2x 2) = 0

tan x =

3
cot 2x = 2

x =

3
+k
2x = arccot 2 +k

x =

3
+k
x =
1
2
arccot 2 +k

2
b) pt tan3x = tan

2
x
2
5

tan 3x = tan

10
x

3x =

10
x +k x =

40
+k

4
c) pt tanx + 2 tanx = 3 tanx = 1 x =

4
+k
2.2.5 Bi tp
Bi 2.5: Gii cc phng trnh sau:
a) sin(3x + 1) =

3
2
b) cos(2x + 2) =
1
2
c) sin 2x = sin(
x
2
+ 1)
Bi 2.6: Gii cc phng trnh sau:
a) cos 2x = sin 5x
b) sin
x
2
= cos 4x
c) cos( sin x) = cos(3 sin x)
d) 2 sin xcos x = cos x 2 sin x + 1
Bi 2.7: Gii cc phng trnh sau:
a) tan x = 1
b) tan(2x 1) = tan(x + 2)
c) cot(x + 1) = tan2x
d) tan x =

2
cot x
+ 1
Ngi son : Th.s Minh Tun Trang 39 Khoa T nhin - Trng CSP Nam nh
T
h
.
s

M
i
n
h
T
u

n
2.3. Cc phng trnh lng gic khc Chng 2. Phng trnh lng gic
2.3 Cc phng trnh lng gic khc
2.3.1 Phng trnh a sin x + b cos x = c
Ta c a sin x +b cos x = c
a

a
2
+b
2
sin x +
b

a
2
+b
2
cos x =
c

a
2
+b
2
.
Gi l gc sao cho

cos =
a

a
2
+b
2
sin =
b

a
2
+b
2
Khi phng trnh tng ng vi:
cos sin x + sin cos x =
c

a
2
+b
2
sin(x +) =
c

a
2
+b
2
iu kin c nghim:

a
2
+b
2

1 [c[

a
2
+b
2
a
2
+b
2
c
2
V d 2.3.1: Gii cc phng trnh sau:
a) sin x =

3 cos x + 1.
b) cos 2x 3 sin xcos x = 3.
c) sin x cos x =

2 cos 3x.
d) 3 sin 3x + 4 cos 3x = 5 sin 4x
Gii: a) pt sin x

3 cos x = 1 2 sin

x

3

= 1 sin

x

3

=
1
2
= sin

x

3
=

6
+k2
x

3
=
5
6
+k2

x =

2
+k2
x =
7
6
+k2
(k Z)
b) pt cos 2x
3
2
sin 2x = 3.
V a
2
+b
2
=
13
4
< c
2
= 9 nn phng trnh v nghim.
c) pt

2 sin

x

4

2 cos 3x sin

x

4

= sin

2
3x

x

4
=

2
3x +k2
x

4
=

2
+ 3x +k2

x =
3
16
+k

2
x =
3
8
k
(k Z)
Ngi son : Th.s Minh Tun Trang 40 Khoa T nhin - Trng CSP Nam nh
T
h
.
s

M
i
n
h
T
u

n
2.3. Cc phng trnh lng gic khc Chng 2. Phng trnh lng gic
d) pt
3
5
sin 3x +
4
5
cos 3x = sin 4x
Gi l gc sao cho cos =
3
5
v sin =
4
5
ta c:
cos sin 3x + sin cos 3x = sin 4x sin(3x +) = sin 4x

3x + = 4x +k2
3x + = 4x +k2

x = k2
x =
+k2
7
(k Z)
2.3.2 Phng trnh ng cp cha sin v cos
Nhn bit: phng trnh cha sin x, cos x tha mn bc ca tt c cc hng t u l s
chn hoc l s l. Chng hn :
~ sin x, cos x bc 1.
sin
2
x, cos
2
x, sin xcos x, cos 2x, sin 2x bc 2.
sin
3
x, sin
2
xcos x, sin xcos
2
x, cos
3
x, sin 3x, cos 3x u c bc 3.
~ Phng trnh a sin x+b cos x = c c th c coi l phng trnh ng cp theo sin
x
2
,
cos
x
2
.
Cch gii: Ta xt 2 trng hp sau:
~ Trng hp 1: cos x = 0.
~ Trng hp 2: cos x = 0. Khi ta s chia c 2 v cho cos
m
x ( m l bc ca
phng trnh ng cp)
~ Phng trnh ng cp bc 2: a sin
2
x +b cos xsin x +c cos
2
x = d.
(sin
2
x, cos
2
x c th thay th bi cos 2x)
Cch gii: Nng bc
a sin
2
x +b cos xsin x +c cos
2
x = d(sin
2
x + cos
2
x). Sau rt gn.
Nu cos x = 0: Thay vo tha mn th ta c nghim x =

2
+k (k Z). Cn nu
thu c sin
2
x = 0 sin x = 0 th phng trnh v nghim v sin
2
x + cos
2
x = 1.
Nu cos x = 0: Chia c 2 v cho cos
2
x ta c phng trnh bc 2 n tan x.
~ Phng trnh ng cp bc 3:
a sin
3
x +b cos xsin
2
x +c sin xcos
2
x +d cos
3
x = e sin x +f cos x.
(Bin th: cc hng t v tri c th thay th bi: cos 3x, sin 3x, sin 2x. cos x, ....)
Cch gii: nng bc
Phng trnh a sin
3
x+b cos xsin
2
x+c sin xcos
2
x+d cos
3
x = (e sin x+f cos x)(sin
2
x+
cos
2
x). Rt gn ta c phng trnh ng cp bc 3.
Lm ging nh phng trnh ng cp bc 2 (Nhng nh rng chia cho cos
3
x ch
khng phi cos
2
x)
V d 2.3.2: Gii phng trnh:
Ngi son : Th.s Minh Tun Trang 41 Khoa T nhin - Trng CSP Nam nh
T
h
.
s

M
i
n
h
T
u

n
2.3. Cc phng trnh lng gic khc Chng 2. Phng trnh lng gic
a) 1 + 2 sin 2x = 6 cos
2
x.
b) 2 sin
3
x = cos x
Gii: a) pt sin
2
x + cos
2
x + 4 sin xcos x = 6 cos
2
x
sin
2
x + 4 sinxcos x 5 cos
2
x = 0.
+) Nu cos x = 0 th ta c sin
2
x = 0 sin x = 0. iu ny mu thun.
+) Nu cos x = 0 chia c 2 v cho cos
2
x = 0 ta c:
tan
2
x + 4 tanx 5 = 0

tan x = 1
tan x = 5

x =

4
+k
x = arctan(5) +k
(k Z)
b) pt 2 sin
3
x = cos x(sin
2
x + cos
2
x) 2 sin
3
x + sin
2
xcos x + cos
3
x = 0.
+) Nu cos x = 0 th 2 sin
3
x = 0 sin x = 0. iu ny mu thun.
+) Nu cos x = 0. Chia c 2 v cho cos
3
x.
pt 2 tan
3
x + tan
2
x + 1 = 0 (tanx + 1)(2 tan
2
x tan x + 1) = 0

tan x = 1
2 tan
2
x tanx + 1 = 0( V nghim )
x =

4
+k (k Z).
2.3.3 i s ha phng trnh lng gic
Nhn dng: phng trnh c th qui v phng trnh bc 2, 3, 4 theo sin, cos, tan, cot l
nhng phng trnh c th i s ha c. Hoc phng trnh khi dng php t n ph
cng qui v c phng trnh i s.
Phng php: s dng cc cng thc gc nhn i, nhn ba, v cng thc tnh sin x, cos x, tan x, cot x
theo t = tan
x
2
, ...
Phng trnh bc 2 theo sin x, cos x
a

cos 2x
cos
2
x
sin
2
x

+b

cos x
sin x

+c = 0.
Cch gii: t t = sin x hoc t = cos x ph thuc vo hng t th 2.
Phng trnh bc 3 theo cos x
a cos 3x +b cos 2x +c cos x +d = 0
Cch gii: Phng trnh a(4 cos
3
x 3 cos x) +b(2 cos
2
x 1) +c cos x +d = 0.
Phng trnh bc 3 theo sin x:
a sin 3x +b cos 2x +c sin x +d = 0
Cch gii: a(3 sin x 4 sin
3
x) +b(1 2 sin
2
x) +c sin x +d = 0.
Phng trnh : a sin 3x +b sin 2x +c sin x = 0
Cch gii: Phng trnh a(3 sin x4 sin
3
x)+2b sin xcos x+c sin x = 0 sin x

a(3 4 sin
2
x) + 2b. cos
0
Ngi son : Th.s Minh Tun Trang 42 Khoa T nhin - Trng CSP Nam nh
T
h
.
s

M
i
n
h
T
u

n
2.3. Cc phng trnh lng gic khc Chng 2. Phng trnh lng gic
Phng trnh dng f(sin 2x, cos 2x, tanx) = 0.
p dng cng thc : sin 2x =
2t
1 +t
2
, cos 2x =
1 t
2
1 +t
2
vi t = tan x.
Ta c phng trnh i s: f

2t
1 +t
2
,
1 t
2
1 +t
2
, t

= 0
V d 2.3.3: Gii cc phng trnh lng gic sau:
a) cos 2x sin x + 1 = 0.
b)
(1 + sin x + cos 2x). sin

x +

4

1 + tanx
=
1

2
cos x (A-2010)
c) sin 3x sin 2x + 4 sin x = 0.
d) sin 3x + 4 cos 2x 4 sin x 1 = 0.
e) cos 3x + cos 2x cos x 1 = 0.
f) cos 3x + 4 cos 2x + 4 cos x + 1 = 0.
V d 2.3.4: Gii cc phng trnh lng gic sau:
a) sin 2x + tanx = 2
b) tanx + 2 sin 2x cos 2x = 1
V d 2.3.5: Gii phng trnh sau:
a) tanx + tan
2
x + tan
3
x + cot x + cot
2
x + cot
3
x = 6.
b) tan
2
x
3
cos x
+ 3 = 0.
2.3.4 Phng trnh i xng sin, cos
Nhn dng: phng trnh c dng : f(sin x, cos x) = 0 hoc f(sin x, cos x) = 0
f(x, y) l mt hm i xng theo x, y hay ni cch khc f(x, y) = f(y, x).
Cch gii: t t = sin x + cos x =

2 sin(x +

4
)
hoc t t = sin x cos x =

2 sin(x

4
)
vi

2 t

2
V d 2.3.6: Gii phng trnh sau:
a) sin x cos x + 2 sin 2x = 1
b) sin
3
x + cos
3
x = 2(sin x + cos x) 1
Gii: a) t t = sin x cos x v gii phng trnh bc 2 n t.
b) t t = sin x + cos x v gii phng trnh bc 3 n t.
Ngi son : Th.s Minh Tun Trang 43 Khoa T nhin - Trng CSP Nam nh
T
h
.
s

M
i
n
h
T
u

n
2.3. Cc phng trnh lng gic khc Chng 2. Phng trnh lng gic
2.3.5 Phn tch thnh nhn t
Nhn dng: Bi ton dng ny rt kh nhn dng thng thng theo trc gic ca ngi
gii ton l chnh. Phng php c th dng gii hoc lm cho bi ton tr nn n
gin hn.
Di y l mt s h c tha s chung hay gp:
f(x) Biu thc cha tha s f(x)
sin x sin 2x, sin 3x, tan x, tan2x, tan3x,
cos x sin 2x, cos 3x, tan 2x, cot 3x, cot x,
1 + cos x cos
2
x
2
, cot
2
x
2
, sin
2
x, tan
2
x
1 cos x sin
2
x
2
, tan
2
x
2
, sin
2
x, tan
2
x
1 + sin x cos
2
x, cot
2
x, cos
2

4

x
2

, sin
2

4
+
x
2

1 sin x cos
2
x, cot
2
x, cos
2

4
+
x
2

, sin
2

4

x
2

sin x + cos x cos 2x, cot 2x, 1 + sin 2x, 1 + tan x, 1 + cot x, tanx cot x
cos x sin x cos 2x, cot 2x, 1 sin 2x, 1 tan x, 1 cot x, tanx cot x
Dng a sin 2x +b cos 2x +c sin x +d cos x +e = 0.
Cch gii: i theo mt trong 3 hng sau:
Hng 1: sin x(2a cos x +c) + [b(2 cos
2
x 1) +d cos x +e] = 0.
Hng 2: cos x(2a sin x +d) +

b(1 2 sin
2
x) +c sin x +e

= 0.
Hng 3: (a sin 2x +b cos 2x) + (c sin x +d cos x) +e = 0
V d 2.3.7: Gii cc phng trnh sau:
a) sin 2x cos 2x + 3 sin x cos x 1 = 0 (D-2010)
b) cos 2x +

3 sin 2x + 3

3 sin x 3 cos x 4 = 0.
c) sin 2x + 2 cos 2x 4 cos x + sin x 1 = 0.
Ngi son : Th.s Minh Tun Trang 44 Khoa T nhin - Trng CSP Nam nh
T
h
.
s

M
i
n
h
T
u

n
2.3. Cc phng trnh lng gic khc Chng 2. Phng trnh lng gic
V d 2.3.8: Gii cc phng trnh sau:
a) (sin 2x + cos 2x) cos x + 2 cos 2x sin x = 0 (B-2010)
b) sin 2(x ) sin(3x ) = sin x
c) cos 3x 2 cos 2x + cos x = 0
d) sin
2
x + sin
2
2x + sin
2
3x =
3
2
e) cos 3xcos 4x + sin 2xsin 5x =
1
2
(cos 2x + cos 4x)
2.3.6 S dng bt ng thc
Phng php: i vi dng A = 0.
Ta chng minh A 0, x t suy ra du bng trong bt ng thc phi xy ra. Gii
nghim t iu kin xy ra du bng .
i vi dng bin th: A = B.
Ta chng minh : A v B , x dn n phng trnh tng ng vi A = = B
v du bng 2 bt ng thc xy ra.
V d 2.3.9: Gii cc phng trnh sau:
a) sin(x +

4
) + 2 sin 2x 3 = 0.
b) sin
4
x + cos
4
x = 2 cos
7
x
Gii: a) sin(x +

4
) + 2 sin 2x 3 0 do phng trnh tng ng vi:

sin(x +

4
) = 1
sin 2x = 1
b) Ta c sin
4
x + cos
4
x sin
2
x + cos
2
x = 1
v 2 cos
7
x 1. Do m ta c h:

sin
4
x = sin
2
x
cos
4
x = cos
2
x
cos
7
x = 1
cos x = 1 x = k2
2.3.7 Loi nghim khng thch hp
t vn : Trong khi gii mt phng trnh lng gic ta gp bi ton m nghim tm
thy khng nm trong tp xc nh. Thiu st trn thng gp khi ta "qun" t iu
kin hoc t iu kin nhng khi gii xong khng i chiu vi iu kin dn n tnh
trng "tha nghim".
Ngi son : Th.s Minh Tun Trang 45 Khoa T nhin - Trng CSP Nam nh
T
h
.
s

M
i
n
h
T
u

n
2.3. Cc phng trnh lng gic khc Chng 2. Phng trnh lng gic
Phng php: c rt nhiu phng php nh gii phng trnh nghim nguyn, th nghim
trc tip vo iu kin, ... nhng vi mc thi i hc hin nay th phng php n
gin d dng nht l biu din tp nghim trn ng trn lng gic. Nhng im b
trng vi im b loi tp xc nh s b loi.
V d 2.3.10: Gii phng trnh sau:
a)
4 sin
6
x + 4 cos
6
x 1
2 sin x

2
= 0
b)

9 x
2
(2 sin x 1) = 0
Gii: a) iu kin: sin x =

2
2
.
pt 4 sin
6
x + 4 cos
6
x 1 = 0
4

1 + cos 2x
2

3
+ 4

1 cos 2x
2

3
1 = 0
1 + 3 cos
2
2x 1 = 0 cos 2x = 0
x =

4
+k

2
.
Biu din trn ng trn lng gic ta c nghim:
x =

4
+k2, x =
5
4
+k2
b) Nghim x = 3.
Trng hp 3 < x < 3: pt sin x =
1
2
Kt hp iu kin ta c nghim:
x =

6
, x =
5
6
2.3.8 Bi tp
Phng trnh dng a sin x +b cos x = c.
Bi 2.8: Gii phng trnh sau:
a) cos
x
2
+

3 sin
x
2
= 1
b) 3 cos x + 4 sinx +
6
3 cos x + 4 sin x + 1
= 6
c) sin 3x

3 cos 3x = 2 sin2x
d) 3 sin 3x

3 cos 9x = 1 + 4 sin
3
3x
e) sin x + cos x =

2 sin 5x
f) cos 7x sin 5x =

3(cos 5x sin 7x)


Bi 2.9: Tm gi tr ln nht, nh nht ca hm s sau:
Ngi son : Th.s Minh Tun Trang 46 Khoa T nhin - Trng CSP Nam nh
T
h
.
s

M
i
n
h
T
u

n
2.3. Cc phng trnh lng gic khc Chng 2. Phng trnh lng gic
a) y =
sin x + 2 cos x + 1
sin x + cos x + 2
.
b) y =
sin x
2 + cos x

Phng trnh ng cp:
Bi 2.10: Gii phng trnh sau:
a) 4 cos
2
x 6 sin
2
x + 5 sin 2x 4 = 0
b) 2 sin
3
x = cos x
c) 6 sin x 2 cos
3
x = 5 sin 2x cos x
d) sin
2
x 2 cos
2
x =
1
2
sin 2x
e) 14 cos
4
x + 16 cos
3
xsin x + 6 sin
2
xcos
2
x cos xsin
3
x + sin
4
x = 2

i s ha phng trnh lng gic:
Bi 2.11: Gii phng trnh sau:
a) cos 2x + sin
2
x + 2 cos x + 1 = 0
b) 2 cos 3x 7 cos 2x + 7 cos x 2 = 0.
c) cos
4
x + sin
4
x = 2 cos(2x +

4
) cos(2x

4
)
d) sin
4
x + cos
4
x cos 2x +
sin
2
2x
4
2 = 0
e) tan
2
x
4
cos x
+ 5 = 0
f)
3
sin
2
x
+ 3 tan
2
x 11(tanx + cot x) 1 = 0
g) sin
4
x + (1 + sin x)
4
= 17
h) 2 cos
2
6x
5
+ 1 = 3 cos
8x
5

Phng trnh i xng:
Bi 2.12: Gii phng trnh sau:
Ngi son : Th.s Minh Tun Trang 47 Khoa T nhin - Trng CSP Nam nh
T
h
.
s

M
i
n
h
T
u

n
2.3. Cc phng trnh lng gic khc Chng 2. Phng trnh lng gic
a) sin
3
x + cos
3
x = 1
sin 2x
2
b) sin 2x + 4(cos x sin x) = 4
c) cos x +
1
cos x
+ sin x +
1
sin x
=
10
3
d) sin xcos x = 6(sin x + cos x 1)
e) 1 + sin 2x = sin x + cos x

Phn tch thnh nhn t:
Bi 2.13: Gii phng trnh sau:
a) sin x + cos x + 1 = 2 cos(
x
2


4
)
b)

2(cos
4
x sin
4
x) = sin x + cos x
c) sin x + 3 sin 2x = sin 3x
d) cos xsin 2xcos 3x =
sin 4x
4
e) cos 2x + sin 2x + cos x + 3 sin x + 1 = 0
f) (2 sin x 1)(2 sin 2x + 1) = 3 4 cos
2
x
g) cot x tan x = sin x + cos x

S dng bt ng thc:
Bi 2.14: Gii phng trnh sau:
a) sin
3
x + cos
3
x = 2 sin
4
x
b) cos 3x +

2 cos
2
3x = 2(1 + sin
2
2x)
c) cos(x) = x
2
4x + 5
d) 2 sin(x +

4
) = tan x + cot x

Loi nghim khng thch hp
Bi 2.15: Gii cc phng trnh sau:
a)
sin
6
x + cos
6
x
tan(x

4
) tan(x +

4
)
=
1
4
Ngi son : Th.s Minh Tun Trang 48 Khoa T nhin - Trng CSP Nam nh
T
h
.
s

M
i
n
h
T
u

n
2.3. Cc phng trnh lng gic khc Chng 2. Phng trnh lng gic
b)
sin
6
x + cos
6
x
cos
2
x sin
2
x
=
tan 2x
4
c)
sin x (sin x + cos x) 1
cos
2
x + sin x + 1
= 0
d) tan x (2 sin
x
3
1) = 0
e)
1 + tan x
1 tanx
= 1 + sin 2x
f) tan
2
x =
1 + cos x
1 sin x

Dng ton tng hp:
Bi 2.16: Gii cc phng trnh sau:
a) sin x + cos 2x + cos
2
x(tan
2
x 1) + 2sin
3
x = 0.
b) cos x. cos 2x. cos 4x. cos 8x =
1
16
c)
sin
4
x + cos
4
x
5 sin 2x
=
1
2
cot 2x
1
8 sin2x
d) 2sin
2
x + sin 2x + 3 sinx cos x + 1 = 0
e) 4sin
2
x
2

3 cos 2x = 1 + 2cos
2

x
3
4

f) 2

2cos
3

x

4

3 cos x sin x = 0
g) tan

2
+x

3tan
2
x =
cos 2x 1
cos
2
x
h) tan

3
2
x

+
sin x
1 + cos x
= 2
i) cos
2
3x. cos 2x cos
2
x = 0
j) 2 sin x(1 + cos 2x) + sin 2x = 1 + 2 cos x
k) sin
3
x

3cos
3
x = sin xcos
2
x

3sin
2
xcos x
l)
1
sin x
+
1
sin

x
3
2
= 4 sin

7
4
x

Ngi son : Th.s Minh Tun Trang 49 Khoa T nhin - Trng CSP Nam nh
T
h
.
s

M
i
n
h
T
u

n
2.3. Cc phng trnh lng gic khc Chng 2. Phng trnh lng gic
m)

2 cos

x
7
4

= cos

2x
7
2

+
1
2
n) sin 2x + sin x
1
2 sinx

1
sin 2x
= 2 cot 2x
o) sin

5x
2


4

cos

x
2

2 cos
3x
2
p) 2

2 sin

x

12

cos x = 1.
q) 2cos
2
x + 2

3 sin xcos x + 1 = 3

sin x +

3 cos x

r)
sin 2x
cos x
+
cos 2x
sin x
= tanx cot x
Ngi son : Th.s Minh Tun Trang 50 Khoa T nhin - Trng CSP Nam nh
T
h
.
s

M
i
n
h
T
u

n
Chng 3. Phng trnh cha cn v du gi tr tuyt i
Chng 3
Phng trnh cha cn v du gi tr
tuyt i
3.1 Phng trnh cha du gi tr tuyt i
3.1.1 Kin thc cn nh
Hc sinh cn nh mt s tnh cht sau ca du gi tr tuyt i:
[A[ =

A Nu A 0
A Nu A < 0
[ A[ = [A[
[A
2
[ = A
2
[A[ A. Du ng thc khi A 0.
[A[ A. Du ng thc khi A 0
[A[ +[B[ [A+B[.
Du ng thc xy ra A.B 0.
[A[ +[B[ [AB[.
Du ng thc xy ra A.B 0.
3.1.2 Cc dng bi tp
Nguyn tc chung: gii mt phng trnh cha du gi tr tuyt i, ta c th bm
vo nh ngha xt du ca biu thc trong du gi tr tuyt i, phn trng hp ph
du gi tr tuyt i. Tuy nhin, nhiu khi vic xt du mt biu thc l kh kh khn,
di dng nn y ti a ra mt s cch bin i tng i vi mt s dng phng
trnh cha du gi tr tuyt i c bn hc sinh c th p dng n lm cc bi tp mt
cch chnh xc ngn gn.
~ Dng 1
[A[ = B

A 0
A = B

A < 0
A = B

A = B
A = B
B 0
Ngi son : Th.s Minh Tun Trang 51 Khoa T nhin - Trng CSP Nam nh
T
h
.
s

M
i
n
h
T
u

n
3.1. Phng trnh cha du gi tr tuyt i Chng 3. Phng trnh cha cn v du gi tr tuyt i
~ Dng 2:
[A[ = [B[ A
2
= B
2
A = B
~ Dng 3: t n ph vi biu thc cha du gi tr tuyt i.
~ Dng 4: S dng bt ng thc nh gi 2 v ca phng trnh :
Chng hn nu ta c phng trnh A = B. Ta chng minh:
A v B t ta c A = = B.
~ Dng 5 : Lp bng xt du vi biu thc trong du gi tr tuyt i ph du gi tr
tuyt i.
3.1.3 Cc v d
V d 3.1.1: Gii cc phng trnh sau:
a) [x
2
+x 1[ = 2 x ()
b) [2x
2
+ 3x 2[ = [x
2
x 3[ ()
Gii: a) ()

2 x 0 (1)

x
2
+x 1 = 2 x (2)
x
2
+x 1 = x 2 (3)
(1) x 2
(2) x
2
+ 2x 3 = 0

x = 1
x = 3

tha mn iu kin

(3) x
2
+ 1 = 0

Phng trnh v nghim

b) ()

2x
2
+ 3x 2 = x
2
x 3 (1)
2x
2
+ 3x 2 = x
2
+x + 3 (2)
(1) x
2
+ 4x + 1 = 0 x = 2

3
(2) 3x
2
+ 2x 5 = 0

x = 1
x =
5
3
V d 3.1.2: Gii phng trnh x
4
+ 4x
2
+ 2 [x
2
2x[ = 4x
3
+ 3
Gii: Phng trnh (x
2
2x)
2
+ 2 [x
2
2x[ 3 = 0
t t = [x
2
2x[ 0. Phng trnh tr thnh:
t
2
+ 2t 3 = 0

t = 1

tha mn

t = 3 (loi)
+)t = 1

x
2
2x

= 1

x
2
2x = 1
x
2
2x = 1

x = 1

2
x = 1
V d 3.1.3: Gii phng trnh [x + 1[ +[2 x[ = x
4
+ 4x
2
1
Gii: Ta c V T = [x + 1[ +[2 x[ [x + 1 + 2 x[ = 3 x (1)
m V P = x
4
+ 4x
2
1 = (x
2
2)
2
+ 3 3 x (2).
T (1) , (2) V T 3 V P x
Do phng trnh cho

(x + 1) . (2 x) 0
x
2
2 = 0
x =

2
Ngi son : Th.s Minh Tun Trang 52 Khoa T nhin - Trng CSP Nam nh
T
h
.
s

M
i
n
h
T
u

n
3.2. Phng trnh cha cn thc Chng 3. Phng trnh cha cn v du gi tr tuyt i
V d 3.1.4: Gii phng trnh 2 [x + 1[ [x
2
2x 8[ = 5 x +x
2
Gii: Ta c bng xt du:
x 2 1 4 +
x + 1 0 + +
x
2
2x 8
+ 0 0 +
0 Trng hp 1: x < 2.
Phng trnh 2 (x 1) (x
2
2x 8) = 5 x +x
2
2x
2
x 11 = 0

x =
1 +

89
4
(loi)
x =
1

89
4
(tha mn)
O Trng hp 2: 2 x < 1
Phng trnh 2 (x 1) + (x
2
2x 8) = 5 x +x
2
3x = 5 x =
5
3
(tha mn)
O Trng hp 3: 1 x < 4
Phng trnh 2 (x + 1) + (x
2
2x 8) = 5 x +x
2
x = 1 (tha mn)
O Trng hp 4: x 4
Phng trnh 2 (x + 1) (x
2
2x 8) = 5 x +x
2
2x
2
5x 15 = 0

x =
5 +

145
4
(tha mn)
x =
5

145
4
(loi)
Vy phng trnh c tp nghim S =

89
4
;
5
3
; 1;
5 +

145
4

3.2 Phng trnh cha cn thc


3.2.1 Cc dng bi tp
0

A = B

A = B
2
B 0
O

A =

A = B
A 0

A = B
B 0
O t n ph. Ta t cn thc hoc biu thc trong cn bng mt n ph lm phng
trnh tr nn n gin hn.
O S dng bt ng thc. Cch lm tng t phn phng trnh cha du gi tr tuyt i.
Ngi son : Th.s Minh Tun Trang 53 Khoa T nhin - Trng CSP Nam nh
T
h
.
s

M
i
n
h
T
u

n
3.2. Phng trnh cha cn thc Chng 3. Phng trnh cha cn v du gi tr tuyt i
3.2.2 Cc v d
V d 3.2.1: Gii phng trnh sau:
a)

x + 3 = 2 x
b)

x
2
+x 1 =

2x + 3
Gii: a) Phng trnh

x + 3 = (2 x)
2
2 x 0

x
2
4x + 4 = x + 3
x 2

x
2
5x + 1 = 0
x 2

x =
5

21
2
x 2
x =
5

21
2
b) Phng trnh

x
2
+x 1 = 2x + 3
2x + 3 0

x
2
x 4 = 0
x
3
2

x =
1

17
2
x
3
2
x =
1 +

17
2
V d 3.2.2: Gii phng trnh sau: 2

x
2
2x 2 = x
2
+ 2x + 5.
Gii: t t =

x
2
2x 2 0 x
2
2x = t
2
+ 2.
Phng trnh 2t = t
2
2 + 5 t
2
+ 2t 3 = 0

t = 1
t = 3 (loi)
Ta c t = 1

x
2
2x 2 = 1 x
2
2x 3 = 0

x = 1
x = 3
Bnh lun: Nu phng trnh ny ta gii theo cch ca phng trnh cn thc c bn th
s gp mt phng trnh bc 4. Khi mi vic tr nn phc tp hn nhiu.
V d 3.2.3: Gii phng trnh :

x + 1 +

3 x =

2 (x
2
2x + 3)
Gii: p dng bt ng thc Bunhiacopxki vi 2 b

x + 1;

3 x

, (1; 1)
ta c: V T
2
=

x + 1.1 +

3 x.1

x + 1
2
+

3 x
2

. (1
2
+ 1
2
).
V T
2
(x + 1 + 3 x) .2 = 8 V T 2

2 (1)
Du bng xy ra khi

x + 1
1
=

3 x
1
x = 1
V P =

2 (x
2
2x + 3) =

2.

(x 1)
2
+ 2

2 (2)
Du bng xy ra khi x = 1.
T (1), (2) ta c phng trnh cho c duy nht nghim x = 1.
Ngi son : Th.s Minh Tun Trang 54 Khoa T nhin - Trng CSP Nam nh
T
h
.
s

M
i
n
h
T
u

n
3.3. Bt phng trnh cha du gi tr tuyt i Chng 3. Phng trnh cha cn v du gi tr tuyt i
3.3 Bt phng trnh cha du gi tr tuyt i
3.3.1 Dng c bn
0 [A[ < B B < A < B
O [A[ > B

A > B
A < B
O [A[ < [B[ A
2
B
2
< 0 (AB) . (A +B) < 0
3.3.2 Cc v d
V d 3.3.1: Gii cc bt phng trnh sau:
a) [x
2
2x 1[ < x + 1
b) [2x
2
+x 3[ 2x + 1
c) [x
2
+x 1[ < [2x
2
+x 2[
Gii: a) Bt phng trnh

x
2
2x 1 < x + 1
x
2
2x 1 > x 1

x
2
3x 2 < 0
x
2
x > 0

17
2
< x <
3 +

17
2
x > 1 x < 0
x

17
2
; 0

1;
3 +

17
2

b) Bt phng trnh

2x
2
+x 3 2x + 1
2x
2
+x 3 2x 1

2x
2
x 4 0
2x
2
+ 3x 2 0

x
1 +

33
4
x
1

33
4
2 x
1
2
x
1
2
x
1 +

33
4
c) Bt phng trnh
(2x
2
+x 2)
2
> (x
2
+x 1)
2
(2x
2
+x 2)
2
(x
2
+x 1)
2
> 0
(x
2
1) (3x
2
+ 2x 3) > 0
(x 1) (x + 1)

x
1

10
3

x
1 +

10
3

> 0
x <
1

10
3
1 < x <
1 +

10
3
x > 1
3.4 Bt phng trnh cha cn thc
3.4.1 Dng c bn
0

A > B

A > B
2
B 0

B < 0
A 0
Ngi son : Th.s Minh Tun Trang 55 Khoa T nhin - Trng CSP Nam nh
T
h
.
s

M
i
n
h
T
u

n
3.4. Bt phng trnh cha cn thc Chng 3. Phng trnh cha cn v du gi tr tuyt i
O

A < B

A < B
2
A 0
B 0
O

A <

A < B
A 0
Ch : Vi cc cn bc chn c lm tng t. Cn i vi cc cn bc l th ta khng
cn iu kin v cch gii dng ny tng i n gin nn chng khng c nhc ti
y.
3.4.2 Cc v d
V d 3.4.1: Gii cc bt phng trnh sau:
a)

x
2
+x 1 > x + 2
b)

2x
2
+ 3x 2 x + 1
c)

x
2
+ 3x + 2 <

3x
2
7x + 4
Gii: a) Bt phng trnh

x
2
+x 1 > (x + 2)
2
x + 2 0

x + 2 < 0
x
2
+x 1 0

3x + 5 < 0
x 2

x < 2
x
1 +

5
2
x
1

5
2

2 x <
5
3
x < 2
x <
5
3
b) Bt phng trnh

x + 1 0
2x
2
+ 3x 2 0
2x
2
+ 3x 2 (x + 1)
2

x 1
x 2 x
1
2
2x
2
+ 3x 2 x
2
+ 2x + 1

x
1
2
x
2
+x 3 0

x
1
2
1

13
2
x
1 +

13
2

1
2
x
1 +

13
2
c) Bt phng trnh

x
2
+ 3x + 2 0
x
2
+ 3x + 2 < 3x
2
7x + 4

x 1 x 2
2x
2
10x + 2 > 0

x 1 x 2
x >
5 +

21
2
x <
5

21
2
1 x <
5

21
2
x >
5 +

21
2
Ngi son : Th.s Minh Tun Trang 56 Khoa T nhin - Trng CSP Nam nh
T
h
.
s

M
i
n
h
T
u

n
3.5. Bi tp Chng 3. Phng trnh cha cn v du gi tr tuyt i
3.5 Bi tp
Phng trnh cha du gi tr tuyt i:
Bi 3.1: Gii cc phng trnh sau:
a) [2x 1[ +[2x + 1[ = 4
b) [x
2
3x + 2[ 2x = 1
c) [x
2
+x 12[ = x
2
x 2
d) [2 [2 x[[ = 1
e) [x
2
2x[ = [2x
2
1[
f) [x
2
5x + 4[ 9x
2
5x + 9 + 10x[x[ = 0
Hng dn. a) x = 1; 1
b) x =
5

21
2
c) x = 5;

7
d) x = 5; 3; 1; 1.
e) x = 1

2;
1
3
; 1.
f) x =
5

259
18
.

Bt phng trnh cha du gi tr tuyt i:
Bi 3.2: Gii cc bt phng trnh:
a) [x 3[ +[5 x[ < 3x
b) [x
2
x 6[ < x
c) [x
2
5x + 4[ > x 2
d) [2x [x 1[[ < 2
e) [3x
2
2x 1[ < [x
2
x[
Hng dn. a) x

8
5
; +

.
b) x

6; 1 +

.
c) x

; 2 +

3 +

3; +

.
Ngi son : Th.s Minh Tun Trang 57 Khoa T nhin - Trng CSP Nam nh
T
h
.
s

M
i
n
h
T
u

n
3.5. Bi tp Chng 3. Phng trnh cha cn v du gi tr tuyt i
d) x

1
3
; 1

.
e) x

1
2
;
1
4

.

Phng trnh cha cn thc:
Bi 3.3: Gii cc phng trnh sau:
a) x

2x + 3 = 0
b)

x + 4

1 x =

1 2x
c) (x 3) (x + 1) + 4 (x 3) .

x + 1
x 3
= 3
d)

3 +x +

6 x

(3 +x) . (6 x) = 3
e) x
3
+ 1 = 2
3

2x 1
Hng dn. a) x = 3.
b) x = 0.
c) x = 1

5; 1

13.
d) x = 3; 6.
e) x = 1;
1
2

5.
Bi 3.4: Gii cc phng trnh sau:
a)

x 2 +

4 x = x
2
6x + 11
b)
3

x + 34
3

x 3 = 1
c)

x + 2

x 1 +

x 2

x 1 =
x + 3
2
d)
x
2

3x 2

3x 2 = 1 x
e) x
2
+

x + 5 = 5
f)
3

x 1 +
3

x 2 =
3

2x 3
Hng dn. a) x = 3
b) x = 61; 30.
Ngi son : Th.s Minh Tun Trang 58 Khoa T nhin - Trng CSP Nam nh
T
h
.
s

M
i
n
h
T
u

n
3.5. Bi tp Chng 3. Phng trnh cha cn v du gi tr tuyt i
c) x = 1; 5.
d) x = 1.
e) x =
1 +

17
2
;
1

21
2
.
f) x = 1; 2.

Bt phng trnh cha cn thc :
Bi 3.5: Gii cc bt phng trnh sau:
a)

x + 9 +

2x + 4 > 5
b) 4(x + 1)
2
< (2x + 10)

3 + 2x

2
c) (x 3)

x
2
+ 4 x
2
9
d)

x + 2

x + 1

x
e)

2x
2
6x + 1 x 2
f)
x
x + 1
2

x + 1
x
> 3
g)

5x
2
+ 10x + 1 7 x
2
2x
h)

1 +x +

1 x 2
x
2
4
Hng dn. a) x (0; +)
b) x

3
2
; 1

(1; 3)
c) x

;
5
6

[3; +)
d) x

1 +
2

3
; +

.
e) x

;
3

7
2

[3; +)
f) x

4
3
; 1

.
g) x (; 3] [1; +).
Ngi son : Th.s Minh Tun Trang 59 Khoa T nhin - Trng CSP Nam nh
T
h
.
s

M
i
n
h
T
u

n
3.5. Bi tp Chng 3. Phng trnh cha cn v du gi tr tuyt i
h) x [1; 1]

Bi tp tng hp:
Bi 3.6: Gii cc phng trnh, bt phng trnh sau:
a)
3

7x + 1 +
4

9x + 7

x + 3 + 6
b)

x
2
+x 1 +
8

2x
2
+x 3 =

x
2
+ 3x 5 +
8

2x
2
+ 4x 9
c)
4

x +
4

17 x = 3
d)

x + 1 <
3

2 x + 3
Hng dn. a) x

7
9
; 1

.
b) x = 2.
c) x = 1; 16.
d) x (; 3)
Ngi son : Th.s Minh Tun Trang 60 Khoa T nhin - Trng CSP Nam nh
T
h
.
s

M
i
n
h
T
u

n
Chng 4. H phng trnh i s
Chng 4
H phng trnh i s
4.1 H phng trnh bc nht
4.1.1 H phng trnh bc nht hai n
Dng:

a
1
x +b
1
y = c
1
a
2
x +b
2
y = c
2
Cch gii:
D =

a
1
b
1
a
2
b
2

, D
x
=

c
1
b
1
c
2
b
2

, D
y
=

a
1
c
1
a
2
c
2

- Nu D = 0: h phng trnh c nghim duy nht

x =
D
x
D
, y =
D
y
D

.
- Nu D = 0 v D
2
x
+D
2
y
= 0: H v nghim.
- Nu D = D
x
= D
y
= 0, h phng trnh v s nghim hoc v nghim, mun bit r, ta
thay gi tr ca tham s vo phng trnh.
V d 4.1.1: Gii v bin lun h phng trnh theo tham s m:

a.x + (2 a) y = 3
(a 2) x + (1 2a) y = a 4
Gii: D =

a 2 a
a 2 1 2a

= a
2
3a + 4 = (a 1) . (a + 4)
D
x
=

3 2 a
a 4 1 2a

= a
2
12a + 11 = (a 1) . (a 11)
D
y
=

a 3
a 2 a 4

= a
2
7a + 6 = (a 1) . (a 6)
+) Trng hp 1: D = 0 (a 1) . (a + 4) = 0

a = 1
a = 4
Nu a = 4, D
x
= 0 th h v nghim.
Nu a = 1, D
x
= D
y
= 0, thay vo h phng trnh ta c:

x +y = 3
x y = 3
S = (x, 3 x) [x R.
Ngi son : Th.s Minh Tun Trang 61 Khoa T nhin - Trng CSP Nam nh
T
h
.
s

M
i
n
h
T
u

n
4.1. H phng trnh bc nht Chng 4. H phng trnh i s
+) Trng hp 2: D = 0

a = 1
a = 4
nn h phng trnh c nghim duy nht:

x =
12 a
a + 4
y =
6 a
4 +a
4.1.2 H phng trnh bc nht ba n
Dng

a
1
x +b
1
y +c
1
z = d
1
a
2
x +b
2
y +c
2
z = d
2
a
3
x +b
3
y +c
3
z = d
3
Cch gii:
0 Kh n bng cch rt 1 n t mt phng trnh th vo 2 phng trnh cn li, c h
phng trnh bc nht 2 n
O S dng my tnh in t gii.
O Dng php bin i Gauss.
V d 4.1.2: Gii h phng trnh:

x + 2y 3z = 2 (1)
2x y + 4z = 4 (2)
5x + 3y 4z = 4 (3)
Gii: C1: Kh n. T (1) x = 2y + 3z + 2. th vo (2), (3) ta c h :

2 (2y + 3z + 2) y + 4z = 4
5 (2y + 3z + 2) + 3y 7z = 4

5y + 10z = 0
7y + 8z = 6

y = 2
z = 1
x = 1.
H c nghim (x = 1, y = 2, z = 1)
C2: Bin i Gauss.
A =

1 2 3 2
2 1 4 4
5 3 7 4

H22H1

H35H1

1 2 3 2
0 5 10 0
0 7 8 6

Ta c h

5y + 10z = 0
7y + 8z = 6
v tip tc gii nh trn.
C3: S dng my tnh in t.
4.1.3 H phng trnh bc nht bn n
Dng :

a
1
x +b
1
y +c
1
z +d
1
t = e
1
a
2
x +b
2
y +c
2
z +d
2
t = e
2
a
3
x +b
3
y +c
3
z +d
3
t = e
3
a
4
x +b
4
y +c
4
z +d
4
t = e
4
Ngi son : Th.s Minh Tun Trang 62 Khoa T nhin - Trng CSP Nam nh
T
h
.
s

M
i
n
h
T
u

n
4.2. H phng trnh bc nht - bc hai: Chng 4. H phng trnh i s
Cch gii:
0 Rt 1 n t mt phng trnh, th vo 3 phng trnh cn li c h phng trnh bc
nht 3 n.
O Dng php bin i Gauss.
V d 4.1.3: Gii h phng trnh:

x 2y + 3z 4t = 5
2x + 3y 5z +t = 10
4x 5y + 6z 7t = 14
x +y +z +t = 3
Gii: - Cch 1: Phng php Gauss:
A =

1 2 3 4 5
2 3 5 1 10
4 5 6 7 14
1 1 1 1 3

H22H1
H34H1

H4H1

1 2 3 4 5
0 7 11 9 20
0 3 6 9 6
0 3 2 5 2

Gii h:

7y 11z + 9t = 20
3y 6z + 9t = 6
3y 2y + 5z = 2

y = 1
z = 2
t = 1
x = 1
H c nghim duy nht: (1; 1; 2; 1).
Cch 2: Rt x = 2y 3z + 4t + 5 th vo 3 phng trnh cn li ta c h phng trnh 3 n
ging nh php bin i Gauss.
4.2 H phng trnh bc nht - bc hai:
Dng: H gm 2 phng trnh trong :
Phng trnh bc nht v phng trnh bc 2.
Cch gii:
0 Rt 1 n t phng trnh bc nht, th vo phng trnh bc 2 cn li.
O Nu c phng trnh ch l phng trnh bc nht theo 1 bin, th ta rt bin ra th
vo phng trnh cn li.
O Nu c 2 phng trnh u khng l bc nht theo 2 bin, ta c th dng php bin i
i s cc phng trnh thu c phng trnh bc nht theo mt bin no . Rt
bin ra th vo phng trnh cn li.
V d 4.2.1: Gii cc h phng trnh:
a)

x + 2y = 1
x
2
+ 3y
2
2x = 2
b)

x
2
2y = 2
2x
2
+xy y = 9
Ngi son : Th.s Minh Tun Trang 63 Khoa T nhin - Trng CSP Nam nh
T
h
.
s

M
i
n
h
T
u

n
4.3. H phng trnh ng cp bc 2: Chng 4. H phng trnh i s
c)

2x
2
+ 4y
2
+x = 19
x
2
+y
2
+y = 7
Gii: a) T (1) ta c: x = 1 2y thay vo phng trnh (2) ta c:
(1 + 2y)
2
+ 3y
2
+ 2 (1 + 2y) = 2
1 + 4y + 4y
2
+ 3y
2
+ 2 + 4y 2 = 0
7y
2
+ 8y + 1 = 0

y = 1 x = 1
y = 1/7 x = 5/7
H c 2 nghim (1; 1) ; (5/7; 1/7).
b) Rt y =
x
2
2
2
t (1) th vo (2) ta c
2x
2
+x.
x
2
2
2

x
2
2
2
= 9
4x
2
+x
3
2x x
2
+ 2 = 18
x
3
+ 3x
2
2x 16 = 0 (x 2) (x
2
+ 5x + 8) = 0

x = 2 y = 1
x
2
+ 5x + 8 = 0

V nghim

Phng trnh c nghim duy nht (2; 1).
c) Ly phng trnh (1) tr i 2 ln phng trnh (2)
2y
2
+x 2y = 5 x = 2y
2
+ 2y + 5
Thay vo phng trnh (2) ta c (2y
2
+ 2y + 5)
2
+y
2
+y = 7
4y
4
+ 4y
2
+ 25 8y
3
+ 20y 20y
2
+y
2
+y 7 = 0
4y
4
8y
3
15y
2
+ 21y + 18 = 0
(y 2) (2y + 3) (2y
2
3y 3) = 0

y = 2
y = 3/2
2y
2
3y 3 = 0

y = 2
y = 3/2
y =
3 +

33
4
y =
3

33
4

x = 1
x = 5/2
x =
5

33
4
x =
5 +

33
4
Vy h c 4 nghim, tp nghim l:
S =

(1; 2) , (3/2; 5/2) ,

33
4
;
3 +

33
4

5 +

33
4
;
3

33
4

4.3 H phng trnh ng cp bc 2:


4.3.1 Phng trnh ng cp bc 2
Dng: a.x
2
+b.xy +c.y
2
= 0
Ngi son : Th.s Minh Tun Trang 64 Khoa T nhin - Trng CSP Nam nh
T
h
.
s

M
i
n
h
T
u

n
4.3. H phng trnh ng cp bc 2: Chng 4. H phng trnh i s
Cch gii:
Coi y nh mt phng trnh bc 2: a.t
2
+b.t +c = 0 nu a = 0
- Nu < 0: Phng trnh bc 2 n t v nghim, nhng phng trnh ng cp bc 2 c
nghim duy nht x = y = 0
- Nu 0: Phng trnh c nghim t nn ta c phng trnh ban u c nghim x = t.y
V d 4.3.1: Gii cc phng trnh sau:
a) x
2
+ 2xy + 2y
2
= 0
b) 2x
2
3xy 2y
2
= 0
Gii: a)

= (1)
2
1.2 = 1 < 0. Phng trnh c nghim duy nht: x = y = 0
b) = 3
2
4.2.(2) = 25 > 0. Nn phng trnh cho tng ng vi:

x =
3 5
4
.y =
1
2
y
x =
3 + 5
4
.y = 2y
4.3.2 H phng trnh ng cp bc 2
Dng ton:

a
1
x
2
+b
1
xy +c
1
y
2
= d
1
a
2
x
2
+b
2
xy +c
2
y
2
= d
2
Cch gii:
0 Cch 1: Quy v phng trnh ng cp bc 2.
- TH1: d
1
d
2
= 0, g/s d
1
= 0. Pt (1) l pt ng cp bc 2. Gii nh trn, tnh c x,
theo y th vo phng trnh cn li.
- TH2: d
1
, d
2
= 0. Nhn phng trnh (1) vi d
2
tr i phng trnh (2) nhn d
1
c
phng trnh ng cp bc 2. Tip tc nh trn.
O Cch 2: Kh y
2
t 2 phng trnh. Rt y t phng trnh . Th vo phng trnh ban
u c phng trnh bc 4 trng phng. Gii phng trnh, t suy ra nghim ca
phng trnh.
V d 4.3.2: Gii cc h phng trnh sau:
a)

x
2
xy +y
2
= 7
x
2
2xy 3y
2
= 0
b)

x
2
+xy y
2
= 1
2x
2
xy + 3y
2
= 12
c)

x
2
+xy 2y
2
= 0
2x
2
3xy +y
2
= 0
Ngi son : Th.s Minh Tun Trang 65 Khoa T nhin - Trng CSP Nam nh
T
h
.
s

M
i
n
h
T
u

n
4.3. H phng trnh ng cp bc 2: Chng 4. H phng trnh i s
Gii: a) C1: Phng trnh (2) ta c:
x = y hoc x = 3y
+) TH1: x = y thay vo pt (1) ta c: 3x
2
= 7 x =

7
3
y =

7
3
+) TH2 : x = 3y thay vo pt (1) ta c : 7y
2
= 7 y = 1 x = 3.
C2: T h phng trnh ta c
3 (x
2
xy +y
2
) +x
2
2xy 3y
2
= 21
4x
2
5xy = 21 y =
4x
2
21
5x
x
2
+x.
21 4x
2
5x
+

4x
2
21
5x

2
= 7
25x
4
+ 5x
2
(21 4x
2
) + (4x
2
21)
2
= 7.25x
2
25x
4
+ 105x
2
20x
4
+ 441 168x
2
+ 16x
4
175x
2
= 0
21x
4
238x
2
+ 441 = 0

x
2
= 9
x
2
=
7
3

x = 3 y = 1
x =

7
3
y =

7
3
H c 4 nghim, tp nghim l:
S =

(3; 1) , (3; 1) ,

7
3
;

7
3

7
3
;

7
3

b) Phng trnh (1) nhn 12 cng phng trnh (2) ta c:


14x
2
+ 11xy 9y
2
= 0

x =
1
2
y
x =
9
7
y
+) TH 1: x =
1
2
y th vo pt (1):
1
4
y
2
+
1
2
y
2
y
2
= 1 y
2
= 4 y = 2 x = 1
+) TH 2: x =
9
7
y thay vo pt (1) ta c:
81
49
y
2

9
7
y
2
y
2
= 1 y
2
=
49
31
y =
7

31
x =
9

31
H c 4 nghim, tp nghim l :
S =

(1; 2) , (1; 2) ,

31
;
7

31

31
;
7

31

c) Phng trnh (1) tng ng vi:


(x y) (x + 2y) = 0

x = y
x = 2y
Ngi son : Th.s Minh Tun Trang 66 Khoa T nhin - Trng CSP Nam nh
T
h
.
s

M
i
n
h
T
u

n
4.4. H i xng Chng 4. H phng trnh i s
+) TH 1: x = y thay vo phng trnh (2) ta c:
0 = 0 (Tha mn vi mi x, y)
+) TH 2: x = 2y thay vo phng trnh (2) ta c:
8y
2
+ 6y
2
+y
2
= 0 15y
2
= 0 y = 0 x = 0
Kt lun: Tp nghim ca phng trnh l: S = (x, x) [x R
4.4 H i xng
4.4.1 H i xng loi I:
Dng:

f (x, y) = 0
g (x, y) = 0
f (x, y), g (x, y) l a thc i xng, tc l:
f (x, y) = f (y, x), g (x, y) = g (y, x) x, y R
Cch gii:
t S = x + y, P = xy. Khi f (x, y) = h (S, P), g (x, y) = k (S, P). T ta c h
mi gm 2 n S, P. Gii h mi ny tm S, P. T ta c h:

x +y = S
x.y = P
Dng nh l o Vi-et x, y l hai nghim ca phng trnh: X
2
S.X +P = 0.
V d 4.4.1: Gii h phng trnh sau:
a)

x +y = 3
x
2
+y
2
= 5
b)

x y +xy = 3
x
2
+y
2
+xy = 7
Gii: a) t S = x +y, P = x.y do h tr thnh:

S = 3
S
2
2P = 5

S = 3
P = 2

x +y = 3
xy = 2
x, y l 2 nghim ca phng trnh : X
2
3X + 2 = 0

X = 1
X = 2
H c 2 nghim S = (1; 2) , (2; 1)
b) t S = x y, P = x.y ta c h :

S +P = 3
S
2
+ 2P +P = 7

P = 3 S
S
2
+ 3 (3 S) = 7

P = 3 S
S
2
3S + 2 = 0

P = 3 S

S = 1
S = 2

S = 1
P = 2
(1)

S = 2
P = 1
(2)
Ngi son : Th.s Minh Tun Trang 67 Khoa T nhin - Trng CSP Nam nh
T
h
.
s

M
i
n
h
T
u

n
4.4. H i xng Chng 4. H phng trnh i s
+) Gii h (1):

S = 1
P = 2

x y = 1
x.y = 2

x + (y) = 1
x. (y) = 2
x, y l 2 nghim ca phng trnh:
X
2
X 2 = 0

X = 1
X = 2
Do h c nghim (x, y) = (1; 2) , (2; 1).
Gii h (2)

S = 2
P = 1

x y = 2
x.y = 1

x + (y) = 2
x. (y) = 1
X
2
2X 1 = 0

X = 1 +

2
X = 1

2
Do h c 2 nghim

1 +

2, 1 +

2, 1

Kt lun vy h c 4 nghim:
(1; 2) , (2; 1) ,

1 +

2, 1 +

2, 1

4.4.2 H i xng loi II:


Dng :
H i xng loi II:

f (x, y) = 0
f (y, x) = 0
H i xng vng quanh:

f (x, y, z) = 0
f (y, z, x) = 0
f (z, x, y) = 0
Cch gii:
~ H i xng loi II:
- Tr v 2 phng trnh ta c: (x y) .P (x, y) = 0
( P (x, y) l mt a thc i xng).
- Nu x = y, thay vo hpt ban u gii.
- Nu P (x, y) = 0 ta c h i xng loi I:

P (x, y) = 0
f (x, y) +f (y, x) = 0
~ H i xng vng quanh:
- S dng phng php hm s gii:
V d 4.4.2: Gii h phng trnh:
Ngi son : Th.s Minh Tun Trang 68 Khoa T nhin - Trng CSP Nam nh
T
h
.
s

M
i
n
h
T
u

n
4.4. H i xng Chng 4. H phng trnh i s
a)

x
2
= 2x +y 2
y
2
2y = x 2
b)

x
3
= x 4y + 6
y
3
= y 4x + 6
c)

x
3
= 4y 3
y
3
= 4z 3
z
3
= 4x 3
Gii: a) Tr v 2 phng trnh ta c:
(x y) (x +y) (x y) = 0 (x y) (x +y 1) = 0
+)TH 1: x = y ta c
x
2
3x + 2 = 0

x = 1 y = 1
x = 2 y = 2
+) TH 2: x +y = 1 y = 1 x
x
2
= 2x + 1 x 2 x
2
x + 1 = 0 (V nghim)
H c 2 nghim phn bit:
(1; 1) , (2; 2)
b) Tr v 2 phng trnh ta c:
(x y) (x
2
+xy +y
2
) = 3 (x y)

x = y
x
2
+xy +y
2
= 3
TH 1: x = y thay vo phng trnh ta c:
x
3
+ 5x 6 = 0 (x 1) (x
2
+x + 6) = 0

x = 1 y = 1
x
2
+x + 6 = 0 (V nghim)
TH 2: x
2
+xy + y
2
= 3
Cng v 2 phng trnh u kt hp vi phng trnh trn ta c h phng trnh:

x
2
+xy +y
2
= 3
x
3
+y
3
= 5 (x +y) + 12
t S = x +y, P = x.y (S
2
4P) th vo h phng trnh ta c h mi:

S
2
P = 3
S
3
3SP = 5S + 12
T pt (1) ta c P = S
2
3 th vo phng trnh (2) ta c
S
3
3S (S
2
3) = 5S + 12 S
3
7S + 6 = 0
(S 1) . (S
2
+S 6) = 0

S = 1 P = 2
S = 3 P = 6 (V nghim)
S = 2 P = 1
+)

S = 1
P = 2

x +y = 1
x.y = 2
Ngi son : Th.s Minh Tun Trang 69 Khoa T nhin - Trng CSP Nam nh
T
h
.
s

M
i
n
h
T
u

n
4.4. H i xng Chng 4. H phng trnh i s
x, y l 2 nghim ca phng trnh: X
2
X 2 = 0

X = 1
X = 2
H c 2 nghim (1; 2) , (2; 1)
+)

S = 2
P = 1

x +y = 2
x.y = 1
x, y l 2 nghim ca phng trnh:
X
2
2X + 1 = 0 X = 1
H phng trnh c nghim (1; 1)
Kt lun: H phng trnh c 3 nghim (1; 1), (1; 2) , (2; 1)
c) H phng trnh tng ng vi:

x =
z
3
+ 3
4
y =
x
3
+ 3
4
z =
y
3
+ 3
4
t f (x) =
x
3
+ 3
4
, f (x) l hm ng bin.
H tr thnh

x = f (z)
y = f (x)
z = f (y)
Gi s x > y ta c:
y = f (x) > f (y) = z
z = f (y) > f (z) = x
x > y > z > x (Mu thun)
Tng t nu x < y ta cng c x < y < z < x (Mu thun)
Vy x = y y = f (x) = f (y) = z z = f (y) = f (z) = x
x = y = z thay vo h ta c:
x
3
= 4x 3 x
3
4x + 3 = 0 (x 1) (x
2
+x 3) = 0

x = 1
x
2
+x 3 = 0

x = 1
x =
1 +

13
2
x =
1

13
2
H c 3 nghim
(1; 1; 1) ,

1 +

13
2
;
1 +

13
2
;
1 +

13
2

13
2
;
1

13
2
;
1

13
2

Ngi son : Th.s Minh Tun Trang 70 Khoa T nhin - Trng CSP Nam nh
T
h
.
s

M
i
n
h
T
u

n
4.5. H phng trnh tng qut Chng 4. H phng trnh i s
4.5 H phng trnh tng qut
Cch gii:
0 t n ph quy v h quen thuc.
O Cng tr i s cc phng trnh.
O Th n.
O Phn tch thnh nhn t.
V d 4.5.1: Gii cc h phng trnh sau:

x
2
+y +x
3
y +xy
2
+xy =
5
4
x
4
+y
2
+xy (1 + 2x) =
5
4
Gii: H phng trnh tng ng

x
2
+y +xy (x
2
+y) +xy =
5
4
(x
2
+y)
2
+xy =
5
4
t u = x
2
+y, v = xy
H tr thnh:

u +u.v +v =
5
4
u
2
+v =
5
4
T phng trnh (2) ta c v =
5
4
u
2
Th vo phng trnh (1) ta c:
u
5
4
u u
3

5
4
u
2
=
5
4
u
3
u
2

1
4
u = 0
u.(2u + 1)
2
= 0

u = 0 v =
5
4
u =
1
2
v =
3
2
+) TH 1: u = 0, v =
5
4

x
2
+y = 0
xy =
5
4
Ta c y = x
2
th vo phng trnh cn li ta c x
3
=
5
4
x =
3

5
4
y =
3

25
16
Ngi son : Th.s Minh Tun Trang 71 Khoa T nhin - Trng CSP Nam nh
T
h
.
s

M
i
n
h
T
u

n
4.5. H phng trnh tng qut Chng 4. H phng trnh i s
+) TH 2: u =
1
2
, v =
3
2
Ta c h:.

x
2
+y =
1
2
xy =
3
2
T phng trnh (1) ta c y = x
2

1
2
x

x
2

1
2

=
3
2
2x
3
+x 3 = 0 (x 1) (2x
2
+ 2x + 3) = 0

x = 1 y =
3
2
2x
2
+ 2x + 3 = 0

Phng trnh v nghim

Kt lun: H c 2 nghim phn bit:

1;
3
2

5
4
;
3

25
16

V d 4.5.2 (D-2008): Gii h phng trnh:

xy +x +y = x
2
2y
2
x

2y y

x 1 = 2x 2y
.
V d 4.5.3 (B-2008): Gii h phng trnh:

x
4
+ 2x
3
y +x
2
y
2
= 2x + 9
x
2
+ 2xy = 6x + 6
V d 4.5.4 (B-2009): Gii h phng trnh:

xy +x + 1 = 7y (1)
x
2
y
2
+xy + 1 = 13y
2
(2)
Gii: Cch 1: Phng trnh ng cp:
Ta c

xy + 1 = x + 7y
(xy + 1)
2
xy = 13y
2
Do ta c (x7y)
2
xy = 13y
2
x
2
14xy +49y
2
xy 13y
2
= 0 x
2
15xy +36y
2

x = 3y
x = 12y
.
+) TH1: Vi x = 3y thay vo h ta c:

3y
2
4y + 1 = 0
9y
4
10y
2
+ 1 = 0

y = 1 x = 3
y =
1
3
x = 1
.
+) TH2: Vi x = 12y thay vo (1) ta c 12y
2
+ 5y + 1 = 0 (V nghim)
Cch 2: Th
T (1) ta c x =
7y 1
y + 1
th vo (2) ta c:
36y
4
33y
3
5y
2
+y + 1 = 0 (y 1)(3y 1)(12y
2
+ 5y + 1) = 0

y = 1 x = 3
y =
1
3
x = 1
12y
2
+ 5y + 1 = 0 (V nghim)
Ngi son : Th.s Minh Tun Trang 72 Khoa T nhin - Trng CSP Nam nh
T
h
.
s

M
i
n
h
T
u

n
4.6. Bi tp Chng 4. H phng trnh i s
Cch 3: t n ph quy v h i xng
Nhn thy y = 0 th phng trnh (2) khng tha mn.
Nu y = 0, chia 2 v ca pt (1) cho y, chia 2 v ca phng trnh (2) cho y
2
ta c:

x +
x
y
+
1
y
= 7
x
2
+
x
y
+
1
y
2
= 13
t u =
1
y
ta c h.

x +xu +u = 7
x
2
+xu +u
2
= 13
.
n y hc sinh t gii tip.
4.6 Bi tp
H phng trnh bc nht:
Bi 4.1: Gii v bin lun h phng trnh:
a)

mx + 3y = m
3x +my = 8
b)

2x + (m1) y = 2m3
m
2
x +my = 4mm
2
Bi 4.2: Tm gi tr ca m h phng trnh :

x my = 1
mx +y = 3
c nghim (x, y) tha mn x.y < 0
Bi 4.3: Gi s (x, y) l 2 s tha mn iu kin:

x my = 2 4m
mx +y = 3m + 1
Tm gi tr ln nht, nh nht ca biu thc: A = x
2
+y
2
2x khi m thay i.
Bi 4.4: Gii h phng trnh bc nht sau:
a)

x + 3y z = 6
2x y + 3z = 3
x + 5y 7z = 2
b)

2x +y 3z 4t = 10
x 3y + 4z 5t = 2
3x 4y + 2z +t = 11
x + 2y 7z + 6t = 2
Ngi son : Th.s Minh Tun Trang 73 Khoa T nhin - Trng CSP Nam nh
T
h
.
s

M
i
n
h
T
u

n
4.6. Bi tp Chng 4. H phng trnh i s
Hng dn. a)

4;
17
8
;
19
8

.
b) (1; 1; 1; 2).

H phng trnh i xng loi 1:
Bi 4.5: Gii cc h phng trnh sau:
a)

x y = 1
x
3
y
3
= 1
b)

x
3
y
3
= 3 (x y)
x +y = 1
c)

x +y +xy = 5
x
2
+y
2
= 5
d)

x +xy +y = 3
x
2
y +xy
2
= 2
e)

xy (x + 2) (y + 2) = 24
x
2
+y
2
+ 2 (x +y) = 11
f)

x
3
+y
3
= 12 (x +y)
x y = 2
g)

x
2
y +xy
2
= 30
x
3
+y
3
= 35
h)

x
2
+y
2
+x +y = 4
x(x +y + 1) +y (y + 1) = 2
i)

x
2
+ 4x +y = 7
x(x + 3) (x +y) = 12
j)

x y +x
2
+y
2
= 12
x y +xy = 5
k)

x + 1 +

y + 1 = 2 +

x +

y =

3 + 1
l)

x 4 +

y 1 = 4
x +y = 73
m)

x
3
y
3
= 7
xy (x y) = 2
Hng dn. a) (1; 0), (0; 1).
Ngi son : Th.s Minh Tun Trang 74 Khoa T nhin - Trng CSP Nam nh
T
h
.
s

M
i
n
h
T
u

n
4.6. Bi tp Chng 4. H phng trnh i s
b) (2; 1),

1
2
;
1
2

, (1; 2).
c) (2; 1), (1; 2).
d) (1; 1).
e) (1; 4), (1; 2), (2; 3), (2; 1), (4; 3), (4; 1), (3; 4), (3; 2).
f) (2; 4), (4; 2), (1; 1).
g) (3; 2), (2; 3).
h) (2; 1), (1; 2),

2;

2;

.
i) (4; 7), (1; 2),

3 +

21
2
;
11

21
2

21
2
;
11 +

21
2

.
j) (3; 1), (2; 3), (1; 3), (3; 2).
k) (1; 3), (3; 1).
l) (38 4

30; 35 + 4

30).
m) (2; 1), (1; 2).

H phng trnh i xng loi 2:
Bi 4.6: a)

2x +

y 1 = 5
2y +

x 1 = 5
b)

3x
2
= 2y +
1
y
3y
2
= 2x +
1
x
c)

x
2
= 3x + 2y
y
2
= 3y + 2x
d)

x + 5 +

y 2 = 7

y + 5 +

x 2 = 7
e)

x 3y = 4
y
x
y 3x = 4
x
y
f)

x +
4

y 1 = 1
y +
4

x 1 = 1
g)

x
3
= 3x + 8y
y
3
= 3y + 8x
Ngi son : Th.s Minh Tun Trang 75 Khoa T nhin - Trng CSP Nam nh
T
h
.
s

M
i
n
h
T
u

n
4.6. Bi tp Chng 4. H phng trnh i s
h)

x +
2xy
3

x
2
2x + 9
= x
2
+y
y +
2xy
3

y
2
2y + 9
= y
2
+x
i)

x = y
2
2y + 2
y = z
2
2z + 2
z = x
2
2x + 2
Hng dn. a) (2; 2).
b) (1; 1).
c) (0; 0), (5; 5), (2; 1), (1; 2).
d) (11; 11).
e) (2; 2).
f) (1; 1).
g) (0; 0),

11;

11

11;

11

.
h) (0; 0), (1; 1).
i) (2; 2; 2), (1; 1; 1).
Bi 4.7: Gii phng trnh : x
3
+ 2 = 3
3

3x 2
Hng dn. x = 1
Bi 4.8: Chng minh rng h c ng 2 nghim tha mn iu kin: x > 0, y > 0

e
x
= 2007
y

y
2
1
e
y
= 2007
x

x
2
1

H phng trnh ng cp bc 2:
Bi 4.9: Gii cc h phng trnh:
a)

x
2
4xy +y
2
= 1
y
2
3xy = 4
b)

x
2
3xy +y
2
= 5
2x
2
xy y
2
= 2
c)

3x
2
+ 5xy 4y
2
= 38
5x
2
9xy 3y
2
= 15
Hng dn. a) (1; 4), (1; 4).
Ngi son : Th.s Minh Tun Trang 76 Khoa T nhin - Trng CSP Nam nh
T
h
.
s

M
i
n
h
T
u

n
4.6. Bi tp Chng 4. H phng trnh i s
b) (1; 1), (1; 1).
c) (3; 1), (3; 1).

H phng trnh bc 1 - bc 2:
Bi 4.10: Gii h phng trnh sau:
a)

2x
2
5xy + 2y
2
= 0
x
2
+y = 5
b)

x + 2y = 1
2x
3
3y
3
= 5
Hng dn. a) (2; 1),

5
2
;
5
4

1 +

6; 2 + 2

6; 2 2

.
b) (1; 1).

H phng trnh tng qut:
Bi 4.11: Gii h phng trnh:
a)

x
y

2
+

x
y

3
= 12
(xy)
2
+xy = 6
b)

y +xy
2
= 6x
2
1 +x
2
y
2
= 5x
2
c)

x
4
x
3
y +x
2
y
2
= 1
x
3
y x
2
+xy = 1
d)

2x +y + 1

x +y = 1
3x + 2y = 4
e)

xy +x +y = x
2
2y
2
x

2y y

x 1 = 2x 2y
f)

x
4
+ 2x
3
y +x
2
y
2
= 2x + 9
x
2
+ 2xy = 6x + 6
g)

2 (y x) +xy = 4
x
2
+y
2
+ 4 (x +y) = 17
Hng dn. a) (2; 1), (2; 1).
b) (1; 2),

1
2
; 1

.
Ngi son : Th.s Minh Tun Trang 77 Khoa T nhin - Trng CSP Nam nh
T
h
.
s

M
i
n
h
T
u

n
4.6. Bi tp Chng 4. H phng trnh i s
c) (1; 1), (1; 1).
d) (2; 1).
e) (5; 2).
f)

4;
17
4

.
g) (5; 2), (1; 2), (2; 7), (2; 3).
Ngi son : Th.s Minh Tun Trang 78 Khoa T nhin - Trng CSP Nam nh
T
h
.
s

M
i
n
h
T
u

n
Chng 5. Gii tch t hp
Chng 5
Gii tch t hp
5.1 Khi qut chung
Trong nhng nm gn y, thi i hc mn Ton thng xut hin mt s dng bi v t
hp sau:
0 Bi ton m: bi ton ny thng c gii quyt bng cch s dng cc quy tc cng,
quy tc nhn, hon v, t hp, chnh hp.
O Gii phng trnh t hp (C cha C
k
n
, A
k
n
, P
n
, ...)
O Chng minh ng thc t hp: dng ny thng c gii quyt bng cch s dng cng
thc nh thc Newton, kt hp vi php tnh o hm, nguyn hm.
O Xc nh s hng khng cha x, cha x
k
, ... trong khai trin mt nh thc Newton.
Tuy nhin khng hn l tt c cc bi u nh vy. Chng c th c trn ln cc dng vo
vi nhau. Chng hn nh c th kt hp vic chng minh ng thc t hp gii phng trnh
t hp tm s m n ca khai trin sau trong ng thc khai trin tm s hng cha x
k
.
5.2 Kin thc c bn
5.2.1 Quy tc cng - nhn
Quy tc cng:
Nu hnh ng H :

H
1
H
2
...
H
n
m H
i
, H
j
khng xy ra ng thi th
[H[ = [H
1
[ +[H
2
[ + +[H
n
[
Cch nhn bit: trong phng n gii c s dng vic chia cc trng hp
Quy tc nhn:
Nu hnh ng H c thc hin bng cch thc hin mt chui cc hnh ng c lp
ni tip nhau:
H : H
1
H
2
H
n
th [H[ = [H
1
[.[H
2
[ [H
n
[
Ngi son : Th.s Minh Tun Trang 79 Khoa T nhin - Trng CSP Nam nh
T
h
.
s

M
i
n
h
T
u

n
5.2. Kin thc c bn Chng 5. Gii tch t hp
5.2.2 T hp - chnh hp - hon v
Hon v:
Cho N = 1, 2, , n, mt b gm n s (x
1
, x
2
, , x
n
) tha mn x
i
= x
j
i = j, x
i
N
gi l mt hon v ca N. S cc hon v ca N:
P
n
= n.(n 1)...2.1 = n!
Chnh hp:
Cho N = 1, 2, , n, k l s t nhin b hn n. Mt b gm k s (x
1
, x
2
, , x
k
) tha
mn x
i
= x
j
i = j, x
i
N c gi l mt chnh hp chp k ca n phn t. S cc chnh
hp chp k ca n phn t l
A
k
n
=
n!
(n k)!
= n.(n 1)...(n k + 1)
V d: Tnh A
2
n
, A
3
n+1
.
T hp :
Cho N = 1, 2, , n, k l s t nhin b hn n. Mt b gm k s [x
1
, x
2
, , x
k
] khng
phn bit th t tha mn x
i
= x
j
i = j, x
i
N c gi l mt t hp chp k ca n
phn t. S cc t hp chp k ca n phn t l
C
k
n
=
A
k
n
k!
=
n!
(n k)!k!
Khng phn bit th t c ngha l i ch cc s cho nhau trong b s khng c tnh
l mt phn t khc. Chng hn [1, 2, 3] = [3, 1, 2].
Hc sinh t chng minh cc cng thc nh mt bi tp (s dng quy tc nhn)
V d: Tnh C
3
n
, C
2
n+1
, C
n1
n+2
Tnh cht:
+) Tnh i xng: C
k
n
= C
nk
n
.
+) Tam gic Pascal: C
k
n
+C
k+1
n
= C
k+1
n+1
Ch : S khc bit ca t hp v chnh hp chnh l tnh c th t hay khng, y l
ch hc sinh hay nhm ln nht. Chng hn nh: ly 3 hc sinh trong mt lp 52 ngi
lm cn s lp th khng c th t nhng nu ly 3 hc sinh trong 1 ngi lm lp
trng, mt lp ph, mt b th th l c th t, thm ch nu ly 3 ngi trong 1 lp
trng v 2 lp ph th li xen ln c th t v khng c th t! (gia lp trng v 2 lp
ph th c th t nhng gia 2 lp ph li khng c th t)
5.2.3 Cng thc nh thc Newton
Mt s cng thc hay dng:
(a +b)
n
=
n

k=0
C
k
n
a
nk
b
k
= C
0
n
a
n
+C
1
n
a
n1
b + +C
n
n
b
n
Ngi son : Th.s Minh Tun Trang 80 Khoa T nhin - Trng CSP Nam nh
T
h
.
s

M
i
n
h
T
u

n
5.3. Cc v d Chng 5. Gii tch t hp
(1 +x)
n
= C
0
n
+C
1
n
x + +C
n
n
x
n
C
k
n
+C
k+1
n
= C
k+1
n+1
C
k
n
= C
nk
n
Cng thc nh thc Newton kh n gin nhng vic p dng n trong tng trng hp l rt
phong ph, iu ny lm cho n tr nn kh kh khn cho hc sinh mi lam quen vi t hp.
Chng hn nh vi ng thc th hai chng qua l ng thc mt nhng vi a = 1, b = x. ng
thc th 2 nu ta thay x = 1, hoc x = 1 th c cc ng thc kh th v, chng xut hin
trong thi i hc nhng nm gn y!
5.3 Cc v d
V d 5.3.1: Tm s cc s t nhin c 4 ch s khc nhau sao cho s l chn.
Gii: Gi s abcd l s cn tm t ta c:
abcd tha mn

a, b, c, d 0, 1, 2, , 9
a = 0, d 0, 2, 4, 6, 8
a, b, c, d khc nhau i mt
Ta xt cc trng hp sau:
TH1: d = 0, a c 9 cch chn, b c 8 cch chn, c c 7 cch chn
c 9.8.7 = 504 cch chn.
TH2: d = 0, d 2, 4, 6, 8 nn d c 4 cch chn, a c 8 cch chn, b c 8 cch chn, c c 7 cch
chn c 4.8.8.7 = 1792 cch chn.
Kt hp ta c : 504 + 1792 = 2296 cch chn.
V d 5.3.2: Tm s cc s t nhin c 4 ch s khc nhau sao cho s nh hn 4653.
Gii: Gi s abcd l s cn tm theo bi ra ta c:
abcd tha mn

a, b, c, d 0, 1, 2, ..., 9
a = 0
abcd < 4653
Ta xt cc trng hp sau:
TH1: a 1, 2, 3
do a c 3 cch chn, b c 9 cch chn, c c 8 cch chn, d c 7 cch chn
c 3.9.8.7 = 1512.
TH2: a = 4, b 0, 1, 2, 3, 5
do b c 5 cch chn, c c 8 cch chn, d c 7 cch chn vy c 5.8.7 = 280
TH3: a = 4, b = 6, c 0, 1, 2, 3
do c c 4 cch chn, d c 7 cch chn.
c 4.7 = 28 cch chn. TH4: a = 4, b = 6, c = 5, d 0, 1, 2 Vy d c 3 cch chn.
Kt lun: C 1512 + 280 + 28 + 3 = 1823 cch
V d 5.3.3: i thanh nin xung kch mt trng ph thng gm 12 hc sinh trong c 5
hc sinh lp A, 4 hc sinh lp B, 3 hc sinh lp C. Cn chn 4 hc sinh i lm nhim v sao
cho 4 hc sinh ny khng qu 2 trong 3 lp trn. Hi c bao nhiu cch chn nh vy?
Ngi son : Th.s Minh Tun Trang 81 Khoa T nhin - Trng CSP Nam nh
T
h
.
s

M
i
n
h
T
u

n
5.3. Cc v d Chng 5. Gii tch t hp
Gii: Cch 1: Gin tip
Cch lm gin tip: Thay v m tp A ta m tp m v phn b ca n l A = `A.
Sau s dng cng thc [A[ = [[

[A[ k hiu s phn t tp A.


Ly 4 hc sinh trong 12 hc sinh: c C
4
12
= 495.
By gi ta chn 4 hc sinh sao cho 4 hs ny trong c 3 lp:
STT Lp A Lp B Lp C S cch chn
5 4 3
1 2 1 1 C
2
5
.C
1
4
.C
1
3
= 120
2 1 2 1 C
1
5
.C
2
4
.C
1
3
= 90
3 1 1 2 C
1
5
.C
1
4
.C
2
3
= 60
Tng: 270
Vy c : 495 270 = 225 cch chn.
Cch 2: Trc tip. C th dng cch lp bng nh trn lm trc tip nhng s di hn cch
ny. Hc sinh t lm nh mt bi tp.
V d 5.3.4: Tm s hng khng cha x trong khai trin nh thc ca

2x +
1
5

n
(x > 0)
bit n tha mn iu kin C
0
n
+ C
1
n+1
+ C
n
n+2
= 10n + 30. Trong khai trin nh thc trn, tm
hng t c h s ln nht.
Gii: T iu kin ta c
1 +n + 1 +
(n + 1) (n + 2)
2
= 10n + 30
n
2
15n 54 = 0

n = 18
n = 3 (loi)
Vy n = 18 v ta c

2x +
1
5

18
=
18

k=0
C
k
18
.(2x)
18k
.

1
5

k
=
18

k=0
C
k
18
.2
18k
x
18k
k
5
S hng khng cha x 18 k
k
5
= 0 k = 15.
H s ca s hng khng cha x l: 2
3
.C
15
18
= 6528
t a
k
= C
k
18
.2
18k
, k = 0, 1, 2, .., 18
Xt bt phng trnh C
k
18
.2
18k
< C
k+1
18
.2
17k
2.
18!
k!. (18 k)!
<
18!
(k + 1)! (17 k)!

2. (k + 1)!
k!
<
(18 k)!
(17 k)!
2 (k + 1) < 18 k
3k < 16 k <
16
3
k = 0; 1; ..; 5
a
0
< a
1
< ... < a
5
< a
6
> a
7
> .. > a
18
Vy hng t c h s ln nht l: C
6
18
.2
186
x
18
6.6
5
= C
6
18
.2
12
.x
54
5
V d 5.3.5: Chng minh cc ng thc t hp sau:
a) C
k
n
+ 3C
k+1
n
+ 3C
k+2
n
+C
k+3
n
= C
k+3
n+3
n, k N, n k + 3.
b) 2C
0
n
+ 3C
1
n
+ 4C
2
n
+ + (n + 2) C
n
n
= (n + 4) 2
n1
Ngi son : Th.s Minh Tun Trang 82 Khoa T nhin - Trng CSP Nam nh
T
h
.
s

M
i
n
h
T
u

n
5.3. Cc v d Chng 5. Gii tch t hp
c)
C
0
n
2
+
C
1
n
3
+ +
C
n
n
n + 1
=
1 + 2
n+1
n
n
2
+ 3 n + 2
Gii: a) S dng phng php ng nht h s:
Ta c (1 +x)
n+3
= (1 +x)
n
(1 +x)
3
(1)
M ta c (1 +x)
n+3
=
n+3

k=0
C
k
n+3
x
k
nn h s ca x
k+3
l C
k+3
n+3
. (2)
Mt khc (1 +x)
n
(1 +x)
3
=

i=0
C
i
n
x
i

j=0
C
j
3
x
j

.
Vi n k + 3, ta i +j = k + 3 vi 0 j 3 c nghim nh sau:
i k + 3 k + 2 k + 1 k
j 0 1 2 3
Do h s ca x
k+3
s l:
C
k
n
.C
3
3
+C
k+1
n
.C
2
3
+C
k+2
n
.C
1
3
+C
k+3
n
.C
0
3
= C
k
n
+ 3C
k+1
n
+ 3C
k+2
n
+C
k+3
n
(3)
T (1), (2), (3) ta c iu phi chng minh.
b) S dng php tnh o hm.
Ta c (1 +x)
n
= C
0
n
+C
1
n
x + +C
n
n
x
n
Do x
2
(1 +x)
n
= C
0
n
x
2
+C
1
n
x
3
+ +C
n
n
x
n+2
o hm 2 v ca ng thc trn ta c:
2x(1 +x)
n
+nx
2
(1 +x)
n1
= 2C
0
n
x + 3C
1
n
x
2
+ + (n + 2) C
n
n
x
n+1
Thay x = 1 ta c:
2C
0
n
+ 3C
1
n
+ + (n + 2) C
n
n
= (n + 4) 2
n1
c) S dng tch phn.
Ta c (1 +x)
n
= C
0
n
+C
1
n
x + +C
n
n
x
n
Do x(1 +x)
n
= C
0
n
x +C
1
n
x
2
+ +C
n
n
x
n+1
.
Ly tch phn 2 v trn on [0; 1] ta c:
1

0
x(1 +x)
n
dx =
1

C
0
n
x +C
1
n
x
2
+ +C
n
n
x
n+1

dx
Mt khc ta c:
1

0
x(1 +x)
n
dx =
1

(1 +x)
n+1
(1 +x)
n

dx
=

(1 +x)
n+2
n + 2

(1 +x)
n+1
n + 2

1
0
=
1 + 2
n+1
n
n
2
+ 3 n + 2
Ngi son : Th.s Minh Tun Trang 83 Khoa T nhin - Trng CSP Nam nh
T
h
.
s

M
i
n
h
T
u

n
5.4. Bi tp Chng 5. Gii tch t hp
M
1

0
(C
0
n
x +C
1
n
x
2
+ +C
n
n
x
n+1
) dx =

C
0
n
x
2
2
+C
1
n
x
3
2
+ +C
n
n
x
n+2
n + 2

1
0
=
C
0
n
2
+
C
1
n
3
+ +
C
n
n
n + 1
nn ta c iu phi chng minh.
5.4 Bi tp
Bi ton m:
Bi 5.1: Trong mt trng hc c 5 em hc sinh khi 12; 3 em hc sinh khi 11; 2 em hc sinh
khi 10 l hc sinh xut sc. Hi c bao nhiu cch c 5 em hc sinh xut sc ca trng tham
gia mt on i biu sao cho mi khi c t nht 1 em.
Bi 5.2: Trong s 16 hc sinh c 3 hc sinh gii, 5 hc sinh kh, 3 trung bnh. C bao nhiu
cch chia s hc sinh trn lm 2 t sao cho:
a) Mi t c ng 8 hc sinh.
b) Mi t c ng 8 hc sinh sao cho mi t u c hc sinh gii v t nht 2 hc sinh kh.
Bi 5.3: Cho cc s 0, 1, 2, 3, 4.
a) C th thnh lp c bao nhiu s c 8 ch s thnh lp t 5 ch s trn.
b) C th thnh lp c bao nhiu s c 8 ch s thnh lp t 5 ch s trn sao cho ch s 3
c mt ng 4 ln, cc ch s khc c mt ng mt ln.
c) C th thnh lp c bao nhiu s c 8 ch s thnh lp t 5 ch s trn sao cho ch s 3
c mt ng 2 ln, ch s 0 c mt 3 ln, cc ch s khc c mt nhiu nht mt ln.
Bi 5.4: Vi cc ch s 0, 1, 2 ,3, 4 ,5 ,6 c th thnh lp c bao nhiu s t nhin m mi
s c 5 ch s khc nhau. Trong nht thit phi c mt ch s 5. Cng cu hi nh vy
nhng thm iu kin s l s chn th kt qu l bao nhiu?
Bi 5.5: C bao nhiu s t nhin chn gm 5 ch s khc nhau sao cho trong khng c mt
ch s 2.
Bi 5.6: C bao nhiu ch s chn gm 4 ch s i mt khc nhau v ln hn 4657
Bi 5.7: C bao nhiu ch s t nhin gm 4 ch s khc nhau chia ht cho 2.
Bi 5.8: T mt tp th gm 14 ngi gm 6 nam v 8 n trong c An v Bnh. Ngi ta
mun chn mt t cng tc gm 6 ngi. Tm s cch chn sao cho:
a) Trong mi t c c nam v n.
b) Trong mi t c 1 t trng v 5 t vin sao cho An v Bnh khng ng thi c mt trong
t.
Ngi son : Th.s Minh Tun Trang 84 Khoa T nhin - Trng CSP Nam nh
T
h
.
s

M
i
n
h
T
u

n
5.4. Bi tp Chng 5. Gii tch t hp
Bi 5.9: Tm tng tt c cc s t nhin c 5 ch s khc nhau thnh lp t 6 ch s: 1, 3, 4,
5, 7, 8? Cng cu hi nh vy nu ta thm ch s 0 vo 6 ch s trn.

Phng trnh t hp
Bi 5.10: Gii phng trnh :
1
A
2
2
+
1
A
2
3
+ +
1
A
2
n
=
2007
2008
Bi 5.11: Gii phng trnh t hp sau:
a) 10C
2
n
+nC
2
10
= 2800
b)
P
n+5
(n k)!
60A
k+2
n+3
n, k N
c) 2P
n
+ 6A
2
n
P
n
.A
2
n
= 12.
d) C
7
n
= 2C
3
n
.
e) C
0
n
+C
1
n
+C
2
n
= 8n + 1
Bi 5.12: Cho tp hp A gm n phn t, n 4. Bit rng s tp con 4 phn t ca A bng 20
ln s tp con gm 2 phn t ca A. Tm k 1, 2, , n sao cho s tp con gm k phn t
ca A l ln nht.
Bi 5.13: Tm cc s nguyn k, n tha mn iu kin
C
k1
n
2
=
C
k
n
9
=
C
k+1
n
24
.
Bi 5.14: Tnh gi tr ca biu thc M =
A
4
n+1
+ 3A
3
n
(n + 1)!
bit rng C
2
n+1
+ 2C
2
n+2
+ 2C
2
n+3
+C
2
n+4
= 149
Bi 5.15: Chng minh rng
n + 1
n + 2

1
C
k
n+1
+
1
C
k+1
n+1

=
1
C
k
n
vi n, k nguyn dng.
Bi 5.16: Tm s m trong dy x
n
=
A
4
n+4
P
n+2

143
4P
n

ng dng cng thc nh thc Newton
Bi 5.17: Chng minh cc ng thc sau:
a) (C
0
n
)
2
+ (C
1
n
)
2
+ + (C
n
n
)
2
= C
n
2n
b) 2.1.C
2
n
+ 3.2.C
3
n
+ +n(n 1) .C
n
n
= n(n 1) .2
n2
c) C
0
2n
+
1
3
C
2
2n
+ +
1
2n + 1
C
2n
2n
=
2
2n+1
2n + 1
.
Ngi son : Th.s Minh Tun Trang 85 Khoa T nhin - Trng CSP Nam nh
T
h
.
s

M
i
n
h
T
u

n
5.4. Bi tp Chng 5. Gii tch t hp
d) C
n
n
+
1
2
C
n1
n
+ +
1
n + 1
C
0
n
=
2
n+1
1
n + 1
Bi 5.18: Bit rng trong khai trin nh thc Newton ca

x +
1
x

n
tng cc h s ca 2 s hng
u tin bng 24, tnh tng cc h s ca cc ly tha bc nguyn dng ca x. Chng minh
tng ny l mt s chnh phng.
Bi 5.19: Gi s (1 + 2x)
n
= a
0
+a
1
x + +a
n
x
n
. Bit rng a
0
+a
1
+ +a
n
= 729. Tm s
t nhin n v s ln nht trong cc s a
0
, a
1
, ..., a
n
.
Bi 5.20: Bit rng (2 +x)
100
= a
0
+ a
1
x + + a
100
x
100
. Chng minh rng a
2
< a
3
. Vi gi
tr no th a
k
< a
k1
.
Bi 5.21: Tm h s ca s hng cha a
4
trong khai trin nh thc Newton

a
2

2
a

n
vi
a = 0. Bit rng tng h s ca 3 s hng u tin trong khai trin bng 97.
Bi 5.22: Tm h s ca s hng cha x
26
trong khai trin nh thc Newton ca

1
x
4
+x
7

n
,
bit rng : C
1
2n+1
+C
2
2n+1
+ +C
n
2n+1
= 2
20
1.
Bi 5.23: Tm h s ca x
7
trong khai trin nh thc ca (2 3x)
2n
, trong n l s nguyn
dng tha mn : C
1
2n+1
+C
3
2n+1
+ +C
2n+1
2n+1
= 1024.
Bi 5.24: Tm s hng khng cha x trong khai trin nh thc Newton ca

3x
2

2
3

n
bit
n tha mn iu kin: 2C
0
n
3C
n1
n
+ 4C
2
n
= (n + 4)
2
.
Bi 5.25: Cho khai trin (1 + 2x)
n
= a
0
+a
1
x + +a
n
x
n
.
Trong cc s a
0
, a
1
, , a
n
tha mn h thc: a
0
+
a
1
2
+ +
a
n
2
n
= 4096. Tm s ln nht
trong cc s a
0
, a
1
, , a
n
.
Ngi son : Th.s Minh Tun Trang 86 Khoa T nhin - Trng CSP Nam nh
T
h
.
s

M
i
n
h
T
u

n
Chng 6. Hnh phng ta
Chng 6
Hnh phng ta
6.1 Vc t, im, ng thng
6.1.1 Kin thc c bn
Ta , vc t
~ (a, b) (a

, b

) = (a a

, b b

)
k (a, b) = (ka, kb)
~ (a, b) = (a

, b

a = a

b = b

~ (a, b) . (a

, b

) = a.a

+b.b

[(a, b)[ =

a
2
+b
2
cos (

v ,

v

) =

v .

v

[

v [ . [

v

[
~

AB = (x
B
x
A
, y
B
y
A
)
AB =

AB

(x
B
x
A
)
2
+ (y
B
y
A
)
2
~ M chia AB theo t s k

MA = k.

MB
x
M
=
x
A
kx
B
1 k
, y
M
=
y
A
ky
B
1 k
(k = 1)
c bit nu M l trung im AB ta c:
x
M
=
x
A
+x
B
2
, y
M
=
y
A
+y
B
2
G l trng tm tam gic ABC x
G
=
x
A
+x
B
+x
C
3
, y
G
=
y
A
+y
B
+y
C
3
ng thng
~ (d) qua im M
0
(x
0
, y
0
), c

u
d
= (a, b) hoc

n
d
= (A, B):
Phng trnh tham s (d) :

x = x
0
+a.t
y = y
0
+b.t
Phng trnh chnh tc: (d) :
x x
0
a
=
y y
0
b
Ngi son : Th.s Minh Tun Trang 87 Khoa T nhin - Trng CSP Nam nh
T
h
.
s

M
i
n
h
T
u

n
6.1. Vc t, im, ng thng Chng 6. Hnh phng ta
Phng trnh tng qut: A(x x
0
) +B(y y
0
) = 0.
~ Mi quan h gia vc t php tuyn v vc t ch phng:
Ta s dng quy tc sau chuyn i gia 2 loi vc t: (x, y) (y, x) hoc (y, x)
V d: n = (1; 2) u = (2; 1) hoc u = (2; 1)
~ Phng trnh ng thng i qua 2 im A(x
A
, y
A
), B(x
B
, y
B
) :
x x
A
x
B
x
A
=
y y
A
y
B
y
A
~ Phng trnh on chn: d i qua 2 im A(a; 0), B(0; b) (a, b = 0)
x
a
+
y
b
= 1
~ Gc gia 2 ng thng d
1
v d
2
c thay bng gc gia 2 vc t ch phng hoc 2 vc
t php tuyn:
cos = [cos (

u
1
,

u
2
)[ = [cos (

n
1
,

n
2
)[
=

(d
1
, d
2
)
Ch : Trng hp 2 ng thng khng song song vi Oy v chng khng vung gc vi
nhau th ta c th tnh bng cng thc: tan =
[k
1
k
2
[
1 +k
1
k
2
. k
1
, k
2
tng ng l h s
gc ca 2 ng thng.
~ Khong cch t im M(x
0
, y
0
) n ng thng d : Ax +By +C = 0:
d(M, d) =
[Ax
0
+By
0
+C[

A
2
+B
2
Ch : Ta thng s dng phng trnh tng qut khi phi tnh gc, khong cch. Cn
ta dng phng trnh tham s khi c mi quan h thuc.
6.1.2 Dng bi
Cc bi tp trong thi i hc thng l nhng bi xc nh ta im, vit phng trnh
ng thng tha mn mt s iu kin no .
Ta im
S dng quan h thuc rt bt n. Chng hn M (C), (C) c phng trnh tham s:
(C) : x = f(t), y = g(t) t R
S dng quan h thuc, cng nh cc quan h khc thnh lp phng trnh.
Chng hn M (C) v (C) : y = f(x), nu M(a, b) (C) th b = f(a).
Hoc G l trng tm tam gic ABC, bit ta im A, B, G tm ta C. Ta c th
thnh lp t cng thc x
G
=
x
A
+x
B
+x
C
3
x
C
= 3x
G
x
A
x
B
.
Ngi son : Th.s Minh Tun Trang 88 Khoa T nhin - Trng CSP Nam nh
T
h
.
s

M
i
n
h
T
u

n
6.1. Vc t, im, ng thng Chng 6. Hnh phng ta
V d 6.1.1: Cho tam gic ABC c A(6; 4) , B(4; 1) , C (2; 4)
a) Tm ta trng tm G ca tam gic ABC v trung im M ca BC.
b) Tm ta D sao cho M l trng tm tam gic ABD v im E sao cho D l trung im
EM.
c) Tm ta im F BC sao d(F, AB) = 2d(F, AC).
d) Tm ta im I sao cho t gic ABCI l hnh bnh hnh.
Gii: a) Ta c x
M
=
x
B
+x
C
2
= 1, y
M
=
y
B
+y
C
2
=
5
2
x
G
=
x
A
+x
B
+x
C
3
=
4
3
, y
G
=
y
A
+y
B
+y
C
3
=
1
3
.
M

1;
5
2

v G

4
3
;
1
3

b) Ta c x
M
=
x
A
+x
B
+x
D
3
x
D
= 3x
M
x
A
x
B
= 3 6 + 4 = 5,
y
D
= 3y
M
y
A
y
B
=
15
2
4 + 1 =
21
2
Ta c x
D
=
x
E
+x
M
2
x
E
= 2x
D
x
M
= 2.(5) (1) = 9,
y
E
= 2y
D
y
M
= 2.
21
2
+
5
2
=
37
2
D

5;
21
2

v E

9;
37
2

c) +)

AC = (4; 8) = 4 (1; 2)
AC :


n
AC
= (2; 1)
A(6; 4)
. Nn phng trnh AC l:
2 (x 6) (y 4) = 0 2x y 8 = 0
+)

AB = (10; 5) = 5 (2; 1)
AB :


n
AB
= (1; 2)
A(6; 4)
. Nn phng trnh AB l:
(x 6) 2 (y 4) = 0 x 2y + 2 = 0
+)

BC = (6; 3) = 3 (2; 1)
BC :


u
BC
= (2; 1)
C (2; 4)
. Nn phng trnh tham s BC l:

x = 2 + 2t
y = 4 t
+) F BC F(2 + 2a; 4 a)
d(F, AB) = 2d(F, AC)
[2 + 2a 2 (4 a) + 2[

1
2
+ (2)
2
= 2.
[2 (2 + 2a) (4 a) 8[

2
2
+ (1)
2
Ngi son : Th.s Minh Tun Trang 89 Khoa T nhin - Trng CSP Nam nh
T
h
.
s

M
i
n
h
T
u

n
6.1. Vc t, im, ng thng Chng 6. Hnh phng ta
[4a + 12[ = 2 [5a[

2a + 6 = 5a
2a + 6 = 5a

3a = 6
7a = 6

a = 2
a =
6
7
+) a = 2: F (6; 6)
+) a =
6
7
: F

2
7
;
22
7

d) T gic ABCI l hnh bnh hnh


AB =

IC (10; 5) = (2 x
I
, 4 y
I
)

2 x
I
= 10
4 y
I
= 5

x
I
= 12
y
I
= 1
I (12; 1)
V d 6.1.2 (C 2009): Trong mt phng vi h ta Oxy, cho tam gic ABC c C(1; 2),
ng trung tuyn k t A v ng cao k t B ln lt c phng trnh l 5x +y 9 = 0 v
x + 3y 5 = 0. Tm ta cc nh A v B.
Gii: Gi M l trung im BC, v H l chn ng cao h t nh B xung AC.
A
B C(1; 2) M
H
5
x
+
y

9
=
0
x
+
3
y

8
=
0
+)

n
BH
= (1; 3)

u
BH
= (3; 1)
Do ACBH

n
AC
=

u
BH
= (3; 1)
V AC :

C (1; 2)

n
AC
= (3; 1)
nn phng trnh AC l:
3 (x + 1) (y + 2) = 0 3x y + 1 = 0
V A = AC AM nn ta A l nghim ca h:

5x +y 9 = 0
3x y + 1 = 0

x = 1
y = 4
A(1; 4)
+) V B BH B(5 3b; b) M

4 3b
2
;
b 2
2

Mt khc ta c M AM 5.
4 3b
2
+
b 2
2
9 = 0 20 15b +b 2 18 = 0
14b = 0 b = 0 B(5; 0)
Phng trnh ng thng
Cc dng bi c th
Vit phng trnh ng thng i qua mt im v mt phng (Phng y l phng
vung gc (php tuyn) hoc phng song song (ch phng))
Ngi son : Th.s Minh Tun Trang 90 Khoa T nhin - Trng CSP Nam nh
T
h
.
s

M
i
n
h
T
u

n
6.1. Vc t, im, ng thng Chng 6. Hnh phng ta
Tm 2 im ca ng thng . Vit phng trnh ng thng i qua 2 im. Trng
hp ny c th quy v trng hp trn bng cch : im i qua l mt trong 2 im v
vc t ch phng l vc t ni 2 im.
Ch :
> Cc dng bi khc thng xoay quanh cc dng trn, chng ch khc nhau v cch din
t. V thng thng nu bi khng hi g thm ta thng vit phng trnh dng tng
qut.
> Phng trnh tng qut th tng ng vi vc t php tuyn. Phng trnh tham s,
chnh tc tng ng vi vc t ch phng.
V d 6.1.3: Vit phng trnh ng thng d tha mn mt trong cc iu kin sau:
a) d i qua im A(1; 2) c vc t ch phng

u = (3; 1).
b) d i qua im A(3; 4) v vung gc vi ng thng : : x 4y + 2000 = 0
c) d i qua im A(1; 4) v song song vi ng thng: :
x 1
2
=
2 y
3
d) d i qua giao im ca 2 ng thng
1
:
x + 1
2
=
y 3
3
,
2
:

x = 2 +t
y = 3 3t
v to vi
ng thng
3
: 3x + 4y 10 = 0 mt gc 45
0
.
Gii: a) u = (3; 1)

n = (1; 3)
V d :

A(1; 2)

n = (1; 3)
nn d c phng trnh:
(x 1) + 3 (y + 2) = 0 x + 3y + 5 = 0
b) Ta c

n

= (1; 4)

u

= (4; 1)
V d

n
d
=

u

= (4; 1)
Ta c d :

A(3; 4)

n
d
= (4; 1)
nn phng trnh d l:
4 (x 3) + (y + 4) = 0 4x +y 8 = 0
c) Ta c :
x 1
2
=
2 y
3

x 1
2
=
y 2
3
nn

u

= (2; 3)

n

= (3; 2)
V d |

n
d
=

n

= (3; 2)
T ta c d :

A(1; 4)

n
d
= (3; 2)
nn phng trnh d l:
3 (x 1) + 2 (y 4) = 0 3x + 2y 11 = 0
Ngi son : Th.s Minh Tun Trang 91 Khoa T nhin - Trng CSP Nam nh
T
h
.
s

M
i
n
h
T
u

n
6.1. Vc t, im, ng thng Chng 6. Hnh phng ta
d) +) Ta c M =
1

2
nn ta M l nghim ca h:

x = 2 +t
y = 3 3t
x + 1
2
=
y 3
3

2 +t + 1
2
=
3 3t 3
3
t + 3 = 2t
t = 1 x = 1, y = 6 M (1; 6)
Ta c

n

3
= (3; 4). Gi

n
d
= (A; B)
V

(d,
3
) = 45
0
cos 45
0
=
[

n
d
.

3
[
[

n
d
[ . [

3
[

2
=
[3A+ 4B[

A
2
+B
2
.

3
2
+ 4
2
5.

A
2
+B
2
=

2. [3A+ 4B[
25 (A
2
+B
2
) = 2 (9A
2
+ 24AB + 16B
2
)
7A
2
48AB 7B
2
= 0

A = 7B
A =
1
7
B
+) Vi A = 7B: Chn B = 1 A = 7. Phng trnh d l:
7 (x 1) + (y 6) = 0 7x +y 13 = 0
+) Vi A =
1
7
B: Chn B = 7 A = 1. Phng trnh d l:
(x 1) 7 (y 6) = 0 x 7y + 41 = 0
V d 6.1.4 (H A-2009): Cho hnh ch nht ABCD c im I(6; 2) l giao im ca 2 ng
cho AC v BD. im M(1; 5) thuc ng thng AB v trung im E ca cnh CD thuc
ng thng : x +y 5 = 0. Vit phng trnh ng thng AB.
Gii: Do ABCD l hnh ch nht nn I l trung im AC, BD v AC = BD. Do tam
gic ICD cn ti I, v ng trung tuyn IE ng thi l ng cao IECD.
A B
C D N
M(1; 5)
I(6; 2)
E

:
x
+
y

5
=
0
Gi N l im i xng vi M qua I I l trung im ca 2 ng AC, MN nn t gic
AMCN l hnh bnh hnh
AM | CN, m AM | CD nn C, N, D thng hng.
Do IECD nn IEEN

IE.

EN = 0.
E : x + y 5 = 0 E (a; 5 a)
Do I l trung im ca MN nn x
I
=
x
M
+x
N
2
x
N
= 2x
I
x
M
= 2.6 1 = 11
Ngi son : Th.s Minh Tun Trang 92 Khoa T nhin - Trng CSP Nam nh
T
h
.
s

M
i
n
h
T
u

n
6.2. ng trn Chng 6. Hnh phng ta
y
N
= 2y
I
y
M
= 2.2 5 = 1 N (11; 1)
V

IE.

NE = 0 (a 6; 5 a 2) . (a 11; 5 a + 1) = 0
(a 6) . (a 11) + (3 a) (6 a) = 0
a
2
17a + 66 +a
2
9a + 18 = 0 2a
2
26a + 84 = 0
a
2
13a + 42 = 0

a = 6
a = 7
+) Vi a = 6:

IE = (a 6; 3 a) = (0; 3) = 3 (0; 1)

IECD
AB | CD
ABIE

n
AB
=

u
IE
= (0; 1)
Ta c AB :

M(1; 5)

n
AB
= (0; 1)
nn phng trnh ca AB l:
0. (x 1) + (y 5) = 0 y 5 = 0
+) Vi a = 7:

IE = (1; 4)

n
AB
=

IE = (1; 4)
T ta c AB :

M(1; 5)

n
AB
= (1; 4)
nn phng trnh AB l:
(x 1) 4 (y 5) = 0 x 4y + 19 = 0
6.2 ng trn
6.2.1 Kin thc c bn
Phng trnh
Phng trnh chnh tc ca ng trn tm I(a, b) bn knh R:
(x a)
2
+ (y b)
2
= R
2
Phng trnh tng qut ca ng trn :
x
2
+y
2
+ 2Ax + 2By +C = 0
tm I(A; B) bn knh R =

A
2
+B
2
C
Phng trnh tham s ca ng trn tm I(a, b) bn knh R:

x = a +R. cos t
y = b +R. sin t
(t R)
Phng tch
nh ngha: Cho ng trn (C) : x
2
+y
2
+ 2Ax + 2By +C = 0.
{
M/(C)
=

MA.

MB khng ph thuc vo phng ca ct tuyn MAB ca ng trn m ch


ph thuc vo v tr im M.
C th nu im M(x
0
, y
0
) th {
M/(C)
= x
2
0
+y
2
0
+ 2Ax
0
+ 2By
0
+C.
Ngi son : Th.s Minh Tun Trang 93 Khoa T nhin - Trng CSP Nam nh
T
h
.
s

M
i
n
h
T
u

n
6.2. ng trn Chng 6. Hnh phng ta
ngha: Phng tch ca im M cho bit v tr tng i ca im vi ng trn.
Nu {
M/(C)
< 0 th im M nm bn trong ng trn.
Nu {
M/(C)
= 0 th im M nm trn ng trn.
Nu {
M/(C)
> 0 th im M nm ngoi ng trn.
Trc ng phng: Cho 2 ng trn (C
1
) v (C
2
) khi :
Tp d =

{
M/(C
1
)
= {
M/(C
2
)

l mt ng thng v gi l trc ng phng ca


2 ng trn.
Nu (C
1
) : x
2
+y
2
+ 2A
1
x + 2B
1
y +C
1
= 0 v (C
2
) : x
2
+y
2
+ 2A
2
x + 2B
2
y +C
2
= 0 th
phng trnh trc ng phng l :
{
M/(C
1
)
= {
M/(C
2
)
x
2
+y
2
+ 2A
1
x + 2B
1
y +C
1
= x
2
+y
2
+ 2A
2
x + 2B
2
y +C
2
2 (A
1
A
2
) x + 2 (B
1
B
2
) y +C
1
C
2
= 0
Ch : Khi 2 ng trn ct nhau ti 2 im A, B th AB chnh l trc ng phng
ca 2 ng trn. Nu 2 ng trn tip xc nhau ti im A th trc ng phng ca
2 ng trn chnh l ng tip tuyn chung ca 2 ng trn ti im A.
V tr tng i gia ng thng v ng trn
Gi s ta c ng thng v ng trn (C) tm I bn knh R. K hiu d = d(I, )
V tr Khng ct nhau Tip xc Ct nhau
iu kin d > R d = R d < R
Hnh v
I

Trng hp ng thng l tip tuyn ca ng trn. Ta c th tm tip tuyn nh iu kin


d = R. Trong trng hp ta bit tip im ta c th dng phng trnh tch ta tm tip
tuyn nh sau:
Gi tip im l M(x
0
, y
0
) (C). Ta s dng quy tc sau tm phng trnh tip tuyn:
x
2
x.x x.x
0
v 2x x +x x +x
0
Nu (C) : x
2
+y
2
+ 2Ax + 2By +C = 0 phng trnh tip tuyn l:
x.x
0
+y.y
0
+A(x +x
0
) +B(y +y
0
) +C = 0
Nu (C) : (x a)
2
+ (y b)
2
= R
2
phng trnh tip tuyn l:
(x
0
a)(x a) + (y
0
b)(y b) = R
2
Ngi son : Th.s Minh Tun Trang 94 Khoa T nhin - Trng CSP Nam nh
T
h
.
s

M
i
n
h
T
u

n
6.2. ng trn Chng 6. Hnh phng ta
V tr tng i gia 2 ng trn
Cho 2 ng trn (C
1
) tm I
1
bn knh R
1
v ng trn (C
2
) tm I
1
bn knh R
2
.
K hiu: d = I
1
I
2
gi l di ng ni tm.
Tip xc: C 2 trng hp
V tr Tip xc trong Tip xc ngoi
iu kin d = [R
1
R
2
[ d = R
1
+R
2
Hnh v
I
1
I
2
I
1
I
2
Ct nhau: iu kin [R
1
R
2
[ < d < R
1
+R
2
I
1
I
2
I
1
I
2
Khng giao nhau: C 2 trng hp
V tr ng nhau ngoi nhau
iu kin d < [R
1
R
2
[ d > R
1
+R
2
Hnh v
I
1
I
2
I
1
I
2
Ngi son : Th.s Minh Tun Trang 95 Khoa T nhin - Trng CSP Nam nh
T
h
.
s

M
i
n
h
T
u

n
6.2. ng trn Chng 6. Hnh phng ta
6.2.2 Cc dng bi
Xc nh phng trnh ng trn v ngc li
V d 6.2.1: Tm tm v bn knh ca cc ng trn sau:
a) (C) : (x 1)
2
+ (y + 3)
2
= 5
b) (C) : x
2
+y
2
3x + 2y 1 = 0
Gii: a) Tm I(1; 3) bn knh R =

5.
b) Tm I(
3
2
; 1) bn knh R =

3
2

2
+ 1
2
(1) =

17
2
.
V d 6.2.2: Vit phng trnh ng trn (C), tm tm v bn knh bit:
a) (C) i qua 3 im A(4; 2), B(1; 3), C(3; 1).
b) (C) i qua 2 im A(1; 5), B(0; 2) v tip xc vi ng thng : 2x y + 2 = 0.
c) (C) i qua im A(4; 7) v tip xc vi 2 ng thng
1
: 3x 4y 42 = 0 v ng
thng
2
: y + 8 = 0.
d) (C) tip xc ngoi vi ng trn (C
1
) : x
2
+y
2
+ 4x 2y + 4 = 0 v i qua 2 im A(1; 5),
B(0; 2).
Gii: a) Gi phng trnh ca (C) : x
2
+y
2
+ax +by +c = 0. Do (C) i qua 3 im A, B, C
nn ta c h phng trnh:

4
2
+ 2
2
+ 4a + 2b +c = 0
1
2
+ 3
2
+a + 3b +c = 0
(3)
2
+ 1
2
3a +b +c = 0

4a + 2b +c = 20
a + 3b +c = 10
3a +b +c = 10

a = 2
b = 4
c = 20
T ta c phng trnh ng trn l: x
2
+y
2
2x + 4y 20 = 0
Tm I(1; 2) bn knh R =

1
2
+ (2)
2
(20) = 5
b) Gi phng trnh ng trn (C) : x
2
+y
2
+ 2ax + 2by +c = 0
(C) i qua 2 im A(1; 5), B(0; 2) nn ta c h phng trnh:

(1)
2
+ 5
2
2a + 10b +c = 0
0
2
+ 2
2
+ 4b +c = 0

2a + 10b +c + 26 = 0
c = 4b 4

c = 4b 4
a =
10b +c + 26
2
= 3b + 11
Do (C) tip xc vi nn ta c :
d (I; ) = R
[2a +b + 2[

5
=

a
2
+b
2
c
4a
2
+b
2
+ 4 4ab + 4b 8a = 5 (a
2
+b
2
c)
a
2
+ 4b
2
+ 4ab 4b + 8a 5c 4 = 0
(3b + 11)
2
+ 4b
2
+ 4b (3b + 11) 4b + 8 (3b + 11) 5 (4b 4) 4 = 0
Ngi son : Th.s Minh Tun Trang 96 Khoa T nhin - Trng CSP Nam nh
T
h
.
s

M
i
n
h
T
u

n
6.2. ng trn Chng 6. Hnh phng ta
25b
2
+ 150b + 225 = 0 b
2
+ 6b + 9 = 0 b = 3
a = 2 v c = 8
Vy phng trnh ca (C) : x
2
+y
2
+ 4x 6y + 8 = 0.
c) Gi phng trnh ng trn (C) : x
2
+y
2
+ 2ax + 2by +c = 0
Do im A(4; 7) (C) nn ta c:
4
2
+ (7)
2
+ 8a 14b +c = 0 c = 8a + 14b 65 (1)
Tm v bn knh ca (C): I (a; b) , R =

a
2
+b
2
c
V
1
,
1
tip xc vi (C) nn ta c d (I;
1
) = d (I;
2
) = R

[3a + 4b 42[
5
=
[b + 8[
1
(2)
[b + 8[
1
=

a
2
+b
2
c (3)
(2)

3a + 4b 42 = 5b + 40
3a + 4b 42 = 5b 40

3a + 9b 82 = 0
3a +b = 2

b =
3a + 82
9
b = 3a 2
(3) b
2
16b + 64 = a
2
+b
2
c c = a
2
+ 16b 64 (4)
(1) , (4) a
2
+ 16b 64 = 8a + 14b 65 a
2
+ 8a + 2b + 1 = 0 (5)
+) Vi b =
3a + 82
9
thay vo (5) ta c:
a
2
+
26
3
a +
173
9
= 0

V nghim

+) Vi b = 3a 2 thay vo (5) ta c:
a
2
+ 2a 3 = 0

a = 1 b = 5 c = 143
a = 3 b = 7 c = 57
V ta c 2 ng trn tha mn l:
(C
1
) : x
2
+y
2
+ 2x 10y 143 = 0 vi I
1
(1; 5), R
1
= 13
(C
2
) : x
2
+y
2
6x + 14y + 57 = 0 vi I
2
(3; 7), R
2
= 1.
d) Gi phng trnh ng trn (C) : x
2
+y
2
+ 2ax + 2by +c = 0
Do (C) i qua 2 im A, B nn ta c:

1
2
+ 5
2
+ 2a + 10b +c = 0
(2)
2
4b +c = 0

c = 4b 4
a =
10b c 26
2
= 7b 11
Tm v bn knh ca (C
1
) l: I
1
(2; 1) , R
1
=

(2)
2
+ 1
2
4 = 1
Tm v bn knh ca (C) l: I (a; b) , R =

a
2
+b
2
c
V 2 ng trn tip xc ngoi nn : II
1
= R
1
+R

(a 2)
2
+ (b + 1)
2
= 1+

a
2
+b
2
c
a
2
+b
2
4a + 2b + 5 = a
2
+b
2
c + 1 + 2

a
2
+b
2
c
4a + 2b +c + 4 = 2

a
2
+b
2
c
Thay a, c gii c trn vo ta c:
Ngi son : Th.s Minh Tun Trang 97 Khoa T nhin - Trng CSP Nam nh
T
h
.
s

M
i
n
h
T
u

n
6.2. ng trn Chng 6. Hnh phng ta
4 (7b 11) + 2b + 4b 4 = 2

(7b 11)
2
+b
2
(4b 4)
34b + 44 = 2

50b
2
+ 150b + 125

(17b + 22)
2
= 50b
2
+ 150b + 125
b
22
17

239b
2
+ 598b + 359 = 0
b
22
17

b = 1

Tha mn

a = 4 c = 8
b =
359
239
(Loi)
Vy phng trnh ca (C) l: x
2
+y
2
8x 2y 8 = 0
Tm I (4; 1), bn knh R = 5.
V tr tng i ca ng thng v ng trn
V d 6.2.3: Cho ng thng d : 2x+y 4 = 0 v ng trn (C) : x
2
+y
2
2x2y +1 = 0.
a) Chng minh d ct (C) ti 2 im phn bit A, B.
b) Vit phng trnh ng trn i qua 2 im A, B c bn knh R = 5.
c) Vit phng trnh ng trn i qua 2 im A, B c tm thuc ng thng : 3x4y2 =
0.
Gii: a) Cch 1: ng trn (C) c tm I(1; 1) bn knh R = 1.
d (I, d) =
[2.1 + 1 4[

2
2
+ 1
=
1

5
< 1 = R
Vy d ct (C) ti 2 im phn bit.
Cch 2: Ta giao im d v (C) l nghim ca h

2x +y 4 = 0 (1)
x
2
+y
2
2x 2y + 1 = 0 (2)
T (1) ta c y = 4 2x th vo (2) ta c :
x
2
+ (4 2x)
2
2x 2 (4 2x) + 1 = 0 5x
2
14x + 9 = 0

x = 1 y = 2
x =
9
5
y =
2
5
A(1; 2) , B

9
5
;
2
5

Vy d ct (C) ti 2 im phn bit A, B.


b) Do (C
1
) i qua giao im ca (C) v (d) nn phng trnh ca n c dng:
x
2
+y
2
2x 2y + 1 +m(2x +y 4) = 0
x
2
+y
2
+ (2m2) x + (m2) y + 1 4m = 0
I

1 m;
2 m
2

, R =

(1 m)
2
+

2 m
2

2
(1 4m) =

5m
2
+ 4m+ 4
2
Theo gi thit : R = 5

5m
2
+ 4m + 4
2
= 5
5m
2
+ 4m+ 4 = 100 5m
2
+ 4m96 = 0

m = 4
m =
24
5
Ngi son : Th.s Minh Tun Trang 98 Khoa T nhin - Trng CSP Nam nh
T
h
.
s

M
i
n
h
T
u

n
6.2. ng trn Chng 6. Hnh phng ta
+) Vi m = 4: Phng trnh (C
1
) l
x
2
+y
2
2x 2y + 5 = 0
+) Vi m =
24
5
: Phng trnh (C
1
) l
x
2
+y
2
2x 2y
19
5
= 0
c) Do (C
2
) i qua giao im ca (C) v (d) nn phng trnh ca n c dng:
x
2
+y
2
2x 2y + 1 +m(2x +y 4) = 0
x
2
+y
2
+ (2m2) x + (m2) y + 1 4m = 0
I

1 m;
2 m
2

Do im I nn ta c:
3 (1 m) 4

2 m
2

2 = 0 3 m = 0 m = 3
Thay vo ta c phng trnh ca (C
2
) l:
x
2
+y
2
8x 5y + 13 = 0
V d 6.2.4: Cho ng trn (C) : x
2
+y
2
2x + 4y 4 = 0 v ng thng
d : 4x 3y 11 = 0.
a) Tm tm v bn knh ca ng trn.
b) Vit phng trnh tip tuyn vi (C) ti im M
0

4
5
;
2
5

.
c) Vit phng trnh tip tuyn vi (C) song song vi ng thng d.
d) Vit phng trnh tip tuyn vi (C) vung gc vi ng thng d. Tm ta tip im khi
.
e) Vit phng trnh tip tuyn vi (C) i qua im A(4; 1).
f) Gi T
1
, T
2
l tip im ca 2 tip tuyn k t im B(2; 3) vi (C). Vit phng trnh ng
thng T
1
T
2
.
Gii: a) Tm I(1; 2), bn knh R = 3.
b) Phng trnh tip tuyn vi (C) ti im M
0

4
5
;
2
5

l:
x.

4
5

+y.
2
5

x +

4
5

+ 2

y +
2
5

4 = 0

9
5
x +
12
5
y
12
5
= 0 3x 4y + 4 = 0
Ngi son : Th.s Minh Tun Trang 99 Khoa T nhin - Trng CSP Nam nh
T
h
.
s

M
i
n
h
T
u

n
6.2. ng trn Chng 6. Hnh phng ta
c) Ta c | d : 4x 3y +m = 0
Do l tip tuyn ca (C) nn ta c d (I, ) = R
[4.1 3. (2) +m[

4
2
+ (3)
2
= 3
[m+ 10[ = 15

m = 5
m = 25
Thay vo phng trnh ta c 2 ng thng tha mn l:

1
: 4x 3y + 5 = 0

2
: 4x 3y 25 = 0
d) Ta c

n
d
= (4; 3)

u
d
= (3; 4)
V d

n

=

u
d
= (3; 4)
T phng trnh c dng 3x + 4y +m = 0.
Do l tip tuyn ca (C) nn ta c d (I, ) = R
[3.1 + 4. (2) +m[

3
2
+ 4
2
= 3
[m5[ = 15

m = 20
m = 10
Thay vo phng trnh ta c 2 ng thng tha mn l:

1
: 3x + 4y + 20 = 0

2
: 3x + 4y 10 = 0
e) Gi : Ax +By +C = 0
Do A nn ta c 4A+ 3B +C = 0 C = 4A3B
Do l tip tuyn ca (C) nn ta c d (I, ) = R
[A2B +C[

A
2
+B
2
= 3
[A2B 4A3B[ = 3

A
2
+B
2
[3A+ 5B[ = 3

A
2
+B
2
9A
2
+ 30AB + 25B
2
= 9A
2
+ 9B
2
30AB + 16B
2
= 0

B = 0
B =
15
8
A
+) Vi B = 0: Chn A = 1 C = 4
1
: y 4 = 0
Gi T
1
(x
1
; y
1
) l tip im ca
1
vi (C). T ta c phng trnh
1
l:
x
1
.x +y
1
.y (x +x
1
) + 2 (y +y
1
) 4 = 0 (x
1
1) x + (y
1
+ 2) y x
1
+ 2y
1
4 = 0

x
1
1
0
=
y
1
+ 2
1
=
x
1
+ 2y
1
4
4

x
1
= 1
4y
1
8 = x
1
+ 2y
1
4

x
1
= 1
y
1
=
1
2
T
1

1;
1
2

+) Vi B =
15
8
A: Chn A = 8 B = 15 C = 13
2
: 8x 15y + 13 = 0
Gi T
2
(x
2
; y
2
) l ta tip im ca
2
vi (C) ta c phng trnh
2
l
Ngi son : Th.s Minh Tun Trang 100 Khoa T nhin - Trng CSP Nam nh
T
h
.
s

M
i
n
h
T
u

n
6.2. ng trn Chng 6. Hnh phng ta
(x
2
1) x + (y
2
+ 2) y x
2
+ 2y
2
4 = 0

x
2
1
8
=
y
2
+ 2
15
=
x
2
+ 2y
2
4
13

15x
2
+ 8y
2
= 1
15x
2
43y
2
= 34

x
2
=
7
17
y
2
=
11
17
T
2

7
17
;
11
17

f) Gi T
1
(x
1
; y
1
), T
2
(x
2
; y
2
)
BT
1
l tip tuyn vi (C) ti T
1
nn ta c phng trnh BT
1
l:
x
1
.x +y
1
.y (x +x
1
) + 2 (y +y
1
) 4 = 0
Do B(2; 3) BT
1
2x
1
+ 3y
1
2 x
1
+ 6 + 2y
1
4 = 0 x
1
+ 5y
1
= 0
Tng t vi im T
2
ta cng c h thc : x
2
+ 5y
2
= 0
Do T
1
, T
2
cng thuc ng thng : x + 5y = 0. Nn phng trnh T
1
T
2
: x + 5y = 0 .
V tr tng i ca ng trn v ng trn
V d 6.2.5: Cho 2 ng trn : (C
1
) : x
2
+y
2
+ 6x 16y + 9 = 0 v
(C
2
) : x
2
+y
2
+ 14x 30y + 18 = 0.
a) Tm tm v bn knh ca 2 ng trn. Chng minh 2 ng trn ct nhau ti 2 im phn
bit A, B. Vit phng trnh ng thng AB.
b) Vit phng trnh ng trn i qua 2 im A, B v c tm nm trn ng thng :
3x + 2y 8 = 0.
c) Vit phng trnh ng tip tuyn chung ca 2 ng trn.
Gii: a) I
1
(3; 8) , R
1
=

(3)
2
+ 8
2
9 = 8
I
2
(7; 15) , R
2
=

(7)
2
+ 15
2
18 = 16

I
1
I
2
= (4; 7) I
1
I
2
=

(4)
2
+ 7
2
=

65
Nh vy R
2
R
1
= 8 < I
1
I
2
=

65 < R
1
+ R
2
= 24 nn 2 ng trn ct nhau ti 2 im
phn bit. Phng trnh AB chnh l phng trnh trc ng phng ca 2 ng trn. Do
phng trnh AB c dng:
x
2
+y
2
+ 6x 16y + 9 = x
2
+y
2
+ 14x 30y + 18 8x 14y + 9 = 0
b) Phng trnh ng trn (C) i qua 2 im A, B c dng:
m(x
2
+y
2
+ 6x 16y + 9) +n(x
2
+y
2
+ 14y 30y + 18) = 0
(m +n) x
2
+ (m +n) y
2
+ (6m+ 14n) x (16m+ 30n) y + 9m+ 18n = 0
x
2
+y
2
+ 2.
3m+ 7n
m +n
x 2.
8m+ 15n
m +n
y +
9m+ 18n
m +n
= 0
I

3m + 7n
m +n
;
8m + 15n
m +n

V I : 3x + 2y 8 = 0 9m21n + 16m+ 30n 8m8n = 0


Ngi son : Th.s Minh Tun Trang 101 Khoa T nhin - Trng CSP Nam nh
T
h
.
s

M
i
n
h
T
u

n
6.2. ng trn Chng 6. Hnh phng ta
m +n = 0 m = n
Chn n = 1 m = 1. Phng trnh ng trn (C) l:
(x
2
+y
2
+ 6x 16y + 9) + (x
2
+y
2
+ 14x 30y + 18) = 0
2x
2
+ 2y
2
+ 20x 36y + 27 = 0 x
2
+y
2
+ 10x 18y +
27
2
= 0
c) Gi : Ax +By +C = 0 l tip tuyn chung ca 2 ng trn. Do ta c h:

d (I
1
, ) = R
1
d (I
2
, ) = R
2

[3A + 8B +C[

A
2
+B
2
= 8
[7A + 15B +C[

A
2
+B
2
= 16

[3A + 8B +C[
8
=

A
2
+B
2
[7A + 15B +C[
16
=

A
2
+B
2
2 [3A + 8B +C[ = [7A + 15B +C[

6A + 16B + 2C = 7A+ 15B +C


6A + 16B + 2C = 7A15B C

C = A B
C =
13A31B
3
~ C = AB:
[3A + 8B A B[ = 8

A
2
+B
2
[4A + 7B[ = 8

A
2
+B
2
16A
2
56AB + 49B
2
= 64A
2
+ 64B
2
48A
2
+ 56AB + 15B
2
= 0

A =
5B
12
A =
3
4
B
+) Vi A =
5B
12
:
Chn B = 12 A = 5 C = 7 : 5x 12y + 7 = 0
+) Vi A =
3
4
B:
Chn B = 4 A = 3 C = 1 : 3x 4y + 1 = 0
~ C =
13A31B
3

3A+ 8B +
13A31B
3

= 8

A
2
+B
2
[4A7B[ = 24

A
2
+B
2
16A
2
56AB + 49B
2
= 576A
2
+ 576B
2
560A
2
+ 56AB + 527B
2
= 0

V nghim

Ch : Trng hp ny phng trnh vn c nghim A = B = 0. Nhng trong thc t


chng c vc t php tuyn no l vc t

0 . Nn trng hp ny khng tn ti ng
thng cn tm.
Ngi son : Th.s Minh Tun Trang 102 Khoa T nhin - Trng CSP Nam nh
T
h
.
s

M
i
n
h
T
u

n
6.3. Ba ng Conic Chng 6. Hnh phng ta
6.3 Ba ng Conic
6.3.1 Kin thc chung v 3 ng Conic
Elip Hyperbol Parabol
nh ngha
MF
1
+ MF
2
= 2a > 2c =
F
1
F
2
[MF
1
MF
2
[ = 2a <
2c = F
1
F
2
MF = d(M, )
th
x
y
O a
a
b
b
(E) :
x
2
a
2
+
y
2
b
2
= 1
F
1
F
2

a
e
a
e
x
=

a e
x
=
a e
x
y
O
F
1
(c; 0) F
2
(c; 0)
b
b
a
a
(H) :
x
2
a
2

y
2
b
2
= 1
x
=

a e
x
=
a e
x
y
O

:
x
=

p 2
(P) : y
2
= 2px
F

p
2
; 0

Mi quan h a
2
= b
2
+c
2
c
2
= a
2
+b
2
p = d(F, )
Tiu im F
1
(c; 0), F
2
(c; 0) F
1
(c; 0), F
2
(c; 0) F(
p
2
; 0)
Tm sai e =
c
a
< 1 e =
c
a
> 1 e = 1
ng
chun
x =
a
e
x =
a
e
x =
p
2
Tim cn
x
a

y
b
= 0
Bn knh
qua tiu

MF
1
= a +e.x
M
MF
2
= a e.x
M

MF
1
= [a +e.x
M
[
MF
2
= [a e.x
M
[
MF = x
M
+
p
2
Cc yu t
Trc ln : 2a
Trc b: 2b
Tiu c: 2c
Trc thc : 2a
Trc o: 2b
Tiu c: 2c
Tip tuyn
ti im
M
0
(x
0
, y
0
)
x
0
.x
a
2
+
y
0
.y
b
2
= 1
x
0
.x
a
2

y
0
.y
b
2
= 1 y
0
.y = p(x +x
0
)
Tip tuyn
Ax + By +
C = 0
a
2
A
2
+b
2
B
2
= C
2
a
2
A
2
b
2
B
2
= C
2
pB
2
= 2AC
6.3.2 Elip
Xc nh Elip
V d 6.3.1: Xc nh di trc ln, trc b, tiu c, tm sai, ta tiu im, phng trnh
cc ng chun ca Elip.
a) 4x
2
+ 9y
2
36 = 0.
b) 9x
2
+ 4y
2
= 5.
Ngi son : Th.s Minh Tun Trang 103 Khoa T nhin - Trng CSP Nam nh
T
h
.
s

M
i
n
h
T
u

n
6.3. Ba ng Conic Chng 6. Hnh phng ta
Gii: a) Ta c 4x
2
+ 9y
2
36 = 0
x
2
9
+
y
2
4
= 1
a
2
= 9, b
2
= 4 c
2
= a
2
b
2
= 5 a = 3, b = 2, c =

5
+) Tm sai: e =
c
a
=

5
3
+) di trc ln: 2a = 6
+) di trc b: 2b = 4
+) Tiu c: 2c = 2

5
+) Tiu im: F
1

5; 0

, F
2

5; 0

+) ng chun: x =
a
e
x =
3

5/3
x =
9

5
b) Ta c 9x
2
+ 4y
2
= 5
x
2
5/9
+
y
2
5/4
= 1.
V
5
9
<
5
4
nn y khng phi l phng trnh chnh tc.
a
2
=
5
4
, b
2
=
5
9
c
2
= a
2
b
2
=
5
4

5
9
=
25
36
a =

5
2
, b =

5
3
, c =
5
6
+) Tm sai: e =
c
a
=

5
3
+) di trc ln: 2a =

5
+) di trc b: 2b =
2

5
3
+) Tiu c: 2c =
5
3
+) Tiu im: F
1

5
6
; 0

, F
2

5
6
; 0

+) ng chun: x =
a
e
x =

5/2

5/3
x =
3
2
V d 6.3.2: Vit phng trnh chnh tc ca Elip (E) bit:
a) Tiu im F
1
(6; 0), tm sai e =
2
3
.
b) di trc ln l 6, tiu c 2

5.
c) di trc ln l 3, tm sai e =

3
3
.
d) (A-2008) Tm sai e =

5
3
v hnh ch nht c s c chu vi bng 20.
e) i qua 2 im A

4;

, B

2; 3

f) i qua im A

2;
5
3

v tm sai e =
2
3
.
Ngi son : Th.s Minh Tun Trang 104 Khoa T nhin - Trng CSP Nam nh
T
h
.
s

M
i
n
h
T
u

n
6.3. Ba ng Conic Chng 6. Hnh phng ta
g) Phng trnh ng chun l 3x 8

3 = 0 v di trc b bng 4.
Gii: a) Do F
1
(6; 0) c = 6
e =
c
a
a =
c
e
=
6
2/3
= 9 b =

a
2
c
2
=

9
2
6
2
= 3

5
Phng trnh Elip l:
x
2
81
+
y
2
45
= 1
b) Theo gi thit ta c:
2a = 6 a = 3,
2c = 2

5 c =

5 b =

a
2
c
2
=

3
2

2
= 2
Phng trnh Elip l:
x
2
9
+
y
2
4
= 1
c) Theo gi thit ta c:
2a = 3 a =
3
2
e =
c
a
c = e.a =

3
3
.
3
2
=

3
2
b =

a
2
c
2
=

3
2

3
2

2
=

3
2
Phng trnh Elip l:
x
2
9/4
+
y
2
3/2
= 1
d) T gi thit ta c:

e =
c
a
=

5
3
4 (a +b) = 20

c =

5
3
a
b = 5 a
V a
2
= b
2
+c
2
a
2
= (5 a)
2
+

5
3
a

2
a
2
= 25 10a +a
2
+
5
9
a
2
a
2
18a + 45 = 0

a = 3 b = 2
a = 15 b = 10 (Loi)
Phng trnh Elip l:
x
2
9
+
y
2
4
= 1
e) Gi phng trnh Elip l:
x
2
a
2
+
y
2
b
2
= 1
V 2 im A

4;

, B

2; 3

(E) nn ta c h :

16
a
2
+
3
b
2
= 1
8
a
2
+
9
b
2
= 1

1
a
2
=
1
20
1
b
2
=
1
15

a
2
= 20
b
2
= 15
T ta c phng trnh ca Elip l:
x
2
20
+
y
2
15
= 1
Ngi son : Th.s Minh Tun Trang 105 Khoa T nhin - Trng CSP Nam nh
T
h
.
s

M
i
n
h
T
u

n
6.3. Ba ng Conic Chng 6. Hnh phng ta
f) Gi phng trnh Elip l:
x
2
a
2
+
y
2
b
2
= 1
Do im A

2;
5
3

(E) nn ta c :
4
a
2
+
25
9b
2
= 1
e =
2
3

c
a
=
2
3
c
2
=
4
9
a
2
b
2
= a
2
c
2
=
5
9
a
2
9b
2
= 5a
2
4
a
2
+
25
5a
2
= 1
9
a
2
= 1 a
2
= 9 b
2
=
5
9
a
2
= 5
Vy phng trnh Elip l :
x
2
9
+
y
2
5
= 1
g) Theo gi thit ta c: 2b = 4 b = 2
3x 8

3 = 0 x =
8

a
e
=
8

3
a
2
=
8

3
c 3a
4
= 64c
2
c
2
= a
2
b
2
= a
2
4 3a
4
= 64 (a
2
4) 3a
4
64a
2
+ 256 = 0

a
2
= 16
a
2
=
16
3
+) Vi a
2
= 16: Phng trnh Elip l
x
2
16
+
y
2
4
= 1
+) Vi a
2
=
16
3
: Phng trnh Elp l
x
2
16/3
+
y
2
4
= 1
Cng thc bn knh qua tiu
V d 6.3.3: Cho Elip (E) :
x
2
100
+
y
2
36
= 1
a) Tm trn (E) nhng im M sao cho MF
1
= 4MF
2
.
b) Tm trn (E) nhng im N sao cho

F
1
NF
2
= 90
0
.
c) Tm trn (E) nhng im K sao cho

F
1
NF
2
= 60
0
.
d) P l mt im ty trn (E). Chng minh rng: PF
1
.PF
2
+OP
2
= const
Gii: a) a
2
= 100, b
2
= 36 a = 10, b = 6 c = 8 e =
4
5
Theo gi thit ta c:
MF
1
= 4MF
2
a +e.x
M
= 4 (a e.x
M
) 5e.x
M
= 3a x
M
=
3a
5e
=
15
2
y
M
=
6
10

100 x
2
M
=
6
10
.

100

15
2

2
=
3

7
2
Ta c 2 im tha mn : M
1

15
2
;
3

7
2

, M
2

15
2
;
3

7
2

Ngi son : Th.s Minh Tun Trang 106 Khoa T nhin - Trng CSP Nam nh
T
h
.
s

M
i
n
h
T
u

n
6.3. Ba ng Conic Chng 6. Hnh phng ta
b) Theo gi thit ta c :

F
1
NF
2
= 90
0
F
1
F
2
2
= NF
2
1
+NF
2
2
4c
2
= (a +e.x
N
)
2
+ (a e.x
N
)
2
4c
2
= 2a
2
+ 2e
2
.x
2
N
x
2
N
=
2c
2
a
2
e
2
=
2c
2
a
2
c
2
.a
2
y
2
N
=
b
2
a
2
. (a
2
x
2
N
) =
b
2
a
2
.

a
2

2c
2
a
2
c
2
.a
2

=
b
2
(a
2
c
2
)
c
2
=
b
4
c
2
x
N
=
5

7
2
, y
N
=
9
2
Ta c 4 im tha mn :
M
1

7
2
;
9
2

, M
2

7
2
;
9
2

, M
3

7
2
;
9
2

, M
4

7
2
;
9
2

c)

F
1
KF
2
= 60
0
F
1
F
2
2
= KF
2
1
+KF
2
2
2KF
1
.KF
2
. cos 60
0
4c
2
= (a +e.x
K
)
2
+ (a e.x
K
)
2
2 (a +e.x
K
) . (a e.x
K
) .
1
2
4c
2
= 2a
2
+ 2e
2
.x
2
K
a
2
+e
2
.x
2
K
3e
2
.x
2
K
= 4c
2
a
2
x
2
K
=
4c
2
a
2
3c
2
.a
2
y
2
K
=
b
2
a
2
. (a
2
x
2
K
) =
b
2
a
2
.

a
2

4c
2
a
2
3c
2
.a
2

=
b
2
(a
2
c
2
)
3c
2
=
b
4
3c
2
x
K
=
5

13
2
, y
K
=
3

3
16
Ta c 4 im K tha mn:
K
1

13
2
;
3

3
16

, K
2

13
2
;
3

3
16

, K
3

13
2
;
3

3
16

, K
4

13
2
;
3

3
16

d) PF
1
.PF
2
+OP
2
= (a +e.x
P
) (a e.x
P
) + (x
2
P
+y
2
P
) = a
2
e
2
.x
2
P
+x
2
P
+
b
2
a
2
(a
2
x
2
P
)
= a
2
+b
2
+
a
2
b
2
c
2
a
2
.x
2
P
= a
2
+b
2
= 136 = const
Tng giao ca ng thng v Elip
V d 6.3.4: Cho Elip (E) : 4x
2
+ 9y
2
= 36
a) Cho im M(

5; y
0
) (E) (y
0
> 0). Vit phng trnh tip tuyn ti im M.
b) Tm m ng thng : mx 2y + 5 = 0 tip xc vi Elip. Tm ta tip im.
c) Vit phng trnh tip tuyn vi (E) bit tip tuyn vung gc vi ng thng 4x3y
1 = 0.
d) Vit phng trnh tip tuyn qua im I(3; 1) vi (E).
e) Qua im S(2; 3) k 2 tip tuyn ST
1
, ST
2
vi (E) vi T
1
, T
2
l tip im. Vit phng
trnh ng thng T
1
T
2
.
Ngi son : Th.s Minh Tun Trang 107 Khoa T nhin - Trng CSP Nam nh
T
h
.
s

M
i
n
h
T
u

n
6.3. Ba ng Conic Chng 6. Hnh phng ta
Gii: a) (E) : 4x
2
+ 9y
2
= 36
x
2
9
+
y
2
4
= 1
a
2
= 9, b
2
= 4 c
2
= 5 a = 3, b = 2, c =

5
9y
2
0
= 36 4.5 = 16 y
0
=
4
3
(y
0
> 0) M

5;
4
3

Phng trnh tip tuyn ti M l: 4.

5.x + 9.
4
3
.y = 36 4

5x + 12y 36 = 0
b) a
2
A
2
+b
2
B
2
= C
2
9.m
2
+ 4. (2)
2
= 5
2
9m
2
= 9 m = 1
+) Trng hp m = 1 :
: x 2y + 5 = 0
Gi T
1
(x
1
, y
1
) l tip im. Do phng trnh l:
4x
1
.x + 9y
1
.y 36 = 0

4x
1
1
=
9y
1
2
=
36
5

x
1
=
9
5
y
1
=
8
5
T
1

9
5
;
8
5

+) Trng hp m = 1:
: x + 2y 5 = 0
Gi T
2
(x
2
, y
2
) l tip im. Do phng trnh l:
4x
2
.x + 9y
2
.y 36 = 0

4x
2
1
=
9y
2
2
=
36
5

x
2
=
9
5
y
2
=
8
5
T
2

9
5
;
8
5

c) a : 4x 3y 1 = 0

n
a
= (4; 3)

u
a
= (3; 4)
da

n
d
=

u
a
= (3; 4) d : 3x + 4y +m = 0
Do d l tip tuyn ca (E) nn 3
2
.9 + 4
2
.4 = m
2
m =

145
Vy phng trnh ca d : 3x + 4y

145 = 0
d) Gi d : Ax +By +C = 0 l ng thng cn tm.
Do I d nn ta c 3A +B +C = 0 C = 3AB
a
2
A
2
+b
2
B
2
= C
2
9A
2
+ 4B
2
= C
2
9A
2
+ 4B
2
= (3AB)
2
9A
2
+ 4B
2
= 9A
2
6AB +B
2
3B
2
+ 6AB = 0 3B(B + 2A) = 0

B = 0
B = 2A
+) Vi B = 0: Chn A = 1 C = 3 d : x + 3 = 0
+) Vi B = 2A: Chn A = 1 B = 2 C = 5
d : x 2y + 5 = 0
Ngi son : Th.s Minh Tun Trang 108 Khoa T nhin - Trng CSP Nam nh
T
h
.
s

M
i
n
h
T
u

n
6.3. Ba ng Conic Chng 6. Hnh phng ta
e) Gi T
1
(x
1
; y
1
) , T
2
(x
2
; y
2
) l ta tip im.
Phng trnh ST
1
l : 4.x
1
.x + 9.y
1
.y 36 = 0
Do S ST
1
8x
1
+ 27y
1
36 = 0
Hon ton tng t ta cng c ng thc: 8x
2
+ 27y
2
36 = 0
Do phng trnh T
1
T
2
l: 8x + 27y 36 = 0
6.3.3 Hyperbol
Xc nh Hyperbol
V d 6.3.5: Tm di cc trc, tiu c, tiu im, tm sai, phng trnh ng tim cn,
phng trnh ng chun ca (H):
a) 9x
2
4y
2
= 36
b) 16x
2
9y
2
= 144
Gii: a) 9x
2
4y
2
= 36
x
2
4

y
2
9
= 1
a
2
= 4, b
2
= 9 a = 2, b = 3 c =

a
2
+b
2
=

13
+) di trc thc : 2a = 4
+) di trc o : 2b = 6
+) Tiu c : 2c = 2

13
+) Tm sai : e =
c
a
=

13
2
+) Tiu im : F
1

13; 0

, F
2

13; 0

+) Phng trnh ng tim cn : 9x


2
4y
2
= 0 3x 2y = 0
+) ng chun : x =
a
e
x =
a
2
c
= x =
4

13
b) 16x
2
9y
2
= 144
y
2
16

x
2
9
= 1
a
2
= 16, b
2
= 9 a = 4, b = 3 c =

a
2
+b
2
= 5
+) di trc thc : 2a = 8
+) di trc o : 2b = 6
+) Tiu c : 2c = 10
+) Tm sai : e =
c
a
=
5
4
+) Tiu im : F
1
(0; 5) , F
2
(0; 5)
+) Phng trnh ng tim cn : 16x
2
9y
2
= 0 4x 3y = 0
+) ng chun : y =
a
e
y =
a
2
c
= y =
16
5
V d 6.3.6: Lp phng trnh chnh tc ca Hyperbol bit:
Ngi son : Th.s Minh Tun Trang 109 Khoa T nhin - Trng CSP Nam nh
T
h
.
s

M
i
n
h
T
u

n
6.3. Ba ng Conic Chng 6. Hnh phng ta
a) C cng tiu im vi Elip (E) :
x
2
35
+
y
2
10
= 1 v i qua im A(4

2; 3).
b) nh A
1
(24; 0), tm sai e =
13
12
.
c) Trc o c di bng 6; tiu c bng 10.
d) Tm sai e =

2 v (H) i qua im M(5; 3).


e) Gc gia 2 tim cn bng 60
0
v (H) i qua M(6; 3).
f) Khong cch gia 2 nh bng 6 v (H) i qua im A(6; 2

3).
g) (H) i qua 2 im A(4;

6), B(

6; 1).
Gii: a) +) Xt (E) :
x
2
35
+
y
2
10
= 1
a
2
= 35, b
2
= 10 c
2
= a
2
b
2
= 25 c = 5 +) Xt Hyperbol:
x
2
a
2

y
2
b
2
= 1
V (H) c chung tiu im vi Elip (E) nn ta c c = 5 b
2
= c
2
a
2
= 25 a
2
V im A (H) nn
32
a
2

9
b
2
= 1
32
a
2

9
25 a
2
= 1 800 32a
2
9a
2
= 25a
2
a
4
a
4
66a
2
+ 800 = 0

a
2
= 50 > c
2
(Loi)
a
2
= 16
a
2
= 16 b
2
= 9 (H) :
x
2
16

y
2
9
= 1
b) A
1
(24; 0) a = 24
e =
c
a
=
13
12
c =
13
12
a = 26 b =

c
2
a
2
= 10
(H) :
x
2
576

y
2
100
= 1
c) +) di trc o bng 6 2b = 6 b = 3
+) Tiu c bng 10 2c = 10 c = 5 a =

c
2
b
2
= 4
Phng trnh Hyperbol (H) :
x
2
16

y
2
9
= 1
d) e =
c
a
=

2 c = a

2 c
2
= a
2
+b
2
= 2a
2
b
2
= a
2
Do (H) i qua im M nn ta c:
25
a
2

9
b
2
= 1
25
a
2

9
a
2
= 1 a
2
= 16 b
2
= 16
Phng trnh Hyperbol l: (H) :
x
2
16

y
2
16
= 1
Ngi son : Th.s Minh Tun Trang 110 Khoa T nhin - Trng CSP Nam nh
T
h
.
s

M
i
n
h
T
u

n
6.3. Ba ng Conic Chng 6. Hnh phng ta
e) V M (H) nn ta c
36
a
2

9
b
2
= 1
+) Hai ng tim cn :
1
: bx +ay = 0,
2
: bx ay = 0


n
1
= (b; a) ,

n
2
= (b; a)
Theo gi thit ta c: cos 60
0
=
[b.b +a. (a)[

b
2
+a
2
.

b
2
+ (a)
2

[b
2
a
2
[
a
2
+b
2
=
1
2

a
2
+b
2
= 2 (b
2
a
2
)
a
2
+b
2
= 2 (b
2
a
2
)

3a
2
= b
2
a
2
= 3b
2
+) Vi 3a
2
= b
2
:

36
a
2

9
b
2
= 1
36
a
2

9
3a
2
= 1 a
2
= 33 b
2
= 99
Phng trnh Hyperbol: (H) :
x
2
33

y
2
99
= 1
+) Vi a
2
= 3b
2
:

36
a
2

9
b
2
= 1
36
3b
2

9
b
2
= 1 b
2
= 3 a
2
= 9
Phng trnh ca Hyperbol : (H) :
x
2
9

y
2
3
= 1
f) 2a = 6 a = 3
im A (H) nn ta c
36
a
2

12
b
2
= 1 4
12
b
2
= 1 b
2
= 4
(H) :
x
2
9

y
2
4
= 1
g) T gi thit ta c :

16
a
2

6
b
2
= 1
6
a
2

1
b
2
= 1

1
a
2
=
1
4
1
b
2
=
1
2

a
2
= 4
b
2
= 2
(H) :
x
2
4

y
2
2
= 1
Cng thc bn knh qua tiu
V d 6.3.7: Cho Hyperbol (H) : 9x
2
16y
2
= 144.
a) Cho M(5; y
0
) (H). Tnh MF
1
, MF
2
.
b) Tm im M (H) sao cho MF
1
= 2MF
2
.
c) Tm im N (H) sao cho

F
1
NF
2
= 60
0
.
Ngi son : Th.s Minh Tun Trang 111 Khoa T nhin - Trng CSP Nam nh
T
h
.
s

M
i
n
h
T
u

n
6.3. Ba ng Conic Chng 6. Hnh phng ta
Gii: a) Phng trnh (H):
x
2
16

y
2
9
= 1
a
2
= 16, b
2
= 9 c
2
= 25 a = 4, b = 3, c = 5, e =
5
4
x
M
< 0

MF
1
= a e.x
M
MF
2
= a e.x
M

MF
1
= 4
5
4
. (5) =
9
4
MF
2
= 4
5
4
. (5) =
41
4
b) MF
1
= 2MF
2
x
M
> 0

MF
1
= a +e.x
M
MF
2
= a +e.x
M
MF
1
= 2MF a +e.x
M
= 2 (a +e.x
M
) e.x
M
= 3a x
M
=
3a
e
= 3.4.
4
5
=
48
5
y
2
M
=
9
16
(x
2
M
16) =
9
16

48
5

2
16

=
1071
25
y
M
=
3

119
5
C 2 im M tha mn : M
1

48
5
;
3

119
5

, M
2

48
5
;
3

119
5

c) F
1
N = [a +e.x
N
[ , F
2
N = [a e.x
N
[ , F
1
N.F
2
N = (a e.x
N
) (a e.x
N
) = e
2
x
2
N
a
2

F
1
NF
2
= 60
0
F
1
F
2
2
= NF
2
1
+NF
2
2
2NF
1
.NF
2
. cos 60
0
4c
2
= [a +e.x
N
[
2
+[a e.x
N
[
2
2 (e
2
x
2
N
a
2
) .
1
2
4c
2
= 2a
2
+ 2e
2
.x
2
N
e
2
.x
2
M
+a
2
e
2
.x
2
N
= 4c
2
a
2
x
2
N
=
4c
2
a
2
c
2
.a
2
y
2
N
=
b
2
a
2
(x
2
N
a
2
) =
b
2
a
2

4c
2
a
2
c
2
.a
2
a
2

= b
2
.
3c
2
a
2
c
2
x
2
N
=
4.25 16
25
.16 =
1344
25
x
N
=
8

21
5
y
2
N
= 9.
3.25 16
25
=
531
25
y
N
=
3

59
5
N
1

21
5
;
3

59
5

, N
2

21
5
;
3

59
5

, N
3

21
5
;
3

59
5

, N
4

21
5
;
3

59
5

Tng giao ca ng thng v Hyperbol


V d 6.3.8: Cho Hyperbol (H) : 9x
2
16y
2
= 144.
a) Vit phng trnh tip tuyn ca (H) bit n vung gc vi : x + 3y 1 = 0.
b) Vit phng trnh tip tuyn ca (H) ti im M(5; y
0
) (y
0
> 0).
c) Vit phng trnh tip tuyn ca (H) i qua im A

2;
3
2

.
Ngi son : Th.s Minh Tun Trang 112 Khoa T nhin - Trng CSP Nam nh
T
h
.
s

M
i
n
h
T
u

n
6.3. Ba ng Conic Chng 6. Hnh phng ta
d) Qua im P(1; 1) k 2 tip tuyn PT
1
, PT
2
ti (H), T
1
v T
2
l tip im. Vit phng
trnh ng thng T
1
T
2
.
Gii: a) Gi d l ng thng cn tm.
: x + 3y 1 = 0

n

= (1; 3)

u

= (3; 1)
d

n
d
=

u

= (3; 1) d : 3x y +m = 0
d tip xc vi (H) nn : 3
2
.16 (1)
2
.9 = m
2
m
2
= 135 m = 3

15
d : 3x y 3

15 = 0
b) y
2
0
=
9x
2
0
144
16
=
9.25 144
16
=
81
16
y
0
=
9
4
y
0
=
9
4
Phng trnh tip tuyn ti M l:
9.5.x 16.
9
4
.y 144 = 0 45x 36y 144 = 0 5x 4y 16 = 0
c) Gi d : Ax +By +C = 0 l ng thng cn tm.
A d 2A+
3
2
B +C = 0 C = 2A
3
2
B
Do d l tip tuyn ca (H) nn ta c:
C
2
= 16A
2
9B
2

2A
3
2
B

2
= 16A
2
9B
2
4A
2
+ 6AB +
9
4
B
2
= 16A
2
9B
2
12A
2
6AB
45
4
B
2
= 0
16A
2
8AB 15B
2
= 0

A =
5
4
B
A =
3
4
B
+) Vi A =
5
4
B:
Chn B = 4 A = 5 C = 16 d : 5x + 4y 16 = 0
+) Vi A =
3
4
B:
Chn B = 4 A = 3 C = 0 d : 3x 4y = 0
d) Gi T
1
(x
1
, y
1
) , T
2
(x
2
, y
2
)
Do PT
1
l tip tuyn ti im T
1
ca (H) nn phng trnh PT
1
l:
PT
1
: 9.x
1
.x 16.y
1
.y 144 = 0
V P PT
1
9x
1
16y
1
144 = 0 9x
1
+ 16y
1
+ 144 = 0
Lp lun tng t ta cng c ng thc : 9x
2
+ 16y
2
+ 144 = 0
Phng trnh T
1
T
2
: 9x + 16y + 144 = 0
Ngi son : Th.s Minh Tun Trang 113 Khoa T nhin - Trng CSP Nam nh
T
h
.
s

M
i
n
h
T
u

n
6.3. Ba ng Conic Chng 6. Hnh phng ta
6.3.4 Parabol
Xc nh Parabol
V d 6.3.9: Tm ta tiu im, phng trnh ng chun ca cc Parabol sau:
a) y
2
= 4x.
b) y
2
= 8x.
c) x
2
= 8y.
d) x
2
= 4y
Gii: a) 2p = 4 p = 2 F (1; 0). ng chun : x = 1
b) 2p = 8 p = 4 F (2; 0). ng chun x = 2.
c) 2p = 8 p = 4 F (0; 2). ng chun y = 2.
d) 2p = 4 p = 2 F (0; 1). ng chun y = 1.
V d 6.3.10: Lp phng trnh ca Parabol (P) bit
a) Tiu im F(2; 0), trc tung l ng chun.
b) Tiu im F(0; 5), ng chun : y + 2 = 0.
c) (P) c trc i xng l Ox, qua gc O v qua im M(1; 3).
d) (P) c trc i xng Oy, qua gc O v qua im M(5; 1).
Gii: a) Oy : x = 0. Nn ta c :
M (x; y) (P) MF = d (M, )

(x 2)
2
+y
2
= [x[
x
2
4x + 4 +y
2
= x
2
y
2
= 4x 4
b) M (x; y) (P) MF = d (M, )

x
2
+ (y 5)
2
= [y + 2[
x
2
+y
2
10y + 25 = y
2
+ 4y + 4 x
2
= 14y 21
c) (P) c trc i xng l Ox, qua gc O nn ta c phng trnh (P) c dng y
2
= ax.
(P) qua im M(1; 3) nn ta c 9 = a (P) : y
2
= 9x (p =
9
2
)
d) (P) c trc i xng Oy, qua gc O nn phng trnh (P) c dng x
2
= ay. Do (P) i qua
im M(5; 1) nn 25 = a a = 25.
Do phng trnh (P) : x
2
= 25y (p =
25
2
).
Ngi son : Th.s Minh Tun Trang 114 Khoa T nhin - Trng CSP Nam nh
T
h
.
s

M
i
n
h
T
u

n
6.3. Ba ng Conic Chng 6. Hnh phng ta
S tng giao gia ng thng v Parabol
V d 6.3.11: Cho Parabol y
2
= 8x.
a) Tm cc im trn (P) cch tiu im F mt on bng 20.
b) Qua F dng mt dy cung bt k ct (P) ti 2 im phn bit A, B. Tm qu tch trung
im I ca on AB. Chng minh ng trn ng knh AB lun tip xc vi ng chun.
c) Vit phng trnh tip tuyn ca (P) ti im M(2; 4).
d) Vit phng trnh tip tuyn vung gc vi (P) bit tip tuyn vung gc vi ng thng
y = x. Tm ta tip im khi .
e) Vit phng trnh tip tuyn vi (P) qua im A(0; 1). Tnh gc to bi hai tip tuyn.
f) Qua im S(3; 1) k 2 tip tuyn ST
1
, ST
2
vi (P). T
1
, T
2
l cc tip im. Vit phng
trnh ng thng T
1
T
2
.
Gii: a) y
2
= 8x 2p = 8 p = 4
MF = x
M
+
p
2
= 20 x
M
+ 2 = 20 x
M
= 18 y
2
M
= 8.18 y
M
= 12
Ta c 2 im tha mn l: M
1
(18; 12) , M
2
(18; 12)
b) F (2; 0). Do ng thng qua F ct parabol ti 2 im phn bit A, B do ng thng
ny khng song song vi Ox nn phng trnh AB : x = ky + 2.
Ta A, B l nghim ca h:

y
2
= 8x
x = ky + 2
y
2
= 8 (ky + 2) y
2
8ky 16 = 0
Phng trnh c a.c < 0 nn lun c 2 nghim phn bit y
A
, y
B
theo nh l Viet ta c:

y
A
+y
B
= 8k
y
A
.y
B
= 16
y
I
=
y
A
+y
B
2
= 4k
Do I l trung im AB I AB x
I
= ky
I
+ 2 = 4k
2
+ 2

x
I
= 4k
2
+ 2
y
I
= 4k

k =
y
I
4
x
I
= 4k
2
+ 2
x
I
= 4.

y
I
4

2
+ 2 y
2
I
= 4 (x
I
2)
I (P

) : y
2
= 4 (x 2)
+) Gi R l bn knh ca ng trn ng knh AB.
Ta c : AB = FA+FB = x
A
+ 2 +x
B
+ 2 = 2x
I
+ 4 = 2 (4k
2
+ 2) + 4 = 8 (k
2
+ 1)
R =
AB
2
= 4 (k
2
+ 1)
ng chun : x + 2 = 0
d (I, ) = [x
I
+ 2[ = [4k
2
+ 2 + 2[ = 4 (k
2
+ 1) = R
iu ny dn n ng trn ng knh AB tip xc vi ng chun.
c) Phng trnh tip tuyn ti M l: 4.y = 4 (x + 2) x +y + 2 = 0
Ngi son : Th.s Minh Tun Trang 115 Khoa T nhin - Trng CSP Nam nh
T
h
.
s

M
i
n
h
T
u

n
6.4. Bi tp Chng 6. Hnh phng ta
d) Gi d l ng thng cn tm. k l h s gc ca d. Theo bi ra ta c:
k.k

= 1 k. (1) = 1 k = 1 d : y = x +m x y +m =
V d l tip tuyn ca (P) nn ta c:
p.B
2
= 2AC 4. (1)
2
= 2.1.m m = 2 d : x y + 2 = 0
Gi M
0
(x
0
, y
0
) l ta tip im. Do phng trnh MM
0
l:
y
0
.y = 4 (x +x
0
) 4x y
0
.y + 4x
0
= 0
V MM
0
d
4
1
=
y
0
1
=
4x
0
2

x
0
= 2
y
0
= 4
M
0
(2; 4)
e) Gi d : Ax +By +C = 0 l ng thng cn tm. V im A d B +C = 0 C = B
Do d tip xc vi (P) nn ta c:
pB
2
= 2AC 4B
2
= 2AB

B = 0
B =
1
2
A
+) Vi B = 0: Chn A = 1 C = 0 d
1
: x = 0
+) Vi B =
1
2
A: Chn A = 2 B = 1 C = 1 d
2
: 2x y + 1 = 0
Gi l gc gia d
1
v d
2
. Ta c :

n
1
= (1; 0) ,

n
2
= (2; 1)
cos =
[

n
1
.

n
2
[
[

n
1
[ . [

n
2
[
=
[1.2 + 0. (1)[

1
2
+ 0
2
.

2
2
+ (1)
2
=
2

5
26
0
33

54

f) Gi T
1
(x
1
, y
1
) , T
2
(x
2
, y
2
) l cc tip im. Phng trnh ST
1
l : y
1
.y = 4 (x +x
1
)
V S ST
1
y
1
.1 = 4 (3 +x
1
) 4x
1
y
1
12 = 0
Lp lun tng t vi im T
2
ta cng c ng thc: 4x
2
y
2
12 = 0
Do phng trnh T
1
T
2
: 4x y 12 = 0
6.4 Bi tp
ng thng, im
Bi 6.1: Cho tam gic ABC c A(4; 1), B(1; 7), C(1; 0). Vit phng trnh tng qut ca:
a) ng cao AH v ng thng BC.
b) Trung tuyn AM v trung trc AB.
Bi 6.2: Cho tam gic ABC c phng trnh cc ng thng AB, BC, CA l:
AB : x 3 = 0, BC : 4x 7y + 23 = 0, 3x + 7y + 5 = 0.
a) Tm ta 3 nh A, B, C v din tch tam gic.
b) Tm ta im A

i xng vi A qua BC.


c) Tm ta trc tm, trng tm ca tam gic.
Ngi son : Th.s Minh Tun Trang 116 Khoa T nhin - Trng CSP Nam nh
T
h
.
s

M
i
n
h
T
u

n
6.4. Bi tp Chng 6. Hnh phng ta
Bi 6.3: Cho 2 im A(5; 2), B(3; 4). Vit phng trnh ng thng d qua im C(1; 1) v
cch u 2 im A, B.
Bi 6.4: Vit phng trnh tng qut ca ng thng d tha mn iu kin:
a) i qua im A(1; 2) v c h s gc bng 3.
b) Qua im B(5; 2) v vung gc vi ng thng 2x 5y + 4 = 0.
c) Qua gc O v vung gc vi ng thng y =
2 3x
4
.
d) Qua im I(4; 5) v hp vi 2 trc ta mt tam gic vung cn.
e) Qua im A(3; 5) v cch im H(1; 2) xa nht.
Bi 6.5: Cho tam gic ABC c phng trnh cc cnh BC : 2x y 4 = 0, ng cao
BH : x + y 2 = 0, ng cao CK : x + 3y + 5 = 0. Vit phng trnh cc cnh cn li ca
tam gic.
Bi 6.6: Cho hnh ch nht ABCD c phng trnh cnh AB : 2x y 1 = 0, AD qua im
M(3; 1) v tm I(1;
1
2
). Vit phng trnh cc cnh AD, BC, CD.
Bi 6.7 (*): Cho tam gic ABC c trung im M ca AB c ta (
1
2
; 0), ng cao CH
vi H(1; 1), ng cao BK vi K(1; 3) v bit B c honh dng.
a) Vit phng trnh cnh AB.
b) Tm ta A, B, C.
Bi 6.8: Cho tam gic ABC vi B(1; 2) v C(4; 2) v din tch tam gic bng 10.
a) Vit phng trnh ng thng BC v tnh di ng cao AH.
b) Tm ta im A bit A thuc trc tung.
Bi 6.9: Cho tam gic ABC c phng trnh cc cnh AB : 2x+y 3 = 0, AC : 2xy +7 = 0,
BC : x y = 0.
a) Tnh bn knh ng trn ngoi tip tam gic ABC.
b) Vit phng trnh ng thng i xng vi AB qua BC.
Bi 6.10: Cho hnh vung c tm I(2; 3), phng trnh AB : 3x + 4y 4 = 0.
a) Tnh cnh hnh vung.
b) Tm phng trnh cc cnh CD, AD, BC.
Bi 6.11: Cho hnh vung ABCD c phng trnh cc cnh l AB : 3x 2y 1 = 0, CD :
3x 2y + 5 = 0 v c tm thuc ng thng d : x +y 1 = 0.
a) Tm ta I.
b) Vit phng trnh ng thng AD, BC.
Ngi son : Th.s Minh Tun Trang 117 Khoa T nhin - Trng CSP Nam nh
T
h
.
s

M
i
n
h
T
u

n
6.4. Bi tp Chng 6. Hnh phng ta
Bi 6.12: Trong mt phng Oxy cho tam gic ABC c A(2; 3), B(3; 2), din tch tam gic
bng
3
2
v trng tm G thuc ng thng d : 3x y 8 = 0. Tm ta nh C
Bi 6.13: Cho ng thng d : x + 2y 4 = 0 v 2 im A(1; 4), B(6; 4)
a) Chng minh A, B nm cng pha vi ng thng d. Tm ta im A

i xng vi A qua
d.
b) Tm im M d sao cho d(M, AB) =

2

ng trn
Bi 6.14: Xc nh tm v bn knh ca cc ng trn sau:
a) (x + 1)
2
+ (y 4)
2
= 1.
b) (x + 2)
2
+y
2
= 5.
c) x
2
+y
2
+ 8x 4y 5 = 0.
d) 3x
2
+ 3y
2
+ 4x + 1 = 0.
e) (2x + 5)
2
+ (2y 3)
2
= 4.
Bi 6.15: Vit phng trnh ng trn:
a) ng knh AB vi A(3; 1), B(2; 2).
b) C tm I(1; 2) v tip xc vi ng thng : x +y 2 = 0.
c) C bn knh 5, tm thuc Ox v qua im A(2; 4).
d) C tm I(2; 1) v tip xc ngoi vi ng trn : (x 5)
2
+ (y 3)
2
= 9.
e) Tip xc vi 2 trc v c tm nm trn ng thng : 2x y 3 = 0.
Bi 6.16: Vit phng trnh ng trn :
a) Qua 3 im A(2; 1), B(1; 4), C(4; 3).
b) Qua A(0; 2), B(1; 1) v c tm nm trn ng thng 2x + 3y = 0.
c) Qua im A(5; 3) v tip xc vi ng thng d : x + 3y + 2 = 0 ti im T(1; 1).
Bi 6.17: a) Vit phng trnh tip tuyn vi ng trn: (x 3)
2
+ (y + 1)
2
= 25 ti im
nm trn ng trn c honh bng 1.
b) Vit phng trnh tip tuyn vi ng trn (C): x
2
+y
2
+4x2y 5 = 0 ti giao im ca
ng trn vi trc Ox.
Bi 6.18: a) Vit phng trnh tip tuyn vi ng trn x
2
+y
2
= 2 bit tip tuyn c h s
gc bng 1.
Ngi son : Th.s Minh Tun Trang 118 Khoa T nhin - Trng CSP Nam nh
T
h
.
s

M
i
n
h
T
u

n
6.4. Bi tp Chng 6. Hnh phng ta
b) Vit phng trnh tip tuyn vi ng trn (C) : x
2
+ (y 1)
2
= 25 bit tip tuyn vung
gc vi ng thng 3x 4y + 7 = 0.
Bi 6.19: Cho ng trn (C) : x
2
+y
2
2x 4y 5 = 0.
a) Vit phng trnh tip tuyn ca (C) vung gc vi ng thng 3x +y = 0.
b) Vit phng trnh tip tuyn vi (C) i qua im A(3; 2). Gi T
1
, T
2
l cc tip im. Vit
phng trnh ng thng T
1
T
2
v vit phng trnh ng trn ngoi tip tam gic AT
1
T
2
.
Bi 6.20: Cho 2 ng trn (C
1
) : x
2
+y
2
2x 2y 2 = 0, (C
2
) : x
2
+y
2
8x 4y +16 = 0.
a) Chng minh 2 ng trn bng nhau v ct nhau.
b) Vit phng trnh ng thng qua giao im ca 2 ng trn.
c) Tm phng trnh tip tuyn chung ca chng.
d) Vit phng trnh ng trn i qua giao im (C
1
), (C
2
) trn v c tm nm trn ng
thng x + 2y 4 = 0.
Bi 6.21: Lp phng trnh ng trn :
a) Qua im A(1; 2) v tip xc 2 trc ta .
b) Tip xc hai ng thng song song
1
: 2x y 3 = 0 v
2
: 2x y + 5 = 0 v c tm
nm trn Oy.
c) Tip xc vi ng thng : 2x +y 5 = 0 ti im T(2; 1) v c bn knh bng 2

5.
d) Tip xc vi 2 ng thng x 2y + 5 = 0 v x + 2y + 1 = 0 v qua gc O.
Bi 6.22: Cho 2 ng trn : (C
1
) : x
2
+y
2
2x4y +1 = 0 v (C
2
) : x
2
+y
2
+4x+4y 1 = 0.
a) Chng minh 2 ng trn tip xc ngoi. Tm ta tip im T.
b) Vit phng trnh tip tuyn chung ti T.

Elip
Bi 6.23: Xc nh di cc trc, tiu c, tiu im, tm sai, phng trnh ng chun ca
cc Elip sau:
a)
x
2
12
+
y
2
9
= 1.
b) 9x
2
+ 4y
2
36 = 0.
Bi 6.24: Tm trn (E) :
x
2
4
+y
2
= 1:
a) im M c tung
1
2
.
b) im N c tung gp i honh .
Ngi son : Th.s Minh Tun Trang 119 Khoa T nhin - Trng CSP Nam nh
T
h
.
s

M
i
n
h
T
u

n
6.4. Bi tp Chng 6. Hnh phng ta
c) im P sao cho 2PF
1
= 3PF
2
.
d) im Q sao cho

F
1
QF
2
= 120
0
.
Bi 6.25: Lp phng trnh (E) bit:
a) di trc ln bng 8 v qua im (2

2; 2).
b) Qua 2 im P

2;
1
3

, Q

2;

5
3

c) C tiu c l 4 v qua im

1;
2

.
d) Qua im M

5
;
4

v

F
1
MF
2
= 90
0
.
e) di trc nh l 4 v mt tiu im c ta (2; 0).
Bi 6.26: Cho 2 Elip (E
1
) : x
2
+ 8y
2
= 16 v (E
2
) : 4x
2
+ 9y
2
= 36. Vit phng trnh ng
trn qua cc giao im ca 2 Elip.
Bi 6.27: Cho Elip (E) :
x
2
25
+
y
2
16
= 1.
a) Vit phng trnh tip tuyn ti im c honh x = 9 v c tung m.
b) Vit phng trnh tip tuyn qua im A(2; 5).
c) Vit phng trnh tip tuyn vung gc vi ng thng 5x 3y + 10 = 0.
d) Gi s qua im S(1; 4) k cc tip tuyn SM, SN. Vit phng trnh ng thng MN.

Hyperbol
Bi 6.28: Xc nh di cc trc, tiu c, tiu im, tm sai, phng trnh ng chun,
phng trnh tim cn ca cc Hyperbol sau:
a)
x
2
4

y
2
5
= 1.
b)
x
2
16

y
2
9
= 1.
Bi 6.29: Cho Hyperbol (H) : x
2

y
2
4
= 1. Tm trn (H):
a) im M c honh 2.
b) im N cch u 2 trc ta .
c) im P sao cho

F
1
PF
2
= 90
0
.
d) im Q sao cho F
2
Q = 2F
1
Q.
Ngi son : Th.s Minh Tun Trang 120 Khoa T nhin - Trng CSP Nam nh
T
h
.
s

M
i
n
h
T
u

n
6.4. Bi tp Chng 6. Hnh phng ta
Bi 6.30: Lp phng trnh chnh tc ca Hyperbol bit:
a) Tiu c c di l 8 v khong cch t nh trn trc thc n tiu im l 1.
b) di trc o l 4 v mt tiu im l (3; 0).
c) Mt tiu im l F
2
(5; 0) v mt tim cn l y = 2x.
d) Mt tim cn l y =

3x v qua im 3;

15
e) Mt tiu im l (2; 0) v qua im (3;

2).
f) Qua 2 im P(

10; 2), Q

5
2
; 1

.
g) Tiu c c di l 4

2 v i qua im (3;

5)
Bi 6.31: Cho Hyperbol (H) : 9x
2
4y
2
= 36.
a) M l mt im ty trn (H). Chng minh rng : (F
1
M +F
2
M)
2
4OM
2
= const
b) Vit phng trnh tip tuyn ti im thuc nhnh tri Hyperbol v c tung y = 1.
c) d : 2x y +m = 0. Tm m d l tip tuyn ca (H).
d) Vit phng trnh tip tuyn ca (H) qua im A

1;
3
2

.
e) Qua S(1; 2) k 2 tip tuyn SP, SQ vi (H). Vit phng trnh ng thng PQ.

Parabol
Bi 6.32: Tm tiu im, ng chun v v Parabol cho bi cc phng trnh sau:
a) y
2
= 5x.
b) x
2
= 6y.
c) y
2
= 4x + 2.
Bi 6.33: Lp phng trnh ca (P) bit:
a) Tiu im l F(3; 0) v ng chun x = 5.
b) Tiu im l F(0; 2) v ng chun l trc Ox.
Bi 6.34: Cho Parabol (P) : y
2
= 4x. Gi d l ng thng i qua tiu im F ct (P) ti 2
im phn bit M, N
a) Tm ta M, N bit FM = 4FN. Vit phng trnh tip tuyn ti M, N vi (P).
b) Gi I l trung im ca MN. Tm qu tch im I.
c) Vit phng trnh tip tuyn ca (P) i qua im A(2; 1).
Ngi son : Th.s Minh Tun Trang 121 Khoa T nhin - Trng CSP Nam nh
T
h
.
s

M
i
n
h
T
u

n
6.4. Bi tp Chng 6. Hnh phng ta
d) Vit phng trnh tip tuyn ca (P) vung gc vi ng thng : 2x + 3y 4 = 0.
e) Gi s qua im S(3; 2) ta k c 2 tip tuyn SC, SD vi (P). Vit phng trnh CD.

Bi tp tng hp
Cc em tham kho trong thi d b v thi chnh thc t nm 2002 n nm 2009.
A. Phng trnh ng thng
Bi 6.35: Lp phng trnh tng qut, tham s, chnh tc ca ng thng (d) bit:
a) i qua im M(1; 2) v c vc t php tuyn

n = (3; 2).
b) i qua im M(3; 1) v c vc t ch phng

u = (4; 1).
c) i qua 2 im A(1; 4), B(2; 1).
d) (d) l trung trc ca on thng AB vi A

1
2
; 1

v B(2; 1).
e) i qua im M(7; 3) v c h s gc k =
2
3
.
Bi 6.36: Chuyn (d) v dng tham s v chnh tc bit (d) c phng trnh tng qut:
a) 2x 3y = 0.
b) 2x 3 = 0.
c) 3x 4y + 5 = 0.
Bi 6.37: Chuyn (d) v dng tng qut bit (d) c phng trnh tham s:
a)

x = 2
y = 3 +t
b)

x = 2 t
y = 5 + 3t
c)

x = 4 + 2t
y = 5t 1
Bi 6.38: Trong cc im A
1
(2; 1), A
2
(1; 2), A
3
(1; 3), A
4
(1; 1), A
5

1
2
; 2

, A
6

7
3
;
1
3

, A
7
(3; 1),
im no nm trn ng thng (d) :

x = 2 t
y = 1 + 2t
.
Bi 6.39: Cho cc im A(2; 1), B(3; 5), C(1; 2).
a) Chng minh rng A, B, C l 3 nh ca mt tam gic.
b) Lp phng trnh cc ng cao ca tam gic ABC.
c) Lp phng trnh cc cnh ca tam gic ABC.
d) Lp phng trnh cc ng trung tuyn ca tam gic ABC.
Ngi son : Th.s Minh Tun Trang 122 Khoa T nhin - Trng CSP Nam nh
T
h
.
s

M
i
n
h
T
u

n
6.4. Bi tp Chng 6. Hnh phng ta
e) Lp phng trnh cc ng trung trc ca tam gic ABC.
Bi 6.40: Cho tam gic ABC c A(1; 2), B(4; 3), C(2; 3).
a) Lp phng trnh ng trung trc ca AB.
b) Lp phng trnh ca ng thng i qua im M(3; 7) v vung gc vi ng trung tuyn
k t A ca tam gic ABC.
Bi 6.41: Lp phng trnh cc cnh v ng trung trc ca tam gic ABC bit trung im
3 cnh BC, CA, AB ln lt l M(2; 3), N(4; 1), P(3; 5).
Bi 6.42: Cho tam gic ABC bit A(3; 3), B(2; 1), C(11; 2). Lp phng trnh ng thng
d i qua A v chia tam gic ABC thnh 2 phn c t s din tch bng 2.
B. ng thng song song, vung gc vi mt ng thng.
Bi 6.43: Lp phng trnh tng qut ca ng thng () i qua A v song song vi ng
thng (d):
a) A(1; 3), d : x y + 1 = 0.
b) A(2; 5), d trc Ox.
c) A(1; 1), d :

x = 1 t
y = 2 + 2t
d) A(3; 5), (d) :
x 2
2
=
y + 3
5
.
Bi 6.44: Lp phng trnh tng qut, tham s ca ng thng () i qua A vung gc vi
ng thng d bit:
a) A(3; 3), (d) : 2x 5y + 1 = 0.
b) A(4; 2), (d) Oy.
c) A(1; 6), (d) :

x = 1 +t
y = 2 + 2t
d) A(2; 5), (d) :
x 3
2
=
y + 1
1
.
Bi 6.45: Lp phng trnh cc cnh ca tam gic ABC bit A(2; 2) v 2 ng cao (d
1
), (d
2
)
c phng trnh l (d
1
) : x +y 2 = 0, (d
2
) : 9x 3y + 4 = 0.
Bi 6.46: Cho tam gic ABC bit phng trnh cnh AB l x +y 9 = 0, cc ng cao qua
nh A v B ln lt l (d
1
) : x + y 1 = 0 v (d
2
) : 7x + 5y 49 = 0. Lp phng trnh cc
cnh AC, BC v ng cao th 3 ca tam gic ABC.
Bi 6.47: Lp phng trnh cc cnh ca tam gic ABC bit A(3; 5), ng cao v ng trung
tuyn k t mt nh c phng trnh ln lt l (d
1
) : 5x + 4y 1 = 0, (d
2
) : 8x +y 7 = 0.
Bi 6.48: Lp phng trnh cc cnh ca tam gic ABC bit A(3; 1) v 2 ng trung tuyn
c phng trnh (d
1
) : 2x y 1 = 0, (d
2
) : x 1 = 0.
Ngi son : Th.s Minh Tun Trang 123 Khoa T nhin - Trng CSP Nam nh
T
h
.
s

M
i
n
h
T
u

n
6.4. Bi tp Chng 6. Hnh phng ta
Bi 6.49: Phng trnh 2 cnh ca tam gic ABC l (d
1
) : x +y 2 = 0, (d
2
) : x + 2y 5 = 0
v trc tm tam gic H(2; 3). Lp phng trnh cnh th 3 ca tam gic .
Bi 6.50: Lp phng trnh cc cnh ca tam gic ABC bit B(2; 3), phng trnh ng
cao h t A v ng trung tuyn t C ln lt l (d
1
) : 3x 2y + 3 = 0, (d
2
) : 7x +y 2 = 0.
Bi 6.51: Xc nh ta cc nh v lp phng trnh cnh BC ca tam gic ABC bit trung
im ca BC l M(2; 3), phng trnh cc cnh AB, AC ln lt l xy 1 = 0 v 2x+y = 0.
Bi 6.52: Xc nh ta cc nh v lp phng trnh cnh BC ca tam gic ABC bit
trng tm tam gic G

4
3
;
2
3

v phng trnh cc cnh AB, AC ln lt l: x 3y + 13 = 0,


12x + y 29 = 0.
Bi 6.53: Lp phng trnh cc cnh ca tam gic ABC bit trung im AB l M(3; 4), 2
ng cao k t A, B ln lt l (d
1
) : 2x 5y + 29 = 0, (d
2
) : 10x 3y + 5 = 0.
C. Hnh chiu vung gc ca im ln ng thng.
Bi 6.54: Tm ta hnh chiu vung gc H ca M ln ng thng (d) v xc nh ta
im M

i xng vi M qua d.
a) M(6; 4), d : 4x 5y + 3 = 0.
b) M(1; 4), d : 3x + 4y 4 = 0.
c) M(3; 5), d :

x = 1 2t
y = 3 + 4t
Bi 6.55: Tm ta trc tm H ca tam gic ABC v xc nh ta im K i xng vi
H qua BC bit A(0; 3), B(3; 0), C(1; 1).
Bi 6.56: Lp phng trnh ng thng () i xng vi ng thng d qua im I bit:
a) I(3; 1), d : 2x +y 3 = 0.
b) I(1; 1), d : 3x 2y + 1 = 0.
c) I(1; 3), d :

x = 2 t
1 2t
d) I(0; 2), d :

x = 3 +t
y = 5 4t
Bi 6.57: Lp phng trnh ng thng d
1
i xng vi ng thng d qua ng thng
bit:
a) d : x + 2y 1 = 0, : 2x y + 3 = 0.
b) d : 2x + 3y + 5 = 0, : 5x y + 4 = 0.
c) d : 5x +y 6 = 0, :
x + 1
2
=
y 3
3
.
Ngi son : Th.s Minh Tun Trang 124 Khoa T nhin - Trng CSP Nam nh
T
h
.
s

M
i
n
h
T
u

n
6.4. Bi tp Chng 6. Hnh phng ta
d) d : 2x +y + 3 = 0, :

x = 1 + 2t
x = 3 +t
.
Bi 6.58: Lp phng trnh cc cnh ca tam gic ABC bit A(0; 3), phng trnh 2 ng
phn gic trong xut pht t B v C ln lt l d
B
: x y = 0, d
C
: x + 2y 8 = 0.
Bi 6.59: Lp phng trnh cc cnh ca tam gic ABC bit A(4; 3), B(9; 2) v phng trnh
ng phn gic trong xut pht t C l d; x y + 3 = 0.
Bi 6.60: Cho tam gic ABC bit phng trnh cnh BC : x + 4y 8 = 0 v phng trnh 2
ng phn gic trong xut pht t B v C ln lt l d
B
: y = 0, d
C
: 5x + 3y 6 = 0.
Bi 6.61: Cho tam gic ABC bit C(3; 3), phng trnh ng cao v ng phn gic trong
xut pht t A ln lt l d
1
: x = 2, d
2
: 3x + 8y 14 = 0.
Bi 6.62: Lp phng trnh cc cnh ca tam gic ABC bit C(5; 2), phng trnh ng
trung tuyn t A l d
1
: 5x +y 7 = 0 v ng phn gic trong gc B l d
2
: x y + 3 = 0.
D. V tr tng i ca 2 ng thng.
Bi 6.63: Xt v tr tng i ca cc cp ng thng sau:
a) d
1
:

x = 1 t
y = 2 +t
, d
2
:

x = 2 u
, y = 5 +u
b) d
1
:

x = 1 +t
y = 3 t
, d
2
:
x 3
2
=
y 2
1
.
c) d
1
:

x = 2 + 3t
y = 1 +t
, d
2
: 2x 3y + 1 = 0.
d) d
1
: 3x + 2y 1 = 0, d
2
: x + 3y 4 = 0.
Bi 6.64: Cho a
2
+b
2
= 0 v 2 ng thng d
1
v d
2
c phng trnh:
d
1
: (a b)x +y = 1, d
2
: (a
2
b
2
)x +ay = b
a) Tm quan h gia a, b d
1
, d
2
ct nhau. Khi hy xc nh ta giao im I ca chng.
b) Tm iu kin gia a, b I thuc trc honh.
Bi 6.65: Cho 2 ng thng d
1
: kx y +k = 0, d
2
: (1 k
2
)x + 2ky 1 k
2
= 0.
a) Chng minh rng d
1
lun i qua mt im c nh vi mi k.
b) Chng minh rng d
1
lun ct d
2
. Xc nh ta giao im ca chng.
E. Gc v khong cch
Bi 6.66: Tm gc gia cc cp ng thng sau:
a) d
1
: 5x + 3y 4 = 0, d
2
: x + 2y + 2 = 0.
b) d
1
: 3x 4y 14 = 0, d
2
:
x 1
2
=
y + 3
1
.
Ngi son : Th.s Minh Tun Trang 125 Khoa T nhin - Trng CSP Nam nh
T
h
.
s

M
i
n
h
T
u

n
6.4. Bi tp Chng 6. Hnh phng ta
c) d
1
:

x = 1 3t
y = 2 +t
, d
2
: 3x + 2y 2 = 0.
d) d
1
: x +my 1 = 0, d
2
: x y + 2m1 = 0.
Bi 6.67: Tnh khong cch t im M im ng thng d trong cc trng hp sau.
a) M(3; 2), d : 3x + 4y 1 = 0.
b) M(2; 5), d : y = 2x + 3.
c) M(4; 1), d Ox.
d) M(3; 2), d : 2x = 3.
e) M(5; 2), d :

x = 2 + 2t
y = 5 t
f) M(3; 2), d :
x 3
1
=
y + 4
2
.
Bi 6.68: Cho 2 ng thng d
1
: 2x 3y + 1 = 0, d
2
: 4x + 6y 3 = 0.
a) Chng minh d
1
//d
2
.
b) Tnh khong cch gia 2 ng thng .
Bi 6.69: Lp phng trnh ng thng d i qua M v to vi mt gc bit:
a) M(1; 2), : x 2y + 3 = 0, = 45
0
.
b) M(2; 0), :

x = 1 3t
y = 1 +t
, = 45
0
.
Bi 6.70: Lp phng trnh ng phn gic ca cc gc to bi d
1
v d
2
bit:
a) d
1
: 2x + 3y 1 = 0, d
2
: 3x + 2y + 2 = 0.
b) d
1
: 4x + 3y 4 = 0, d
2
:

x = 1 5t
y = 3 + 12t
c) d
1
: 5x + 3y 4 = 0, d
2
:
x + 1
3
=
y + 1
5
.
d) d
1
: 3x 4y + 5 = 0, d Ox.
Bi 6.71: Lp phng trnh ng thng d i qua im M(2; 5) v cch N(4; 1) mt on bng
2.
Bi 6.72: Lp phng trnh ng thng d i qua M(2; 3) v cch u 2 im A(5; 1),
B(3; 7).
Bi 6.73: Cho 2 ng thng d
1
: x 3y + 5 = 0, d
2
: 3x + y 2 = 0. Tm im M nm trn
Ox v cch u hai ng thng d
1
, d
2
.
Ngi son : Th.s Minh Tun Trang 126 Khoa T nhin - Trng CSP Nam nh
T
h
.
s

M
i
n
h
T
u

n
6.4. Bi tp Chng 6. Hnh phng ta
Bi 6.74: Cho 3 ng thng d
1
:

x = 1 2t
y = 1 +t
, d
2
: 5x + 12y 1 = 0, d
3
: 4x 3y + 2 = 0.
Tm M nm trn d
1
v cch u d
2
, d
3
.
Bi 6.75: Cho 2 im A(2; 1), B(3; 2) v ng thng d : 4x + 3y + 5 = 0. Tm im M cch
u 2 im A, B ng thi khong cch t M n d bng 2.
Bi 6.76: Cho 2 ng thng d
1
: 2x y + 1 = 0, d
2
: x + 2y 7 = 0. Lp phng trnh ng
thng d i qua 2 gc ta sao cho d to vi d
1
v d
2
tam gic cn c nh l giao im ca d
1
v d
2
.
Bi 6.77: Cho 2 im A(0; 5), B(4; 1) v ng thng d : x4y +7 = 0. Tm trn ng thng
d im C sao cho tam gic ABC cn ti C.
Bi 6.78: Cho im A(3; 1). Xc nh im B, C sao cho OABC l hnh vung v B nm trong
gc phn t th nht. Lp phng trnh ng cho ca hnh vung .
Ngi son : Th.s Minh Tun Trang 127 Khoa T nhin - Trng CSP Nam nh
T
h
.
s

M
i
n
h
T
u

n
Chng 7. Gii hn
Chng 7
Gii hn
7.1 Gii hn dy s
7.1.1 Cc tnh cht c bn ca gii hn
0 lim
n
1
n
k
= 0 (k = 1, 2, )
O lim
n
1
a
n
= 0 (a R, [a[ > 1)
O Php ton vi i lng v hn:
+a = +, a =
++ (+) = +, + () = =
a. () =

Nu a > 0
Nu a < 0
kx Nu a = 0
a

= 0,

= kx. C th nh sau:
a > 0 :
a
0
+
= +,
a
0

=
a < 0 :
a
0
+
= ,
a
0

= +
0. = kx
. =
O nh l gii hn kp: u
n
v
n
w
n
n
Nu lim
n
u
n
= lim
n
w
n
= l lim
n
v
n
= l
O Php ton trn gii hn: Gii hn ca tng, hiu, tch, thng cc dy s tng ng l
tng, hiu, tch, thng cc gii hn.
Ngi son : Th.s Minh Tun Trang 128 Khoa T nhin - Trng CSP Nam nh
T
h
.
s

M
i
n
h
T
u

n
7.1. Gii hn dy s Chng 7. Gii hn
7.1.2 Cc v d
V d 7.1.1: Tnh cc gii hn sau:
a) lim
n
2n + 1
1 n
2
b) lim
n
2n
2
n + 1
1 +n 3n
2
c) lim
n
3n
3
2n
2
+ 1
1 +n 3n
2
d) lim
n
2n + 3 cos n
3n 2 sin n
Gii: a) lim
n
2n + 1
1 n
2
= lim
n
2
n
+
1
n
2
1
n
2
1
=
0
1
= 0 .
b) lim
n
2n
2
n + 1
1 +n 3n
2
= lim
n
2
1
n
+
1
n
2
1 +
1
n

3
n
2
=
2
1
= 2 .
c) lim
n+
3n
3
2n
2
+ 1
1 +n 3n
2
= lim
n+
n
3

3
2
n
+
1
n
3

n
2

1
n
2
+
1
n
3
= lim
n+
n

3
2
n
+
1
n
3

1
n
2
+
1
n
3
= .
d) Ta c lim
n+
2n + 3 cos n
3n 2 sin n
= lim
n+
2 + 3
cos n
n
3 2
sin n
n
.
Mt khc ta c:
1
n

cos n
n

1
n
v lim
n+

1
n

= lim
n+
1
n
= 0 lim
n+
cos n
n
= 0.
Tng t ta c : lim
n+
sin n
n
= 0.
T ta c: lim
n+
2n + 3 cos n
3n 2 sin n
=
2
3
V d 7.1.2: Tnh cc gii hn sau:
a) lim
n+
2
n+1
3.5
n
(3)
n1
+ 5
n+1
.
Ngi son : Th.s Minh Tun Trang 129 Khoa T nhin - Trng CSP Nam nh
T
h
.
s

M
i
n
h
T
u

n
7.2. Gii hn hm s Chng 7. Gii hn
b) lim
n+
n + 2
n
3 2
n1
.
c) lim
n+

n
2
+n + 1 n

.
d) lim
n+

2n
2
+n + 1 2n + 1

.
Gii: a) lim
n+
2
n+1
3.5
n
(3)
n1
+ 5
n+1
= lim
n+
2.2
n
3.5
n

1
3
.(3)
n
+ 5.5
n
= lim
n+
2.

2
5

n
3
1
3
.

3
5

n
+ 5
=
3
5
b) lim
n+
n + 2
n
3 2
n1
= lim
n+
n + 2
n
3
1
2
.2
n
= lim
n+
n
2
n
+ 1
3.

1
2

1
2
Ta c : 2
n
= (1 + 1)
n
= C
0
n
+C
1
n
+C
2
n
+ +C
n
n
C
0
n
+C
1
n
+C
2
n
= 1 +n +
n(n 1)
2
=
n
2
+n + 2
2
n 2
0 <
n
2
n

2n
n
2
+n + 2
n 2
m lim
n+
0 = lim
n+
2n
n
2
+n + 2
= 0.
lim
n+
n
2
n
= 0 lim
n+
n + 2
n
3 2
n1
=
1
1/2
= 2 .
c) lim
n+

n
2
+n + 1 n

= lim
n+
(n
2
+n + 1 n
2
)

n
2
+n + 1 +n
= lim
n+
n + 1

n
2
+n + 1 +n
.
= lim
n+
1 +
1
n

1 +
1
n
+
1
n
2
+ 1
=
1
2
d) lim
n+

2n
2
+n + 1 2n + 1

= lim
n+
n

2 +
1
n
+
1
n
2
2 +
1
n

=
7.2 Gii hn hm s
Ngoi mt s cc tnh cht ca gii hn dy s, gii hn hm s cn c mt s gii hn c bn
sau:
7.2.1 Gii hn c bn
lim
x0
sin x
x
= 1.
lim
x0
tan x
x
= 1.
Ngi son : Th.s Minh Tun Trang 130 Khoa T nhin - Trng CSP Nam nh
T
h
.
s

M
i
n
h
T
u

n
7.2. Gii hn hm s Chng 7. Gii hn
lim
x0
(1 +x)
1
x
= e hoc lim
x

1 +
1
x

x
= e.
lim
x0
e
x
1
x
= 1.
lim
x0
ln (1 +x)
x
= 1.
7.2.2 Phng php tnh gii hn
0 S dng tng ng hm s:
f (x) g (x) khi x x
0
nu lim
xx
0
f (x)
g (x)
= 1.
V d: sin x x khi x 0 v sin 2x 2x khi x 0 ?
O Phn tch a thc thnh nhn t :
p dng vi gii hn dng
0
0
khi x x
0
vi x
0
hu hn.
O Nhn lin hp:
p dng vi gii hn cha cn thc: cn bc 2 hoc bc 3.
O Chia cho hng t bc cao nht :
p dng vi gii hn khi x
7.2.3 Cc v d
V d 7.2.1: Tnh cc gii hn sau:
a) lim
x(1)
+
3x + 2
x
2
1
b) lim
x1
x
3
5x + 4
2x
2
x 1
c) lim
x2

x + 2
3

3x + 2
4 x
2
Gii: a) lim
x(1)
+
3x + 2
x
2
1
= lim
x(1)
+
3x + 2
(x 1)(x + 1)
=
1
2.0
+
= .
b) lim
x1
x
3
5x + 4
2x
2
x 1
= lim
x1
(x 1)(x
2
+x 4)
(x 1)(2x + 1)
= lim
x1
x
2
+x 4
2x + 1
=
2
3
.
c) l = lim
x2

x + 2
3

3x + 2
4 x
2
= lim
x2

x + 2
3

3x + 2
(2 x) (2 +x)
= lim
x2
3

3x + 2

x + 2
x 2
. lim
x2
1
x + 2
lim
x2
3

3x + 2 2
x 2
= lim
x2
3x + 2 8
(x 2)

3x + 2
2
+ 2
3

3x + 2 + 4

Ngi son : Th.s Minh Tun Trang 131 Khoa T nhin - Trng CSP Nam nh
T
h
.
s

M
i
n
h
T
u

n
7.2. Gii hn hm s Chng 7. Gii hn
= lim
x2
3
3

3x + 2
2
+ 2
3

3x + 2 + 4
=
3
12
=
1
4
lim
x2
2

x + 2
x 2
= lim
x2
4 x 2
(x 2)

2 +

x + 2
= lim
x2
1
2 +

x + 2
=
1
4
l =
1
4

1
4
+
1
4

= 0
V d 7.2.2: Tnh cc gii hn sau:
a) lim
x
x
3
+ 3x + 1
2x
2
+x + 1
b) lim
x

x
2
+x + 1 +x + 1

v lim
x+

x
2
+x + 1 +x + 1

Gii: a) lim
x
x
3
+ 3x + 1
2x
2
+x + 1
= lim
x
x
3

1 +
3
x
2
+
1
x
3

x
2

2 +
1
x
+
1
x
2

= lim
x
x

1 +
3
x
2
+
1
x
3

2 +
1
x
+
1
x
2
= +
b) Dng ++ (+)
lim
x+

x
2
+x + 1 +x + 1

= lim
x+

x
2

1 +
1
x
+
1
x
2

+x + 1

= lim
x+

[x[

1 +
1
x
+
1
x
2
+x + 1

= lim
x+

1 +
1
x
+
1
x
2
+ 1 +
1
x

= +
Dng ++ ()
lim
x

x
2
+x + 1 +x + 1

= lim
x
x
2
+x + 1 (x + 1)
2

x
2
+x + 1 x 1
= lim
x
x

x
2
.

1 +
1
x
+
1
x
2
x 1
= lim
x
x
[x[ .

1 +
1
x
+
1
x
2
x 1
= lim
x
x
x

1 +
1
x
+
1
x
2
+ 1 +
1
x

= lim
x
1

1 +
1
x
+
1
x
2
+ 1 +
1
x
=
1
2
V d 7.2.3: Tnh cc gii hn sau:
a) lim
x0
sin 2x
x
Ngi son : Th.s Minh Tun Trang 132 Khoa T nhin - Trng CSP Nam nh
T
h
.
s

M
i
n
h
T
u

n
7.2. Gii hn hm s Chng 7. Gii hn
b) lim
x0
sin 2x. tan 3x
(e
x
1) ln (1 + 5x)
c) lim
x1

tan

2
x

. (x
2
+x 2)

Gii: a) lim
x0
sin 2x
x
= lim
x0
sin 2x
2x
.2 = 1.2 = 2
Ch : y ta s dng sin 2x 2x khi x 0
b) lim
x0
sin 2x. tan 3x
(e
x
1) ln (1 + 5x)
= lim
x0
sin 2x
2x
.
tan3x
3x
.
x
e
x
1
.
5x
ln (1 + 5x)
.
2x.3x
x.5x
= 1.1.1.1.
6
5
=
6
5
Ch : y ta s dng sin 2x 2x, tan 3x 3x, e
x
x, ln(1 + 5x) 5x khi x 0.
c) i bin x = t + 1, khi x 1 th t 0.
l = lim
x1

tan

2
x

. (x
2
+x 2)

= lim
t0

tan

2
t +

2

(t + 1)
2
+t + 1 2

= lim
t0
[cot t. (t
2
+ 3t)] = lim
t0

t
sin t
. (t + 3) . cos t

= 1.3.1 = 3
V d 7.2.4: Tnh cc gii hn sau:
a) lim
x0
(1 + 2x)
cot 3x
b) lim
x

2x + 1
2x + 5
x
2
x+2
3x+1
Gii: a) A = (1 + 2x)
cot 3x
lnA = cot 3x. ln (1 + 2x)
lim
x0
ln A = lim
x0

ln (1 + 2x)
2x
.
3x
sin 3x
. cos 3x.
2x
3x

= 1.1.1.
2
3
=
2
3
lim
x0
A = e
2
3
b) A =

2x + 1
2x + 5
x
2
x+2
3x+1
ln A =
x
2
x + 2
3x + 1
. ln
2x + 1
2x + 5
=
x
2
x + 2
3x + 1
. ln

1 +
4
2x + 5

lim
x
ln A = lim
x

ln

1 +
4
2x + 5

4
2x + 5
.
x
2
x + 2
3x + 1
.
4
2x + 5

= 1. lim
x
4 (x
2
x + 2)
(3x + 1) . (2x + 5)
= 4 lim
x
x
2

1
1
x
+
2
x
2

3 +
1
x

.x

2 +
5
x

Ngi son : Th.s Minh Tun Trang 133 Khoa T nhin - Trng CSP Nam nh
T
h
.
s

M
i
n
h
T
u

n
7.3. Bi tp Chng 7. Gii hn
= 4 lim
x
1
1
x
+
2
x
2

3 +
1
x

2 +
5
x
= 4.
1
3.2
=
2
3
lim
x0
A = e

2
3
7.3 Bi tp
Bi 7.1: Tnh cc gii hn sau:
a) lim
x3
2x
3
5x
2
2x 3
4x
3
13x
2
+ 4x 3
b) lim
x0

1 +x + x
2

1 x +x
2
x
2
x
c) lim
x7
3

x + 20 3
4

x + 9 2
d) lim
x2
3

1 +x +

3 +x
x
3
2x + 4
Hng dn. a)
11
7
.
b) 1.
c)
32
27
.
d)
1
12
.
Bi 7.2: Tnh cc gii hn sau:
a) lim
x0
1 cos 5x
1 cos 3x
b) lim
x

2
cos x
2x
c) lim
x0

cos x
3

cos 3x
1 cos x. cos 2x
d) lim
x

3
sin

x

3

2 cos x 1
e) lim
x0
1 cos x

cos 2x
sin
2
x
f) lim
x0
7
x
6
x
5
x
4
x
Ngi son : Th.s Minh Tun Trang 134 Khoa T nhin - Trng CSP Nam nh
T
h
.
s

M
i
n
h
T
u

n
7.3. Bi tp Chng 7. Gii hn
Hng dn. a)
25
9
.
b)
1
2
.
c)
1
2
.
d)
1

3
.
e)
3
2
.
f) log5
3

7
6

.
Bi 7.3: Tnh cc gii hn sau:
a) lim
x
x
3
+ 2x
2
+ 3x + 4
4x
3
+ 3x
2
+ 2x + 1
b) lim
x
2x
3
+ 5x
2
x 1
3x
5
4x
4
+x
2
+ 3
c) lim
x

x
4
+x
2
+ 1
3

x
5
+x
2
1
d) lim
x+

x
3
+x
2

x
2
x

e) lim
x+
(3x
5
4x
3
+x 1) v lim
x
(3x
5
4x
3
+x 1)
Hng dn. a)
1
4
.
b) 0.
c) .
d)
5
6
.
e) + v .
Bi 7.4: Tnh cc gii hn ca hm s sau:
a) lim
x3
+
2x
2
5x 4
x
2
9
v lim
x3

2x
2
5x 4
x
2
9
b) lim
x2
+

x
2
4
2 x
.
x
1 3x

Ngi son : Th.s Minh Tun Trang 135 Khoa T nhin - Trng CSP Nam nh
T
h
.
s

M
i
n
h
T
u

n
7.3. Bi tp Chng 7. Gii hn
c) lim
x1

x 1
(x + 1)
2
.
2x + 1
2x 3

d) lim
x1
+
[x
2
+x 2[
[x
3
2x[ 1
e) lim
x0
+
1

cos x
1 cos

x
Hng dn. a) v +.
b) +.
c) .
d) 3.
e) 0.
Bi 7.5: Tnh cc gii hn ca hm s sau:
a) lim
x0
(1 x)
1
x
b) lim
x

3x + 8
3x 2

2x+3
c) lim
x

x
2
+ 5x + 4
x
2
3x + 7

x+2
Hng dn. a) e
1
.
b) e
20
3
.
c) e
8
.
Ngi son : Th.s Minh Tun Trang 136 Khoa T nhin - Trng CSP Nam nh
T
h
.
s

M
i
n
h
T
u

n
Chng 8. Bt ng thc
Chng 8
Bt ng thc
8.1 Cc bt ng thc c bn
- Bt ng thc l mt vn rt kh, chng kh khng phi v kin thc nhiu m v ta khng
bit bt u. Bt ng thc ni chung l mt dng bi tp khng c thut ton trong thi
i hc. Ti liu ny ra i nhm cung cp cho hc sinh mt s thut ton nht nh chng
minh bt ng thc hay tm gi tr ln nht, nh nht.
nh l 4 (Bt ng thc Cauchy). Vi cc s x
1
, x
2
, , x
n
khng m ta c :
n

x
1
x
2
x
n

x
1
+x
2
+ +x
n
n
x
1
+x
2
+ +x
n
n.
n

x
1
x
2
x
n
Du bng xy ra khi x
1
= x
2
= = x
n
.
Chng minh. Ta chng minh bt ng thc bng phng php qui np theo n.
0 Vi n = 2: Bt ng thc tr thnh:

x
1
x
2

x
1
+x
2
2
(

x
1

x
2
)
2
0
Du bng xy ra khi x
1
= x
2
.
O Gi s bt ng thc ng vi n ta chng minh bt ng thc ng vi 2n. Gi s
x
1
, x
2
, , x
2n
0, p dng bt ng thc Cauchy vi n s x
1
, x
2
, , x
n
0 ta c:
x
1
+x
2
+ +x
n
n

n

x
1
x
2
x
n
p dng bt ng thc Cauchy vi n s x
n+1
, x
n+2
, , x
2n
0 ta c:
x
n+1
+x
n+2
+ +x
2n
n

n

x
n+1
x
n+2
x
2n
T ta c:
x
1
+x
2
+ +x
2n
2n
=
1
2

x
1
+x
2
+ +x
n
n
+
x
n+1
+x
n+2
+ +x
2n
n

Ngi son : Th.s Minh Tun Trang 137 Khoa T nhin - Trng CSP Nam nh
T
h
.
s

M
i
n
h
T
u

n
8.1. Cc bt ng thc c bn Chng 8. Bt ng thc

1
2

x
1
x
2
x
n
+
n

x
n+1
x
n+2
x
2n

x
1
x
2
x
n
.
n

x
n+1
x
n+2
x
2n
=
2n

x
1
x
2
x
2n
(p dng bt ng thc Cauchy vi 2 s
n

x
1
x
2
x
n
,
n

x
n+1
x
n+2
x
2n
)
O Chng minh tnh li, tc chng minh bt ng thc ng vi n th ng vi n 1.
Tht vy gi s bt ng thc ng vi n, ly n 1 s x
1
, x
2
, , x
n1
.
p dng bt ng thc Cauchy vi n s x
1
, x
2
, , x
n1
, t =
x
1
+x
2
+ +x
n1
n 1
ta c:
x
1
+x
2
+ +x
n1
+t
n

n

x
1
.x
2
. .x
n1
.t

(n 1) t +t
n

n

x
1
.x
2
. .x
n1
.t t
n

x
1
.x
2
. .x
n1
.t
t
n
x
1
.x
2
. .x
n1
.t t
n1
x
1
.x
2
. .x
n1
t
n1

x
1
.x
2
. .x
n1

x
1
+x
2
+ +x
n1
n 1

n1

x
1
.x
2
. .x
n1
Vy bt ng thc ng vi n 1.
O Gi s bt ng thc ng vi n = k, theo bc 2 th bt ng thc ng vi n = 2k. p
dng lin tip kt qu ca bc 3 ta c bt ng thc ng vi n = 2k 1 do ng vi
n = 2k 2, ... , cui cng bt ng thc ng vi n = k + 1.
T kt qu ca bc 1, 4 v theo nguyn l qui np ta c bt ng thc ng vi mi s t
nhin n 2.
Phng php chng minh trn gi l phng php chng minh qui np li. Phng php
chng minh th v ny l ca nh ton hc ngi Php Cauchy. Tuy nhin ng khng phi l
ngi pht minh ra bt ng thc ny. Tn ca bt ng thc ny theo cc ti liu nc ngoi
l AM GM (Arithmetic Means - Geometric Means):
Trong thc t ta thng p dng bt ng thc vi n = 2, n = 3.
Vi mi a 0, b 0 ta c:

ab
a +b
2
ab

a +b
2

2
a +b 2

ab
Vi mi a, b, c 0 ta c:
3

abc
a +b +c
3
abc

a +b +c
3

3
a +b +c 3
3

abc
H qu 8.1 (Cauchy). Nu tch khng i, tng nh nht khi cc s bng nhau. Nu tng khng
i, tch ln nht khi cc s bng nhau.
nh l 5 (Bunhiacopxki). Cho 2 b s (a
1
, a
2
, , a
n
) , (b
1
, b
2
, , b
n
). Khi ta c bt ng
thc:

a
2
1
+a
2
2
+ +a
2
n

b
2
1
+b
2
2
+ +b
2
n

(a
1
b
1
+a
2
b
2
+ +a
n
b
n
)
2
Ngi son : Th.s Minh Tun Trang 138 Khoa T nhin - Trng CSP Nam nh
T
h
.
s

M
i
n
h
T
u

n
8.1. Cc bt ng thc c bn Chng 8. Bt ng thc
Du ng thc xy ra khi 2 b t l, tc l: (a
1
, a
2
, , a
n
) = t. (b
1
, b
2
, , b
n
)
Hoc
a
1
b
1
=
a
2
b
2
= =
a
n
b
n
Qui c: nu a
j
= 0 th b
j
= 0 v ngc li.
Chng minh. +) Nu a
2
1
+a
2
2
+ +a
2
n
= 0 a
1
= a
2
= = a
n
= 0 bt ng thc ng.
+) Nu a
2
1
+a
2
2
+ +a
2
n
= 0:
Ta c P = (a
1
t b
1
)
2
+ (a
2
t b
2
)
2
+ + (a
n
t b
n
)
2
= (a
2
1
+a
2
2
+ +a
2
n
) t
2
2 (a
1
b
1
+a
2
b
2
+ +a
n
b
n
) t + (b
2
1
+b
2
2
+ +b
2
n
)
P 0 t 0, P l a thc bc 2.
Do

0, ta c bt ng thc:
(a
1
b
1
+a
2
b
2
+ +a
n
b
n
)
2
(a
2
1
+a
2
2
+ +a
2
n
) (b
2
1
+b
2
2
+ +b
2
n
) 0
(a
2
1
+a
2
2
+ +a
2
n
) (b
2
1
+b
2
2
+ +b
2
n
) (a
1
b
1
+a
2
b
2
+ +a
n
b
n
)
2
.
Du bng xy ra khi tn ti t sao cho a
1
t b
1
= a
2
t b
2
= = a
n
t b
n
= 0.
a
1
b
1
=
a
2
b
2
= =
a
n
b
n
= t (a
1
, a
2
, , a
n
) = t. (b
1
, b
2
, , b
n
)
Ta thng p dng bt ng thc Bunhicopxki trong trng hp n = 2, n = 3.

a
2
+b
2

x
2
+y
2

(ax +by)
2
Du bng xy ra khi
a
x
=
b
y
t : (a, b) = t (x, y).

a
2
+b
2
+c
2

x
2
+y
2
+z
2

(ax +by +cz)


2
Du bng xy ra khi
a
x
=
b
y
=
c
z
t : (a, b, c) = t (x, y, z)
nh l 6 (Cauchy - Schwarz). Cho 2 b s (a
1
, a
2
, , a
n
) , (b
1
, b
2
, , b
n
), vi b
i
> 0 i =
1, 2, .., n. Ta c bt ng thc:
a
2
1
b
1
+
a
2
2
b
2
+ +
a
2
n
b
n

(a
1
+a
2
+ +a
n
)
2
b
1
+b
2
+ +b
n
Du bng xy ra khi 2 b t l ging bt ng thc Bunhiacopxki.
Chng minh. p dng bt ng thc Bunhiacopxki cho 2 b s:

a
1

b
1
,
a
2

b
2
, ,
a
n

b
n

b
1
,

b
2
, ,

b
n

ta c bt ng thc :

a
1

b
1

2
+

a
2

b
2

2
+ +

a
n

b
n

b
1

2
+

b
2

2
+ +

b
n

a
1

b
1
.

b
1
+
a
2

b
2
.

b
2
+ +
a
n

b
n
.

b
n

2
Ngi son : Th.s Minh Tun Trang 139 Khoa T nhin - Trng CSP Nam nh
T
h
.
s

M
i
n
h
T
u

n
8.2. Bt ng thc Cauchy Chng 8. Bt ng thc

a
2
1
b
1
+
a
2
2
b
2
+ +
a
2
n
b
n

(b
1
+b
2
+ +b
n
) (a
1
+a
2
+ +a
n
)
2

a
2
1
b
1
+
a
2
2
b
2
+ +
a
2
n
b
n

(a
1
+a
2
+ +a
n
)
2
b
1
+b
2
+ +b
n
(pcm)
8.2 Bt ng thc Cauchy
Trong thc t vic p dng bt ng thc Cauchy c mun hnh vn trng nhng tt c u tp
trung quanh phng php im ri. Nm c tng ca phng php im ri khng phi
l mt vic d dng nhng hc sinh cn nh rng c th thm bt nhng du bng phi xy ra
c, hn th na hc sinh phi cch bit c du bng xy ra khi no? Mc ny nhm
mc ch hc sinh c th nm vng hn c phng php ny. Hy vng l thng qua vic
thc hnh hc sinh s nm c ct yu ca vn .
8.2.1 Tm min tng, max ca tch
Cch gii: S dng h qu ca Bt ng thc Cauchy.
V d 8.2.1: Tm gi tr nh nht ca biu thc:
a) A = 2x +
1
x
(x > 0)
b) B = 4x +
1
x 1
(x > 1)
c) C = x
2
+
3
x
(x > 0)
d) D = x
2
+
1
x
3
(x > 0).
Gii: a) p dng BT Cauchy cho 2 s dng 2x,
1
x
ta c:
A = 2x +
1
x
2

2x.
1
x
= 2

2
Vy A
min
= 2

2, t khi 2x =
1
x
x
2
=
1
2
x =
1

2
(Do x > 0)
b) Ta c B = 4 (x 1) +
1
x 1
+ 4
p dng BT Cauchy cho 2 s dng 4 (x 1) ,
1
x 1
B = 4 (x 1) +
1
x 1
+ 4 2

4 (x 1)
1
x 1
+ 4 = 8
Ngi son : Th.s Minh Tun Trang 140 Khoa T nhin - Trng CSP Nam nh
T
h
.
s

M
i
n
h
T
u

n
8.2. Bt ng thc Cauchy Chng 8. Bt ng thc
Vy ta c B
min
= 8
4 (x 1) =
1
x 1
(x 1)
2
=
1
4
x 1 =
1
2
x =
3
2
c) Ta c C = x
2
+
3
2x
+
3
2x
3
3

x
2
.
3
2x
.
3
2x
= 3
3

9
4
.
Vy C
min
= 3
3

9
4
t khi x
2
=
3
2x
x
3
=
3
2
x =
3

3
2
d) D =
x
2
3
+
x
2
3
+
x
2
3
+
1
2x
3
+
1
2x
3
5
5

x
2
3
.
x
2
3
.
x
2
3
.
1
2x
3
.
1
2x
3
D
5
5

108
D
min
=
5
5

108
t khi
x
2
3
=
1
2x
3
x
5
=
3
2
x =
5

3
2
.
V d 8.2.2: Tm gi tr ln nht ca biu thc:
a) P = x(3 x)
2
(0 < x < 3).
b) Q = x
2
(2 5x)
3

0 < x <
2
5

.
c) R = sin
3
xcos
5
x.
Gii: a) P =
1
2
.2x(3 x) (3 x)
1
2

2x + 3 x + 3 x
3

3
= 4.
P
max
= 4 t khi 2x = 3 x x = 1.
b) Q =
5x
2
.
5x
2
.
2 5x
3
.
2 5x
3
.
2 5x
3
.
3
3
.2
2
5
2

108
25

5x
2
+
5x
2
+
2 5x
3
+
2 5x
3
+
2 5x
3
5

5
=
3456
78125
Q
max
=
3456
78125
t khi
5x
2
=
2 5x
3
25x = 2 x =
2
25
c) R
2
=

sin
2
x

3
.(cos
2
x)
5
=
sin
2
x
3
.
sin
2
x
3
.
sin
2
x
3
.
cos
2
x
5
.
cos
2
x
5
.
cos
2
x
5
.
cos
2
x
5
.
cos
2
x
5
.3
3
.5
5
3
3
.5
5

sin
2
x
3
+
sin
2
x
3
+
sin
2
x
3
+
cos
2
x
5
+
cos
2
x
5
+
cos
2
x
5
+
cos
2
x
5
+
cos
2
x
5
8

8
Ngi son : Th.s Minh Tun Trang 141 Khoa T nhin - Trng CSP Nam nh
T
h
.
s

M
i
n
h
T
u

n
8.2. Bt ng thc Cauchy Chng 8. Bt ng thc
=
3
3
.5
5
8
8
R

3
3
.5
5
8
8
R
max
=

3
3
.5
5
8
8
sin
2
x
3
=
cos
2
x
5
tan
2
x =
3
5
tanx =

3
5
. (Do R > 0 nn tan x > 0)
8.2.2 Bt ng thc i xng
Dng bi tp : chng minh bt ng thc nhng cc bin i xng vi nhau v hn
th na chng cn ng cp vi nhau.Dng bi ny thng d bi v du bng ca chng
thng xy ra khi cc bin bng nhau. Da vo phng php im ri ta s bit cch p
dng bt ng thc Cauchy th no cho thch hp.
V d 8.2.3: Cho a, b, c > 0. Chng minh cc bt ng thc sau:
a)
a
2
b
+
b
2
c
+
c
2
a
a +b +c.
b)
a
2
3b + 2c
+
b
2
3c + 2a
+
c
2
3a + 2b

a +b +c
5
c)
a
3
b (2c +a)
+
b
3
c (2a +b)
+
c
3
a (2b +c)

a +b +c
3
.
d)
a
5
b
2
+
b
5
c
2
+
c
5
a
2
a
3
+b
3
+c
3
.
e)
a
5
b
2
c
+
b
5
c
2
a
+
c
5
a
2
b
a
2
+b
2
+c
2
f)
a
3
3b + 2c
+
b
3
3c + 2a
+
c
3
3a + 2b

a
2
+b
2
+c
2
5
g)
a
5
b
2
(c + 2a)
+
b
5
c
2
(a + 2b)
+
c
5
a
2
(b + 2c)

a
2
+b
2
+c
2
3
h) a
4
+b
4
+c
4
a
3
b +b
3
c +c
3
a.
Gii: a) p dng BT Cauchy ta c:
+

a
2
b
+b 2

a
2
b
.b = 2a
b
2
c
+ c 2

b
2
c
.c = 2b
c
2
a
+a 2

c
2
a
.a = 2c
a
2
b
+
b
2
c
+
c
2
a
+ (a +b +c) 2 (a +b +c)

a
2
b
+
b
2
c
+
c
2
a
a +b +c
Du bng xy ra a = b = c.
Ngi son : Th.s Minh Tun Trang 142 Khoa T nhin - Trng CSP Nam nh
T
h
.
s

M
i
n
h
T
u

n
8.2. Bt ng thc Cauchy Chng 8. Bt ng thc
b) p dng BT Cauchy ta c:
a
2
3b + 2c
+
3b + 2c
25
2

a
2
3b + 2c
.
3b + 2c
25
=
2a
5

a
2
3b + 2c

2a
5

3b + 2c
25
T
2
:
b
2
3c + 2a

2b
5

3c + 2a
25
c
2
3a + 2b

2c
5

3a + 2b
25

a
2
3b + 2c
+
b
2
3c + 2a
+
c
2
3a + 2b

2
5
(a +b +c)
5
25
(a +b +c)

a
2
3b + 2c
+
b
2
3c + 2a
+
c
2
3a + 2b

a +b +c
5
Du bng xy ra khi:

a
2
3b + 2c
=
3b + 2c
25
b
2
3c + 2a
=
3c + 2a
25
c
2
3a + 2b
=
3a + 2b
25
a = b = c
c) p dng BT Cauchy ta c:
a
3
b (2c +a)
+
b
3
+
2c +a
9
3
3

a
3
b (2c +a)
.
b
3
.
2c +a
9
= a

a
3
b (2c +a)

8
9
a
1
3
b
2
9
c
T
2
:
b
3
c (2a +b)

8
9
b
1
3
c
2
9
a
c
3
a (2b +c)

8
9
c
1
3
a
2
9
b

a
3
b (2c +a)
+
b
3
c (2a +b)
+
c
3
a (2b +c)

8
9

a
1
3

a
2
9

a
3
b (2c +a)
+
b
3
c (2a +b)
+
c
3
a (2b +c)

1
3

a =
a +b +c
3
.
Du bng xy ra khi:

a
3
b (2c +a)
=
b
3
=
2c +a
9
b
3
c (2a +b)
=
c
3
=
2a +b
9
c
3
a (2b +c)
=
a
3
=
2b +c
9
a = b = c
d) p dng BT Cauchy ta c:
a
5
b
2
+
a
5
b
2
+
a
5
b
2
+b
3
+b
3
5
5

a
5
b
2
.
a
5
b
2
.
a
5
b
2
.b
3
.b
3
= 5a
3
Ngi son : Th.s Minh Tun Trang 143 Khoa T nhin - Trng CSP Nam nh
T
h
.
s

M
i
n
h
T
u

n
8.2. Bt ng thc Cauchy Chng 8. Bt ng thc
3
a
5
b
2
+ 2b
3
5a
3
T
2
: 3
b
5
c
2
+ 2c
3
5b
3
3
c
5
a
2
+ 2a
3
5c
3

+
3

a
5
b
2
+
b
5
c
2
+
c
5
a
2

+ 2 (a
3
+b
3
+c
3
) 5 (a
3
+b
3
+c
3
)
3

a
5
b
2
+
b
5
c
2
+
c
5
a
2

3 (a
3
+b
3
+c
3
)
a
5
b
2
+
b
5
c
2
+
c
5
a
2
a
3
+b
3
+c
3
Du bng xy ra khi:

a
5
b
2
= b
3
b
5
c
2
= c
3
c
5
a
2
= a
3
a = b = c
e) p dng BT Cauchy ta c:
a
5
b
2
c
+
a
5
b
2
c
+b
2
+b
2
+c
2
5
5

a
5
b
2
c
.
a
5
b
2
c
.b
2
.b
2
.c
2
= 5a
2
2
a
5
b
2
c
5a
2
2b
2
c
2
T
2
: 2
b
5
c
2
a
5b
2
2c
2
a
2
2
c
5
a
2
b
5c
2
2a
2
b
2

+
2

a
5
b
2
c
+
b
5
c
2
a
+
c
5
a
2
b

a
2
2

a
2

a
2
= 2

a
2

a
5
b
2
c
+
b
5
c
2
a
+
c
5
a
2
b
a
2
+b
2
+c
2
Du bng xy ra khi :

a
5
b
2
c
= b
2
= c
2
b
5
c
2
a
= c
2
= a
2
c
5
a
2
b
= a
2
= b
2
a = b = c
f) p dng BT Cauchy ta c:
a
3
3b + 2c
+
a (3b + 2c)
25
2

a
3
3b + 2c
.
a (3b + 2c)
25
Ngi son : Th.s Minh Tun Trang 144 Khoa T nhin - Trng CSP Nam nh
T
h
.
s

M
i
n
h
T
u

n
8.2. Bt ng thc Cauchy Chng 8. Bt ng thc

a
3
3b + 2c
+
3ab + 2ca
25

2a
2
5
T
2
:
b
3
3c + 2a
+
3bc + 2ab
25

2b
2
5
c
3
3a + 2b
+
3ca + 2bc
25

2c
2
5

+
a
3
3b + 2c
+
b
3
3c + 2a
+
c
3
3a + 2b
+
ab +bc +ca
5

2a
2
5
+
2b
2
5
+
2c
2
5
Do a
2
+b
2
+c
2
ab +bc +ca

a
3
3b + 2c
+
b
3
3c + 2a
+
c
3
3a + 2b
+
a
2
+b
2
+c
2
5

2 (a
2
+b
2
+c
2
)
5

a
3
3b + 2c
+
b
3
3c + 2a
+
c
3
3a + 2b

a
2
+b
2
+c
2
5
Du bng xy ra khi:

a
3
3b + 2c
=
a (3b + 2c)
25
b
3
3c + 2a
=
3bc + 2ab
25
c
3
3a + 2b
=
3ca + 2bc
25
a
2
+b
2
+c
2
= ab +bc +ca
a = b = c
g) p dng BT Cauchy ta c:
a
5
b
2
(c + 2a)
+
a
5
b
2
(c + 2a)
+
b
2
3
+
b
2
3
+
(c + 2a)
2
27
5
5

a
5
b
2
(c + 2a)

2
.

b
2
3

2
.
(c + 2a)
2
27

2a
5
b
2
(c + 2a)
+
2b
2
3
+
(c + 2a)
2
27

5a
2
3

2a
5
b
2
(c + 2a)

5a
2
3

2b
2
3

c
2
+ 4ca + 4a
2
27
Do 4ca 2c
2
+ 2a
2
nn ta c:
2a
5
b
2
(c + 2a)

5a
2
3

2b
2
3

c
2
+ 2c
2
+ 2a
2
+ 4a
2
27

2a
5
b
2
(c + 2a)

13a
2
6b
2
c
2
9
T
2
:
2b
5
c
2
(a + 2b)

13b
2
6c
2
a
2
9
2c
5
a
2
(b + 2c)

13c
2
6a
2
b
2
9

+
2

a
5
b
2
(c + 2a)
+
b
5
c
2
(a + 2b)
+
c
5
a
2
(b + 2c)

6 (a
2
+b
2
+c
2
)
9

a
5
b
2
(c + 2a)
+
b
5
c
2
(a + 2b)
+
c
5
a
2
(b + 2c)

a
2
+b
2
+c
2
3
.
Du bng xy ra khi a = b = c.
Ngi son : Th.s Minh Tun Trang 145 Khoa T nhin - Trng CSP Nam nh
T
h
.
s

M
i
n
h
T
u

n
8.2. Bt ng thc Cauchy Chng 8. Bt ng thc
h) p dng BT Cauchy ta c:
a
4
+a
4
+a
4
+b
4
4
4

a
4
.a
4
.a
4
.b
4
3a
4
+b
4
4a
3
b
T
2
: 3b
4
+c
4
4b
3
c
3c
4
+a
4
4c
3
a

+
4 (a
4
+b
4
+c
4
) 4 (a
3
b +b
3
c +c
3
a) a
4
+b
4
+c
4
a
3
b +b
3
c +c
3
a
Du bng xy ra khi a = b = c.
8.2.3 Cc tr c iu kin
Cc tr c iu kin: l dng bi tm gi tr ln nht, nh nht ca biu thc hoc chng minh
mt bt ng thc vi iu kin cho trc. iu kin y khng phi l rng buc t nhin
nh kiu cc bin khng m m l cc ng thc, hay kh hn l bt ng thc cho mi lin h
rng buc gia cc bin.
V d 8.2.4: Cho a, b, c > 0. Chng minh rng:
a) Nu a
4
+b
4
+c
4
33(a +b +c) 150 th ta c a +b +c 6.
b) Nu a
4
+b
4
+c
4
2(a +b +c) 3 th ta c a +b +c 3.
Gii: a) p dng BT Cauchy ta c:
a
4
+ 16 + 16 + 16 4
4

a
4
.16.16.16
a
4
+ 48 32a
T
2
: b
4
+ 48 32b
c
4
+ 48 32c

+
a
4
+b
4
+c
4
+ 144 32 (a +b +c)
Theo gi thit ta c : a
4
+b
4
+c
4
33(a +b +c) 150
33(a +b +c) 150 + 144 32 (a +b + c)
a +b +c 6.
Du bng xy ra khi a = b = c = 2.
b) p dng BT Cauchy ta c:
a
4
+ 1 + 1 + 1 4
4

a
4
.1.1.1
a
4
+ 3 4a
T
2
: b
4
+ 3 4b
c
4
+ 3 4c

+
a
4
+b
4
+c
4
+ 9 4 (a +b +c)
Theo gi thit ta c: a
4
+b
4
+c
4
2(a +b +c) 3.
2(a +b +c) 3 + 9 4 (a +b +c) a +b +c 3.
Du bng xy ra khi : a = b = c = 1.
Ngi son : Th.s Minh Tun Trang 146 Khoa T nhin - Trng CSP Nam nh
T
h
.
s

M
i
n
h
T
u

n
8.2. Bt ng thc Cauchy Chng 8. Bt ng thc
Bnh lun: cu a) vn chnh l ta phi bit c du bng xy ra khi a = b = c = 2. Cu
b) xy ra du bng khi a = b = c = 1. Ci ny hc sinh t mnh suy ngm.
V d 8.2.5: Cho 0 y x 1. Chng minh rng:
x

y y

x
1
4
Gii: Ta c : x

y y

x =

x.

= 2.

1
2

x.

2.

x
2
+

y +

3
=
1
4
.(

x)
3

1
4
Du bng xy ra khi:

1
2

x =

y =

x = 1

x = 1
y =
1
4
Bnh lun: Vn t ra y l ti sao trong bt ng thc Cauchy ta li p dng cho
1
2

x
ch khng phi l

x. Ta rng mt

y th nht thit phi c mt s

y v

x

y.
Cho 2 s ny bng nhau th

y =
1
2

x.
V d 8.2.6: Chng minh rng vi mi x, y > 0 ta u c:
(1 +x)

1 +
y
x

1 +
9

2
256
Gii: Bt ng thc Cauchy dng tng qut:
mx +ny (m +n)
m+n

x
m
y
n
hoc x + (1 ) y x

.y
1
(0 < < 1).
- Phn tch bi ton: 1 +x = (1 ) .
1
1
+.
x

1
1

1
= k
1
.x

Tng t ta cng c :1 +
y
x
k
2

y
x

, 1 +
9

y
k
3
.

(k
i
y l hng s)
(1 +x)

1 +
y
x

1 +
9

2
k
4
x

y
x

2
= k
5
.x

.y

Do = = 0 = = .
Ta tm chng bng cch xem du bng xy ra khi no?

=
1
1
y
x
=
1
1
9

y
=
1
1

x =

1
y =

2
(1 )
2

2
(1 )
2
= 9
= 3 (1 ) =
3
4
1 =
1
4
.
Cch gii: 1 +
x
3
+
x
3
+
x
3
4
4

1.
x
3
.
x
3
.
x
3
= 4.
4

x
3
27
Ngi son : Th.s Minh Tun Trang 147 Khoa T nhin - Trng CSP Nam nh
T
h
.
s

M
i
n
h
T
u

n
8.2. Bt ng thc Cauchy Chng 8. Bt ng thc
1 +x 4.
4

x
3
27
Tng t ta cng c: 1 +
y
x
4.
4

y
3
27x
3

1 +
9

4.
4

27

y
3

2
= 16.
4

27
2
y
3
Nhn v cc bt phng trnh ta c:
(1 +x)

1 +
y
x

1 +
9

2
256
Du ng thc xy ra khi:

x
3
= 1
y
3x
= 1
3

y
= 1

x = 3
y = 9
V d 8.2.7: Cho a, b, c > 0 tha mn a +b +c = 1. Chng minh rng:
a)

1 +
1
a

1 +
1
b

1 +
1
c

64.
b)

2 +
1
a

2 +
1
b

2 +
1
c

45 + 29

2.
Gii: a) Ta c: V T =
(a + 1) (b + 1) (c + 1)
abc
.
p dng BT Cauchy ta c:
a + 1 = a +a +b +c 4
4

a.a.b.c
a + 1 4
4

a
2
bc
T
2
: b + 1 4
4

ab
2
c
c + 1 4
4

abc
2

+
V T
4
4

a
2
bc.4
4

ab
2
c.4
4

abc
2
abc
= 64
Du bng xy ra khi a = b = c =
1
3
.
b) Ta chng minh bt ng thc sau:
3

(k +X) (k +Y ) (k +Z) k +
3

XY Z k, X, Y, Z > 0
p dng BT Cauchy cho 3 s ta c:
Ngi son : Th.s Minh Tun Trang 148 Khoa T nhin - Trng CSP Nam nh
T
h
.
s

M
i
n
h
T
u

n
8.3. Bt ng thc Bunhiacopxki Chng 8. Bt ng thc
3

k
k +X
.
k
k +Y
.
k
k +Z

1
3

k
k +X
+
k
k +Y
+
k
k +Z

X
k +X
.
Y
k +Y
.
Z
k +Z

1
3

X
k +X
+
Y
k +Y
+
Z
k +Z

+
3

k
k +X
.
k
k +Y
.
k
k +Z
+
3

X
k +X
.
Y
k +Y
.
Z
k +Z
1

(k +X) (k +Y ) (k +Z) k +
3

XY Z (pcm)
p dng BT trn vi k =

2, X =
1
a
, Y =
1
b
, Z =
1
c
> 0 ta c:
3

2 +
1
a

2 +
1
b

2 +
1
c

2 +
3

1
a
.
1
b
.
1
c

2 +
1
3

abc

2 +
1
1
3
(a +b +c)
= 3 +

2
V T

3 +

3
= 45 + 29

2
Du bng xy ra khi a = b = c =
1
3
8.3 Bt ng thc Bunhiacopxki
V d 8.3.1: Cho a, b l cc s thc . Tm gi tr nh nht ca:
a) P = 2a
2
+ 3b
2
nu a +b = 1.
b) Q = 3a
2
+
b
2
2
nu 2a + 5b = 1.
Gii: a) p dng BT Bunhiacopxki vi 2 b

a

2, b

2
,
1

ta c:

2
+

2
+

2.
1

2
+b

3.
1

2
(2a
2
+ 3b
2
)

1
2
+
1
3

(a +b)
2
= 1 P
6
5
Vy P
min
=
6
5
t khi:

2
1/

2
=
b

3
1/

3
a +b = 1

2a = 3b
a +b = 1

a =
3
5
b =
2
5
Ngi son : Th.s Minh Tun Trang 149 Khoa T nhin - Trng CSP Nam nh
T
h
.
s

M
i
n
h
T
u

n
8.3. Bt ng thc Bunhiacopxki Chng 8. Bt ng thc
b) p dng BT Bunhiacopxki vi 2 b

3a,
b

3
, 5

ta c:

3a

2
+

2
+

3a.
2

3
+
b

2
.5

3a
2
+
b
2
2

4
3
+ 50

(2a + 5b)
2
Q.
154
3
1 Q
3
154
Q
min
=
3
154
Du bng xy ra khi :

3a
2/

3
=
b/

2
5

2
2a + 5b = 1

15a = b
2a + 5b = 1

a =
1
77
b =
15
77
V d 8.3.2: Cho a, b, c, p, q l cc s thc dng. Chng minh rng:
a) a
2
+b
2
+c
2
ab +bc +ca.
b)
a
3
pb +qc
+
b
3
pc +qa
+
c
3
pa +qb

a
2
+b
2
+c
2
p +q
.
Gii: a) p dng bt ng thc Bunhiacopxki cho 2 b (a, b, c) , (b, c, a) ta c
(a
2
+b
2
+c
2
) (b
2
+c
2
+a
2
) (ab +bc +ca)
2
(a
2
+b
2
+c
2
)
2
(ab +bc +ca)
2
a
2
+b
2
+c
2
[ab +bc +ca[
a
2
+b
2
+c
2
ab +bc +ca
Du bng xy ra khi:

a
b
=
b
c
=
c
a
ab +bc +ca 0
a = b = c
b) p dng BT Bunhiacopxki vi 2 b s:

a
3
pb + qc
,

b
3
pc +qa
,

c
3
pa +qb

a (pb +qc),

b (pc +qa),

c (pa +qb)

ta c :

a
3
pb +qc
+
b
3
pc +qa
+
c
3
pa +qb

. (a (pb +qc) +b (pc +qa) +c (pa +qb))

a
3
pb +qc
.

a (pb +qc) +

b
3
pc +qa
.

b (pc +qa) +

c
3
pa +qb
.

c (pa +qb)

2
Do a
2
+b
2
+c
2
ab +bc +ca nn ta c:

a
3
pb +qc
+
b
3
pc +qa
+
c
3
pa +qb

a
2
+b
2
+c
2
p +q
.
a
2
+b
2
+c
2
ab +bc +ca

a
2
+b
2
+c
2
p +q
Du bng xy ra khi a = b = c.
Ngi son : Th.s Minh Tun Trang 150 Khoa T nhin - Trng CSP Nam nh
T
h
.
s

M
i
n
h
T
u

n
8.4. Bt ng thc Cauchy - Schwarz Chng 8. Bt ng thc
8.4 Bt ng thc Cauchy - Schwarz
V d 8.4.1: Cho x, y, z > 0, chng minh rng:
a)
1
x
+
1
y

4
x +y
b)
1
x
+
1
y
+
1
z

9
x +y +z
c) Nu x, y, z l 3 cnh ca tam gic th ta c:
1
y +z x
+
1
z +x y
+
1
x +y z

1
x
+
1
y
+
1
z
Gii: a) p dng BT Cauchy - Schwarz (C-S) vi 2 b (1; 1) , (x, y) ta c:
1
2
x
+
1
2
y

(1 + 1)
2
x +y

1
x
+
1
y

4
x +y
Du bng xy ra khi x = y.
Ch : C th chng minh bt ng thc bng php bin i tng ng.
b) p dng BT C-S vi 2 b (1; 1; 1) , (x, y, z) ta c:
1
2
x
+
1
2
y
+
1
2
z

(1 + 1 + 1)
2
x +y +z

1
x
+
1
y
+
1
z

9
x +y +z
Du bng xy ra khi x = y = z.
c) p dng cu a) vi 2 s y +z x, z +x y ta c:
1
y +z x
+
1
z +x y

4
y +z x +z +x y
=
4
2z

1
y +z x
+
1
z +x y

2
z
T
2
:
1
z +x y
+
1
x +y z

2
x
1
y +z x
+
1
x +y z

2
y

+
2

1
y +z x
+
1
z +x y
+
1
x +y z

1
x
+
1
y
+
1
z

1
y +z x
+
1
z +x y
+
1
x +y z

1
x
+
1
y
+
1
z
Du bng xy ra khi x = y = z.
Ngi son : Th.s Minh Tun Trang 151 Khoa T nhin - Trng CSP Nam nh
T
h
.
s

M
i
n
h
T
u

n
8.5. Bi tp Chng 8. Bt ng thc
V d 8.4.2 (H - A - 2005): Cho x, y, z > 0 tha mn:
1
x
+
1
y
+
1
z
= 4. Chng minh rng:
1
2x +y +z
+
1
2y +z +x
+
1
2z +x +y
1
Gii: p dng BT Cauchy - Schwarz ta c:
1
2
x
+
1
2
x
+
1
2
y
+
1
2
z

(1 + 1 + 1 + 1)
2
x +x +y +z

2
x
+
1
y
+
1
z

16
2x +y +z
T
2
:
2
y
+
1
z
+
1
x

16
2y +z +x
2
z
+
1
x
+
1
y

16
2z +x +y

+
4

1
x
+
1
y
+
1
z

16

1
2x +y +z
+
1
2y +z +x
+
1
2z +x +y

1
2x +y +z
+
1
2y +z +x
+
1
2z +x +y

1
4

1
x
+
1
y
+
1
z

= 1
Du bng xy ra khi :

x = y = z
1
x
+
1
y
+
1
z
= 4
x = y = z =
3
4
8.5 Bi tp
Bi 8.1 (H-B-2005): Chng minh rng x R, ta c:

12
5

x
+

15
4

y
+

20
3

z
3
x
+ 4
x
+ 5
x
Bi 8.2 (H-D-2005): Cho cc s dng x, y, z tha mn xyz = 1. Chng minh rng:

1 +x
3
+y
3
xy
+

1 +y
3
+z
3
yz
+

1 +z
3
+x
3
zx
Du ng thc xy ra khi no?
Bi 8.3 (H-B-2006): Cho cc s thc x, y thay i. Tm gi tr nh nht ca biu thc:
A =

(x 1)
2
+y
2
+

(x + 1)
2
+y
2
+[y 2[
Bi 8.4 (H-A-2007): Cho x, y, z l cc s thc dng tha mn iu kin xyz = 1. Tm gi
tr nh nht ca biu thc:
P =
x
2
(y +z)
y

y + 2z

z
+
y
2
(z +x)
z

z + 2x

x
+
z
2
(x +y)
x

x + 2y

y
Ngi son : Th.s Minh Tun Trang 152 Khoa T nhin - Trng CSP Nam nh
T
h
.
s

M
i
n
h
T
u

n
8.5. Bi tp Chng 8. Bt ng thc
Bi 8.5 (H-B-2007): Cho x, y, z > 0. Tm gi tr nh nht ca biu thc:
P = x

x
2
+
1
yz

+y

y
2
+
1
zx

+z

z
2
+
1
xy

Bi 8.6 (H-D-2008): Cho x, y l cc s thc khng m thay i. Tm min, max ca biu thc:
P =
(x y) (1 xy)
(1 +x)
2
(1 + y)
2
Bi 8.7 (H-A-2006): Cho cc s thc x, y = 0 thay i v tha mn iu kin:
xy(x +y) = x
2
xy +y
2
. Tm gi tr ln nht ca biu thc: A =
1
x
3
+
1
y
3
Bi 8.8 (H-A-2003): Cho x, y, z > 0 tha mn x +y +z 1. Chng minh rng:

x
2
+
1
x
2
+

y
2
+
1
y
2
+

z
2
+
1
z
2

82
Bi 8.9 (DB2-D-2005): Cho x, y, z > 0 tha mn xyz = 1. Chng minh rng:
x
2
1 +y
+
y
2
1 +z
+
z
2
1 +x

3
2
Bi 8.10 (DB2-B-2008): Cho s nguyn n (n 2) v 2 s thc khng m x, y. Chng minh
rng:
n

x
n
+y
n

n+1

x
n+1
+y
n+1
Bi 8.11 (DB1-B-2008): Cho 3 s dng x, y, z tha mn x +y +z =
yz
3x
. Chng minh rng:
x
2

3 3
6
(y +z)
Bi 8.12 (DB1-D-2007): Cho a, b, c > 0 tha mn ab +a +b = 3. Chng minh:
3a
b + 1
+
3b
a + 1
+
ab
a +b
a
2
+b
2
+
3
2
Bi 8.13 (DB2-A-2007): Cho x, y, z > 0. Tm gi tr nh nht ca biu thc:
P =
3

4 (x
3
+y
3
) +
3

4 (y
3
+z
3
) +
3

4 (z
3
+x
3
) + 2

x
y
2
+
y
z
2
+
z
x
2

Bi 8.14 (DB2-B-2006): Cho x, y > 0 tha mn x +y 4. Tm gi tr nh nht ca biu thc


: A =
3x
2
+ 4
4x
+
2 +y
3
y
2
Bi 8.15 (DB2-A-2006): Cho cc s thc x, y, z tha mn iu kin 3
x
+3
y
+3
z
= 1. Chng
minh rng:
9
x
3
x
+ 3
y+z
+
9
y
3
y
+ 3
z+x
+
9
z
3
z
+ 3
x+y

3
x
+ 3
y
+ 3
z
4
Ngi son : Th.s Minh Tun Trang 153 Khoa T nhin - Trng CSP Nam nh
T
h
.
s

M
i
n
h
T
u

n
8.5. Bi tp Chng 8. Bt ng thc
Bi 8.16 (DB1-A-2005): Cho x, y, z tha mn x +y +z = 0. Chng minh rng:

3 + 4
x
+

3 + 4
y
+

3 + 4
z
6
Bi 8.17 (DB2-A-2003): Tm gi tr ln nht, nh nht ca hm s: y = sin
5
x +

3 cos x.
Bi 8.18 (DB1-B-2002): Gi s x, y > 0 tha mn iu kin x + y =
5
4
. Tm gi tr nh nht
ca biu thc : S =
4
x
+
1
4y
Bi 8.19 (DB1-A-2002): Gi s a, b, c, d l bn s nguyn thay i tha mn 1 a < b < c <
d 50. Chng minh :
a
b
+
c
d

b
2
+b + 50
50b
v tm gi tr nh nht ca biu thc S =
a
b
+
c
d
Ngi son : Th.s Minh Tun Trang 154 Khoa T nhin - Trng CSP Nam nh
T
h
.
s

M
i
n
h
T
u

n
Chng 9. Hm s v th
Chng 9
Hm s v th
9.1 Kho st v v th hm s
9.1.1 Kin thc cn nh
Tnh ng bin nghch bin
nh ngha 9.1. Gi s hm f(x) xc nh trn tp D. Hm s f(x) gi l ng bin (t.
nghch bin trn tp D) nu x
1
, x
2
D v x
1
< x
2
th f(x
1
) < f(x
2
) (t. f(x
1
) > f(x
2
)).
Ch : f(x) ng bin (t. nghch bin) trn D
f(x
1
) f(x
2
)
x
1
x
2
> 0
(t.
f(x
1
) f(x
2
)
x
1
x
2
< 0) vi mi x
1
= x
2
nh l 7 (Tiu chun n iu). Hm s f(x) c o hm trn khong (a; b).
a) Hm s f(x) ng bin trn khong (a; b) nu f

(x) 0 x (a; b).


b) Hm s f(x) nghch bin trn khong (a; b) nu f

(x) 0 x (a; b)
f

(x) ch c hu hn khng im trn khong (a; b).


V d 9.1.1: Hm f(x) = x
3
l ng bin trn R v f

(x) = 3x
2
0 x R v f

(x) = 0
x = 0.
Hm g(x) =
1
5
x
5
+
1
2
x
4

1
3
x
3
ng bin trn R v g

(x) = x
4
+ 2x
3
x
2
= (x
2
x)
2
0
x R, g

(x) = 0 x = 0 hoc x = 1.
Cc tr ca hm s
Di y ta gii thiu 2 cch tm nhanh ta im cc tr thng dng trong bi tp:
Hm bc 3: y = ax
3
+bx
2
+cx +d (a = 0). Gi s (x
1
, y
1
) v (x
2
, y
2
) l ta 2 im cc
tr ca hm s. Khi ta c cng thc :

y
1
= Ax
1
+B
y
2
= Ax
2
+B
Ax+B l phn d trong php chia ca y = ax
3
+bx
2
+cx+d cho y

= 3ax
2
+2bx+c.
V khi y = Ax +B cng chnh l ng thng ni 2 im cc tr.
Ngi son : Th.s Minh Tun Trang 155 Khoa T nhin - Trng CSP Nam nh
T
h
.
s

M
i
n
h
T
u

n
9.1. Kho st v v th hm s Chng 9. Hm s v th
Vi hm phn thc y =
P(x)
Q(x)
,
Gi s (x
0
, y
0
) l 2 im cc tr ca hm s th: y
0
=
P

(x
0
)
Q

(x
0
)
.
Ni ring vi hm phn thc: y =
ax
2
+bx +c
px +q
a.p = 0 v nu hm s t cc i cc tiu
th y =
2ax +b
p
l ng thng i qua 2 im cc tr.
im un ca hm s
nh ngha 9.2 (Khong li - lm). Cho hm s y = f(x). Hm s gi l li (t. lm) trn
(a, b) nu f

(x) < 0 (t. f

(x) > 0) x (a; b).


nh ngha 9.3 (im un). Hm s y = f(x) gi l c im un ti im M nm trn ng
cong c honh x
0
, nu nh n tha mn ng thi 2 iu kin sau:
(i) f

(x
0
) = 0.
(ii) f

(x) i du khi qua im x


0
.
Tim cn ca hm phn thc
Cho hm phn thc y =
P(x)
Q(x)
, vit dng ti gin (khng c chung nhn t).
Hm phn thc c tim cn ngang nu nh degP(x) degQ(x) ( degP(x) l bc ca
a thc P(x)). V khi y = a l tim cn ngang ca hm s vi a = lim
x
P(x)
Q(x)
.
Hm phn thc c tim cn xin nu nh degP(x) = degQ(x) + 1 v khi y = ax +b
vi a = lim
x
P(x)
Q(x)
, b = lim
x

P(x)
Q(x)
ax

.
Ta c th tm a, b bng cch chia P(x) cho Q(x). Thng ca php chia chnh l ax +b:
P(x) = Q(x).(ax + b) + R(x) degR(x) < degQ(x) nn lim
x
P(x)
Q(x)
(ax +b) =
lim
x

R(x)
Q(x)

= 0
Hm phn thc c tim cn ng nu Q(x) c nghim. Nu x
0
l mt nghim ca Q(x) th
x = x
0
l mt tim cn ng ca hm s v lim
xx
0
P(x)
Q(x)
= . Q(x) c bao nhiu nghim
th hm s c by nhiu tim cn ng.
Cc php bin i th c bn
Cn nh : Cc php bin i
~ (x, y) (x, y): i xng qua trc tung.
Ngi son : Th.s Minh Tun Trang 156 Khoa T nhin - Trng CSP Nam nh
T
h
.
s

M
i
n
h
T
u

n
9.1. Kho st v v th hm s Chng 9. Hm s v th
~ (x, y) (x, y): i xng qua trc honh.
~ (x, y) (x, y): i xng qua gc ta .
~ (x, y) (x a, y): Tnh tin theo trc honh a n v.
~ (x, y) (x, y b): Tnh tin theo trc tung b n v.
~ (x, y) (x a, y b): Tnh tin theo vc t

u = (a; b).
th hm y = f(x) th hm y = [f(x)[ th hm y = f([x[)
y
x
0 1 2
-2
-1
0
1
2
y = x
3
3x
2
+ 2
y
x
0 1 2
-2
-1
0
1
2
y = [x
3
3x
2
+ 2[
y
x
-2 -1 0 1 2
-2
-1
0
1
2
y = [x[
3
3[x[
2
+ 2
th hm y = f(x).g(x) th hm y = f(x).[g(x)[
y
x
-1 0 1 2 3
-3
-2
-1
0
1
2
3
y = x
3
3x
2
y
x
-1 0 1 2 3
-3
-2
-1
0
1
2
3
y = x[x
2
3x[
a = 1
th gc Tnh tin theo trc honh Tnh tin theo trc tung
Ngi son : Th.s Minh Tun Trang 157 Khoa T nhin - Trng CSP Nam nh
T
h
.
s

M
i
n
h
T
u

n
9.1. Kho st v v th hm s Chng 9. Hm s v th
y
x
-1 0 1 2 3
-3
-2
-1
0
1
y = x
3
3x
2
+ 1
y
x
0 1 2 3 4
-3
-2
-1
0
1
y = x
3
6x
2
+ 9x 3
y
x
0 1 2
-2
-1
0
1
2
y = x
3
3x
2
+ 2
9.1.2 Cc bc kho st hm s
Tm tp xc nh ca hm s (xc nh tnh chn l, tun hon).
S bin thin.
~ Chiu bin thin ca hm s: Tnh o hm y

. Tm nghim ca phng trnh y

= 0
v cc im khng xc nh ca y

. Xt du ca y

v suy ra chiu bin thin.


~ Tm cc tr ca hm s.
~ Tnh gii hn ti , + v ti cc im m hm s khng xc nh. Tm tim
cn.
~ Lp bng bin thin.
V th. Nn tnh thm giao vi trc tung, v cc im ln cn ca cc cc tr.
Ch : v chnh xc tim cn, cc tr.
9.1.3 Hm a thc
V d 9.1.2: Kho st v v th hm s y = x
3
+ 3x
2
.
Gii: +) Tp xc nh : R
+) Gii hn : lim
x+
y = lim
x+
(x
3
+ 3x
2
) = lim
x+
x
3

1 +
3
x

=
lim
x
y = lim
x
(x
3
+ 3x
2
) = lim
x
x
3

1 +
3
x

= +
+) Chiu bin thin: y

= 3x
2
+ 6x. y

= 0 3x
2
+ 6x = 0

x = 0 y = 0
x = 2 y = 4
Bng bin thin :
x 0 2 +
f

(x) 0 + 0
f(x)
+
0
4

Ngi son : Th.s Minh Tun Trang 158 Khoa T nhin - Trng CSP Nam nh
T
h
.
s

M
i
n
h
T
u

n
9.1. Kho st v v th hm s Chng 9. Hm s v th
im cc tiu (0; 0). im cc i (2; 4).
Hm s ng bin trn khong (0; 2).
Hm s nghch bin trn 2 khong (; 0) v (2; +).
+) th:
y
x
-1 0 1 2 3 0
1
2
3
4
y = x
3
+ 3x
2
Bng cc gi tr c bit
x 1 0 1 2 3
y 4 0 2 4 0
V d 9.1.3: Kho st v v th hm s : y = x
4
2x
2
1
Gii: +) Tp xc nh : R
+) Gii hn : lim
x
(x
4
2x
2
1) = lim
x
x
4

1
2
x
2

1
x
4

= +
+) Chiu bin thin: y

= 4x
3
4x = 0

x = 0 y = 0
x = 1 y = 2
Bng bin thin :
x 1 0 1 +
f

(x) 0 + 0 0 +
f(x)
+
2
1
2
+
Hm s ng bin trn 2 khong : (1; 0), (1; +)
Hm s nghch bin trn 2 khong : (; 1), (0; 1)
Hm s c 2 im cc tiu : (1; 2) v (1; 2)
Hm s c im cc i : (0; 1) +) th:
Bng cc gi tr c bit
x 2 1 0 1 2
y 7 2 1 2 7
Ngi son : Th.s Minh Tun Trang 159 Khoa T nhin - Trng CSP Nam nh
T
h
.
s

M
i
n
h
T
u

n
9.1. Kho st v v th hm s Chng 9. Hm s v th
y
x
-2 -1 0 1 2
-2
-1
0
1
2
y = x
4
2x
2
1
9.1.4 Hm phn thc
V d 9.1.4: Kho st v v th hm s y =
x
2
+x 1
x 1
Gii: y =
x
2
+x 1
x 1
y = x + 2 +
1
x 1
+) Tp xc nh: R` 1
+) Tim cn: lim
x1
+
y = lim
x1
+
x
2
+x 1
x 1
= +, lim
x1

y = lim
x1

x
2
+x 1
x 1
=
nn tim cn ng l x = 1.
lim
x
(y (x + 2)) = lim
x
1
x 1
= 0
y = x + 2 l tim cn xin ca hm s.
+) Chiu bin thin: y

= 1
1
(x 1)
2
=
x
2
2x
(x 1)
2
y

= 0
x
2
2x
(x 1)
2
= 0

x = 0 y = 1
x = 2 y = 5
Bng bin thin :
x 0 1 2 +
f

(x) + 0 0 +
f(x)

+
5
+
Hm s ng bin trn khong: (; 0) v (2; +).
Hm s nghch bin trn khong: (0; 1) v (1; 2).
Hm s t cc tiu ti im (2; 5).
Hm s t cc i ti im (0; 1)
+) th: Bng cc gi tr c bit
x 1 0
1
2
3
2
2 3
y
1
2
1
1
2
11
2
5
11
2
Ngi son : Th.s Minh Tun Trang 160 Khoa T nhin - Trng CSP Nam nh
T
h
.
s

M
i
n
h
T
u

n
9.2. Cc tr v tim cn ca hm s Chng 9. Hm s v th
y
x
-2 -1 0 1 2 3 4
-1
0
1
2
3
4
5
6
7
y =
x
2
+x 1
x 1
9.2 Cc tr v tim cn ca hm s
9.2.1 Quy tc tm cc i v cc tiu ca hm s
nh ngha 9.4. Cho hm f(x) xc nh trn khong (a, b).
x
0
gi l im cc tiu (t. cc i) ca hm s f(x) nu f(x) f(x
0
) (t. f(x) f(x
0
)) vi
mi x nm trong mt ln cn no ca x
0
.
im cc i v cc tiu c gi chung l cc tr.
Tuy nhin ta khng th dng nh ngha ny tm im cc tr. Ty tng bi tp m ta s
dng cc iu kin di y tm cc tr.
nh l 8 (iu kin cn - Fermat). Hm s f(x) xc nh trn khong (a, b). Nu f(x) t
cc tr ti x
0
(a; b) v f(x) kh vi (c o hm) ti im x
0
th f

(x
0
) = 0.
Cc im c o hm bng 0 gi l im dng (Xem ngha vt l ca o hm).
nh l 9 (iu kin - Du hiu 1). Gi s f(x) l hm s kh vi trn khong (a, b),
x
0
l mt im dng ca f(x).
a) Nu f

(x) i du t sang + khi qua im x


0
th hm s t cc tiu ti x
0
.
b) Nu f

(x) i du t + sang khi qua im x


0
th hm s t cc i ti x
0
.
nh l 10 (iu kin -Du hiu 2). f(x) l mt hm s xc nh trn khong (a, b). Gi s
x
0
l mt im dng ca hm s f(x). f(x) kh vi cp 1 v cp 2 ti x
0
. Khi :
a) Nu f

(x
0
) > 0 th x
0
l im cc tiu.
b) Nu f

(x
0
) > 0 th x
0
l im cc i.
Ch :
iu kin cn + Du hiu 1: p dng trong trng hp hm s c th d xt du.
Ngi son : Th.s Minh Tun Trang 161 Khoa T nhin - Trng CSP Nam nh
T
h
.
s

M
i
n
h
T
u

n
9.2. Cc tr v tim cn ca hm s Chng 9. Hm s v th
iu kin cn + Du hiu 2: p dng trong trng hp hm s cha tham s, hoc hm
s kh xt du o hm (chng hn: hm lng gic).
V d 9.2.1: Cho hm s y = sin
2
x sin x xt trn on [0; 2].
Gii: y

= 2 sin xcos x cos x.


y

= 0 cos x(2 sin x 1) = 0

cos x = 0
2 sinx 1 = 0

x =

2
+k
x =

6
+k2
x =
5
6
+k2
x [0; 2] x
1
=

6
, x
2
=

2
, x
3
=
5
6
, x
4
=
3
2
Ta c y

= 2 cos 2x + sin x
y

(x
1
) = y

=
3
2
> 0 x
1
nn x
1
l honh im cc tiu.
y

(x
2
) = y

= 1 < 0 x
2
nn x
2
l honh im cc i.
y

(x
3
) = y

5
6

=
3
2
> 0 x
3
nn x
3
l im honh cc tiu.
y

(x
4
) = y

3
2

= 3 < 0 x
4
nn x
4
l im honh cc i.
Kt lun: im cc tiu

6
;
1
4

2
; 0

im cc i

5
6
;
1
4

3
2
; 0

9.2.2 Cc tr hm s
Cc bi ton n thun tm cc tr
V d 9.2.2: Cho hm s y = (x m)
3
3x.
Tm m hm s t cc tiu ti im c honh x = 0.
Gii: Ta c y

= 3(x m)
2
3, y

= 6(x m).
Hm s t cc tiu ti im c honh bng khng khi:

(0) = 0
y

(0) > 0

3m
2
3 = 0
6m > 0

m = 1
m = 1
m < 0
m = 1
V d 9.2.3: Cho hm s y = x
4
+ 2. Tm cc tr ca hm s.
Gii: Ta c y

= 4x
3
, y

= 0 x = 0. Ta c bng bin thin


Ngi son : Th.s Minh Tun Trang 162 Khoa T nhin - Trng CSP Nam nh
T
h
.
s

M
i
n
h
T
u

n
9.2. Cc tr v tim cn ca hm s Chng 9. Hm s v th
x 0 +
f

(x) 0 +
f(x)
+
2
+
Do im cc tiu (0; 2) .
Ch : Nu dng quy tc 2 th khng n bi ny.
Cc bi ton nh tnh v cc tr
Cc bi tp ny thng c dng sau: Tm iu kin mt hm s cho c cc tr v
cc tr tha mn mt iu kin no (Tnh cht ny thng c cho di dng mt h
thc c th l ng thc hoc bt ng thc).
Lc chung gii bi ton ny nh sau:
0 Trc ht tm iu kin hm s cho c cc tr (ni cch khc tm iu kin cn
tn ti li gii). Xin lu vi cc bn rng: hc sinh hay qun iu kin ny v
h cho rng khi u bi yu cu tm iu kin cc tr ca hm s tha mn mt
iu kin no hc mc nhin cng nhn l cc tr tn ti. Chnh v th dn n
chuyn trong cc nghim tm c rt c kh nng gp phi nghim ngoi lai.
O Vn dng cc kin thc khc y (hay dng nh l Viet) chng minh h thng
cc tr ca hm s cn tha mn.
Ta rt hay s dng mt s kin thc sau:
~ Hm s y = ax
3
+ bx
2
+ cx + d (a = 0) c cc i, cc tiu khi v ch khi y

= 0
(3ax
2
+ 2bx +c = 0) c 2 nghim phn bit.
~ Hm s y =
ax
2
+bx +c
px +q
(a.p = 0) c cc i cc tiu khi v ch khi y

= 0 c 2
nghim phn bit x =
q
p
.
~ Hm s y =
ax
2
+bx +c
px +q
c cc tr ti x
0
th y(x
0
) =
2ax
0
+b
p
.
Tng qut y =
P(x)
Q(x)
c cc tr ti x
0
th y(x
0
) =
P

(x
0
)
Q

(x
0
)
.
V d 9.2.4: Cho hm s y = x
3
+ 2(m1)x
2
+ (m
2
4m+ 1)x 2(m
2
+ 1).
Tm m hm s t cc tr ti 2 im c honh x
1
, x
2
sao cho:
1
x
1
+
1
x
2
=
1
2
(x
1
+x
2
).
Gii: Trc ht hm s c cc tr ta cn c phng trnh : y

= 0 c 2 nghim phn bit.


Ta c y

= 0 3x
2
+ 4(m1)x +m
2
4m + 1 = 0 (1)
(1) c 2 nghim phn bit khi

> 0
m
2
+ 4m+ 1 > 0

m < 2

3
m > 2 +

3
(2)
Ngi son : Th.s Minh Tun Trang 163 Khoa T nhin - Trng CSP Nam nh
T
h
.
s

M
i
n
h
T
u

n
9.2. Cc tr v tim cn ca hm s Chng 9. Hm s v th
Vy (2) l iu kin ng cong c cc tr.
Khi c cc tr x
1
, x
2
l 2 nghim ca phng trnh (1) theo nh l Viet ta c:

x
1
+x
2
=
4 (1 m)
3
x
1
x
2
=
m
2
4m+ 1
3
1
x
1
+
1
x
2
=
1
2
(x
1
+x
2
)
x
1
+x
2
x
1
x
2
=
1
2
(x
1
+x
2
)
(x
1
+x
2
) (2 x
1
x
2
) = 0

x
1
+x
2
= 0
x
1
x
2
= 2

4 (1 m)
3
= 0
m
2
4m+ 1
3
= 2

m = 1
m
2
4m5 = 0

m = 1

Tha mn

m = 1 (Loi)
m = 5

Tha mn

Kt lun: Vi m = 1 v m = 5 tha mn iu kin bi ton.


V d 9.2.5: Cho f (x) =
x
3
3

x
2
2
+mx + 1, g (x) =
x
3
3
+x
2
+ 3mx +m.
Tm m mi hm s c 2 cc tr v gia honh 2 cc tr ca hm s ny c honh cc
tr ca hm s kia.
Gii: Ta c f

(x) = x
2
x+m, g

(x) = x
2
+2x+3m. Trc ht ta cn tm iu kin f(x),
g(x), mi hm s u c cc tr. iu l cc phng trnh f

(x) = 0 v g

(x) = 0 u c 2
nghim phn bit.


1
= 1 4m > 0

2
= 1 3m > 0

m <
1
4
m <
1
3
m <
1
4
(1)
Vi iu kin (1) th : f

(x) c 2 nghim phn bit x


1
< x
2
. g

(x) c 2 nghim phn bit x


3
< x
4
.
Theo bi ra ta cn c :

x
3
< x
1
< x
4
< x
2
x
1
< x
3
< x
4
< x
2

(x
3
) .f

(x
4
) < 0 (2)
g

(x
1
) .g

(x
2
) < 0 (3)
Theo nh l Viet ta c:

x
3
+x
4
= 2
x
3
.x
4
= 3m
v

x
1
+x
2
= 1
x
1
.x
2
= m
V x
1
+x
2
< x
3
+x
4
(3) khng xy ra.
f

(x
3
) = x
2
3
x
3
+m = x
2
3
+ 2x
3
+ 3m(3x
3
+ 2m) = (3x
3
+ 2m)
f

(x
4
) = (3x
4
+ 2m)
(2) (3x
3
+ 2m) (3x
4
+ 2m) < 0 9x
3
x
4
+ 6m(x
3
+x
4
) + 4m
2
< 0
9.3m+ 6m. (2) + 4m
2
< 0 4m
2
+ 15m < 0
15
4
< m < 0
Kt lun: Kt hp vi iu kin (1) ta c
15
4
< m < 0
V d 9.2.6: Cho hm s : y =
x
2
+mx
1 x
.
Tm m khong cch gia 2 im cc tr bng 10.
Ngi son : Th.s Minh Tun Trang 164 Khoa T nhin - Trng CSP Nam nh
T
h
.
s

M
i
n
h
T
u

n
9.2. Cc tr v tim cn ca hm s Chng 9. Hm s v th
Gii: Ta c : y

=
x
2
+ 2x +m
(1 x)
2
. Trc ht tm iu kin ng cong c 2 cc tr:
iu ny xy ra khi y

= 0 c 2 nghim phn bit khc 1.


t f(x) = x
2
+ 2x +m. iu kin ny tng ng vi

f (1) = 0

f
= 1 +m > 0

m + 1 = 0
m + 1 > 0
m > 1 (1)
Vi iu kin (1), gi s ng cong c 2 cc tr ti cc im x
1
, x
2
. Khi x
1
, x
2
l 2 nghim
ca phng trnh x
2
+ 2x +m = 0 (2)
Gi s M(x
1
, y
1
), N(x
2
, y
2
) l cc im cc tr.
Ta c y
1
=
2x
1
+m
1
= 2x
1
m. Tng t y
2
= 2x
2
m.
Theo nh l Viet ta c:

x
1
+x
2
= 2
x
1
.x
2
= m
Ta c MN = 10 MN
2
= 100 (x
1
x
2
)
2
+ (y
1
y
2
)
2
= 100
(x
1
x
2
)
2
+ (2x
1
m + 2x
2
+m)
2
= 100 (x
1
x
2
)
2
+ 4 (x
1
x
2
)
2
= 100
(x
1
x
2
)
2
= 20 (x
1
+x
2
)
2
4x
1
.x
2
= 20 4 4. (m) = 20 m = 4
Kt hp vi (1) ta c: m = 4 .
V d 9.2.7: Cho hm s y =
x
2
+ 2mx + 5
x 1
.
Tm m cc i, cc tiu ca hm s nm v 2 pha ca y = 2x.
Gii: Ta c y

=
x
2
+ 2x 2m + 5
(x
1
)
2
. t f(x) = x
2
+ 2x 2m + 5
Trc ht tm iu kin hm s c cc tr. Phng trnh y

= 0 c 2 nghim phn bit khc


1.

f (1) = 0

f
= 2m + 6 > 0

2m + 6 = 0
2m + 6 > 0
m < 3 (1)
Vi iu kin (1) hm s c 2 cc tr: M(x
1
, y
1
), N(x
2
, y
2
) vi x
1
, x
2
l 2 nghim ca phng
trnh : x
2
+ 2x 2m + 5 = 0 (2).
Theo nh l Viet ta c:

x
1
+x
2
= 2
x
1
.x
2
= 2m5
Ta c y
1
=
2x
1
+ 2m
1
= 2x
1
+ 2m, y
2
= 2x
2
+ 2m.
Vy M(x
1
, 2x
1
+ 2m), N(x
2
, 2x
2
+ 2m) l 2 im cc tr. M, N nm v 2 pha ca ng
thng 2x y = 0 nn ta c :
(2x
1
y
1
) . (2x
2
y
2
) < 0 (2x
1
+ 2x
1
2m) . (2x
2
+ 2x
2
2m) < 0
4. (2m5) 2m.2 +m
2
< 0 m
2
+ 4m20 < 0 2 2

6 < m < 2 + 2

6.
Kt hp iu kin ta c: 2 2

6 < m < 3 .
9.2.3 Cc bi ton tim cn
Tim cn l mt c trng ca hm phn thc, v l lp cc bi ton v tim cn i vi
hm phn thc kh a dng. Ta hy xt trc tin cc bi ton m t tnh cht cc tim cn.
V d 9.2.8: Cho hm s y =
2x + 1
x 2
(C). M l mt im ty trn (C).
Tip tuyn vi (C) ti M ct tim cn ngang v tim cn ng ti A v B.
Ngi son : Th.s Minh Tun Trang 165 Khoa T nhin - Trng CSP Nam nh
T
h
.
s

M
i
n
h
T
u

n
9.2. Cc tr v tim cn ca hm s Chng 9. Hm s v th
a) Chng minh rng M l trung im AB.
b) Chng minh rng khi M di ng trn (C) th tip tuyn to vi 2 ng tim cn ngang v
ng mt tam gic c din tch khng i.
c) Chng minh rng khng c tip tuyn no ca (C) li i qua giao im ca 2 ng tim
cn.
Gii: a) D thy (C) c 2 ng tim cn x = 2 v y = 2.
y
x
-3 -2 -1 0 1 2 3 4 5 6 7
-3
-2
-1
0
1
2
3
4
5
6
7
y =
2x + 1
x 2
I
M
B
A
y

=
5
(x
0
2)
2
.
M
0
(x
0
,
2x
0
+ 1
x
0
2
) (C). Tip tuyn ti M
0
c dng:
y = y

(x
0
) (x x
0
) +y (x
0
) y =
5
(x
0
2)
2
(x x
0
) +
2x
0
+ 1
x
0
2
Ta im A l nghim ca h :

y =
5
(x
0
2)
2
(x x
0
) +
2x
0
+ 1
x
0
2
y = 2

x = 2x
0
2
y = 2
A(2x
0
2; 2)
Ta im B l nghim ca h:

y =
5
(x
0
2)
2
(x x
0
) +
2x
0
+ 1
x
0
2
x = 2

x = 2
y =
2x
0
+ 6
x
0
2
B

2;
2x
0
+ 6
x
0
2

Ta c :
x
A
+x
B
2
=
2x
0
2 + 2
2
= x
0
= x
M
y
A
+y
B
2
=
2 +
2x
0
+ 6
x
0
2
2
=
2x
0
+ 1
x
0
2
= y
M
M l trung im AB.
b) Gi I l giao im 2 tim cn. iu ny dn n I (2; 2)
Do tam gic IAB vung ti I nn ta c : S
IAB
=
1
2
IA.IB
Ngi son : Th.s Minh Tun Trang 166 Khoa T nhin - Trng CSP Nam nh
T
h
.
s

M
i
n
h
T
u

n
9.2. Cc tr v tim cn ca hm s Chng 9. Hm s v th
T trn ta c: IA = [2x
0
4[ , IB =

2x
0
+ 6
x
0
2
2

=
10
[x
0
2[
S
IAB
=
1
2
. [2x
0
4[ .
10
[x
0
2[
= 10 khng i.
c) Gi s d l tip tuyn qua I ca (C) gi M
0
(x
0
,
2x
0
+ 1
x
0
2
) l tip im.
Do phng trnh d c dng : y =
5
(x
0
2)
2
(x x
0
) +
2x
0
+ 1
x
0
2
I d nn ta c: 2 =
5
(x
0
2)
2
(2 x
0
) +
2x
0
+ 1
x
0
2
2 =
5
x
0
2
+
2x
0
+ 1
x
0
2
2x
0
4 = 5 + 2x
0
+ 1 4 = 6 ><
Vy khng tn ti tip tuyn i qua giao im 2 ng tim cn.
Nhn xt: Hm phn thc quen thuc y =
ax +b
px +q
v y =
ax
2
+bx +c
px +q
cng c tnh cht
nh trn. Cch chng minh cng ging nh cch chng minh v d trn.
V d 9.2.9: Cho ng cong y =
x
2
+ 3x 1
x 2
(C). Chng minh rng:
Tch khong cch t mt im M bt k trn (C) n 2 tim cn (C) l mt hng s.
Gii: D thy: lim
x2
+
x
2
+ 3x 1
x 2
= +, lim
x2
x
2
+ 3x 1
x 2
=
1
: x 2 = 0 l tim
cn ng ca hm s.
y = x + 5 +
9
x 2
lim
x
(y (x + 5)) = lim
x
9
x 2
= 0

2
: x y + 5 = 0 l tim cn xin ca hm s.
Ly M

x
0
; x
0
+ 5 +
9
x
0
2

(C) bt k. Ta c:
d (M,
1
) = [x
0
2[
d (M,
2
) =

x
0

x
0
+ 5 +
9
x
0
2

+ 5

1
2
+ (1)
2
=
9

2. [x
0
2[
d (M,
1
) .d (M,
2
) =
9

2
(pcm)
V d 9.2.10: Cho y =
x
2
x + 1
x 1
.
Tm M (C) sao cho khong cch t M n giao im I ca 2 ng tim cn l b nht.
Gii: y = x +
1
x 1
Bng php tnh tng t nh trn, d dng thy rng (C) nhn x 1 = 0 l tim cn ng v
x y = 0 l tim cn xin.
Ngi son : Th.s Minh Tun Trang 167 Khoa T nhin - Trng CSP Nam nh
T
h
.
s

M
i
n
h
T
u

n
9.2. Cc tr v tim cn ca hm s Chng 9. Hm s v th

x 1 = 0
x y = 0

x = 1
y = 1
I (1; 1)
Ly M

x
0
; x
0
+
1
x
0
1

(C)
MI =

(x
0
1)
2
+

x
0
+
1
x
0
1
1

2
=

2 (x
0
1)
2
+
1
(x
0
1)
2
+ 2
MI

2 (x
0
1)
2
.
1
(x
0
1)
2
+ 2 =

2 + 2 (1)
MI
min
=

2 + 2 2 (x
0
1)
2
=
1
(x
0
1)
2
(x
0
1)
4
=
1
2

x
0
= 1 +
1
4

2
y
0
= 1 +
4

2 +
1
4

2
x
0
= 1
1
4

2
y
0
= 1
4

2
1
4

2
M
1

1 +
1
4

2
; 1 +
4

2 +
1
4

, M
2

1
1
4

2
; 1
4

2
1
4

l ta 2 im cn tm.
9.2.4 Cng c kin thc
V d 9.2.11 (H B- 2002): Cho hm s y = mx
4
+ (m
2
9)x
2
+ 10.
Tm m hm s c 3 im cc tr.
Gii: Ta c y

= 4mx
3
+ 2(m
2
9)x.
+) Nu m = 0, th y

= 18x. Ta c bng bin thin:


x 0 +
f

(x) + 0
f(x)

10

Do hm s ch c 1 im cc tr (Khng tha mn).


+) Nu m = 0, hm s c 3 cc tr khi v ch khi y

= 0 c 3 nghim phn bit


4mx
3
+ 2(m
2
9)x = 0 (1) c 3 nghim phn bit.
(1) 2x[2mx
2
+m
2
9] = 0

x = 0
2mx
2
+m
2
9 = 0 (2)
t f (x) = 2mx
2
+m
2
9
(1) c 3 nghim khi v ch khi (2) c 2 nghim phn bit khc 0

f (0) = 0

f
> 0

m
2
9 = 0
4.2m. (m
2
9) > 0
m(m
2
9) < 0
m(m3) (m+ 3) < 0
m 3 0 3 +
m.(m
2
9) 0 + 0 0 +
Ngi son : Th.s Minh Tun Trang 168 Khoa T nhin - Trng CSP Nam nh
T
h
.
s

M
i
n
h
T
u

n
9.2. Cc tr v tim cn ca hm s Chng 9. Hm s v th
m (; 3) (0; 3)
Kt lun: hm s c 3 im cc tr khi v ch khi: m (; 3) (0; 3)
V d 9.2.12 (H - A - 2005): Cho ng cong y = mx +
1
x
.
Tm m ng cong c cc tr v khong cch t im cc tiu n tim cn xin bng
1

2
.
Gii: Ta c y

= m
1
x
2
trc ht ng cong c gi tr cc tr th y

= 0 c 2 nghim phn
bit. iu ny c khi v ch khi m > 0.
V bng bin thin ta thy im cc tiu l M

m
; 2

.
x
1

m
0
1

m
+
y

+ 0 0 +
y

+
2

m
+
Mt khc, tim cn xin : y = mx hay mx y = 0.
d (M, ) =

m.
1

m
2

m
2
+ (1)
2
=

m
2
+ 1
Theo gi thit ta c: d (M, ) =
1

m
2
+ 1
=
1

2
m
2
+ 1 = 2m
m
2
2m+ 1 = 0 m = 1 (Tha mn iu kin m > 0)
V d 9.2.13 (H - B - 2005): Xt ng cong y =
x
2
+ (m+ 1)x +m + 1
x + 1
(C
m
).
Chng minh rng vi m bt k, th (C
m
) lun c 2 im cc i, cc tiu v khong cch
gia 2 im bng

20.
Gii: Ta c y

=
[2x + (m+ 1)] . (x + 1) [x
2
+ (m + 1)x +m+ 1] .1
(x + 1)
2
=
x
2
+ 2x
(x + 1)
2
y

= 0

x = 0 y = m+ 1
x = 2 y = m3
. T ta c bng bin thin:
x 2 1 0 +
f

(x) + 0 0 +
f(x)

m3

+
m+ 1
+
Vy ta thy 2 im cc tr l M(0, m+ 1), v N(2, m3).
MN =

(2 0)
2
+ (m3 m1)
2
=

20 (pcm)
Ngi son : Th.s Minh Tun Trang 169 Khoa T nhin - Trng CSP Nam nh
T
h
.
s

M
i
n
h
T
u

n
9.3. Gi tr ln nht, nh nht ca hm s Chng 9. Hm s v th
9.3 Gi tr ln nht, nh nht ca hm s
9.3.1 Kin thc c bn
nh ngha 9.5. Gi s f(x) l hm s xc nh trn min D. M gi l gi tr ln nht ca
hm f(x) trn tp D (K hiu l M = max
D
f(x)) khi:

f (x) M
x
0
D : f (x
0
) = M
Tng t ta c khi nim gi tr nh nht:
min
D
f (x) = m

f (x) m
x
0
D : f (x
0
) = m
Ch :Hc sinh thng ch ch n iu kin th nht m qun i iu kin th 2.
V d 9.3.1: Tm gi tr nh nht ca hm f(x) = x +
2
x
vi x 2.
Gii: Nu hc sinh p dng bt ng thc Cauchy gii quyt bi ton nh sau:
Do x +
2
x
2

x.
2
x
= 2

2
sau kt lun l min f(x) = 2

2 l sai.
V sao li vy? Bi v hc sinh khng ch n du bng ca bt ng thc. Bt ng thc xy
ra du bng khi x =
2
x
x =

2 < 2. iu ny mu thun.
Li gii ng phi nh sau:
x +
2
x
=
x
2
+

x
2
+
2
x

x
2
+ 2

x
2
.
2
x
=
x
2
+ 2
2
2
+ 2 = 3
min
[2;+)
f (x) = 3 x = 2
Ch : Cc bn cn phn bit gi tr ln nht trn min D v gi tr cc i trn min
D. Cng nh phn bit gi tr b nht vi gi tr cc tiu. C nhng trng hp chng
trng nhau nhng cng c nhng trng hp chng khc nhau.
V d 9.3.2: Tm gi tr ln nht, gi tr nh nht, cc i, cc tiu ca hm s
f(x) = x
3
3x
2
trn min D = [2; 4].
Gii: y

= 3x
2
6x, y

= 0

x = 0 y = 0
x = 2 y = 4
Ta c bng bin thin:
x 2 0 2 4
f

(x) + 0 0 +
f(x)
20
0
4
16
Ngi son : Th.s Minh Tun Trang 170 Khoa T nhin - Trng CSP Nam nh
T
h
.
s

M
i
n
h
T
u

n
9.3. Gi tr ln nht, nh nht ca hm s Chng 9. Hm s v th
Da vo bng bin thin ta c:
max
D
f (x) = 12 x = 4
min
D
f (x) = 20 x = 2
y
CT
= 4 x = 2
y
C
= 0 x = 0
Hin nhin ta thy: y
CT
= min
D
f (x) , y
C
= max
D
f (x)
z Nu lm mt php so snh cc i vi gi tr ln nht th cc i ging nh "X m thng
cht lm vua". N ch mang tnh cc b a phng. Cn gi tr ln nht th mang tnh
ton cc.
S dng o hm tm gi tr ln nht, nh nht ca hm s
o hm l cng c duy nht tm cc i, cc tiu.
tm gi tr ln nht v gi tr nh nht ca mt hm s f(x) trn min D ta c th s
dng o hm v kt hp vi vic so snh gi tr cc i, cc tiu vi cc gi tr c bit
(ta gi l cc gi tr ti hn).
Gi tr ti hn ny thng l gi tr ti u mt ca cc on (m trn cn tm gi tr
ln nht, nh nht ca mt hm s) hoc l gi tr ca hm s ti nhng im m khng
tn ti o hm.
Lc chung ca phng php o hm tm gi tr ln nht, nh nht ca hm s
f(x) trn min D cho trc nh sau:
0 Tm o hm f

(x) v t tm cc i, cc tiu ca f(x) (d nhin l ta ch quan


tm n cc i, cc tiu thuc min D).
O So snh gi tr cc i, cc tiu vi cc gi tr ti hn trn min D.
O T suy ra kt lun cn tm.
9.3.2 Cc bi ton n thun
.
V d 9.3.3: Cho x +y = 1, x 0, y 0. Tm gi tr ln nht, nh nht ca biu thc.
P = 3
2x
+ 3
y
Gii: T x +y = 1 nn y = 1 x. Thay vo P ta c:
P = 3
2x
+ 3
1x
= 3
2x
+
3
3
x
. t t = 3
x
.
Do x +y = 1, x 0, y 0 nn 0 x 1 do 1 t 3.
Khi P = f(t) = t
2
+
3
t
vi 1 t 3.
f

(t) = 2t
3
t
2
=
2t
2
3
t
2
. T ta c bng bin thin:
Ngi son : Th.s Minh Tun Trang 171 Khoa T nhin - Trng CSP Nam nh
T
h
.
s

M
i
n
h
T
u

n
9.3. Gi tr ln nht, nh nht ca hm s Chng 9. Hm s v th
t 1
3

12
2
3
f

(t) 0 +
f(t)
4
3
2
3

18
10
T bng bin thin ta c:
min
1t3
f (t) = min

4,
3
2
.
3

18, 10

=
3
2
.
3

18 t =
3

12
2
x = log
3

12
2

y = 1 log
3

12
2

max
1t3
f (t) = max

4,
3
2
.
3

18, 10

= 10 t = 3 x = 1 y = 0
Nhn xt: Ngi ta thng hay dng phng php i bin trong qu trnh tm gi tr
min, max a v mt bi ton mi c cu trc n gin hn. Ch lu l phi i min
xc nh ca bi ton. Chng hn nh bi ton trn: min xc inh c l 0 x 1, cn
min xc nh mi l 1 t 3.
V d 9.3.4: Cho hm s y = sin
2x
1 +x
2
+ cos
4x
1 +x
2
+ 1 vi x R.
Tm gi tr min, max ca hm s trn R.
Gii: Ta c f(x) = 2 sin
2
2x
1 +x
2
+ sin
2x
1 +x
2
+ 2 . t t =
2x
1 +x
2
.
Vi mi x R ta c: 1
2x
1 +x
2
1 do sin 1 t sin 1.
Bi ton a v tm gi tr min, max ca hm s : g(t) = 2t
2
+t + 2 vi sin 1 t sin 1
Ta c g

(t) = 4t + 1. g

(t) = 0 t =
1
4
. Ta c bng bin thin:
t sin1
1
4
sin1
g

(t) + 0
g(t)
g(sin1)
17
8
g(sin 1)
Do min
sin 1tsin 1
g (t) = min

g (sin 1) ,
17
8
, g (sin 1)

= g (sin 1) = 2 sin
2
1 sin 1 + 2
t = sin 1
2x
1 +x
2
= 1 x = 1
max
sin1tsin 1
g (t) = max

g (sin 1) ,
17
8
, g (sin 1)

=
17
8
t =
1
4

2x
1 +x
2
= arcsin
1
4
x
2

2
arcsin
1
4
x + 1 = 0
Ngi son : Th.s Minh Tun Trang 172 Khoa T nhin - Trng CSP Nam nh
T
h
.
s

M
i
n
h
T
u

n
9.3. Gi tr ln nht, nh nht ca hm s Chng 9. Hm s v th
t k =
1
arcsin
1
4
x
2
2k.x + 1 = 0

x = k +

k
2
+ 1
x = k

k
2
+ 1
9.3.3 Bi ton gi tr ln nht, nh nht cha tham s
Trong cc bi ton ny, gi tr min, max ca mt hm s f(x) trn mt min D s phi
ph thuc vo tham s m. Khi m bin thin, ni chung cc gi tr ny cng thay i. Cn
nhn mnh rng phng php dng o hm t ra c hiu lc r rt vi loi bi ton ny.
Cc loi bi ton chnh thng gp:
> Tm gi tr min, max ca hm s f(x) trn min D theo tham s m.
> Xt bi ton khc sau khi tm xong gi tr min, max.
Chng ta hy xt cc v d sau:
V d 9.3.5: Cho hm s y = sin
4
x + cos
4
x +msin xcos x vi x R.
Tm gi tr min, max ca hm s v bin lun theo m.
Gii: Ta c y = 1
1
2
sin
2
2x +
m
2
sin 2x.
t t = sin 2x. Bi ton quy v: Tm gi tr min, max ca hm s:
f(t) =
1
2
t
2
+
m
2
t + 1 vi 1 t 1.
f

(t) = t +
m
2
. Ta c f

(t) = 0 t =
m
2
. Ta xt cc trng hp sau:
a) Nu m 2 khi
m
2
1. Ta c bng bin thin sau:
t 1 1
m
2
f

(t) +
f(t)
1m
2
m+1
2
T ta c: max
1t1
f (t) = f (1) =
m + 1
2
b) Vi m 2 khi
m
2
1. Ta c bng bin thin sau
t
m
2
1 1
f

(t)
f(t)
1m
2
m+1
2
Ngi son : Th.s Minh Tun Trang 173 Khoa T nhin - Trng CSP Nam nh
T
h
.
s

M
i
n
h
T
u

n
9.3. Gi tr ln nht, nh nht ca hm s Chng 9. Hm s v th
max
1t1
f (t) = f (1) =
1 m
2
min
1t1
f (t) = f (1) =
m+ 1
2
c) Nu 2 < m < 2 hay 1 <
m
2
< 1. Khi ta c bng bin thin sau:
t 1
m
2
1
f

(t) + 0
f(t)
1m
2
m
2
+8
8
m+1
2
Khi max
1t1
f (t) = f

m
2

=
m
2
+ 8
8
min
1t1
f (t) = min f (1) , f (1) = min

1 m
2
,
1 +m
2

=
1 +[m[
2
V d 9.3.6: Cho hm s f(x) = 4x
2
4ax+a
2
2a khi 2 x 0. Tm a min
2x0
f (x) = 2.
Gii: Ta c f

(x) = 8x 4a. Nn f

(x) = 0 khi x =
a
2
. Ta xt cc trng hp sau:
a) Nu a > 0 tc l
a
2
> 0. Ta c bng bin thin sau:
x 2 0
a
2
f

(x)
f(x)
a
2
+ 6a + 16
a
2
2a
Ta c min
2x0
f (x) = f (0) = a
2
2a.
a
2
2a = 2 a
2
2a 2 = 0

a = 1 +

Tha mn

a = 1

3 (Loi)
b) Nu a < 4. Tc
a
2
< 2. Ta c bng bin thin sau:
x
a
2
2 0
f

(x)
f(x)
a
2
+ 6a + 16
a
2
2a
Ngi son : Th.s Minh Tun Trang 174 Khoa T nhin - Trng CSP Nam nh
T
h
.
s

M
i
n
h
T
u

n
9.3. Gi tr ln nht, nh nht ca hm s Chng 9. Hm s v th
Vy min
2x0
f (x) = f (2) = a
2
+ 6a + 16.
a
2
+ 6a + 16 = 2 a
2
+ 6a + 14 = 0

V nghim

c) Nu 4 a 0 hay 2
a
2
0. Ta c bng bin thin:
x 2
a
2
0
f

(x) + 0
f(x)
a
2
+ 6a + 16
2a
a
2
+ 6a + 16
T ta c: min
2x0
f (x) = f

a
2

= 2a
2a = 2 a = 1

Tha mn

Kt lun: a = 1 +

3 , a = 1 tha mn iu kin bi ton.


9.3.4 Phng php min gi tr ca hm s
Xt bi ton tm gi tr ln nht, nh nht ca hm s f(x) ... ? Mt min D cho ... ? Gi y
0
l mt gi tr ty ca f(x) trn D, th h sau y ca x:

f (x) = y
0
x D
c nghim. Ty dng
ca h trn m ta c cc iu kin c nghim tng ng. Trong nhiu trng hp, iu kin y
(sau khi c bin i) a c v dng a y
0
b. V y
0
l mt gi tr bt k ca f(x) nn
ta c min
D
f (x) = a, max
D
f (x) = b. Nh vy khi s dng phng php ny tm gi tr ln
nht, nh nht ca hm s quy v vic tm iu kin mt phng trnh c nghim.
Ta xt thm cc v d sau:
V d 9.3.7: Tm gi tr ln nht, nh nht ca biu thc :
f (x) =
2 sin x + cos x + 1
sin x 2 cos x + 3
vi x R
.
Gii: Ta c sin x2 cos x+3 =

5. sin (x +) +3 3

5 > 0 nn hm s xc nh trn R.
t y =
2 sin x + cos x + 1
sin x 2 cos x + 3
(2 y) sin x + (1 + 2y) cos x = 3y 1
Do phng trnh trn c nghim nn:
(2 y)
2
+ (1 + 2y)
2
(3y 1)
2
4 4y +y
2
+ 1 + 4y + 4y
2
9y
2
6y + 1
4y
2
6y 4 0 2y
2
3y 2 0
1
2
y 2
Do ta c:
y
min
=
1
2

5
2
sin x =
5
2
sin x = 1 x =

2
+k2
y
max
= 2 5 cos x = 5 cos x = 1 x = k2
Ngi son : Th.s Minh Tun Trang 175 Khoa T nhin - Trng CSP Nam nh
T
h
.
s

M
i
n
h
T
u

n
9.3. Gi tr ln nht, nh nht ca hm s Chng 9. Hm s v th
V d 9.3.8: Tm gi tr ln nht v nh nht ca hm s: f(x) =
2x
2
+ 7x + 23
x
2
+ 2x + 10
, x R.
Gii: V x
2
+ 2x + 10 = (x + 1)
2
+ 9 9 > 0 nn hm s xc nh trn ton b R.
y =
2x
2
+ 7x + 23
x
2
+ 2x + 10
(2 y) x
2
+ (7 2y) x + 23 10y = 0 +) Nu 2 y = 0 y = 2
3x + 3 = 0 x = 1
+ Nu 2 y = 0: = (7 2y)
2
4 (2 y) (23 10y) 0
49 28y + 4y
2
184 + 172y 40y
2
0 36y
2
+ 144y 135 0
4y
2
16y + 15 0
3
2
y
5
2
y
min
=
3
2
x =
2y 7
2 (2 y)
= 4
y
max
=
5
2
x =
2y 7
2 (2 y)
= 2
V d 9.3.9: Tm gi tr ln nht, nh nht ca biu thc P = x
2
+y
2
trn min
D =

x
2
y
2
+ 1

2
+ 4x
2
y
2
x
2
y
2
= 0

Gii: Xt h

x
2
+y
2
= P (1)
(x
2
y
2
+ 1)
2
+ 4x
2
y
2
x
2
y
2
= 0 (2)

x
2
+y
2
= P
(x
2
+y
2
)
2
+ 2 (x
2
y
2
) + 1 x
2
y
2
= 0

x
2
+y
2
= P
(x
2
+y
2
)
2
3 (x
2
+y
2
) + 1 + 4x
2
= 0

x
2
+y
2
= P (3)
P
2
3P + 1 + 4x
2
= 0 (4)
T (4) x
2
=
P
2
3P + 1
4
P
2
3P + 1 0
3

5
2
P
3 +

5
2
(5)
T (3) y
2
= P x
2
= P +
P
2
3P + 1
4
=
P
2
+P + 1
4
> 0. Phng trnh ny lun c nghim.
Vy (5) l iu kin cn v h c nghim.
Vy ta c: P
min
=
3

5
2

x = 0
y =
3

5
2
P
max
=
3 +

5
2

x = 0
y =
3 +

5
2
V d 9.3.10: Tm gi tr ln nht, nh nht ca P = x
2
xy 3y
2
, trn min
D =

(x, y)

x
2
+xy +y
2
3

Gii: Xt h

x
2
+xy +y
2
3
x
2
xy 3y
2
= P
Xt phng trnh : 3. (x
2
xy 3y
2
) = t (x
2
+xy +y
2
)
(t 3) x
2
+ (t + 3) xy + (t + 9) y
2
= 0
= 0 (t + 3)
2
4 (t + 9) (t 3) = 0
Ngi son : Th.s Minh Tun Trang 176 Khoa T nhin - Trng CSP Nam nh
T
h
.
s

M
i
n
h
T
u

n
9.3. Gi tr ln nht, nh nht ca hm s Chng 9. Hm s v th
3.t
2
18t + 117 = 0

t = 3 4

3
t = 3 + 4

3
+) Vi t = 3 4

3: Ta c
3. (x
2
xy 3y
2
)

3 4

(x
2
+xy +y
2
) =

6 + 4

.x
2
+ 4

3.xy +

3 6

y
2
=

6 + 4

3

x
2
+ 2

xy +

7 4

y
2

6 + 4

x +

2
0
3P

3 + 4

(x
2
+xy +y
2
)
V x
2
+xy +y
2
3

3 + 4

(x
2
+xy +y
2
) 3

3 + 4

3P 3

3 + 4

P 3 4

3
Vy P
min
= 3 4

x =

3 2

y
x
2
+xy +y
2
= 3

x =

3 2

6 3

y
2
= 3

x =

3 2

y
y
2
= 2 +

x =

6
2
y =

6 +

2
2
+) Vi t = 3 + 4

3: Xt
3. (x
2
xy 3y
2
)

3 + 4

(x
2
+xy +y
2
) =

6 4

.x
2
4

3.xy +

3 6

y
2
=

6 4

3

x
2
+ 2

2 +

xy +

7 + 4

y
2

6 4

x +

2 +

2
0
3P

3 + 4

(x
2
+xy +y
2
)
Ta c x
2
+xy +y
2
3

3 + 4

(x
2
+xy +y
2
) 3

3 + 4

3P 3

3 + 4

P 3 + 4

3
Vy P
max
= 3 + 4

x =

3 2

y
x
2
+xy +y
2
= 3

x =

3 2

6 + 3

y
2
= 3

x =

3 2

y
y
2
= 2

x =

6 +

2
2
y =

2
2
9.3.5 Phng php chiu bin thin
Phng php ny s dng kt hp vi vic s dng o hm kho st tnh ng bin
v nghch bin ca hm s, vi vic so snh cc gi tr c bit ca hm s (cc im cc
tr, cc im ti hn).
Xt cc th d minh ha sau.
V d 9.3.11: Tm gi tr nh nht ca P = x +y +z +
1
x
+
1
y
+
1
z
trn min
D =

(x, y)

x > 0, y > 0, z > 0, x +y +z


3
2

Gii: Theo bt ng thc Cauchy ta c:


(x +y +z) .

1
x
+
1
y
+
1
z

3
3

xyz.3
3

1
x
.
1
y
.
1
z
= 9
1
x
+
1
y
+
1
z

9
x +y +z
t t = x +y +z. Khi P f (t) = t +
9
t
Ngi son : Th.s Minh Tun Trang 177 Khoa T nhin - Trng CSP Nam nh
T
h
.
s

M
i
n
h
T
u

n
9.3. Gi tr ln nht, nh nht ca hm s Chng 9. Hm s v th
Xt hm f (t) = t +
9
t
vi 0 t
3
2
.
f

(t) = 1
9
t
2
, f

(t) = 0 t = 3. Ta c bng bin thin:


t 3 0
3
2
3
f

(t)
f(t)
+
15
2
T bng bin thin ta c: min
0<t
3
2
f (t) = f

3
2

=
15
2
t =
3
2

x +y +z =
3
2
x = y = z
x = y = z =
1
2
.
Ch :Nu vit P =

x +
1
x

y +
1
y

z +
1
z

6
P = 6 x = y = z = 1
x +y +z = 3 >
3
2
. iu ny v l.
V d 9.3.12: Tm gi tr ln nht v nh nht ca P =
x
y + 1
+
y
x + 1
trn min
D = (x, y) [x +y = 1, x 0, y 0
Gii: Ta c : P =
x
2
+x +y
2
+y
xy +x +y + 1
=
(x +y)
2
2xy +x +y
xy +x +y + 1
=
2 2xy
2 +xy
t t = xy. Ta c 0 xy
(x +y)
2
4
=
1
4
0 t
1
4
.
f

(t) =
6
(2 +t)
2
, nn ta c bng bin thin:
t 0
1
4
f

(t)
f(t)
1
2
3
T ta c: min
0t1/4
f (t) = f

1
4

=
2
3
t =
1
4
x = y =
1
2
max
0t1/4
f (t) = f (0) = 1 t = 0

x = 0, y = 1
x = 1, y = 0
V d 9.3.13: Tm gi tr ln nht, nh nht ca f(x) = x
6
+ 4(1 x
2
)
3
khi x [1; 1]
Ngi son : Th.s Minh Tun Trang 178 Khoa T nhin - Trng CSP Nam nh
T
h
.
s

M
i
n
h
T
u

n
9.3. Gi tr ln nht, nh nht ca hm s Chng 9. Hm s v th
Gii: t t = x
2
th 0 t 1.
Ta c g(t) = f(x) = t
3
+ 4(1 t)
3
= 3t
3
+ 12t
2
12t + 4.
min
1x1
f (x) = min
0t1
g (t) , max
1x1
f (x) = max
0t1
g (t)
Xt hm g(t) = 3t
3
+ 12t
2
12t + 4 trn on [0; 1].
g

(t) = 9t
2
+ 24t 12, g

(t) = 0 9t
2
+ 24t 12 = 0

t = 2
t =
2
3
Ta c bng bin thin:
t 0
2
3
1 2
g

(t) 0 +
g(t)
4
4
9
1
T ta c: max
1x1
f (x) = max
0t1
g (t) = g (0) = 4 t = 0 x = 0
min
1x1
f (x) = min
0t1
g (t) = g

2
3

=
4
9
t =
2
3
x =

6
3
9.3.6 Cng c kin thc
V d 9.3.14: Tm gi tr ln nht, nh nht ca hm s: f (x) =

1 + sin x +

1 + cos x.
Gii: Do f (x) 0 x R max
xR
f (x) =

max
xR
f
2
(x), min
xR
f (x) =

min
xR
f
2
(x)
f
2
(x) = 1 + sin x + 1 + cos x + 2

1 + sin x.

1 + cos x
= 2 + sin x + cos x + 2

1 + sin x + cos x + sin x. cos x


t t = sin x + cos x =

2 sin

x +

2 t

2
Ta c : t
2
= 1 + 2 sin xcos x sin xcos x =
t
2
1
2
g (t) = f
2
(x) = 2 +t + 2

1 +t +
t
2
1
2
= 2 +t + 2

t
2
+ 2t + 1
2
= 2 +t +

2 [t + 1[
g (t) =

2 +

2 +

1 +

t Nu 1 t

2
2

2 +

t Nu

2 t 1
g

(t) =

1 +

2 Nu 1 < t <

2
1

2 Nu

2 < t < 1
.
Ta c bng bin thin:
t

2
1

2
g

(t) +
g(t)
4 2

2
1
4 + 2

2
T ta c:
Ngi son : Th.s Minh Tun Trang 179 Khoa T nhin - Trng CSP Nam nh
T
h
.
s

M
i
n
h
T
u

n
9.4. Vit phng trnh tip tuyn th Chng 9. Hm s v th
max

2t

2
g (t) = g

= 4 + 2

2 t =

2 sin

x +

= 1
max
xR
f (x) =

4 + 2

2 x +

4
=

2
+k2 x =

4
+k2
min

2t

2
g (t) = g (1) = 1 t = 1 sin

x +

=
1

2
max
xR
f (x) = 1

x +

4
=

4
+k2
x +

4
=
5
4
+k2

x =

2
+k2
x = +k2
V d 9.3.15 (H - B - 2002): Tm gi tr ln nht ca hm s y = x +

4 x
2
.
Gii: Tp xc nh : 2 x 2.
Ta c f

(x) =

4 x
2
x

4 x
2
.
f

(x) = 0

4 x
2
= x

x > 0
4 x
2
= x
2
x =

2
Ta c bng bin thin:
x 2

2 2
f

(x) + 0
f(x)
2
2

2
2
Do ta c: min
2x2
f (x) = f (2) = 2 x = 2
max
2x2
f (x) = f

= 2

2 x =

2
9.4 Vit phng trnh tip tuyn th
9.4.1 Kin thc cn nh
Hiu cc cng thc c bn: a
tt
= f

(x
0
). x
0
l honh ca tip im. Phng trnh
tip tuyn:
y = f

(x
0
) (x x
0
) +f (x
0
)
Cn phn bit r 2 khi nim:
+) Tip tuyn ti im M nm trn ng cong. M l tip im.
+) Tip tuyn ca ng cong i qua im M. (cha chc M l tip im).
Ngi son : Th.s Minh Tun Trang 180 Khoa T nhin - Trng CSP Nam nh
T
h
.
s

M
i
n
h
T
u

n
9.4. Vit phng trnh tip tuyn th Chng 9. Hm s v th
9.4.2 Tip tuyn vi ng cong ti im M
V d 9.4.1: Vit phng trnh tip tuyn ti ng cong y = 2x
3
3x
2
. Bit rng tip tuyn
song song vi ng thng y = 12x + 1
Gii: Gi honh tip im l x
0
. Khi a
tt
= y

(x
0
) = 6x
2
0
6x
0
.
V tip tuyn song song vi ng thng y = 12x + 1 nn a
tt
= 12.
Vy ta c : 6x
2
0
6x
0
= 12 x
2
0
x
0
2 = 0

x
0
= 1 y
0
= 5
x
0
= 2 y
0
= 4
Vy phng trnh 2 tip tuyn l:
y = 12 (x + 1) 5 y = 12x + 7
y = 12 (x 2) + 4 y = 12x 20
Nhn xt: Trc ht tm tip im sau s dng cng thc vit phng trnh tip tuyn
ti M nm trn ng cong.
9.4.3 Tip tuyn vi ng cong i qua im M
Mnh 9.1. Cho 2 ng cong y = f(x) v y = g(x) tip xc nhau khi v ch khi h sau c
nghim:

f (x) = g (x)
f

(x) = g

(x)
v nghim ca h chnh l honh x
0
ca tip im.
V d 9.4.2: Cho ng cong y = 3x 4x
3
. Vit phng trnh tip tuyn bit rng tip tuyn
i qua im M(1; 3).
Gii: +) Nhn xt: im M(1; 3) khng nm trn ng cong cho v khi x = 1, y = 1 = 3.
Do nu ai my mc p dng cng thc : y = f

(x
0
)(x x
0
) +f(x
0
) y l sai.
+) gii bi ton: Ta gi tip tuyn cn tm l y = ax +b. V tip tuyn i qua im M(1; 3)
nn ta c 3 = a +b b = 3 a. Do tip tuyn cn tm c dng y = ax + 3 a.
Vy h sau phi c nghim:

3x 4x
3
= ax + 3 a
3 12x
2
= a
3x 4x
3
= x(3 12x
2
) + 3 (3 12x
2
)
3x 4x
3
= 3x 12x
3
+ 12x
2
8x
3
12x
2
= 0

x = 0 a = 3
x =
3
2
a = 24
+) Vi a = 3 y = 3x
+) Vi a = 24 y = 24x + 27
V d 9.4.3: Cho ng cong y = x
3
+ 2x
2
. Vit phng trnh tip tuyn vi ng cong, bit
rng tip tuyn i qua im M(1; 3).
Gii: Hy xem v bnh lun li gii sau:
V im M(1; 3) nm trn ng cong y = x
3
+ 2x
2
. Vy p dng cng thc v phng php
tip tuyn hc ta c: y y
0
= f

(x
0
).(x x
0
), y = 7(x 1) + 3 hay y = 7x 4.
Li gii trn l ng nu u bi l: Vit phng trnh tip tuyn vi ng cong ti
im M(1; 3)
Ngi son : Th.s Minh Tun Trang 181 Khoa T nhin - Trng CSP Nam nh
T
h
.
s

M
i
n
h
T
u

n
9.4. Vit phng trnh tip tuyn th Chng 9. Hm s v th
Tuy nhin li gii l cha ng vi yu cu bi ton (tip tuyn i qua im M). Li
gii trn thiu nghim l tip tuyn m tip im khng phi l M
Tip tuyn phi tm c dng y = ax +b, trong 3 = a +b ( do tip tuyn i qua M(1; 3)).
Vy y = ax + 3 a l phng trnh tip tuyn. H sau phi c nghim:

x
3
+ 2x
2
= ax + 3 a
3x
2
+ 4x = a
x
3
+ 2x
2
= x(3x
2
+ 4x) + 3 (3x
2
+ 4x)
x
3
+ 2x
2
= 3x
3
+ 4x
2
+ 3 3x
2
4x 2x
3
x
2
4x + 3 = 0
(x 1)
2
(2x + 3) = 0

x = 1 a = 7
x =
3
2
a =
3
4
+) Vi a = 7 y = 7x 4
+) Vi a =
3
4
y =
3
4
x +
9
4
Ta thy r hn iu ny vi hnh v:
y
x
-3 -2 -1 0 1
-2
-1
0
1
2
3
4
y = x
3
+ 2x
2
M(1; 3)
y
=
7
x

4
y
=
3
4
x
+
9
4
9.4.4 Lp cc bi ton v s tip xc rt a dng
C th lit k ra y cc bi ton thng dng nht:
0 Tm iu kin 2 ng cong tip xc vi nhau.
O Bi ton tip tuyn xut pht t mt im
O Bi ton v tip tuyn chung.
O Cc bi ton nh tnh v tip tuyn.
Xin a ra vi v d mu:
V d 9.4.4: Cho 2 ng cong y = x
2
5x + 6 v y = x
3
+ 3x 10. Vit phng trnh tip
tuyn chung.
Ngi son : Th.s Minh Tun Trang 182 Khoa T nhin - Trng CSP Nam nh
T
h
.
s

M
i
n
h
T
u

n
9.4. Vit phng trnh tip tuyn th Chng 9. Hm s v th
Gii: Gi y = ax +b l tip tuyn chung. Gi x
1
, x
2
l 2 tip im ta c h phng trnh:

ax
1
+b = x
2
1
5x
1
+ 6
a = 2x
1
5
ax
2
+b = x
3
2
+ 3x
2
10
a = 3x
2
2
+ 3
(1)
(2)
(3)
(4)
T (2) , (4) 2x
1
5 = 3x
2
2
+ 3 x
1
=
3x
2
2
+ 8
2
(5)
T (1) , (2) x
2
1
5x
1
+ 6 = x
1
(2x
1
5) +b b = 6 x
2
1
(6)
Thay (5) , (6) vo (2) , (3)
(3x
2
2
+ 3) .x
2
+ 6

3x
2
2
+ 8
2

2
= x
3
2
+ 3x
2
10
9
4
x
4
2
+ 2x
3
2
12.x
2
2
= 0

1
4
x
2
2
(9x
2
2
8x
2
+ 48) = 0

x
2
= 0
9x
2
2
8x
2
+ 48 = 0

V nghim

x
2
= 0 x
1
= 4 b = 10, a = 3
y = 3x 10 l phng trnh tip tuyn chung cn tm.
9.4.5 Cng c kin thc
V d 9.4.5 (H - B - 2004): Cho hm s y =
1
3
x
3
2x
2
+ 3x, (C).
Vit phng trnh tip tuyn vi (C) ti im un v chng minh l tip tuyn ca (C) c
h s gc b nht.
Gii: y

= x
2
4x + 3, y

= 2x 4. T y

= 0 ta c x = 2, y =
2
3
, y

= 1. Vy im un
A

2;
2
3

. Khi tip tuyn ti A l y = 1.(x 2) +


2
3
hay y = x +
8
3
. V ta c a
tt
= 1. H
s gc ti mt bt k ti im M c honh x
0
nm trn (C) l: k = y

(x
0
) = x
2
0
4x
0
+3. Ta
c k = (x
0
2)
2
1 1 = a
tt
. Do h s gc ca tip tuyn ca (C) c h s gc b nht.
V d 9.4.6 (H - D - nm 2005): Gi (C
m
) : y =
1
3
x
3

m
2
x
2
+
1
3
, m l tham s.
Gi M l im thuc (C
m
) c honh bng 1. Tm m tip tuyn ca (C
m
) ti im M
song song vi ng thng 5x y = 0.
Gii: y =
1
3
x
3

m
2
x
2
+
1
3
y

= x
2
mx
Ta c a
tt
= y

(1) = 1 +m. Phng trnh tip tuyn l:


y = (1 +m) (x + 1)
m
2
y = (m+ 1) x +
m + 2
2
Ta c 5x y = 0 y = 5x

m+ 1 = 5
m+ 2
2
= 0
m = 4
V d 9.4.7 (H - B - 2006): Cho hm s y =
x
2
+x 1
x + 2
(C).
Vit phng trnh tip tuyn ca (C), bit tip tuyn ny vung gc vi tim cn xin ca (C).
Ngi son : Th.s Minh Tun Trang 183 Khoa T nhin - Trng CSP Nam nh
T
h
.
s

M
i
n
h
T
u

n
9.5. Xc nh im tha mn iu kin cho trc Chng 9. Hm s v th
Gii: y = x 1 +
1
x + 2
lim
x
[y (x 1)] = lim
x
1
x + 2
= 0
y = x 1 l tim cn xin ca th (C). y

= 1
1
(x + 2)
2
=
x
2
+ 4x + 3
(x + 2)
2
V tip tuyn ca (C) vung gc vi tim cn xin y = x 1, nn a
tt
= 1.
Gi x
0
l honh tip im. T ta c:
x
2
0
+ 4x
0
+ 3
(x
0
+ 2)
2
= 1 x
2
0
+ 4x
0
+ 3 = x
2
0
4x
0
4 2x
2
0
+ 8x
0
+ 7 = 0

x
0
= 2 +

2
2
y
0
=
3

2
2
3 y = x + 2

2 5
x
0
= 2

2
2
y
0
=
3

2
2
3 y = x 2

2 5
9.5 Xc nh im tha mn iu kin cho trc
9.5.1 Kin thc c bn
nh l 11. Gi s P(x) = a
n
.x
n
+a
n1
x
n1
+ +a
2
x
2
+a
1
x +a
0
.
Nu P(x) c t nht n + 1 nghim th a
n
= a
n1
= = a
1
= a
0
= 0
H qu 9.1. Nu a
n
.x
n
+a
n1
x
n1
+ +a
2
x
2
+a
1
x +a
0
= 0 x R th
a
n
= a
n1
= = a
1
= a
0
= 0
V d 9.5.1: Cho h ng cong y = x
3
+ 2(m1)x
2
+ (m
2
4m+ 1)x 2(m
2
+ 1). Tm cc
im trn mt phng ta m h ng cong i qua vi mi m.
Gii: Gi (x
0
, y
0
) l im cn tm. Khi ta c:
y
0
= x
3
0
+ 2(m1)x
2
0
+ (m
2
4m+ 1)x
0
2(m
2
+ 1) m
(x
0
2) m
2
+ (2x
2
0
4x
0
) m+x
3
0
2x
2
0
+x
0
2 y
0
= 0 m

x
0
2 = 0
2x
2
0
4x
0
= 0
x
3
0
2x
2
0
+x
0
2 y
0
= 0

x
0
= 2
y
0
= 0
Vy h ng cong cho i qua mt im c nh l (2; 0).
Nhn xt:
0 Nh vy lc chung gii bi ton tm im c nh m h ng cong lun i qua
l: gi (x
0
, y
0
) l im m h ng cong y = f(x, m) lun i qua vi mi m. T h thc
y
0
= f(x
0
, m) vi mi m v h qu trn th ta c a
n
= a
n1
= = a
1
= a
0
= 0. T
ta c mt h phng trnh theo x
0
v y
0
. Gii h ny ta c im c nh cn tm.
O Ta nhn thy (2; 0) l im c nh trong v d trn. iu c ngha l : Phng trnh
x
3
+ 2(m1)x
2
+ (m
2
4x + 1)x 2(m
2
+ 1) = 0 vi mi m lun c 1 nghim x = 2. T
suy ra trong nhiu trng hp c th s dng vic tm im c nh nhm nghim
ca phng trnh bc cao c tham s.
Gi s ta phi bin lun (hoc gii) phng trnh bc cao c tham s sau: y = f(x, m) = 0. Nh
ta bit, nu bit trc mt nghim x
0
ca n, th ta c th h bc phng trnh khi mi
Ngi son : Th.s Minh Tun Trang 184 Khoa T nhin - Trng CSP Nam nh
T
h
.
s

M
i
n
h
T
u

n
9.5. Xc nh im tha mn iu kin cho trc Chng 9. Hm s v th
vic s n gin hn. Ta c th lm nh sau: Tm im c nh (x
0
, y
0
) ca h y = f(x, m) theo
cch trn. Nu nh tn ti im c nh (x
0
, 0) th x = x
0
chnh l mt nghim. D nhin khng
phi iu ny lc no cng c. Nu nh ta p dng vic tm im c nh ca mt h ng
cong ph thuc vo tham s nhm nghim phng trnh bc cao vi tham s.
V d 9.5.2 (Bi ton 3 im c nh thng hng): Chng minh rng cc h ng cong sau
vi mi m lun i qua 3 im c nh thng hng:
a) y = (m + 2)x
3
+ 2(m + 2)x
2
(m+ 3)x 2m+ 1.
b) y = (m + 1)x
3
(2m1)x m+ 1.
c) y = (m3)x
3
4(m3)x
2
(m+ 1)x +m.
Gii: a) Xt h ng cong y = (m+ 2)x
3
+ 2(m+ 2)x
2
(m+ 3)x 2m+ 1. Gi (x
0
, y
0
) l
im c nh cn tm. Khi ta c:
y
0
= (m+ 2)x
3
0
+ 2(m+ 2)x
2
0
(m+ 3)x
0
2m + 1 m
(x
3
0
+ 2x
2
0
x
0
2) m + (2x
3
0
+ 4x
2
0
3x
0
+ 1 y
0
) = 0 m

x
3
0
+ 2x
2
0
x
0
2 = 0
2x
3
0
+ 4x
2
0
3x
0
+ 1 y
0
= 0
(1)
(2)
(1) (x
0
+ 2) . (x
2
0
1) = 0

x
0
= 2 y
0
= 7
x
0
= 1 y
0
= 6
x
0
= 1 y
0
= 4
Vy ta c 3 im c nh l : A(2; 7) , B(1; 6) , C (1; 4)

AB = (1; 1) ,

AC = (3; 3)

AC = 3

AB
nn A, B, C thng hng.
Nhn xt: t (2) ta c
y
0
= 2 (x
3
0
+ 2x
2
0
x
0
2) x
0
+ 5
(1) y
0
= x
0
+ 5 A, B, C d : y = x + 5
D nhin vi ng cong cho th cch lm trc tip n gin hn (mi tnh ton u d
dng). Tuy nhin khng phi khi no cch lm trc tip cng sun s.
b) Xt h ng cong y = (m+ 1)x
3
(2m1)x m + 1.
Gi (x
0
, y
0
) l im c nh cn tm. Tnh ton nh trn dn n h phng trnh sau tm
x
0
, y
0
.

x
3
0
2x
0
1 = 0
y
0
x
3
0
+x
0
1 = 0
(1)
(2)
(2) y
0
= x
3
0
x
0
+ 1 = (x
3
0
2x
0
1) +x
0
+ 2 = x
0
+ 2
(1) (x
0
+ 1) (x
2
0
x
0
1) = 0 (1) c 3 nghim phn bit.
Nn 3 im u tha mn: y
0
= x
0
+ 2. Vy 3 im cng thuc ng thng y = x + 2.
Nhn xt: D nhin c th gii trc tip bng cch tm nghim ca (1)
Ngi son : Th.s Minh Tun Trang 185 Khoa T nhin - Trng CSP Nam nh
T
h
.
s

M
i
n
h
T
u

n
9.5. Xc nh im tha mn iu kin cho trc Chng 9. Hm s v th
(1) x
0
= 1, x
0
=
1

5
2
. Vic tm y
0
theo x
0
y phc tp hn v x
0
c nghim di
dng cn. Sau li phi dng cc php tnh v vct c 3 im c nh thng hng.
R rng gii trc tip c th, nhng chc chn phc tp hn hn cch ta trnh by trn.
c) Xt h ng cong y = (m3)x
3
4(m3)x
2
(m + 1)x +m.
Gi (x
0
, y
0
) l im c nh cn tm. Ta c h sau y xc nh x
0
, y
0
.

x
3
0
4x
2
0
x
0
+ 1 = 0
y
0
+ 3x
3
0
12x
2
0
+x
0
= 0
(1)
(2)
(2) y
0
= 3x
3
0
+ 12x
2
0
x
0
= 3 (x
3
0
4x
2
0
x
0
+ 1) 4x
0
+ 3 = 4x
0
+ 3
Ta chng minh (1) c 3 nghim phn bit.
Xt hm f (x) = x
3
4x
2
x + 1
f (1) = 3, f (0) = 1, f (1) = 3, f (5) = 21
f (1) .f (0) = 3 < 0 x
1
(1; 0) : f (x
1
) = 0
f (0) .f (1) = 3 < 0 x
2
(0; 1) : f (x
2
) = 0
f (1) .f (5) = 63 < 0 x
3
(1; 5) : f (x
3
) = 0
V x
1
< x
2
< x
3
nn phng trnh (1) c 3 nghim phn bit.
Vy 3 im c nh thng hng v chng u nm trn ng thng y = 4x + 3.
Nhn xt:
Trong th d ny phng php tm cc im c nh c th, ri chng minh chng thng hng
l hon ton khng th lm c (mc d bit (1) c 3 nghim phn bit nhng lm th no
tm 3 nghim ). Nhn trn hnh v thy r iu ny:
y
x
-1 0 1 2 3 4
-11
-10
-9
-8
-7
-6
-5
-4
-3
-2
-1
0
1
2
y = x
3
4x
2
x + 1
9.5.2 Tm im khng thuc mi ng cong trong h y = f(x, m)
Lc chung gii chng nh sau:
0 Gi (x
0
, y
0
) l im cn tm, th phng trnh sau y (n m): y
0
= f(x
0
, m) (1) v
nghim.
Ngi son : Th.s Minh Tun Trang 186 Khoa T nhin - Trng CSP Nam nh
T
h
.
s

M
i
n
h
T
u

n
9.5. Xc nh im tha mn iu kin cho trc Chng 9. Hm s v th
O Vy ta qui bi ton v vic tm iu kin phng trnh (1) (n m) v nghim. Ty dng
ca (1) m ta c cc iu kin v nghim tng ng. T cc iu kin ny s cho ta li
gii ca bi ton.
Lu vi cc bn lm sng t cc kt qu tm c, trong cc trng hp c th c, bn
hy biu din hnh hc cc kt qu tm c trn mt phng ta . lm c iu ny, bn
ch cn nm c cch biu din min trn mt phng ta t cc h thc cho trc. Ta hy
ln lt xt cc v d sau.
V d 9.5.3: Cho h ng cong y = mx
3
+ (1 m)x ph thuc tham s m.
Tm cc im trn mt phng ta sao cho khng c ng cong no ca h i qua.
Gii: Gi (x
0
, y
0
) l im phi tm. Khi phng trnh sau y (n m)
y
0
= mx
3
0
+ (1 m)x
0
(1) v nghim.
D thy (1) y
0
x
0
= m(x
3
0
x
0
) (2).
Ta c (2) v nghim khi v ch khi h sau tha mn:

x
3
0
x
0
= 0
y
0
x
0
= 0

x
0
= 0
x
0
= 1
y
0
= x
0
Vy cc im cn tm gm 3 ng thng x = 0, x = 1, x = 1 b i 3 im A(0; 0), B(1; 1),
C(1; 1).
V d 9.5.4: Cho h ng thng y = x
3
m
3
x
2
+ 2mx +m
2
1.
Tm cc im trn mt phng ta m h ng cong khng i qua vi mi m.
Gii: Gi (x
0
, y
0
) l im cn tm khi phng trnh sau (n m).
y
0
= x
3
0
m
3
x
2
0
+ 2mx
0
+m
2
1 (1) v nghim.
Ta vit li m
3
x
2
0
m
2
2mx
0
+y
0
+ 1 x
3
0
= 0 (2).
Nu x
0
= 0, th (2) l phng trnh bc 3. Ta bit rng mi phng trnh bc 3 u c t nht
mt nghim. V th (2) v nghim th x
0
= 0.
Vi x
0
= 0 th (2) tr thnh m
2
+y
0
+ 1 = 0 m
2
= y
0
+ 1 (3).
(3) v nghim y
0
+ 1 < 0 y
0
< 1. Vy tp cc im l na ng thng: x = 0 vi
y < 1.
V d 9.5.5: Cho h ng cong y = x
3
+ 2(m1)x
2
+ (m
2
4m+ 1)x 2(m
2
+ 1). Tm cc
im trn mt phng ta sao cho mi ng thng ca h u khng i qua im y.
Gii: Gi (x
0
, y
0
) l im cn tm. Phng trnh sau y (n m):
y
0
= x
3
0
+2(m1)x
2
0
+(m
2
4m+1)x
0
2(m
2
+1) (1) v nghim. Vit li (1) di dng sau:
(x
0
2)m
2
+ 2x
0
(x
0
2)m+x
3
0
2x
2
0
+x
0
2 y
0
= 0 (2).
Xt cc kh nng sau:
a) Nu x
0
= 2. Khi (2) y
0
= 0. Vy trong trng hp ny phng trnh (1) v nghim
khi:

x
0
= 2
y
0
= 0
b) Nu x
0
= 2. Khi (2) v nghim khi:

= x
2
0
. (x
0
2)
2
(x
3
0
2x
2
0
+x
0
2 y
0
) (x
0
2) < 0
(x
0
2) (x
3
0
2x
2
0
x
3
0
+ 2x
2
0
x
0
+ 2 +y
0
) < 0
Ngi son : Th.s Minh Tun Trang 187 Khoa T nhin - Trng CSP Nam nh
T
h
.
s

M
i
n
h
T
u

n
9.6. S tng giao Chng 9. Hm s v th
(x
0
2) (x
0
+y
0
+ 2) < 0

x
0
2 < 0
x
0
+y
0
+ 2 > 0

x
0
2 > 0
x
0
+y
0
+ 2 < 0
y
x
0 1 2 3 4
-2
-1
0
1
2
3
Tp cc im l 2 gc i nh nh hnh v.
9.6 S tng giao
9.6.1 Kin thc c bn
tm giao im ca 2 ng cong y = f(x), v y = g(x). Xt phng trnh honh
giao im : f(x) = g(x) (1).
Nhn chung (1) u l phng trnh bc cao (c bc 3). Nu c th bn nn tm cch h
bc ca (1). Ta lun s dng kt qu sau:
Nu x = a l mt nghim ca (1) th (1) c a v dng sau: (x a).H(x) = 0.
H(x) c bc gim i 1 so vi phng trnh gc. H(x) c th tm bng cch s dng lc
Hooc - ne.
Nu s dng cc kt qu v gi tr ln nht, nh nht ca hm bc 3, ta c kt qu thng
dng sau:
Xt phng trnh sau: f(x) = ax
3
+bx
2
+cx +d, (a = 0) (2).
Khi :
0 (2) c 3 nghim phn bit khi v ch khi f(x) c cc i, cc tiu v y
C
.y
CT
< 0.
O (2) c 2 nghim phn bit khi v ch khi f(x) c cc i, cc tiu v y
C
.y
CT
= 0.
O (2) c 1 nghim khi v ch khi:
+) Hoc l f(x) khng c cc i, cc tiu.
+) Hoc l f(x) c cc i, cc tiu v y
C
.y
CT
> 0
Cn nhn mnh rng vi bi ton ngoi vic i hi tnh giao nhau ca cc ng cong bc ba
vi mt ng cong khc c bc khng qu ba, ta cn quan tm n tnh cht ca cc giao im
th kt qu va dn ra trn ch c th xem nh mt iu kin cn. N cha sc mnh
gii quyt hon ton bi ton. gii quyt trn vn, ta cn s dng thm cc kin thc khc.
Ngi son : Th.s Minh Tun Trang 188 Khoa T nhin - Trng CSP Nam nh
T
h
.
s

M
i
n
h
T
u

n
9.6. S tng giao Chng 9. Hm s v th
9.6.2 S tng giao ca hm a thc vi trc Ox
V d 9.6.1: Cho h ng cong ph thuc tham s m:
y = x
3
3(m+ 1)x
2
+ 2(m
2
+ 4m + 1)x 4m(m+ 1)
Tm m ng cong ct trc honh ti 3 im phn bit c honh ln hn 1
Gii: ng cong ct trc honh ti 3 im phn bit c honh ln hn 1 khi v ch khi
phng trnh x
3
3(m+ 1)x
2
+ 2(m
2
+ 4m+ 1)x 4m(m+ 1) = 0 (1) c 3 nghim phn bit
ln hn 1.
Do x = 2 l nghim ca (1), nn (1) c th vit di dng sau:
(x 2) [x
2
(3m + 1) x + 2m(m + 1)] = 0 (1)
(2) c 3 nghim phn bit ln hn 1 th iu kin cn v l phng trnh
f (x) = x
2
(3m+ 1) x + 2m(m+ 1) = 0 c 2 nghim phn bit ln hn 1 v khc 2.
Theo nh l o v du ca tam thc bc 2 iu xy ra khi v ch khi:

> 0
a.f (1) > 0
S
2
> 1
f (2) = 0

m
2
2m+ 1 > 0
2m
2
m > 0
(3m+ 1) > 2
2m
2
4m+ 2 = 0

m = 1

m >
1
2
m < 0
m >
1
3
m = 1

m >
1
2
m = 1
Nhn xt:
~ nh l o v du ca tam thc bc 2 ni chung l cng c hu hiu gii cc bi ton
thuc loi ny.
~ Tuy nhin trong v d trn (2) c th vit di dng :
(x 2)(x 2m)(x m1) = 0 x = 2, x = 2m, x = m + 1
V th ta cn c:

2m > 1
2m = 2
m + 1 > 1
m + 1 = 2; 2m = m+ 1

m >
1
2
m = 1
m > 0
m = 1; m = 1

m >
1
2
m = 1
V d 9.6.2: Bin lun theo m s giao im ca vi trc honh ca ng cong :
y = x
3
3x
2
+ 3(1 m)x + 3m + 1
Gii: Ta c y

= 3x
2
6x + 3(1 m) = 3(x
2
2x + 1 m).
ng cong c cc tr khi phng trnh y

= 3(x
2
2x + 1 m) = 0 c 2 nghim phn bit

= 1 (1 m) = m > 0. (1)
Ta c nhn xt sau:
x
3
3x
2
+ 3(1 m)x + 3m+ 1 = (x
2
2x + 1 m)(x 1) + 2(mx +m+ 1)
Hay y =
y

3
.(x 1) + 2(mx +m + 1) (2)
Ngi son : Th.s Minh Tun Trang 189 Khoa T nhin - Trng CSP Nam nh
T
h
.
s

M
i
n
h
T
u

n
9.6. S tng giao Chng 9. Hm s v th
ng thc (2) chng t rng : Nu (x
1
, y
1
) v (x
2
, y
2
) l cc im cc tr ca hm s th:

y
1
= 2(mx
1
+ 1 +m)
y
2
= 2(mx
2
+ 1 +m)
By gi ta bin lun s giao im ca ng cong vi trc honh nh sau:
1) ng cong ct trc honh ti mt im duy nht khi:
a) Hoc l ng cong khng c cc i, cc tiu:

0 m 0.
b) Hoc l c cc i, cc tiu nhng y
1
.y
2
> 0. iu xy ra khi:

m > 0
y
1
.y
2
> 0

m > 0
m
2
x
1
x
2
m(m+ 1)(x
1
+x
2
) + (1 +m)
2
> 0
(3)
(4)
Do x
1
, x
2
l 2 nghim ca phng trnh x
2
2x + 1 m = 0 nn theo nh l Viet ta c:
x
1
+x
2
= 2, x
1
.x
2
= 1 m thay vo (4) ta c h:

m > 0
m
3
+ 1 > 0
0 < m < 1
Kt hp 2 trng hp ta c : ng cong ct trc honh ti im duy nht khi m < 1
2) ng cong ct trc honh ti 2 im phn bit khi v ch khi ng cong c 2 im cc tr
v y
1
.y
2
= 0. iu ny xy ra khi:

m > 0
y
1
.y
2
= 0

m > 0
m
3
+ 1 = 0
m = 1
3) Tng t ng cong ct trc honh ti 3 im phn bit khi m > 1.
V d 9.6.3: Cho ng cong y = x
3
3x
2
+ (2m 2)x + m 3. Tm m ng cong ct
trc honh ti 3 im phn bit c honh x
1
, x
2
, x
3
tha mn iu kin x
1
< 1 < x
2
< x
3
.
Gii: iu kin cn:
Gi s m l gi tr tha mn yu cu bi ton. Khi ta c:
f(x) = x
3
3x
2
+ (2m2)x +m3 = (x x
1
)(x x
2
)(x x
3
).
Ta c bng xt du:
x
x
1
x
2
x
3 +
f(x) 0 + 0 0 +
T gi thit : x
1
< 1 < x
2
< x
3
v bng xt du suy ra f(1) > 0.
m5 > 0 m < 5.
iu kin :
Gi s m < 5. Ta c:
f(1) = m5 > 0, f(0) = m3 < 0 (Do m < 5)
V lim
x
f (x) = b < 1 : f (b) < 0
lim
x+
f (x) = + a > 0 : f (a) > 0
Da vo tnh lin tc ca hm f(x) ta c :
f (b) .f (1) < 0 x
1
(b; 1) : f (x
1
) = 0
f (1) .f (0) < 0 x
2
(1; 0) : f (x
2
) = 0
f (0) .f (a) < 0 x
3
(0; a) : f (x
3
) = 0
x
1
< x
2
< x
3
: f (x
1
) = f (x
2
) = f (x
3
) = 0
Vy phng trnh c 3 nghim phn bit.
Ngi son : Th.s Minh Tun Trang 190 Khoa T nhin - Trng CSP Nam nh
T
h
.
s

M
i
n
h
T
u

n
9.6. S tng giao Chng 9. Hm s v th
Nhn xt:
Ba v d trn cho ta cc cch gii khc nhau, v cng chnh l cc cch thng gp nht:
> H bc phng trnh ri dng nh l o v du ca tam thc bc 2
> S dng mi lin h gia gi tr ln nht v gi tr nh nht ca hm s.
> S dng cc kin thc khc.
chnh l cc lc chung nht xt cc bi ton v im ct i vi cc ng cong a
thc bc 3.
V d 9.6.4: Cho ng cong y = x
3
3mx
2
+2m(m4)x +9m
2
m. Tm m ng cong
chn trn trc honh 2 on bng nhau.
Gii: iu kin cn:
Gi s ng cong chn trn trc honh 2 on bng nhau, tc l ng cong ct trc honh
ti 3 im phn bit A, B, C sao cho : BA = BC. Gi s x
1
, x
2
, x
3
tng ng l honh ca
A, B, C. Khi ta c: x
2
x
1
= x
3
x
2
x
3
+x
1
= 2x
2
x
1
+x
2
+x
3
= 3x
2
.
V x
1
, x
2
, x
3
l 3 nghim ca phng trnh x
3
3mx
2
+ 2m(m4)x + 9m
2
m = 0 nn ta c
x
1
+x
2
+x
3
= 3m x
2
= m. Do m l nghim ca (1) nn thay vo (1) ta c:
m
3
3m
3
+ 2m
2
(m4) + 9m
2
m = 0 m
2
m = 0

m = 0
m = 1
iu kin :
+) m = 0: ng cong tr thnh y = x
3
. R rng y = x
3
ch ct trc honh ti mt im nn
trng hp ny loi.
+) m = 1: ng cong tr thnh y = x
3
3x
2
6x + 8
y = 0 (x 1).(x
2
2x 8) = 0 x
1
= 1, x
2
= 2, x
3
= 4.
R rng x
2
x
1
= x
3
x
2
nn m = 1 l tham s cn tm.
9.6.3 S tng giao ca hm phn thc
Cc bi ton thuc dng ny thng c dng sau:
Tm iu kin ng cong (C) biu din hm phn thc v mt ng (C

) cho trc
ct nhau v honh cc giao im tha mn iu kin cho trc no .
Hy xt cc v d sau y:
V d 9.6.5: Chng minh rng ng cong y =
x
2
+ 2x
x + 1
v ng thng y = x 3 ct nhau
ti 2 im phn bit i xng nhau qua ng thng y = x.
Gii: Xt phng trnh
x
2
+ 2x
x + 1
= x 3
x
2
+ 2x = x
2
4x 3 2x
2
+ 6x + 3 = 0. (1)
R rng (1) c 2 nghim phn bit v

= 3 > 0.
Gi M
1
(x
1
, x
1
3), M
2
(x
2
, x
2
3) l 2 giao im ca 2 ng trn. ng thng qua M
1
M
2
c h s gc : k =
(x
2
3) (x
1
3)
x
2
x
1
= 1. V vy M
1
M
2
ng thng y = x.
Ngi son : Th.s Minh Tun Trang 191 Khoa T nhin - Trng CSP Nam nh
T
h
.
s

M
i
n
h
T
u

n
9.6. S tng giao Chng 9. Hm s v th
Gi I l trung im ca M
1
M
2
th ta (x
0
, y
0
) ca I l :

x
0
=
x
1
+ x
2
2
y
0
=
(x
1
3) + (x
2
3)
2
=
x
1
+x
2
2
3
Theo nh l Viet ta c x
1
+x
2
= 3 x
0
=
3
2
, y
0
=
3
2
I nm trn ng thng y = x.
Hay ni cch khc M, N i xng vi nhau qua ng thng y = x.
V d 9.6.6: Cho y =
x
2
+ 3
x + 1
(C). Vit phng trnh ng thng d i qua M

2;
2
5

sao cho
d ct (C) ti 2 im phn bit A, B sao cho M l trung im AB
Gii: H

x = 2
y =
x
2
+ 3
x + 1
ch c 1 mt nghim nn ng thng x = 2 khng th ct (C) ti 2
im phn bit. Vy phng trnh ng thng d c dng: y = k(x 2) +
2
5
. Trc ht ta tm k
d ct (C) ti 2 im phn bit. Mun vy xt phng trnh honh giao im:
x
2
+ 3
x + 1
= k(x 2) +
2
5
5(1 k)x
2
+ (5k 2)x + 10k + 13 = 0 (1)
(Do x = 1 khng l nghim ca (1))
(1) c 2 nghim phn bit x
1
, x
2
ta cn c : (5k 2)
2
20(1 k)(10k + 3) (2).
Khi 2 giao im ca d v (C) l I(x
1
, y
1
) v J(x
2
, y
2
). R rng M, I, J cng nm trn d nn
M l trung im I, J khi v ch khi:
2x
M
= x
1
+x
2
4 = x
1
+x
2
4 =
5k 2
5(k 1)
20k 20 = 5k 2
k =
6
5
tha mn iu kin (2). Vy k =
6
5
tha mn yu cu ca bi ra.
Xem hnh v di y:
y
x
-4 -3 -2 -1 0 1 2 3 4 5 6 7 8
-10
-9
-8
-7
-6
-5
-4
-3
-2
-1
0
1
2
3
4
5
6
7
8
y =
x
2
+3
x+1
M
I
J
y
=
6
5
(
x

2
)
+
2
5
V d 9.6.7: Cho y =
x
2
+x 1
x 1
(C). Tm m (C) ct y = x + m ti 2 im phn bit
A, B. Chng minh rng khi y: A, B thuc cng mt nhnh ca th (C)
Ngi son : Th.s Minh Tun Trang 192 Khoa T nhin - Trng CSP Nam nh
T
h
.
s

M
i
n
h
T
u

n
9.6. S tng giao Chng 9. Hm s v th
Gii: y = x +m ct (C) ti 2 im phn bit, iu kin l phng trnh:
x
2
+x 1
x 1
= x +m (1) c 2 nghim phn bit.
(1) x
2
+x 1 = x
2
+ (m+ 1)x m f(x) = 2x
2
mx +m+ 1 = 0 (2)
(x = 1 khng l nghim ca (2) nn (1), (2) l tng ng)
(2) c 2 nghim phn bit khi v ch khi
= m
2
8m + 8 > 0

m < 4 2

2
m > 4 + 2

2
(3)
Vi iu kin (3) ta c a.f(1) = 2 > 0. Vy 1 / [x
1
, x
2
], y x
1
, x
2
l 2 nghim ca (2). iu
ny chng t 2 giao im A, B gia (C) v y = x +m nm v cng mt pha ca ng thng
x = 1. Tc l A, B thuc cng mt nhnh ca th.
9.6.4 Cng c kin thc
V d 9.6.8: Cho y =
x
2
+mx 8
x m
(C
m
). Tm m (C
m
) ct trc honh ti 2 im phn bit
A, B sao cho cc tip tuyn ti A, B vung gc vi nhau.
Gii: ng cong (C
m
) ct trc honh ti 2 im phn bit A, B khi v ch khi :
x
2
+mx 8
x m
= 0 (1) c 2 nghim phn bit
Hay f(x) = x
2
+mx 8 = 0 (2) c 2 nghim phn bit khc m.
iu ny tng ng vi :

= m
2
+ 32 > 0
f(m) = 2m
2
8 = 0
m = 2.
Gi x
1
, x
2
tng ng l honh 2 im A, B. Khi x
1
, x
2
l 2 nghim ca phng trnh (2).
Theo nh l Viet ta c

x
1
+x
2
= m
x
1
.x
2
= 8
Ta c y = x + 2m+
2m
2
8
x m
y

= 1 +
8 2m
2
(x m)
2
.
Gi k
1
, k
2
l h s gc ca tip tuyn ti A, B: k
1
= y

(x
1
), k
2
= y

(x
2
).
Ta c (x m)
2
= x
2
2mx +m
2
= x
2
+mx 8 + (3mx + m
2
+ 8) nn ta c:
k
1
= 1 +
2m
2
8
3mx
1
m
2
8
, k
2
= 1 +
2m
2
8
3mx
2
m
2
8
Theo gi thuyt 2 tip tuyn vung gc vi nhau nn ta c: k
1
.k
2
= 1

1 +
2m
2
8
3mx
1
m
2
8

1 +
2m
2
8
3mx
2
m
2
8

= 1
1 +
2m
2
8
3mx
1
m
2
8
+
2m
2
8
3mx
2
m
2
8
+
(2m
2
8)
2
(3mx
1
m
2
8) . (3mx
2
m
2
8)
= 1
2 +
(2m
2
8) . (3m(x
1
+x
2
) 2m
2
16) + (2m
2
8)
2
9m
2
x
1
x
2
3m(m
2
+ 8) (x
1
+x
2
) + (m
2
+ 8)
2
= 0
2 +
(2m
2
8) . (3m. (m) 2m
2
16) + (2m
2
8)
2
72m
2
+ 3m
2
(m
2
+ 8) + (m
2
+ 8)
2
= 0
2 +
6m
4
24m
2
+ 192
4m
4
32m
2
+ 64
= 0

2m
4
88m
2
+ 320 = 0
4m
4
32m
2
+ 64 = 0
Ngi son : Th.s Minh Tun Trang 193 Khoa T nhin - Trng CSP Nam nh
T
h
.
s

M
i
n
h
T
u

n
9.7. S tip xc ca 2 ng cong Chng 9. Hm s v th

m
2
= 4
m
2
= 40
m
2
= 4
m
2
= 40 m = 2

10
V d 9.6.9 (H - D - 2003): Tm m ng thng y = mx + 2 2m ct ng cong
y =
x
2
2x + 4
x 2
ti 2 im phn bit.
Gii: ng cong y =
x
2
2x + 4
x 2
ct y = mx + 2 2m ti 2 im phn bit khi v ch khi:
Phng trnh
x
2
2x + 4
x 2
= mx + 2 2m (1) c 2 nghim phn bit,
tc l phng trnh: x
2
2x + 4 = (x 2).(mx + 2 m) (2) c 2 nghim phn bit.
(Do (2) khng c nghim x = 2)
(m1)(x 2)
2
= 4 c 2 nghim phn bit
m1 > 0 m > 1 .
V d 9.6.10 (H - A - 2004): Cho y =
x
2
+ 3x 3
2(x 1)
(C). Tm m ng thng y = m
ct (C) ti 2 im phn bit A, B sao cho AB = 1.
Gii: Xt phng trnh
x
2
+ 3x 3
2(x 1)
= m (1)
x
2
+ 3x 3 = 2m(x 1) (2) (Do (2) khng c nghim x = 1).
x
2
+ (2m3)x + 3 2m = 0 (3)
Gi x
1
, x
2
l 2 nghim phn bit ca (3), c iu ny ta phi c:
= (2m3)
2
4(3 2m) > 0 4m
2
4m3 > 0

m >
3
2
m <
1
2
(4)
Theo nh l Viet ta c : x
1
+x
2
= 3 2m, x
1
.x
2
= 3 2m
Ta c ta 2 giao im l A(x
1
, m), B(x
2
, m). T ta c :
AB = 1 [x
1
x
2
[ = 1 (x
1
x
2
)
2
= 1 (x
1
+x
2
)
2
4x
1
x
2
= 1
(3 2m)
2
4(3 2m) = 1 4m
2
4m4 = 0 m =
1

5
2
(5)
T (4) v (5) ta c m =
1

5
2
.
9.7 S tip xc ca 2 ng cong
9.7.1 Kin thc c bn
Hai ng cong y = f(x) v y = g(x) tip xc vi nhau khi h sau c nghim:

f(x) = g(x)
f

(x) = g

(x)
Nghim ca h phng trnh chnh l honh tip im.
Ngi son : Th.s Minh Tun Trang 194 Khoa T nhin - Trng CSP Nam nh
T
h
.
s

M
i
n
h
T
u

n
9.7. S tip xc ca 2 ng cong Chng 9. Hm s v th
9.7.2 Cc v d
V d 9.7.1: Cho y = x
3
3x
2
+ 2. Tm trn ng thng y = 2 cc im m t c th k
c 2 tip tuyn ti th hm s v 2 tip tuyn vung gc vi nhau.
Gii: Gi im cn tm l: M(m, 2). rng ng thng x = m i qua M ct ng cong
v song song vi trc tung v n khng th l tip tuyn nn mi tip tuyn vi ng cong i
qua M u c dng: y = k(x m) + 2. Vy h sau phi c nghim:

x
3
3x
2
+ 2 = k (x m) + 2
3x
2
6x = k
x
3
3x
2
+ 2 = (x m) . (3x
2
6x) + 2
2x
3
(3m+ 3) x
2
+ 6mx = 0

x = 0 k = 0
2x
2
3 (m+ 1) x + 6m = 0 (1)
+) k = 0: tip tuyn l y = 2, ng thng vung gc vi n c dng x = c. Vy khng c mt
tip tuyn no ca ng cong vung gc vi tip tuyn ny.
Vy ng cong c 2 tip tuyn vung gc th phng trnh (1) phi c 2 nghim x
1
, x
2
sao
cho: y

(x
1
).y

(x
2
) = 1 (2).
Phng trnh (1) c 2 nghim phn bit khi > 0
9m
2
+ 18m+ 9 48m > 0 9m
2
30m+ 9 > 0

m > 3
m <
1
3
(2)
Theo nh l Viet ta c : x
1
+x
2
=
3m + 3
2
, x
1
.x
2
= 3m
Ta c : 3x
2
6x =
3
2
(2x
2
3 (m+ 1) x + 6m) +
3
2
(3m1) x 9m
y

(x
1
) = 3x
2
1
6x
1
=
3
2
(3m1) x
1
9m, y

(x
2
) =
3
2
(3m1) x
2
9m
(2) y

(x
1
) .y

(x
2
) = 1

3
2
(3m1) x
1
9m

3
2
(3m1) x
2
9m

= 1

9
4
(3m1)
2
x
1
x
2

27
2
m(3m1) . (x
1
+x
2
) + 81m
2
= 1

9
4
(3m1)
2
.3m
27
2
m(3m1) .
3m+ 3
2
+ 81m
2
= 1
27m+ 1 = 0 m =
1
27
(Tha mn iu kin (2))
Kt lun: im M

2;
1
27

tha mn iu kin bi ton.


y
x
-1 0 1 2 3
-2
-1
0
1
2
y = x
3
3x
2
+ 2
M
Ngi son : Th.s Minh Tun Trang 195 Khoa T nhin - Trng CSP Nam nh
T
h
.
s

M
i
n
h
T
u

n
9.7. S tip xc ca 2 ng cong Chng 9. Hm s v th
V d 9.7.2: Cho y = x
3
3x
2
. Tm tt c cc im M nm trn ng cong sao cho t M ch
c th v c mt tip tuyn duy nht ti ng cong cho.
Gii: Gi M(a; a
3
3a
2
) l im cn tm. Tip tuyn qua M ch c th c dng
y = k(x a) +a
3
3a
2
Vy h sau phi c nghim:

x
3
3x
2
= k (x a) +a
3
3a
2
3x
2
6x = k
x
3
3x
2
= (3x
2
6x) . (x a) +a
3
3a
2
(1)
(x a) . (3x
2
6x) (x a) (x
2
+ax +a
2
) + 3 (x a) (x +a) = 0
(x a) . (3x
2
6x (x
2
+ax +a
2
) + 3 (x +a)) = 0
(x a) . (2x
2
(a + 3) x a
2
+ 3a) = 0 (x a)
2
. (2x +a 3) = 0

x = a
x =
3 a
2
V qua M ch k c mt tip tuyn duy nht nn phng trnh (1) phi c nghim duy nht.
iu ny tng ng vi :
a =
3 a
2
a = 1 M (1; 2)
Nhn xt:
> Ta thy im M(1; 2) chnh l im un ca ng cong cho.
> Bng cc php ton tng t bn c c th d dng chng minh kt qu tng qut: "Vi
mt ng cong bc 3 ty y = ax
3
+ bx
2
+ cx + d, a = 0, im un l im duy nht
trn ng cong c th k c ng mt tip tuyn vi ng cong"
> Chnh v im M ng cong nn phng trnh (1) chc chn c nghim x = a, nn ta
c th h bc nh lm trn.
> Trong bi s dng tnh cht: "Vi mi ng cong bc ba mi tip tuyn ch tip xc
vi ng cong ti mt im" (d chng minh). Tnh cht ny xin lu l khng cn ng
vi ng cong bc 4.
C th xt v d sau:
V d 9.7.3: Cho hm s y = x
4
2x
2
. Khi ng thng y = 1 s tip xc vi th hm
s ti 2 im
Gii: Ta c y

= 4x
3
4x = 4x(x
2
1), y

= 0 x = 0, x = 1. Ta c bng bin thin:


x 1 0 1 +
y

0 + 0 0 +
y
+
1
0
1
+
R rng l ng thng y = 1 tip xc vi ng cong ti 2 im cc tr.
Ngi son : Th.s Minh Tun Trang 196 Khoa T nhin - Trng CSP Nam nh
T
h
.
s

M
i
n
h
T
u

n
9.7. S tip xc ca 2 ng cong Chng 9. Hm s v th
y
x
-1 0 1
-1
0
1
2
3
y = x
4
2x
2
y = 1
V d 9.7.4: Cho ng cong y =
x
2
+x + 1
x + 1
(C). Chng minh rng t im A(1; 1) lun
k c 2 tip tuyn vung gc vi nhau n th (C).
Gii: V ng thng x = 1 khng th l tip tuyn ca (C) nn mi tip tuyn qua A(1; 1)
u c dng : y = k(x 1) 1. H sau c nghim:

x
2
+x + 1
x + 1
= k (x 1) 1
1
1
(x + 1)
2
= k

x
2
+x + 1
x + 1
=

1
1
(x + 1)
2

(x 1) 1
x
2
+ 3x + 1
(x + 1)
2
= 0
x
2
+ 3x + 1 = 0 (1)
Ta c = 3 > 0 x
1
, x
2
l 2 nghim ca x
1
, x
2
.
Theo nh l Viet ta c x
1
+x
2
= 3, x
1
.x
2
= 1.
Gi k
1
, k
2
l 2 h s gc ca tip tuyn tng ng vi 2 nghim.
V (x + 1)
2
= x
2
+ 2x + 1 = (x
2
+ 3x + 1) x
k
1
= 1 +
1
x
1
, k
2
= 1 +
1
x
2
k
1
.k
2
= 1 +
1
x
1
+
1
x
2
+
1
x
1
x
2
= 1 +
x
1
+x
2
x
1
x
2
+
1
x
1
x
2
k
1
k
2
= 1 +
3
1
+
1
1
= 1 hai tip tuyn vung gc vi nhau.
nh l 12. Mi tip tuyn ca ng cong y =
ax
2
+bx +c
px +q
(p, q = 0) ch tip xc vi ng
cong ti mt im duy nht
V d 9.7.5: Cho ng cong y =
x
2
+x 3
x + 2
(C). Tm cc im trn trc honh, nu t
k c mt tip tuyn ca (C).
Gii: Gi im cn tm l M(a, 0). Do x = a khng l tip tuyn ca (C) nn mi tip tuyn
vi (C) qua M c dng y = k(x a). Do h sau c nghim:

x
2
+x 3
x + 2
= k (x a) (1)
1 +
1
(x + 2)
2
= k (2)

x
2
+x 3
x + 2
=

1 +
1
(x + 2)
2

. (x a)
Ngi son : Th.s Minh Tun Trang 197 Khoa T nhin - Trng CSP Nam nh
T
h
.
s

M
i
n
h
T
u

n
9.7. S tip xc ca 2 ng cong Chng 9. Hm s v th
x
2
6x 6 +x
2
a + 4xa + 5a = 0 c nghim x = 2
(a 1) x
2
+ 2 (2a 3) x + 5a 6 = 0 (3) c nghim x = 2
+) Vi a = 1 :
(3) f (x) = 2x 1 = 0 x =
1
2
tha mn x = 2.
+) Vi a = 1 :
(3) c nghim duy nht khc 2 khi v ch khi:

= 0
f (2) = 0

> 0
f (2) = 0

= (2a 3)
2
(a 1) (5a 6) = 0
f (2) = 0

a
2
a + 3 > 0
2 +a = 0

a
2
a + 3 = 0
2 +a = 0

13
2
< a <
1 +

13
2
a = 2

a =
1

13
2
a = 2
M
1
(1; 0) , M
2

13
2
; 0

, M
3

1 +

13
2
; 0

, M
4
(2; 0) l 4 im tha mn
9.7.3 Cng c
V d 9.7.6 (H - D - 2002): Tm m ng cong (C) c phng trnh
y =
(2m1)x m
2
x 1
(C)
tip xc vi ng thng y = x
Gii: Xt y =
(2m1)x m
2
x 1
, ta c y

=
(m1)
2
(x 1)
2
. Vy h sau phi c nghim:

(2m1)x m
2
x 1
= x (1)
(m1)
2
(x 1)
2
= 1 (2)
(2)

x 1 = m1
x 1 = 1 m
x = 1

x = m
x = 2 m
m = 1
Vi x = m
(2m1) mm
2
m1
= m m
2
m = m
2
m m
m = 1 h c nghim.
Vy m = 1 l gi tr cn tm.
V d 9.7.7: Tm m ng cong y = x
4
6x
3
+ 12x
2
14x + 2m
2
+m v
y = 2x
3
10x
2
+ 10x + 1 tip xc vi nhau.
Gii: V 2 ng cong tip xc nn h sau c nghim.

x
4
6x
3
+ 12x
2
14x + 2m
2
+m = 2x
3
10x
2
+ 10x + 1
4x
3
18x
2
+ 24x 14 = 6x
2
20x + 10
(1)
(2)
Ngi son : Th.s Minh Tun Trang 198 Khoa T nhin - Trng CSP Nam nh
T
h
.
s

M
i
n
h
T
u

n
9.8. Bin lun s nghim bng th Chng 9. Hm s v th
(2) 4x
3
24x
2
+ 44x 24 = 0

x = 1
x = 2
x = 3
+) Vi x = 1 2m
2
+m10 = 0

m = 2
m =
5
2
+) Vi x = 2 2m
2
+m9 = 0

m =
1 +

73
4
m =
1

73
4
+) Vi x = 3 2m
2
+m10 = 0

m = 2
m =
5
2
+) Kt lun : m

2;
5
2
;
1 +

73
4
;
1

73
4

tha mn iu kin bi ton.


9.8 Bin lun s nghim bng th
9.8.1 Kin thc c bn
Da vo nhn xt: "S nghim ca phng trnh f(x) = g(x) chnh l s giao im ca 2
ng cong y = f(x) v y = g(x)", bi ton bin lun phng trnh trong nhiu trng
hp c cch gii n gin, r rng nu da vo cc th bit ca ng cho trc
(thng da vo kt qu ca v th hm s trong cc phn trc).
Nu m ng s giao im ca 2 ng y = f(x) v y = g(x) ngi ta s dng n cc
im ti hn, v cc v tr ti hn ca cc ng (thng l cc v tr m cc ng tip
xc nhau). V th cc kt qu trong mc ny c lin quan n cc kt qu v tnh tip xc
ca cc ng.
9.8.2 Cc v d
Xt cc v d sau:
V d 9.8.1: Cho hm s y = 4x
3
3x.
a) Kho st v v th hm s.
b) Bin lun s nghim ca phng trnh theo m: 4[x[
3
3[x[ = m.
c) Chng minh rng phng trnh : 4x
3
3x =

1 x
2
c 3 nghim.
Gii: a) Ta c y

= 12x
2
3, y

= 24x, vy c bng bin thin sau:


x
1
2
1
2
+
y

+ 0 0 +
y

1
1
+
Ngi son : Th.s Minh Tun Trang 199 Khoa T nhin - Trng CSP Nam nh
T
h
.
s

M
i
n
h
T
u

n
9.8. Bin lun s nghim bng th Chng 9. Hm s v th
+) im cc i

1
2
; 1

, im cc tiu

1
2
; 1

+) Khong ng bin :

;
1
2

1
2
; +

Khong nghch bin :

1
2
;
1
2

Ta c th nh hnh v:
y
x
-1 0 1
-1
0
1
2
y = 4x
3
3x
b) S nghim ca phng trnh 4[x[
3
3[x[ = m bng s giao im ca th hm s y =
4[x[
3
3[x[ v ng thng y = m. Ta c hnh v:
y
x
-1 0 1
-1
0
1
y = 4[x[
3
3[x[
Da vo hnh v ta c :
+) Nu m > 0: phng trnh c 2 nghim
+) Nu m = 0: Phng trnh c 3 nghim.
+) Nu 1 < m < 0: Phng trnh c 4 nghim.
+) Nu m = 1: Phng trnh c 3 nghim.
+) Nu m < 1 : Phng trnh v nghim.
c) V th 2 hm s ta c:
Ngi son : Th.s Minh Tun Trang 200 Khoa T nhin - Trng CSP Nam nh
T
h
.
s

M
i
n
h
T
u

n
9.8. Bin lun s nghim bng th Chng 9. Hm s v th
y
x
-1 0 1
-1
0
1
y
x
-1 0 1
-1
0
1
Da vo th ta c 2 th y = 4x
3
3x v y =

1 x
2
ct nhau ti 3 im phn bit.
Do phng trnh c 3 nghim phn bit.
V d 9.8.2: Tm m phng trnh 4[x[
3
3[x[ 1 = mx m c 4 nghim phn bit.
Gii: th hm y = 4[x[
3
3[x[ 1:
y
x
-1 0 1
-2
-1
0
y = 4[x[
3
3[x[ 1
A(1; 0)
B(0; 1)
y
=
x

1
y
=
(
6
3

9
)
(
x

1
)
ng thng y = m(x 1) lun i qua im c nh A(1; 0) v c h s gc m. Ta xt 2 v tr
ti hn ca h ng thng y = m(x 1).
Trc ht l ng thng i qua A(1; 0) v B(0; 1) l y = x 1 c h s gc 1.
Th 2 xt tip tuyn vi ng cong y = 4[x[
3
3[x[ 1 v qua A. R rng tip tuyn ny tip
xc vi nhnh ca ng cong vi x < 0. (Khi y = 4x
3
+3x1). Khi h sau c nghim:

4x
3
+ 3x 1 = m(x 1)
12x
2
+ 3 = m
x < 0
4x
3
+ 3x = (12x
2
+ 3) . (x 1)
4x
3
+ 3x 1 = 12x
3
+ 12x
2
+ 3x 3 8x
3
12x
2
+ 2 = 0
(2x 1) . (4x
2
4x 2) = 0

x =
1
2
(Loi)
4x
2
4x 2 = 0

x =
1 +

3
2
(Loi)
x =
1

3
2
Ta c x =
1 +

3
2
m = 9 + 6

3
Vy phng trnh c 4 nghim khi ng thng y = m(x 1) nm gia 2 ng thng ti hn
trn.
T ta c : 1 < m < 6

3 9 l cc gi tr ca tham s m cn tm.
V d 9.8.3 (H - A - 2002): Cho hm s y = x
3
+ 3x
2
(C).
a) Kho st v v th (C) ca hm s.
Ngi son : Th.s Minh Tun Trang 201 Khoa T nhin - Trng CSP Nam nh
T
h
.
s

M
i
n
h
T
u

n
9.9. Bi tp Chng 9. Hm s v th
b) Tm k phng trnh x
3
+ 3x
2
+k
3
3k
2
= 0 c 3 nghim phn bit.
Gii: a) Ta c y

= 3x
2
+ 6x, y

= 6x + 6, ta c bng bin thin sau:


x 0 2 +
y

0 + 0
y
+
0
4

th ca (C) nh sau:
y
x
-1 0 1 2 3
-1
0
1
2
3
4
y = x
3
+ 3x
2
b) Ta thy x
3
+ 3x
2
+k
3
3k
2
= 0 x
3
+ 3x
2
= k
3
+ 3k
2
(1)
T (1) suy ra (1) c 3 nghim phn bit (da vo th (C)) khi v ch khi:
0 < k
3
+ 3k
2
< 4 (2)
T (2) v da vo th ca (C) cu trn ta suy ra :
f
1
[(0; 4)] = (1; 0) (0; 2) (2; 3). Vy k (1; 0) (0; 2) (2; 3).
9.9 Bi tp
Kho st v v th hm s
Bi 9.1: Kho st v v th hm di y:
a) y =
3 x
3x 1
.
b) y =
x
2
3x + 2
x + 1
.
c) y = 2 x
2
2 3x
.
Ngi son : Th.s Minh Tun Trang 202 Khoa T nhin - Trng CSP Nam nh
T
h
.
s

M
i
n
h
T
u

n
9.9. Bi tp Chng 9. Hm s v th
d) y =
x
2
x
[x 2[
.
e) y = x
3
3x + 1.
f) y = x
4
2x
2
+ 4.

Cc tr v tim cn
Bi 9.2: Tm m hm s y =
1
3
x
3
+(m2)x
2
+(5m+4)x +m
2
+1 t cc tr ti x
1
, x
2
sao
cho x
1
< 1 < x
2
.
Hng dn. m < 3.
Bi 9.3: Cho y =
2
3
x
3
+ (cos 3 sin )x
2
8(1 + cos 2)x + 1.
a) Chng minh rng hm s lun c cc tr.
b) Gi s hm s c 2 cc tr x
1
, x
2
. Chng minh , ta lun c x
2
1
+x
2
2
18.
Bi 9.4: Cho y =
1
3
x
3
mx
2
x+m+1. Tm m khong cch gia 2 im cc tr l b nht.
Hng dn. y
min
=
2

3
m = 0.
Bi 9.5: Cho y = x
3
3x
2
+m
2
x+m. Tm m ng cong c cc i, cc tiu i xng nhau
qua ng thng y =
1
2
x
5
2
Hng dn. m = 0
Bi 9.6: Tm m ng cong y = x
4
+ 4mx
3
+ 3 (m + 1) x
2
+ 1 ch c cc tiu m khng c
cc i.
Hng dn. m = 1 hoc
1

7
3
m
1 +

7
3
Bi 9.7: Cho y = x
4
+ (m+ 3) x
3
+ 2 (m+ 1) x
2
. Chng minh rng vi mi m = 1, th hm s
lun lun c cc i ti im c honh 0
Bi 9.8: Cho y =
x
2
+ (2m + 1)x +m
2
+m+ 4
2(x +m)
. Tm m hm s c cc tr v tm khong
cch gia 2 im cc tr.
Hng dn. 4

2
Bi 9.9: Cho y =
x
2
+ 3x +m
x 4
. Tm m ng cong c cc tr v tha mn h thc
[y
C
y
CT
[ = 4.
Hng dn. m = 3
Ngi son : Th.s Minh Tun Trang 203 Khoa T nhin - Trng CSP Nam nh
T
h
.
s

M
i
n
h
T
u

n
9.9. Bi tp Chng 9. Hm s v th
Bi 9.10: a) Kho st v v th hm s : y =
x
2
+x +m
x +m
.
b) Tm m ng tim cn (C) i qua im A(3; 0).
Bi 9.11: Cho hm s y =
x
2
+mx 1
x + 1
(C
m
).
a) Kho st v v th hm s khi m = 1.
b) Tm m tim cn xin ca (C
m
) to vi 2 trc ta 1 tam gic c din tch bng 8.
Bi 9.12: Cho y =

x
2
x + 1 (C). Tm cc tim cn ca (C).
Gi tr ln nht, nh nht ca hm s
Bi 9.13: Tm gi tr ln nht ca hm s f(x) =
4

1 x
2
+
4

1 x +
4

1 +x.
Hng dn. max
xD
f (x) = 3.
Bi 9.14: Tm gi tr b nht ca biu thc P = (xyz + 1)

1
x
+
1
y
+
1
z

+
x
y
+
y
z
+
z
x
xy z.
trn min D = (x, y, z) [x > 0, y > 0, z > 0.
Hng dn. min P = 6
Bi 9.15: Tm gi tr ln nht ca biu thc : P = xyz trn min
D =

(x, y, z)

x 0, y 0, z 0,
1
1 +x
+
1
1 +y
+
1
1 +z
= 2

.
Hng dn. max P =
1
8
Bi 9.16: Tm gi tr ln nht, nh nht ca P = x
2
y(4 x y) trn min
D = (x, y) [x 0, y 0, x +y 6
.
Hng dn. max P = 4, min P = 64
Bi 9.17: Tm gi tr ln nht, nh nht ca : P =
x
2
(x 4y)
2
x
2
+y
2
trn min
D =

(x, y)

x
2
+y
2
> 0

.
Hng dn. max P = 2

2 2, min P = 2

2 2.
Bi 9.18: Tm gi tr ln nht, nh nht ca : P =
x + 2y + 1
x
2
+y
2
+ 7
, x, y R.
Ngi son : Th.s Minh Tun Trang 204 Khoa T nhin - Trng CSP Nam nh
T
h
.
s

M
i
n
h
T
u

n
9.9. Bi tp Chng 9. Hm s v th
Hng dn. max P =
1
2
, min P =
5
14
Bi 9.19: Tm gi tr ln nht, nh nht ca hm s:
f(x) =

3 +x +

6 x

18 + 3x x
2
trn min D = x[3 x 6.
Hng dn. max
xD
f (x) = 3; min
xD
f (x) =
9 3

2
2
.
Bi 9.20: Cho f (x) = 4x
2
4ax +a
2
2a xt trn min D = x R[2 x 0. Tm a
min
xD
f (x) = 2.
Hng dn. a = 1 hoc a = 1 +

3.

Vit phng trnh tip tuyn th
Bi 9.21: Vit phng trnh tip tuyn ca ng cong y =
x
2
9
x
, bit rng n i qua im
M (1; 8).
Hng dn. y = 2x + 6 v y = 50x 42.
Bi 9.22: Vit phng trnh tip tuyn ca ng cong y = x
4
4x
2
, bit rng n i qua im
M (2; 0).
Hng dn. y = 0; y = 16x 32 v y =
32
27
x
64
27
Bi 9.23: Tm m ng cong y = 2x
3
3 (m + 3) x
2
+ 18mx 8 tip xc vi trc honh.
Hng dn. m =
35
27
; m = 1; m = 4 + 2

6 v m = 4 2

6
Bi 9.24: Cho ng cong y = x
3
3x + 2 (C). Tm im M trn ng thng y = 2, sao
cho t M c th v c hai tip tuyn ti (C) sao cho hai tip tuyn vung gc vi nhau.
Hng dn. M

55
27
; 2

Bi 9.25: Cho ng cong y = x


2
5x + 6 .Vit phng trnh tip tuyn vi ng cong bit
rng n song song vi ng thng y = 3x + 1.
Bi 9.26: Cho y = x
2
5x + 6 v im M (5; 5). Vit phng trnh tip tuyn vi ng cong
i qua M.
Bi 9.27: Cho y = x
2
3x v y = 2x
2
+5x. Vit phng trnh tip tuyn chung ca hai ng
cong .
Bi 9.28: Cho hm s :y =
1
3
x
3
2x
2
+ 3x c th l (C) Vit phng trnh tip tuyn ti
im trn (C) c honh x = 2
Ngi son : Th.s Minh Tun Trang 205 Khoa T nhin - Trng CSP Nam nh
T
h
.
s

M
i
n
h
T
u

n
9.9. Bi tp Chng 9. Hm s v th
Bi 9.29: Cho hm s : y =
1
3
x
3
x +
2
3
c th l (C) Vit phng trnh tip tuyn vi (C)
bit tip tuyn vung gc vi ng thng () : y = x 2
Bi 9.30: Cho hm s : y = x
3
+ 3x
2
+ 4 (C) Vit phng trnh tip tuyn vi (C) bit tip
tuyn i qua m A(0; 1)
Bi 9.31: Cho hm s : y =
2x 5
x 2
(C) Vit phng trnh tip tuyn vi (C) bit tip tuyn
i qua im A(2; 0)

Bi ton xc nh im tha mn iu kin cho trc
Bi 9.32: Tm im M trn th hm s: y =
x + 2
x 3
sao cho khong cch t M n tim cn
ng bng khong cch t M n tim cn ngang.
Bi 9.33: Tm trn th hm s: y =
x
2
+ 3x + 6
x + 2
cc im c ta nguyn.
Bi 9.34: Tm trn th y =
x
3
3
+x
2
+3x
11
3
hai im phn bit M, N i xng nhau qua
trc tung.
Bi 9.35: Tm trn th y =
x
2
+x + 2
x 1
hai im M, N i xng nhau qua im I

0;
5
2

.
Bi 9.36: Tm trn th ca hm s y =
x
2
+ 4x + 5
x + 2
nhng im c tng khong cch t
n ng thng (d) : y + 3x + 6 = 0 l nh nht.
Bi 9.37: Tm ta im M thuc y =
2x
x + 1
(C) ,bit tip tuyn ca (C) ti M ct hai trc
Ox, Oy ti A, B v OAB c din tch bng
1
4
.
Hng dn. M
1
(
1
2
; 2), M
2
(1; 1)
Bi 9.38: Cho hm s :y =
x
2
+ 2x + 2
x + 1
a) Kho st s bin thin v v th (C) ca hm s
b) Tm im M thuc th (C) ca hm s sao cho khong cch t n trc honh bng
hai ln khong cch t n trc tung
Bi 9.39: Cho hm s :y =
2x + 1
x + 1
a) Kho st s bin thin v v th (C) ca hm s
Ngi son : Th.s Minh Tun Trang 206 Khoa T nhin - Trng CSP Nam nh
T
h
.
s

M
i
n
h
T
u

n
9.9. Bi tp Chng 9. Hm s v th
b) Tm trn th (C) ca hm s nhng c tng khong cch t n hai tim cn l nh
nht.
Bi 9.40: Cho hm s :y =
x
2
+ 2x 2
x 1
a) Kho st s bin thin v v th (C) ca hm s
b) Tm trn th (C) ca hm s nhng c tng khong cch t n hai tim cn l nh
nht.
Bi 9.41: Cho hm s :y =
x
2
+ 4x + 5
x + 2
a) Kho st s bin thin v v th (C) ca hm s
b) Tm trn th (C) ca hm s nhng c tng khong cch t n ng thng (d) :
y + 3x + 6 = 0 l nh nht.
Bi 9.42: Cho hm s : y =
x
2
x + 1
x 1
a) Kho st s bin thin v v th (C) ca hm s
b) Xc nh im A trn (C) c honh x = a > 1 sao cho khong cch t A n giao im
hai tim cn l nh nht
Bi 9.43: Cho hm s: y = 2x
3
+ 3x
2
12x 1
a) Kho st s bin thin v v th (C) ca hm s
b) Tm im M thuc th (C) ca hm s sao cho tip tuyn ca (C) ti im M i qua gc
ta
Bi 9.44: Cho hm s :y =
x
3
3
+x
2
+ 3x
11
3
a) Kho st s bin thin v v th (C) ca hm s cho.
b) Tm trn th (C) hai im phn bit M, N i xng nhau qua trc tung.
Hng dn.

3;
16
3

3;
16
3

Bi 9.45: Cho hm s :y =
x
2
+x + 2
x 1
c th l (C)
a) Kho st s bin thin v v th (C) ca hm s cho.
b) Tm trn th (C) hai im M, N i xng nhau qua im I

0;
5
2

.
Hng dn. (3; 2) ; (3; 7)

S tng giao
Ngi son : Th.s Minh Tun Trang 207 Khoa T nhin - Trng CSP Nam nh
T
h
.
s

M
i
n
h
T
u

n
9.9. Bi tp Chng 9. Hm s v th
Bi 9.46: Chng minh rng vi mi m, ng thng y =
1
2
x m lun ct (C) : y =
x + 2
x 1
ti
2 im phn bit A, B. Tm m sao cho AB l nh nht
Hng dn. AB =

10 khi m =
3
2
Bi 9.47: Tm m (C) : y =
x
2
+x +m
x +m
ct ng thng d : y = x 1 ti 2 im phn
bit.
Hng dn. m < 6 4

2 hoc

m > 6 + 4

2
m = 0
Bi 9.48: Tm m th hm s y =
x
2
+mx 1
x + 1
ct y = m ti 2 im phn bit A, B sao
cho ABOB.
Hng dn. m =
1

5
2
.
Bi 9.49: Cho ng cong y = x
3
x
2
+18mx 2m. Tm m ng cong ct trc honh ti
3 im phn bit c honh x
1
, x
2
, x
3
sao cho x
1
< 0 < x
2
< x
3
.
Hng dn. m < 0
Bi 9.50: Cho hm s y = 2x
3
3x
2
1. Gi d l ng thng i qua im M(0; 1) v c h
s gc l k. Tm k ng thng d ct (C) ti 3 im phn bit.
Bi 9.51: Cho hm s y =
x
2
2x + 4
x 2
(1) v ng thng y = mx+22m. Tm m ng
thng d ct th hm s (1) ti 2 im phn bit.
Bi 9.52: Cho hm s : y = (x 1)(x
2
+ mx + m) (1) Xc nh m sao cho th hm s (1)
ct trc honh ti 3 im phn bit.
Bi 9.53: Cho hm s : y = x
3
3x + 2 Gi (d) l ng thng i qua im M(3; 20) v c h
s gc l m. Tm m ng thng (d) ct (C) to 3 im phn bit.
Bi 9.54: Cho hm s : y = x
4
mx
2
+ m 1 (1) Xc nh m sao cho th hm s (1) ct
trc honh ti 4 im phn bit.
Bi 9.55: Cho hm s : y =
x
2
2x + 4
x 2
(1) v ng thng (d) : y = mx + 2 2m. Tm m
ng thng (d) ct th hm s (1) ti hai im phn bit.

S tip xc ca 2 ng cong
Bi 9.56: Cho y =
x
2
x + 1
x 1
(C). Tm trn Oy cc im c th k c t nht mt tip
tuyn n (C)
Ngi son : Th.s Minh Tun Trang 208 Khoa T nhin - Trng CSP Nam nh
T
h
.
s

M
i
n
h
T
u

n
9.9. Bi tp Chng 9. Hm s v th
Bi 9.57: Cho y =
2x
2
+x + 1
x + 1
. Tm trn Oy cc im c th k n (C) 2 tip tuyn vung
gc vi nhau.
Bi 9.58: Cho hm s y = x
3
3x
2
+ 3mx + 4. Xc nh m th hm s trn tip xc vi
trc honh.
Bi 9.59: Cho ng cong (C
m
) : y =
2x
2
+ (1 m)x + 1 +m
x m
. Tm m (C
m
) ct trc Ox ti
2 im v tip tuyn vi (C) ti 2 im vung gc vi nhau.

Bin lun s nghim ca phng trnh bng th
Bi 9.60: Cho hm s y = x
3
4x
2
+ 4x c th (C). Bin lun theo k s giao im ca
th (C) vi ng thng y = k
Bi 9.61 (H - A - 2006): a) Kho st v v th hm s y = 2x
3
9x
2
+ 12x 4 (C).
b) Tm m phng trnh 2[x[
3
9x
2
+ 12[x[ = m c 6 nghim phn bit.
Bi 9.62: Bin lun theo a s nghim ca phng trnh :
x
2
2[x[ + 3
[x[ 1
= a
Bi 9.63: a) Kho st v v th hm s y =
(x + 1)
2
x + 2
b) Bin lun theo tham s m v s nghim ca phng trnh sau:
(x + 1)
2
m.[x + 2[ = 0
Ngi son : Th.s Minh Tun Trang 209 Khoa T nhin - Trng CSP Nam nh
T
h
.
s

M
i
n
h
T
u

n
Chng 10. Hnh khng gian ta
Chng 10
Hnh khng gian ta
10.1 H ta , vc t, im
10.1.1 H ta Cc
Cho cc trc Ox, Oy, Oz i mt vung gc vi nhau.
Ly I, J, K ln lt trn Ox, Oy, Oz sao cho OI = OJ = OK = 1.
t

i =

OI,

j =

OJ,

k =

OK.
Tnh cht


i .

j =

j .

k =

k .

i = 0
[

i [ = [

j [ = [

k [ = 1 hay

i
2
=

j
2
=

k
2
= 1
3 vc t

i ,

j ,

k khng ng phng
nh l 13. Nu

a ,

b ,

c khng ng phng th

u , !(x, y, z) sao cho

u = x.

a +y.

b +z.

c
H qu 10.1.

u , !(x, y, z) sao cho



u = x.

i +y.

j +z.

k .
Da vo H qu 10.1 ta c nh ngha:
nh ngha 10.1. B 3 s (x, y, z) nh trong H qu 10.1 c gi l ta ca vc t

u .
K hiu

u = (x, y, z) hay

u (x, y, z)
10.2 Php ton trn vc t
Gi s

u = (x, y, z) v

v = (x

, y

, z

).

v = (x x

, y y

, z z

).
.

u = (x, y, z) = (x, y, z)


u .

v = x.x

+y.y

+z.z

u [ =

x
2
+y
2
+z
2
.
Ngi son : Th.s Minh Tun Trang 210 Khoa T nhin - Trng CSP Nam nh
T
h
.
s

M
i
n
h
T
u

n
10.3. im Chng 10. Hnh khng gian ta
cos(

u ,

v ) =

u .

v
[

u [.[

v [
=
xx

+yy

+zz

x
2
+y
2
+z
2
.

x
2
+y
2
+z
2
V d 10.2.1: Cho

u = (2, 1, 3),

v = (0, 1, 2).
a) Biu din

u ,

v theo cc vc t

i ,

j .
b) Tnh 2

u + 3

v ,

u

v v gc gia chng.
V d 10.2.2: Cho

u = 2

j ,

v =

i +

k .
a) Vit ta

u ,

v .
b) Tnh gc gia

u

v v

u +

v .
c) Tm

w = (a 1, b + 2, a 2b) sao cho


u .

w = 2

v .

w = 3
10.3 im
nh ngha 10.2. Ta im M l ta ca vc t

OM.
Tnh cht
Gi s A(x
A
, y
A
, z
A
), B(x
B
, y
B
, z
B
).


AB = (x
B
x
A
, y
B
y
A
, z
B
z
A
).
M l trung im AB th x
M
=
x
A
+x
B
2
, y
M
=
y
A
+y
B
2
, z
M
=
z
A
+z
B
2
.
G l trng tm tam gic ABC th:

x
G
=
x
A
+x
B
+x
C
3
y
G
=
y
A
+y
B
+y
C
3
z
G
=
z
A
+z
B
+z
C
3
G l trng tm t din ABCD c lm tng t.
10.3.1 Tch c hng ca 2 vc t v ngha
nh ngha 10.3. Cho

u = (x, y, z),

v = (x

, y

, z

).
Vc t: [

u ,

v ] =

y z
y

z x
z

x y
x

gi l tch c hng ca 2 vc t

u ,

v .
ngha
[

u ,

v ] =

0

u ,

v cng phng.
[

u ,

v ]

u ,

v .
Cho hnh bnh hnh ABCD th S
ABCD
= [[

AB,

AD][.
Ngi son : Th.s Minh Tun Trang 211 Khoa T nhin - Trng CSP Nam nh
T
h
.
s

M
i
n
h
T
u

n
10.3. im Chng 10. Hnh khng gian ta
Cho tam gic ABC th S
ABC
=
1
2
[[

AB,

AC][.
Cho hnh hp ABCD.A

th V
ABCD.A

D
= [[

AB,

AD].

AA

[.
Cho t din ABCD th V
ABCD
=
1
6

AB,

AC].

AD

A, B, C thng hng [

AB,

AC] =

0

AB,

AC cng phng.
A, B, C, D ng phng [

AB,

AC].

AD = 0
Ch : Cn phn bit gia du gi tr tuyt i v di vc t.
[

u [: di ca vc t

u .
[u[: Tr tuyt i ca s u.
V d 10.3.1: Cho

u = (2; 1; 3),

v = (0; 1; 2),

w = (1; 1; 1).
a) Tnh [

u ,

v ], [

v ,

w], [

w,

u ] v di ca chng.
b) Kim tra ng thc: [

u ,

v ].

u = 0, [

u ,

v ].

v = 0
c) Tnh [

u ,

v ].

w, [

v ,

w].

u , [

w,

u ].

v v so snh chng.
V d 10.3.2: Cho A(1; 3; 1), B(2; 1; 1), C(1; 0; 2), D(0; 1; 1).
a) Chng minh rng A, B, C, D ng phng v tnh th tch t din ABCD.
b) Tm ta trng tm A

ca BCD v trng tm G ca t din. Chng minh rng: A, A

, G
thng hng.
c) Trong mt phng (BCD) h BHCD. Tnh S
BCD
, BH.
d) Tnh h
A
l di ng cao h t nh A ca t din ABCD.
10.3.2 Bi tp
Bi 10.1: Cho A(2; 1; 3), B(0; 1; 2), C(1; 1; 0), D(3; 1; 2).
a) Tnh gc gia AB, CD v gc gia BC, AD.
b) Chng minh rng: ABCD l mt t din v tnh th tch ca t din khi .
c) Tnh din tch ABC v di ng cao nh D ca t din.
d) Tm ta trung im M, N ca AB, CD v trng tm G ca t din ABCD. Chng minh
rng: M, N, G thng hng.
Bi 10.2: Cho ABC c A(1; 1; 0), B(0; 1; 1), trng tm G(1; 0; 3).
a) Tm ta im C.
b) Tm im D sao cho ABCD l hnh bnh hnh.
c) Tm E sao cho A l trung im ca BE.
d) Tm di chiu cao h t nh A ca ABC.
Ngi son : Th.s Minh Tun Trang 212 Khoa T nhin - Trng CSP Nam nh
T
h
.
s

M
i
n
h
T
u

n
10.4. Phng trnh mt phng Chng 10. Hnh khng gian ta
10.4 Phng trnh mt phng
10.4.1 Phng trnh tng qut ca mt phng
Input: () i qua im M
0
(x
0
, y
0
, z
0
) c

n = (A, B, C) l vc t php tuyn.
Output: M(x, y, z) () M tha mn iu kin g?
M ()

M
0
M.

n = 0 A(x x
0
) +B(y y
0
) +C(z z
0
) = 0
Ax +By +Cz +D = 0 vi D = (Ax
0
+By
0
+Cz
0
)
10.4.2 Phng php xc nh mt phng
Mt s kt qu v phng nh sau:
Mt phng s xc nh v phng nu c mt vc t php tuyn

n hoc 2 vc t ch
phng

u
1
,

u
2
v c th tnh c

n = [

u
1
,

u
2
].
Nu ()() th

n

l mt ch phng ca ().
Nu ()//() th

n

=

n

.
Nu ng thng d() th

n
()
=

u
d
Nu ng thng d//() th

u
d
l mt ch phng ca ()
Mt phng () tha mn iu kin.
i qua 3 im A, B, C th


n = [

AB,

AC]
i qua im A
Cha im A v ng thng d th


n = [

u
d
,

AM
0
]
i qua im A
vi M
0
l mt im trn d,

u
d
l
vc t ch phng ca d (vc t c gi song song vi ng thng d)
Cha d
1
v d
2
ct nhau th


n = [

u
d
1
,

u
d
2
]
i qua im M d
1
Cha d
1
v d
2
song song vi nhau th


n = [

u
d
1
,

M
1
M
2
]
i qua M
1
vi M
1
d
1
v M
2
d
2
.
i qua Ad th


n =

u
d
i qua im A
V d 10.4.1: Vit phng trnh mt phng (ABC) bit A(1; 1; 3), B(0; 2; 1), C(1; 0; 2).
V d 10.4.2: Vit phng trnh mt phng () i qua A(2; 1; 1) v song song vi
() : x 2y +z 3 = 0.
V d 10.4.3: Vit phng trnh mt phng () i qua A(1; 1; 3) v B(0; 2; 1) v vung gc
vi mt phng () : 2x y z 1 = 0.
V d 10.4.4: Vit phng trnh mt phng () i qua A(1; 2; 1) v vung gc vi 2 mt
phng (P) : x +y z 1 = 0 v (Q) : 2x y z 2 = 0.
Ngi son : Th.s Minh Tun Trang 213 Khoa T nhin - Trng CSP Nam nh
T
h
.
s

M
i
n
h
T
u

n
10.5. Phng trnh ng thng Chng 10. Hnh khng gian ta
10.4.3 V tr tng i ca 2 mt phng
() song song vi ()

cng phng [

] =

0
M
0
() nhng M
0
/ ()
() ()

cng phng
M
0
() ()
() ct ()

n

khng cng phng.


V d 10.4.5: Tm v tr tng i ca 2 mt phng trong cc trng hp sau:
a) (P) : 2x y z 3 = 0, (Q) : x + y z 2 = 0.
b) (P) : 2x + 3y z 5 = 0, (Q) : 4x 6y + 2z + 3 = 0.
c) (P) : 3x 6y + 12z 3 = 0, (Q) : 2x + 4y 8z + 2 = 0.
10.4.4 Bi tp
Bi 10.3: Vit phng trnh mt phng i qua 3 im A(1; 0; 3), B(2; 1; 0), C(0; 3; 1).
Bi 10.4: Vit phng trnh mt phng trung trc ca on AB vi A(1; 1; 2), B(5; 3; 0).
Bi 10.5: Cho t din ABCD c A(1; 0; 2), B(0; 2; 1), C(5; 1; 0), D(1; 1; 1)
a) Vit phng trnh mt phng qua A song song vi mt phng (BCD).
b) Vit phng trnh mt phng cha AB v song song vi CD.
c) Vit phng trnh mt phng qua A vung gc vi CD.
d) Vit phng trnh mt phng qua trng tm G ca t din vung gc 2 mt phng (P) :
x y z 1 = 0, (Q) : 2x +y z 3 = 0.
Bi 10.6: Xc nh v tr tng i ca cc mt phng:
a) (P) : 3x y +z 3 = 0, (Q) : 2x +y z = 0.
b) (P) : 5x 10y + 15z 1 = 0, (Q) : x 2y + 3z + 4 = 0.
10.5 Phng trnh ng thng
Cch xc nh phng ca ng thng d:
Phng ca d xc nh nu c mt ch phng

u hoc 2 php tuyn

n
1
,

n
2
v

u = [

n
1
,

n
2
].
Nu d mt phng (P) th

u
d
=

n
P
.
Nu d song song vi mt phng (P) th

n
P
l mt php tuyn ca d.
Nu d th

u

l mt php tuyn ca d.
Nu d song song vi th

u
d
=

.
Ngi son : Th.s Minh Tun Trang 214 Khoa T nhin - Trng CSP Nam nh
T
h
.
s

M
i
n
h
T
u

n
10.5. Phng trnh ng thng Chng 10. Hnh khng gian ta
Bi ton: Cho ng thng d :


u = (a, b, c)
i qua im M
0
(x
0
, y
0
, z
0
)
M(x, y, z) d x, y, z tha mn iu kin g?
Li gii: M(x, y, z) d

M
0
M,

u cng phng.
t sao cho

M
0
M = t

x = x
0
+at
y = y
0
+bt
z = z
0
+ct
vi t l tham s (Phng trnh tham s)

x x
0
a
=
y y
0
b
=
z z
0
c
(Phng trnh chnh tc)
V d 10.5.1: Vit phng trnh ng thng i qua A(1; 1; 0) v song song vi ng thng
d :
x 1
2
=
y 1
3
=
2 z
1
V d 10.5.2: Vit phng trnh ng thng i qua A(0; 1; 2) v song song vi 2 mt phng
(P) : x 2y z 1 = 0, (Q) : 2x +y 3z + 1 = 0.
V d 10.5.3: Vit phng trnh ng thng l giao tuyn ca mt phng () : 2xyz3 = 0,
() : y + 3z 1 = 0.
10.5.1 V tr tng i ca ng thng v mt phng
Cho ng thng d v mt phng (). Khi :
d song song vi ()

u
d


n

u
d
= 0
M
0
d nhng M
0
/ ()
d ()

u
d
= 0
M
0
d ()
d ct ()

n

u
d
= 0
V d 10.5.4: Xc nh v tr tng i ca cc cp ng thng v mt phng sau:
a) d :
x + 1
2
=
1 y
1
=
z + 2
4
vi () : 3x 2y z + 3 = 0.
b) d :
x 2
3
=
y + 1
4
=
z 1
1
vi () : 2x y z + 3 = 0.
c) d :

x = 2 t
y = 3 +t
z = 4 2t
vi : 2x 4y 3z + 1 = 0.
V d 10.5.5: Vit phng trnh mt phng () trong cc trng hp:
a) () i qua im A(1; 1; 0) v vung gc vi ng thng d :
x 1
2
=
y + 1
1
=
z 3
5
.
b) () cha im A(2; 0; 1) v ng thng d :

x = 1 t
y = 3
z = 4 +t
c) () cha d
1
:
x 2
1
=
y 1
3
=
z + 1
1
v song song vi ng thng d
2
:

x = 2 t
y = 3 +t
z = 1 2t
Ngi son : Th.s Minh Tun Trang 215 Khoa T nhin - Trng CSP Nam nh
T
h
.
s

M
i
n
h
T
u

n
10.5. Phng trnh ng thng Chng 10. Hnh khng gian ta
10.5.2 V tr tng i ca ng thng v ng thng
Cho 2 ng thng d
1
, d
2
vi M
1
d
1
, M
2
d
2
. Khi :
d
1
song song vi d
2


u
d
1
,

u
d
2
cng phng [

u
d
1
,

u
d
2
] =

0
M
1
d
1
nhng M
1
/ d
2
d
1
d
2

u
d
1
,

u
d
2
] =

0
M
1
d
1
d
2
d
1
ct d
2

u
d
1
,

u
d
2
] =

0
[

u
d
1
,

u
d
2
].

M
1
M
2
= 0
d
1
cho d
2
[

u
d
1
,

u
d
2
].

M
1
M
2
= 0
V d 10.5.6: Xt v tr tng i ca cc ng thng sau:
a) d
1
:
x + 1
2
=
y + 1
1
=
z 3
4
, d
2
:

x = 3 + 2t
y = 1 t
z = 1 4t
b) d
1
:

x = 1 t
y = 1
z = 2t
, d
2
:
x 5
5
=
y 2
3
=
1 z
1
c) d
1
:
x 1
4
=
y + 1
1
=
z 3
2
, d
2
l giao tuyn ca 2 mt phng: (
1
) : x + 4y + 3 = 0,
(
2
) : 2y +z 1 = 0.
d) d
1
:
x 1
2
=
y + 1
3
=
z 1
4
, d
2
:
x 2
1
=
y + 1
3
=
z 1
4
.
V d 10.5.7: Vit phng trnh mt phng () tha mn:
a) () cha 2 ng thng ct nhau d
1
:

5 + 5t
y = 2 + 3t
z = 1 t
, d
2
:

x = 2 +t
y = 1
z = 2 2t
b) () cha 2 ng thng song song d
1
:
x 1
2
=
y + 1
3
=
z 3
1
v d
2
:
x + 1
2
=
y + 1
3
=
z 1
1
.
10.5.3 Mt s dng ton v ng thng v mt phng
~ ng thng i qua mt im ct c 2 ng thng cho trc
Thut ton: ng thng d i qua im A ct d
1
v d
2
.
+) () = mp(d, d
1
) = mp(A, d
1
), () = mp(d, d
2
) = mp(A, d
2
).
+) d = () ().
V d 10.5.8: Vit phng trnh ng thng d i qua A(1; 2; 0) ct c 2 ng thng
d
1
:
x 1
2
=
y + 1
3
=
z 1
4
v d
2
:

x = 1 t
y = 2 +t
z = 3 3t
Ngi son : Th.s Minh Tun Trang 216 Khoa T nhin - Trng CSP Nam nh
T
h
.
s

M
i
n
h
T
u

n
10.5. Phng trnh ng thng Chng 10. Hnh khng gian ta
~ ng thng song song vi mt ng thng v ct 2 ng thng khc
Thut ton: ng thng d song song vi d
3
v ct d
1
, d
2
.
+) () = mp(d, d
1
) c

n

= [

u
d
,

u
d
1
] = [

u
d
3
,

u
d
1
] v i qua im M
1
d
1
.
+) () = mp(d, d
2
) c

n

= [

u
d
,

u
d
2
] = [

u
d
3
,

u
d
2
] v i qua im M
2
d
2
.
+) d = () ().
V d 10.5.9: Vit phng trnh ng thng d song song vi d
3
v ct d
1
, d
2
bit:
d
1
:
x 2
1
=
y + 1
2
=
z 3
3
, d
2
:

x = 5 t
y = 2 + 2t
z = 1 + 3t
, d
3
:
x + 4
1
=
y 2
3
=
z + 6
2
.
~ Tm hnh chiu vung gc ca mt im xung mt ng thng
Thut ton: Hnh chiu vung gc H ca im A ln ng thng d.
+) Vit phng trnh mt phng () qua A v vung gc d (tc l

n

=

u
d
)
+) H = d ().
V d 10.5.10: Cho im A(2; 1; 2) tm im i xng A

ca A qua ng thng
d :
x 1
1
=
y 1
2
=
z + 3
1
.
~ Tm hnh chiu vung gc ca mt im xung mt mt phng
Thut ton: Tm hnh chiu vung gc H ca im A xung mt phng ()
+) Vit phng trnh ng thng d i qua im A v vung gc vi () (Tc l

u
d
=

n

).
+) H = d ().
V d 10.5.11: Cho A(1; 3; 0), tm hnh chiu vung gc ca A ln mt phng
() : x + 2y +z 1 = 0 t tm im i xng A

ca A qua mt phng ().


~ Tm hnh chiu vung gc ca mt ng thng xung mt mt phng
Thut ton: Hnh chiu vung gc ca d ln mt phng (). Ta c th 2 im ca ng thng
v ly hnh chiu ca 2 im ln mt phng v vit phng trnh ng thng qua 2 hnh
chiu l ng thng cn tm. Ngoi ra ta c th lm theo cch sau y tm hnh chiu d

ca d ln ():
+) () = mp(d, d

) tc l mt phng cha d v vung gc vi () s c



n

= [

u
d
,

u
d
] = [

u
d
,

]
v i qua im M
0
d.
+) d

= () ().
V d 10.5.12: Tm hnh chiu vung gc ca ng thng d :
x 1
2
=
y + 1
3
=
z 1
2
ln mt
phng () : 2x y z + 1 = 0.
~ Tm hnh chiu ca im A vi php chiu song song theo phng d ca mt
phng ()
Thut ton:
+) Vit phng trnh ng thng qua A song song vi d.
+) Ta hnh chiu H = ().
Ngi son : Th.s Minh Tun Trang 217 Khoa T nhin - Trng CSP Nam nh
T
h
.
s

M
i
n
h
T
u

n
10.5. Phng trnh ng thng Chng 10. Hnh khng gian ta
V d 10.5.13: Cho im A(1; 1; 0) v ng thng d :
x 1
2
=
y + 1
3
=
z 1
4
, mt phng
() : x + 2y z 1 = 0
Tm hnh chiu song song ca im A theo phng d ln mt phng ()
~ Tm hnh chiu ca ng thng a vi php chiu song song theo phng d ca
mt phng ()
Thut ton: Gi hnh chiu l d

.
+) Vit phng trnh mt phng () = mp(d

, a) tha mn

n

= [

u
a
,

u
d
] v i qua im M
0
a.
+) d

= () ().
V d 10.5.14: Cho ng thng :
x 1
2
=
y + 1
1
=
z 2
4
, d :
x + 1
2
=
y 1
3
=
z + 1
1
v
() : 2x +y z 3 = 0.
Vit phng trnh hnh chiu ca vi php chiu song song theo phng d ln mt phng ().
10.5.4 Bi tp
Bi 10.7: Xt v tr tng i ca ng thng d vi mt phng ():
a) d :
x 1
1
=
y + 1
1
=
z
3
v mt phng () : x 2y +z 3 = 0.
b) d :
x + 1
2
=
y 2
3
=
z 1
1
vi mt phng () : x +y +z 1 = 0.
c) d :

x = 1 + 2t
y = 1 + 3t
z = 1 +t
vi mt phng () : 2x y z 3 = 0
Bi 10.8: Cho d :
x 1
1
=
y 1
1
=
z
5
, () : 3x 2y +z 1 = 0.
a) Chng minh rng d ().
b) Tm hnh chiu vung gc ca A
1
, A
2
ca im A(2; 1; 3) ln d v ().
Bi 10.9: Xt v tr tng i ca 2 ng thng trong cc trng hp sau:
a) d
1
:
x 1
2
=
y + 1
1
=
z 1
3
, d
2
:

x = 1 2t
y = 5 + 5t
z = 2 2t
b) d
1
:

x = 1 t
y = 2 +t
z = 3 2t
v d
2
:
x
1
=
y 3
1
=
z 1
2
.
c) d
1
:
x + 1
2
=
y 1
1
=
z + 2
1
v d
2
:
x 1
3
=
y 2
1
=
z + 1
2
.
d) d
1
:
x 2
1
=
y + 1
3
=
z 2
1
v d
2
:

x = 3 t
y = 1 + 3t
z = 2 t
Ngi son : Th.s Minh Tun Trang 218 Khoa T nhin - Trng CSP Nam nh
T
h
.
s

M
i
n
h
T
u

n
10.5. Phng trnh ng thng Chng 10. Hnh khng gian ta
Bi 10.10: Vit phng trnh ng thng d trong cc trng hp sau:
a) d l giao tuyn ca () : x y z + 3 = 0, () : 2x +y 3z 1 = 0
b) d i qua im A(1; 1; 1) v vung gc vi mt phng () : x +y 3z 1 = 0.
c) d i qua im A(2; 1; 3) v song song vi 2 mt phng () : x y z + 3 = 0, () :
2x + 3y z 1 = 0.
d) d i qua im A(1; 2; 3) v song song vi ng thng () :
x + 1
2
=
1 y
2
=
z 2
3
.
e) d i qua im A(0; 1; 2) v vung gc vi 2 ng thng d
1
:
x 1
2
=
y + 1
1
=
z 1
3
v
d
2
:
x 1
1
=
y + 2
1
=
z 2
2
Bi 10.11: Vit phng trnh ng thng d trong cc trng hp sau:
a) d l hnh chiu vung gc ca ng thng :
x 1
2
=
y + 1
1
1 =
z 3
2
ln mt phng
() : x + 3y 2z 5 = 0.
b) d l hnh chiu song song ca ng thng a :

x = 2 +t
y = 1 2t
z = 1 + 3t
ln mt phng
() : x y z 1 = 0 theo phng :
x 1
3
=
y + 1
2
=
z 1
1
c) d l ng thng i qua im A(6; 2; 1) v ct c 2 ng thng d
1
:

x = 1 +t
y = 1 +t
z = 2 t
v
d
2
:
x
2
=
y 2
1
=
z + 1
1
.
d) d l ng thng song song vi d
3
:
x + 1
3
=
y 1
1
=
z 3
4
v ct c 2 ng thng
d
1
:

x = 2 t
y = 1 +t
z = t
, d
2
:
x 1
2
=
y 1
1
=
z 1
3
Bi 10.12: Vit phng trnh mt phng () trong cc trng hp sau:
a) () cha im A(2; 1; 2) v ng thng d :
x 1
1
=
y + 1
1
=
z 2
3
.
b) () cha 2 ng thng ct nhau d
1
:

x = t
y = 1
z = 1 +t
v d
2
:
x 1
1
=
y
1
=
z 2
1
.
c) () cha 2 ng thng song song: d
1
:

x = 2 +t
y = 3 t
z = 2 + 3t
, d
2
:
x 3
1
=
y + 1
1
=
z 2
3
Ngi son : Th.s Minh Tun Trang 219 Khoa T nhin - Trng CSP Nam nh
T
h
.
s

M
i
n
h
T
u

n
10.6. Gc Chng 10. Hnh khng gian ta
Bi 10.13: Tm im i xng A

ca A(1; 2; 1) qua:
a) ng thng d :
x 1
1
=
y 4
3
=
z 4
1
b) mt phng () : x +y +z 4 = 0
Bi 10.14: Tm hnh chiu song song ca im A(3, 1, 1) ln mt phng x y z 3 = 0
theo phng d :
x 3
2
=
y + 1
1
=
z 1
2
Bi 10.15 (D-2006): Cho im A(1; 2; 3) v 2 ng thng d
1
:
x 2
2
=
y + 2
1
=
z 3
1
v
d
2
:
x 1
1
=
y 1
2
=
z + 1
1
a) Tm ta im A

i xng vi A qua d
1
.
b) Vit phng trnh ng thng i qua A vung gc vi d
1
v ct d
2
.
10.6 Gc
10.6.1 Kin thc c bn
Gc gia ng thng d v mt phng (). t =

(d, ()). Khi
sin =
[

u
d
.

[
[

u
d
[.[

[
Gc gia mt phng () v mt phng (). t =

((), ()).
cos =
[

[
[

[.[

[
Gc gia ng thng d
1
v ng thng d
2
. t =

(d
1
, d
2
).
cos =
[

u
d
1
.

u
d
2
[
[

u
d
1
[.[

u
d
2
[
10.6.2 Cc v d
V d 10.6.1: Tnh gc gia
a) ng thng d :
x 1
2
=
y + 1
3
=
z 1
4
v () : 8x y 3z 1 = 0.
b) Mt phng () : x y + 3z 1 = 0 v () : 2x y z + 5 = 0.
c) ng thng d
1
:
x 1
2
=
y + 1
1
=
z 1
3
v d
2
:
x + 2
3
=
y 1
2
=
z + 1
4
.
Ngi son : Th.s Minh Tun Trang 220 Khoa T nhin - Trng CSP Nam nh
T
h
.
s

M
i
n
h
T
u

n
10.7. Khong cch Chng 10. Hnh khng gian ta
V d 10.6.2: Vit phng trnh mt phng () cha ng thng d :
x 2
1
=
y 3
3
=
z 3
2
v to vi mt phng (P) : 2x y z 1 = 0 mt gc tha mn: cos =

2
3
.
V d 10.6.3: Vit phng trnh mt phng () cha d :
x 2
1
=
y 1
3
=
z 3
5
v to vi
:
x 1
2
=
y + 1
3
=
z 1
1
mt gc sao cho cos =
2

21
10.6.3 Bi tp
Bi 10.16: Tnh gc gia:
a) ng thng d :
x 1
2
=
y + 2
3
=
z 1
1
v :

x = 1
y = 3 +t
z = 2 3t
b) ng thng d l giao tuyn ca 2 mt phng () : x + 3y z 3 = 0 v y z 1 = 0 vi
mt phng (P) : 2x y 3z 4 = 0.
c) Mt phng () : 3x y 2 = 0 v () : x 2y z + 3 = 0.
Bi 10.17: Vit phng trnh mt phng () trong cc trng hp sau:
a) () cha d :
x
1
=
y + 2
3
=
z + 1
5
v to vi mt phng :
x 1
3
=
y + 1
1
=
z 2
2
mt gc
tha mn cos =

31

33
.
b) () cha ng thng d :

x = 1 t
y = t
z = 2 t
v to vi mt phng () : 2x +y z 4 = 0 mt
gc 60
0
.
10.7 Khong cch
10.7.1 Cc cng thc khong cch
Cng thc c bn:
Khong cch gia 2 im A(x
A
, y
A
, z
A
) v B(x
B
, y
B
, z
B
) l
d(A, B) = AB =

(x
A
x
B
)
2
+ (y
A
y
B
)
2
+ (z
A
z
B
)
2
Khong cch t im M(x
0
, y
0
, z
0
) n mt phng () : Ax +By +Cz +D = 0 l
d(M
0
, ()) =
[Ax
0
+By
0
+Cz
0
+D[

A
2
+B
2
+C
2
Ngi son : Th.s Minh Tun Trang 221 Khoa T nhin - Trng CSP Nam nh
T
h
.
s

M
i
n
h
T
u

n
10.7. Khong cch Chng 10. Hnh khng gian ta
Khong cch t im A n ng thng .
d(A, ) =
[[

u
d
,

AM
0
][
[

u
d
[
, M
0

Khong cch gia 2 ng thng cho nhau d
1
, d
2
:
d(d
1
, d
2
) =
[[

u
d
1
,

u
d
2
].

M
1
M
2
[
[[

u
d
1
,

u
d
2
][
Cc khong cch khc:
Khong cch gia 2 ng thng song song d
1
, d
2
:
d(d
1
, d
2
) = d(M
1
, d
2
), M
1
d
1
Khong cch gia 2 mt phng song song (P), (Q):
d((P), (Q)) = d(M, (Q)), M (P) no
Khong cch gia ng thng d v mt phng (P) bit d song song vi (P):
d(d, (P)) = d(M, (P)), vi M d
10.7.2 Cc v d
V d 10.7.1: Cho im A(1; 1; 2) v () : 2x y z + 3 = 0.
a) Tnh khong cch t A n ().
b) Vit phng trnh () song song vi () bit d(A, ()) = 2d(A, ()).
c) Cho () : 4x + 2y + 2z 1 = 0. Tnh d((), ())
V d 10.7.2: Cho A(2; 1; 3), d :
x 1
2
=
y + 1
1
=
z 2
1
.
a) Tnh d(A, d).
b) Cho :

x = 2 + 2t
y = t
z = 1 +t
. Tnh d(d, ()).
c) Cho () : x y z + 1 = 0. Chng minh d song song vi () v tnh d(d, ()).
V d 10.7.3: Cho d
1
=
x 1
1
=
y + 1
1
=
z 2
3
v d
2
:

x = 2 t
y = 3 + 2t
z = 1 t
a) Chng minh d
1
, d
2
cho nhau.
b) Tnh d(d
1
, d
2
).
c) Vit phng trnh () cha d
1
v song song vi d
2
. T chng minh rng d(d
2
, ()) =
d(d
1
, d
2
).
Ngi son : Th.s Minh Tun Trang 222 Khoa T nhin - Trng CSP Nam nh
T
h
.
s

M
i
n
h
T
u

n
10.7. Khong cch Chng 10. Hnh khng gian ta
10.7.3 Bi tp
Bi 10.18: Tnh khong cch gia
a) im A(1; 1; 2) v ng thng d :
x 1
2
=
y + 1
3
=
z 1
2
.
b) im A(2; 1; 1) v mt phng () : x +y 2z 3 = 0.
c) ng thng d
1
:
x 1
2
=
y + 1
3
=
z 3
4
v d
2
:

x = 2 + 3t
y = 1 t
z = 2 +t
.
Bi 10.19: Tnh khong cch gia:
a) Hai mt phng () : 2x y z + 3 = 0 v () : 4x 2y 2z 1 = 0.
b) Hai ng thng d
1
:

x = 1 + 3t
y = 2 t
z = 3 + 2t
v d
2
:
x + 1
3
=
y 1
1
=
z + 2
2
.
c) ng thng d :

x = 3 +t
y = 2t
z = 1 +t
v () : x y 3z + 2 = 0.
Bi 10.20: Tm M d :
x 1
2
=
y + 1
3
=
z 1
1
sao cho
d(M, ()) =

2
7
, () : x 3y 2z + 5 = 0.
Bi 10.21: Tm M d :

x = 2 t
y = 3 + 2t
z = 1 t
sao cho d(M, ) =

6, :

x = 2t
y = t
z = 5 + 4t
.
Bi 10.22: Vit phng trnh mt phng cha ng thng d :

x = 1 +t
y = 2 +t
z = t
sao cho khong
cch t im A(1; 1; 2) n mt phng ny l
2

6
3
.
Bi 10.23: Cho A(2; 5; 3) v ng thng d :
x 1
2
=
y
1
=
z 2
2
. Vit phng trnh mt phng
() cha d sao cho d(A, ())
max
Bi 10.24: Cho A(1; 2; 1), B(2; 1; 3), C(2; 1; 1), D(0; 3; 1). Vit phng trnh mt phng (P)
qua A, B sao cho d(C, (P)) = d(D, (P)).
Bi 10.25: Cho :
x 1
2
=
y
1
=
z + 2
1
v mt phng (P) : x 2y + z = 0. Gi C = (P),
M . Tnh d(M, (P)) bit MC =

6.
Bi 10.26: Cho A(1; 0; 0), B(0; b; 0), C(0; 0; c) trong b, c > 0 v mt phng (P) : yz+1 = 0.
Tm b, c bit (ABC)(P) v d(0, (ABC)) =
1
3
Ngi son : Th.s Minh Tun Trang 223 Khoa T nhin - Trng CSP Nam nh
T
h
.
s

M
i
n
h
T
u

n
10.8. Mt cu v ng trn Chng 10. Hnh khng gian ta
Bi 10.27 (A-2009): Cho mt phng (P) : x2y+2z1 = 0 v 2 ng thng
1
:
x + 1
1
=
y
1
=
z + 9
6
,
2
:
x 1
2
=
y 3
1
=
z + 1
2
. Xc nh ta im M
1
sao cho d(M,
2
) = d(M, (P))
10.8 Mt cu v ng trn
10.8.1 Mt cu
Cc phng trnh
Mt cu tm I(a, b, c), bn knh R c phng trnh chnh tc l:
(x a)
2
+ (y b)
2
+ (z c)
2
= R
2
Phng trnh tng qut: x
2
+y
2
+ 2Ax + 2By +Cz +D = 0.
tm I(A, B, C) v R =

A
2
+B
2
+C
2
D
V d 10.8.1: Cho mt cu c phng trnh x
2
+ y
2
+ z
2
2x 4y 1 = 0. Tm tm v bn
knh ca mt cu.
V d 10.8.2: Vit phng trnh mt cu i qua 4 im A(4; 2; 3), B(5; 4; 0), C(5; 2; 2),
D(1; 7; 9).
V d 10.8.3: Vit phng trnh mt cu i qua 3 im A(1; 5; 9), B(3; 3; 1), C(1; 8; 7)
v c tm thuc mt phng () : 2x 3y +z + 15 = 0.
V d 10.8.4: Vit phng trnh mt cu i qua 2 im A(5; 5; 14), B(2; 13; 3) v c tm
thuc ng thng: d :

x = 1 + 3t
y = 5 + 4t
z = 3 t
V d 10.8.5: Vit phng trnh mt cu i qua 3 im A(1; 2; 7), B(4; 10; 10), C(16; 1; 1) v
c bn knh R = 13.
10.8.2 V tr tng i ca mt phng v mt cu
Xt mt cu tm I bn knh R v mt phng () v t d = d(I, ()). Khi :
(I; R) ct () d < R v giao ca chng l mt ng trn.
(I; R) tip xc vi () d = R.
(I; R) khng ct () d > R.
Tip din ca mt cu: mt phng tip xc vi mt cu th c gi l tip din ca mt cu.
iu kin mt phng () l tip din mt cu (S):
d(I, ()) = R vi I l tm v R l bn knh ca mt cu.
Ngi son : Th.s Minh Tun Trang 224 Khoa T nhin - Trng CSP Nam nh
T
h
.
s

M
i
n
h
T
u

n
10.8. Mt cu v ng trn Chng 10. Hnh khng gian ta
Phng trnh tip din ca mt cu nu bit tip im M
0
(x
0
, y
0
, z
0
) (S):
Nu (S) : (x a)
2
+ (y b)
2
+ (z c)
2
= R
2
th phng trnh tip din c dng
(x
0
a)(x a) + (y
0
b)(y b) + (z
0
c)(z c) = R
2
Nu (S) : x
2
+y
2
+z
2
+ 2Ax + 2By + 2Cz +D = 0 th phng trnh tip din c dng:
x.x
0
+y.y
0
+z.z
0
+ A(x +x
0
) +B(y +y
0
) +C(z +z
0
) +D = 0
Ch : y l phng php tch ta vi cng thc bin i nh sau:
x
2
x.x
0
, 2x x +x
0
, C C
V d 10.8.6: Tm v tr tng i ca mt cu (S) v mt phng () trong cc trng hp
sau:
a) (S) : x
2
+y
2
+z
2
2x + 6y 4z 1 = 0 v () : x + 2y z 3 = 0
b) (S) : x
2
+y
2
+z
2
+ 2x 4y 2z 3 = 0 v () : 2x y 3z + 1 = 0
c) (S) : x
2
+y
2
+z
2
2x + 4y 6z 15 = 0 v () : 3x + 4y + 2z 30 = 0
V d 10.8.7: Vit phng trnh tip din ca mt cu (S) : x
2
+y
2
+z
2
2x+6y 2z 23 = 0
ti im A(4; 7; 2)
V d 10.8.8: Vit phng trnh tip din ca mt cu (S) : x
2
+y
2
+z
2
4x+2y 6z 20 = 0
bit n
a) Cha ng thng d :
x 6
6
=
y + 5
3
=
z 1
2
.
b) Song song vi mt phng 3x 3y + 4z 10 = 0
Tm ta tip im.
V d 10.8.9: Vit phng trnh mt cu i qua 3 im A(4, 5, 7), B(1, 3, 4), C(0, 4, 8)
v tip xc vi mt phng 6x 5y 42 = 0
10.8.3 ng trn
Cho mt phng () : ax+by+cz+d = 0 v mt cu (S) : x
2
+y
2
+z
2
+2Ax+2By+2Cz+D = 0.
Nu d(I, ()) < R th mt phng () ct mt cu (S) theo giao tuyn l mt ng trn
(C) ( I l tm ca mt cu (S) v R l bn knh ca n). Khi h sau c gi l
phng trnh ng trn:

ax +by +cz +d = 0
x
2
+y
2
+z
2
+ 2Ax + 2By + 2Cz +D = 0
Tm J v bn knh r ca (C).
I(A, B, C), R =

A
2
+B
2
+C
2
D.
t h = d(I, ()) =
[a.A b.B c.C +D[

a
2
+b
2
+c
2
v r =

R
2
h
2
.
Gi l ng thng qua I vung gc vi (). Khi J = ().
Ngi son : Th.s Minh Tun Trang 225 Khoa T nhin - Trng CSP Nam nh
T
h
.
s

M
i
n
h
T
u

n
10.8. Mt cu v ng trn Chng 10. Hnh khng gian ta
V d 10.8.10: Cho mt phng (P) : 2x 2y z 4 = 0 v mt cu
(S) : x
2
+y
2
+z
2
2x 4y 6z 11 = 0
a) Chng minh rng mt phng (P) ct mt cu theo mt ng trn (C). Xc nh ta
tm v bn knh ca ng trn (C).
b) Vit phng trnh mt cu cha ng trn (C) v i qua im A(2; 1; 4).
c) Vit phng trnh mt cu cha ng trn (C) v tip xc vi mt phng
(Q) : 2x + 2y + 5z 35 = 0
V d 10.8.11: Tm tm v bn knh ca ng trn ngoi tip tam gic ABC vi A(1; 2; 3),
B(1; 1; 2), C(2; 2; 2).
10.8.4 Bi tp
Bi 10.28: Vit phng trnh mt cu i qua 4 im A(0; 4; 5), B(6; 1; 6), C(4; 7; 4),
D(3; 5; 7). T vit phng trnh tip din vi mt cu ti D.
Bi 10.29: Vit phng trnh mt cu i qua 3 im A(1, 7, 3), B(4, 3, 6), C(2, 1, 2) v c
tm thuc mt phng 2x y z + 3 = 0.
Bi 10.30: Vit phng trnh mt cu i qua 2 im A(2, 2, 1) v B(1, 7, 5) ng thi c tm
nm trn ng thng d :
x + 1
1
=
y + 1
2
=
z
1
Bi 10.31: Vit phng trnh mt cu i qua 3 im A(4, 11, 7), B(1, 1, 10), C(8, 6, 2)
c bn knh R = 13.
Bi 10.32: Vit phng trnh tip din ca mt cu (S) : x
2
+y
2
+z
2
2x + 4y + 2z 23 = 0
trong cc trng hp sau:
a) vung gc vi ng thng () :
x 1
2
=
y + 1
3
=
z 2
4
.
b) cha 2 im A(2, 17, 2) v B(4, 8, 1)
Bi 10.33: Tm tm v bn knh ng trn ngoi tip tam gic ABC vi A(2; 10; 10),
B(3; 2; 7), C(10; 7; 5).
Bi 10.34: Cho mt cu (S) : x
2
+ y
2
+ z
2
2x 4y + 6z 15 = 0 v mt phng (P) :
x + 7y + 8z + 2 = 0.
a) Tm tm v bn knh ca (S). Chng minh (S) ct (P) theo giao tuyn l mt ng trn
(C).
b) Tm tm v bn knh ca ng trn (C).
c) Vit phng trnh mt cu cha ng trn (C) v i qua im A(4; 2; 3),
d) Vit phng trnh mt cu cha ng trn (C) v c bn knh R =

161.
Ngi son : Th.s Minh Tun Trang 226 Khoa T nhin - Trng CSP Nam nh
T
h
.
s

M
i
n
h
T
u

n
10.9. Ta ha hnh hc khng gian Chng 10. Hnh khng gian ta
e) Vit phng trnh mt cu cha ng trn (C) v tip xc vi mt phng
() : x + 4y + 12z 7 = 0
Bi 10.35 (DB1-B-08): Cho mt phng () : 2x y + 2z + 1 = 0 v ng thng
d :
x 1
1
=
y 1
2
=
y
2
Vit phng trnh mt cu c tm thuc d tip xc vi 2 mt phng () v Oxy.
Bi 10.36 (DB2-A-08): Cho mt phng (P) : 2x + 3y 3z + 1 = 0, ng thng
d :
x 3
2
=
y
9
=
z + 5
1
v 3 im A(4; 0; 3), B(1; 1; 3), C(3; 2; 6).
a) Vit phng trnh mt cu (S) i qua 3 im A, B, C v c tm thuc mt phng (P).
b) Vit phng trnh mt phng (Q) cha d v ct mt cu (S) theo mt ng trn c bn
knh ln nht.
Bi 10.37 (DB2-B-2006): Cho mt phng (P) : 2x + y z + 5 = 0 v cc im A(0; 0; 4),
B(2; 0; 0).
a) Vit phng trnh hnh chiu vung gc ca AB trn mt phng (P).
b) Vit phng trnh mt cu i qua O, A, B v tip xc vi (P).
Bi 10.38 (DB1-D-2005): Cho lng tr ng A(2; 0; 0), B(0; 4; 0), O
1
(0; 0; 4). Tm ta A
1
, B
1
v vit phng trnh mt cu i qua 4 im O, A, B, O
1
.
Bi 10.39 (DB2-A-2005): Trong khng gian cho 3 m A(2; 0; 0), B(0; 4; 0), S(0; 0; 4). Tm ta
im B (Oxy) sao cho t gic OABC l hnh ch nht. Vit phng trnh mt cu i qua
4 im O, B, S, C.
Bi 10.40 (A-2010): Cho im A(0; 0; 2) v ng thng :
x + 2
2
=
y 2
3
=
z + 3
2
. Tnh
d(A, ). Vit phng trnh mt cu tm A ct ti 2 im B, C sao cho BC = 8.
10.9 Ta ha hnh hc khng gian
Phng php chung: Pht hin c gc tam din vung v t gc ta ti .
10.9.1 Hnh chp
H ng cao t nh S ca chp. t gc ta ti chn ng cao.
V d 10.9.1 (B-2006): Cho hnh chp S.ABCD c y ABCD l hnh ch nht vi AB = a,
AD = a

2, SA = a vung gc vi y. Gi M, N ln lt l trung im ca AD v SC. Gi


I = BM AC. Chng minh rng (SAC)(SMB). Tnh V
ANIB
.
Ngi son : Th.s Minh Tun Trang 227 Khoa T nhin - Trng CSP Nam nh
T
h
.
s

M
i
n
h
T
u

n
10.9. Ta ha hnh hc khng gian Chng 10. Hnh khng gian ta
Gii: t gc ta ti O A (chn ng cao h t nh S xung y), cc trc Ox AB,
Oy AD, Oz AS.
Khi A(0; 0; 0), S(0; 0; a), B(a; 0; 0), D(0; a

2; 0), C(a; a

2; 0).
M l trung im AD nn M

0;
a

2
; 0

.
N l trung im SC nn N

a
2
;
a

2
;
a
2

.
Ngi son : Th.s Minh Tun Trang 228 Khoa T nhin - Trng CSP Nam nh
T
h
.
s

M
i
n
h
T
u

n
Chng 11. Tch phn
Chng 11
Tch phn
11.1 Vi phn
11.1.1 nh ngha
nh ngha 11.1. Cho hm f : (a; b) R, tha mn c o hm ti im x (a, b).
K hiu df(x) = f

(x).dx gi l vi phn ca hm f ti im x.
V d 11.1.1: d(x
2
) = (x
2
)

.dx = 2x.dx.
d(?) = x
3
.dx th ? =
x
4
4
+C.
11.1.2 Cc tnh cht
Vi u, v l cc hm theo bin x; C, l cc hng s. Ta c cc cng thc sau:
0 d(C) = 0.
O d(u v) = du dv.
O d(u.v) = udv +vdu. c bit d(.u) = .du.
O d

u
v

=
vdu udv
v
2
.
O d(f(u)) = f

(u)du. (Cng thc bt bin vi phn).


11.1.3 Bng o hm cc hm s s cp thng gp
0 (C)

= 0.
O (x

= .x
1
. c bit:
x

= 1

1
x

=
1
x
2
(

x)

=
1
2

x
Ngi son : Th.s Minh Tun Trang 229 Khoa T nhin - Trng CSP Nam nh
T
h
.
s

M
i
n
h
T
u

n
11.2. Nguyn hm v tch phn bt nh Chng 11. Tch phn
O (sin x)

= cos x, (cos x)

= sin x.
O (tan x)

=
1
cos
2
x
= 1 + tan
2
x, (cot x)

=
1
sin
2
x
= (1 + cot
2
x).
O (e
x
)

= e
x
(a
x
)

= a
x
. lna.
O (ln x)

=
1
x
(log
a
(x))

=
1
x. ln a
.
11.2 Nguyn hm v tch phn bt nh
11.2.1 nh ngha
nh ngha 11.2. Cho hm f : (a, b) R xc nh trn khong (a, b). Hm F : (a, b) R c
o hm trn khong (a, b) v tha mn F

(x) = f(x), x (a, b) c gi l nguyn hm ca


hm f trn khong (a, b).
V d 11.2.1: f(x) = sin x th F(x) = cos x l mt nguyn hm ca hm f(x).
V d 11.2.2: f(x) = 1 th F(x) = x + 2 l mt nguyn hm ca hm f(x).
11.2.2 Cc tnh cht
nh l 14. Nu f

(x) = 0 th f(x) l hm hng.


nh l 15. Nu F(x), G(x) l 2 nguyn hm no ca hm f(x) th C sao cho F(x) =
G(x) +C, x.
Chng minh. Theo gi thit ta c F

(x) = G

(x) = f(x), x. Do (F G)

(x) = 0, x. Theo
nh l 14 th C sao cho F(x) G(x) = C hay F(x) = G(x) +C.
nh ngha 11.3. Ta k hiu tch phn bt nh

f(x)dx l h tt c cc nguyn hm ca
hm f(x). Theo nh l 11.2.2, th

f(x)dx = F(x) + C vi F(x) l mt nguyn hm no
ca f(x).
nh l 16. f(x), g(x), u(x) l cc hm v , C l hng s. Ta c cc tnh cht sau:
0

0.dx = C.
O

f(x)dx = .

f(x)dx, R.
O

(f(x) g(x)) dx =

f(x)dx

g(x)dx.
O Nu

f(x)dx = F(x) + C th

f(u)du =

f(u).u

dx = F(u) + C. (Cng thc bt bin


tch phn - Tch phn khng thay i khi ta thay bin x bi mt hm u theo bin x bt k).
Ngi son : Th.s Minh Tun Trang 230 Khoa T nhin - Trng CSP Nam nh
T
h
.
s

M
i
n
h
T
u

n
11.2. Nguyn hm v tch phn bt nh Chng 11. Tch phn
11.2.3 Bng nguyn hm cc hm s s cp thng gp
Vi C l hng s, u l hm bt k theo bin x. Ta c
TT Hm f(x) Nguyn hm ca f(x) Bt bin tch phn
1 0

0.dx = C
2 1

dx = x +C

du = u +C
3 x

dx =
x
+1
+ 1
+C, ( = 1)

u

du =
u
+1
+ 1
+C
4
1
x
dx
x
= ln [x[ +C
du
u
= ln [u[ +C
5 e
x

e
x
dx = e
x
+C

e
u
du = e
u
+C
a
x

a
x
dx =
a
x
ln a
+C

a
u
du =
a
u
ln a
+C
6 sin x

sin xdx = cos x +C

sin udu = cos u +C


7 cos x

cos xdx = sin x +C

cos udu = sin u +C


8
1
cos
2
x
dx
cos
2
x
= tanx +C
du
cos
2
u
= tanu +C
9
1
sin
2
x
dx
sin
2
x
= cot x +C
du
sin
2
u
= cot u +C
V d 11.2.3: Tnh cc tch phn sau:
a) I =
dx
x
2
b) I =

x
3
+ 2x
2
x
1
3

x
+ 5

dx
c) I =

sin
5
xcos xdx.
d) I =

sin
5
xdx
Gii: a) I =

x
2
dx =
x
2+1
2 + 1
=
1
x
b) I =

x
3
+ 2x
2
x x
1/3
+ 5

=
x
4
4
+2.
x
3
3

x
2
2

x
2/3
2/3
+5x+C =
x
4
4
+2.
x
3
3

x
2
2

3
2
.
3

x
2
+
5x +C.
Ngi son : Th.s Minh Tun Trang 231 Khoa T nhin - Trng CSP Nam nh
T
h
.
s

M
i
n
h
T
u

n
11.3. Cc phng php tnh tch phn Chng 11. Tch phn
c) I =

sin
5
x.d(sin x) =
sin
6
x
6
+C.
d) I =

sin
4
x.(sin xdx) =

(1 cos
2
x)
2
.d(cos x) =

(1 + 2 cos
2
x cos
4
x) d(cos x) =
cos x +
2
3
cos
3
x
1
5
cos
5
x +C.
V d 11.2.4: Tnh cc tch phn sau:
a) I =

(2x + 1)
100
dx.
b) I =

e
x
.(e
x
+ 1)
2
dx.
c) I =

(tan x + tan
2
x)dx.
Gii: a) I =
1
2
.

(2x + 1)
100
.d(2x + 1) =
1
2
.
(1 + 2x)
101
101
=
(1 + 2x)
101
202
.
b) I =

(e
x
+ 1)
2
d(e
x
+ 1) =
1
3
(e
x
+ 1)
3
c) I =
sin x
cos x
dx +

(1 + tan
2
x 1)dx
=
d(cos x)
cos x
+
dx
cos
2
x

dx = ln [ cos x[ + tanx x +C.


11.3 Cc phng php tnh tch phn
11.3.1 Php i bin s
Php i xui: Xt I =

f(x)dx. t x = (t) th dx =

(t)dt.
Ta c I =

f((t)).

(t)dt.
Mt s trng hp:
+)

a
2
x
2
: t x = a sin t (

2
t

2
) hoc x = a cos t vi 0 t .
+)

a
2
+x
2
,
1
a
2
+x
2
: t x = a tan t vi

2
< t <

2
.
Php i ngc: Xt I =

f((x)).

(x)dx.
t t = (x) dt =

(x).dx. Khi I =

f(t)dt
Mt s php i ngc:
STT Dng chun Dng thu gn i bin
1

f(x

)..x
1
dx

f(x

).x
1
dx t = x

Ngi son : Th.s Minh Tun Trang 232 Khoa T nhin - Trng CSP Nam nh
T
h
.
s

M
i
n
h
T
u

n
11.3. Cc phng php tnh tch phn Chng 11. Tch phn
2

f(
n

x).
1
n
n

x
n1
dx
dx

f(
n

x)dx t =
n

x
3

f(sin x) cos xdx

f(sin x) cos xdx t = sin x


4

f(cos x)(sin x)dx

f(cos x) sin xdx t = cos x


5

f(tanx).(1 + tan
2
x)dx

f(tanx)dx t = tanx
6

f(cot x).[(1 + cot
2
x)]dx

f(cot x)dx t = tanx


7

f(e
x
).e
x
dx

f(e
x
)dx t = e
x
8

f(lnx)
dx
x

f(lnx)
dx
x
t = ln x
V d 11.3.1: Tnh cc tch phn sau:
a) I =
x
3
dx
x
8
1
b) J =
dx
3

x + 2
c) K =

(tan
3
x 2 tan
2
x + tan x 2) dx.
d) L =

(ln
2
x ln x 1)
dx
x
.
e) M =
dx
e
x
1
.
Gii: a) t t = x
4
dt = 3x
3
dx x
3
dx =
dt
3
.
Do I =
dt
3(t
2
1)
=
1
6

1
t 1

1
t + 1

dt =
1
6
(ln [t 1[ ln [t + 1[)
=
1
6
. ln

x
4
1
x
4
+ 1

+C
b) t t =
3

x + 2 x = (t 2)
3
dx = 3(t 2)
2
dt
J =
3(t 2)
2
dt
t
=

3t 12 +
12
t

dt
=
3t
2
2
12t + 12 ln [t[ =
3 (
3

x + 2)
2
2
12(
3

x + 2) + ln[
3

x + 2[ +C
Ngi son : Th.s Minh Tun Trang 233 Khoa T nhin - Trng CSP Nam nh
T
h
.
s

M
i
n
h
T
u

n
11.3. Cc phng php tnh tch phn Chng 11. Tch phn
c) K =

(tan x 2) (tan
2
x + 1) dx
t t = tanx dt = (tan
2
x + 1)dx
K =

(t 2)dt =
t
2
2
2t =
tan
2
x
2
2 tanx +C
d) t t = ln x dt =
dx
x
L =

(t
2
t 1)dt =
t
3
3

t
2
2
t =
ln
3
x
3

ln
2
x
2
ln x +C
e) M =
e
x
dx
e
x
(e
x
1)
t e
x
= t e
x
dx = dt
M =
dt
t(t 1)
=

1
t 1

1
t

dt = ln [t 1[ ln [t[ = ln(e
x
1) x +C.
11.3.2 Tch phn tng phn
~ Cng thc:

udv = uv

vdu
Trong :
Php bin i Php ton
u du o hm (hay vi phn)
dv v Nguyn hm (hay tch phn)
~ Ch :
- Hai php ton trn l ngc ca nhau
- Php tnh o hm d thc hin hn php tnh nguyn hm.
- Th t u tin t u:
1. Cc hm ngc: ln, arcsin, arctan, cn bc 2, cn bc 3, ...
2. Cc hm a thc: (khi o hm a thc gim i mt bc)
3. Cc hm tnh o hm hay nguyn hm u n gin: e
x
, sin x, cos x, ...
V d 11.3.2: Tnh cc tch phn sau:
a) I =

xsin xdx.
b) J =

x
2
ln xdx.
c) K =

e
x
. sin xdx.
Gii: a) t

u = x
dv = sin xdx

du = dx
v = cos x
Do I = xcos x +

cos xdx = xcos x + sin x +C.


Ngi son : Th.s Minh Tun Trang 234 Khoa T nhin - Trng CSP Nam nh
T
h
.
s

M
i
n
h
T
u

n
11.3. Cc phng php tnh tch phn Chng 11. Tch phn
b) t

u = ln x
dv = x
2
dx

du =
dx
x
v =
x
3
3
Do I =
1
3
x
3
ln x
1
3

x
2
dx =
1
3
x
3
ln x
1
9
x
3
+C.
c) Cch 1:
t

u = e
x
dv = sin xdx

du = e
x
dx
v = cos x
Do K = e
x
. cos x +

e
x
cos xdx.
t

u = e
x
dv = cos xdx

du = e
x
dx
v = sin x
Do K = e
x
cos x +e
x
sin x

e
x
sin xdx = e
x
cos x +e
x
sin x K
K =
1
2
e
x
cos x +
1
2
e
x
sin x +C.
Cch 2: ng nht h s.
Gi F(x) = (a. sin x +b cos x)e
x
l nguyn hm ca f(x) = e
x
sin x.
Ta c F

(x) = [(a b) sin x + (a +b) cos x]e


x
Do ta c h:

a b = 1
a +b = 0

a =
1
2
b =
1
2
Do K =
1
2
(sin x cos x) e
x
+C.
11.3.3 Tch phn hm phn thc
Dng I =
P(x)
Q(x)
dx.
Khng mt tnh tng c th gi s: deg P < deg Q
Tht vy, nu deg P deg Q th ta c P(x) = Q(x).M(x) +R(x).
(M l thng v N l d).
I =

M(x) +
R(x)
Q(x)

vi deg R < deg Q.


cho n gin ta xt tch phn sau:
1.
ax +b
x
2
+px +q
dx vi = p
2
4q > 0 v 2 nghim x
1
, x
2
.
ax +b
x
2
+px +q
=
ax + b
(x x
1
)(x x
2
)
=
A
x x
1
+
B
x x
2
(Cc bn t tng qut cho trng hp Mu l mt a thc bc n c n nghim phn bit)
2.
ax +b
x
2
+px +q
dx vi = p
2
4q = 0 v nghim kp x
0
.
Ngi son : Th.s Minh Tun Trang 235 Khoa T nhin - Trng CSP Nam nh
T
h
.
s

M
i
n
h
T
u

n
11.3. Cc phng php tnh tch phn Chng 11. Tch phn
ax +b
x
2
+px +q
=
ax +b
(x x
0
)
2
=
A
x x
0
+
B
(x x
0
)
2
.
(Cc bn t tng qut cho trng hp Mu l mt a thc bc n c nghim bi n)
3.
ax +b
x
2
+px +q
dx. vi = p
2
4q < 0.
ax +b
x
2
+px +q
=
ax +b
(x +m)
2
+n
(vi n > 0)
=
A(2x +p)
x
2
+px +q
+
B
(x +m)
2
+n
.
V d 11.3.3: Tnh cc tch phn sau:
a) I =
x
3
+ 2x + 3
x
2
x 6
dx.
b) I =
3x
2
x 1
x
3
3x
2
+ 4
dx
Gii: a) Ta c
x
3
+ 2x + 3
x
2
x 6
= x + 1 +
10x + 9
x
2
x 6
.
Mt khc
10x + 9
x
2
x 6
=
10x + 9
(x 3)(x + 2)
=
A
x 3
+
B
x + 2
=
(A +B)x + 2A3B
x
2
x 6
.
ng nht h s ta c:

A+B = 10
2A3B = 9

A =
39
5
B =
11
5
Do I =
1
2
x
2
+x +
39
5
dx
x 3
+
11
5
dx
x + 2
=
1
2
x
2
+x +
39
5
ln [x 3[ +
11
5
ln [x + 2[ +C.
b) Ta c
3x
2
x 1
x
3
3x
2
+ 4
=
3x
2
x 1
(x + 1)(x 2)
2
=
A
x + 1
+
B
x 2
+
C
(x 2)
2
=
(A +B)x
2
+ (4AB +C)x + 4A 2B +C
(x + 1)(x 2)
2
ng nht h s ta c:

A+B = 3
4AB +C = 1
4A2B +C = 1

A =
1
3
B =
8
3
C = 3
Vy I =
1
3
dx
x + 1
+
8
3
dx
x 2
+ 3
dx
(x 2)
2
=
1
3
ln [x + 1[ +
8
3
ln [x 2[
3
x 2
+C.
Ngi son : Th.s Minh Tun Trang 236 Khoa T nhin - Trng CSP Nam nh
T
h
.
s

M
i
n
h
T
u

n
11.4. Tch phn xc nh Chng 11. Tch phn
11.4 Tch phn xc nh
Cch tnh gn ging nh tch phn bt nh, ta ch vic thm bc th cn.
V d 11.4.1: Tnh tch phn sau:
a) I =
2

0
(x
3
+x 1)dx
x
2
+ 4
b) J =
9

0
(2x 1)

9 x
2
dx
Gii: a) I =
2

x
3x + 1
x
2
+ 4

=
1
2
x
2
[
2
0
3
2

0
xdx
x
2
+ 4

0
dx
x
2
+ 4
= 2 3I
1
I
2
.
Vi I
1
=
2

0
xdx
x
2
+ 4
i bin t = x
2
+ 4 dt = 2xdx xdx =
dt
2
.
Cn: x = 0 t = 4, x = 2 t = 8.
I
1
=
8

4
dt
2t
=
1
2
ln t [
8
4
= ln 2.
I
2
=
2

0
dx
x
2
+ 4
.
i bin x = 2 tant dx = 2(1 + tan
2
t)dt.
Cn x = 0 t = 0, x = 2 t =

4
.
Do I
2
=

0
2(1 + tan
2
t)dt
4 + 4 tan
2
t
=
t
2

4
0
=

8
Vy I = 2 3 ln 2

8
b) J =
3

0
2x

9 x
2
dx
3

9 x
2
dx = J
1
J
2
.
Xt J
1
=
3

0
2x

9 x
2
dx.
i bin t =

9 x
2
t
2
= 9 x
2
2tdt = 2xdx xdx = tdt.
Cn: x = 0 t = 3, x = 3 t = 0.
Do J
1
= 2
0

3
t.(tdt) =
2t
3
3
[
0
3
= 18
Xt J
2
=
3

9 x
2
dx.
i bin x = 3 sint dx = 3 cos tdt.
Ngi son : Th.s Minh Tun Trang 237 Khoa T nhin - Trng CSP Nam nh
T
h
.
s

M
i
n
h
T
u

n
11.5. ng dng ca tch phn Chng 11. Tch phn
Cn: x = 0 t = 0, x = 3 t =

2
.
J
2
=

9 9 sin
2
t cos tdt =

0
3 cos
2
tdt =
3
2

0
(1 + cos 2t)dt
=
3
2

t +
1
2
sin 2t

2
0
=
9
4
Vy J = 18
9
4
V d 11.4.2 (A-2008): Tnh tch phn sau: I =

0
tan
4
xdx
cos 2x
Gii: I =

0
tan
4
xdx
1 tan
2
x
1 + tan
2
x
=

0
tan
4
x.(1 + tan
2
x)dx
1 tan
2
x
.
i bin t = tanx dt = (1 + tan
2
x)dx
Cn: x = 0 t = 0, x =

6
t =
1

3
I =
1

0
t
4
dt
1 t
2
=
1

t
2
1
1
t
2
1

dt =

t
3
3
t

3
0

1
2
1

1
t 1

1
t + 1

dt
=
10

3
27

1
2
ln

t 1
t + 1

3
0
=
10

3
27
+
1
2
ln

3 + 1

3 1

V d 11.4.3 (B-2009): Tnh tch phn sau: I =


3

1
3 + ln x
(x + 1)
2
dx
Gii: t

u = 3 + ln x
dv =
dx
(x + 1)
2

du =
dx
x
v =
1
x + 1
Do I =
3 + ln x
x + 1
[
3
1
+
3

1
dx
x(x + 1)
=
3
4

1
4
ln 3 +
3

1
x

1
x + 1

dx
=
3
4

1
4
ln 3 + ln

x
x + 1

[
3
1
=
3
4

1
4
ln 3 + ln 3 ln 2 =
3
4
+
3
4
ln 3 ln 2
11.5 ng dng ca tch phn
Cng thc c bn: S(D) =
b

a
f(x)dx
V
Ox
(D) =
b

a
f
2
(x)dx
Vi D = (x, y) [a x b, 0 y f(x)
Ngi son : Th.s Minh Tun Trang 238 Khoa T nhin - Trng CSP Nam nh
T
h
.
s

M
i
n
h
T
u

n
11.5. ng dng ca tch phn Chng 11. Tch phn
11.5.1 Tnh din tch
V d 11.5.1 (A-2007): Tnh din tch hnh phng gii hn bi cc ng y = (e + 1)x v
y = (1 +e
x
)x
Gii: Xt phng trnh: (e + 1)x = (1 +e
x
)x

x = 0
e
x
= e x = 1
Ch rng nu x =
1
2
th y
1
=
e + 1
2
cn y
2
=
e
1/2
+ 1
2
v y
1
> y
2
nn
S =
1

0
[(e + 1)x (1 +e
x
)x] dx =
1

0
(e.x x.e
x
) dx = e.
x
2
2
[
1
0
I
I =
1

0
x.e
x
dx
t

u = x
dv = e
x
dx

du = dx
v = e
x
Do I = x.e
x
[
1
0

1

0
e
x
dx = e e
x
[
1
0
= e (e 1) = 1
Vy S =
e
2
1 (vdt)
11.5.2 Tnh th tch vt th trn xoay
V d 11.5.2 (B-2007): Cho hnh phng (H) gii hn bi cc ng: y = xln x, y = 0, x = e.
Tnh th tch vt th trn xoay khi quay hnh (H) quanh trc Ox.
Gii: Xt phng trnh xln x = 0 x = 1 (v x > 0).
Do V =
e

1
x
2
ln
2
xdx = I
t

u = ln
2
x
dv = x
2
dx

du = 2
ln x
x
dx
v =
x
3
3
Do I =
x
3
ln
2
x
3
[
e
1

2
3
e

1
x
2
ln xdx =
e
3
3

2
3
I
1
.
I
1
=
e

1
x
2
lnxdx

u = ln x
dv = x
2
dx

du =
dx
x
v =
x
3
3
I
1
=
x
3
ln x
3
[
e
1

1
3
e

1
x
2
dx =
e
3
3

1
9
x
3
[
e
1
=
2e
3
+ 1
9
Do I =
e
3
3

4e
3
+ 2
27
=
5e
3
2
27
Vy V
Ox
(H) =
5e
3
2
27

Ngi son : Th.s Minh Tun Trang 239 Khoa T nhin - Trng CSP Nam nh
T
h
.
s

M
i
n
h
T
u

n
11.6. Bi tp Chng 11. Tch phn
11.6 Bi tp
Tm nguyn hm
Bi 11.1: Xc nh a, b, c, d sao cho F(x) = (ax
3
+ bx
2
+ cx + d).e
x
l mt nguyn hm ca
hm s f(x) = (2x
3
x
2
+ 1)e
x
.
Bi 11.2: Xc nh a, b, c sao cho F(x) = (ax
2
+bx +c).

2x 3 l mt nguyn hm ca hm
s: f(x) =
2x
2
x 1

2x 3
Tm G(x) l nguyn hm ca f(x) tha mn G(2) = 1.
Bi 11.3: Xc nh a, b, c, d sao cho F(x) = (ax + b) sin x + (cx + d) cos x l nguyn hm ca
hm s f(x) = (x + 2) sinx + (2x 1) cos x
Bi 11.4: Tm nguyn hm ca cc hm s:
a) f(x) = cos
2
x
2
b) f(x) = sin
3
2x.
Bi 11.5: Tnh cc tch phn:
a)

e
x
.(2 e
x
)dx.
b)
e
x
2
x
dx
c)
2
x
.3
x
.5
x
10
x
dx.
d)
e
25x
+ 1
e
x
dx.
e)
e
x
e
x
+ 2
dx
Bi 11.6: Tnh cc tch phn:
a)

x
4
+x
4
+ 2dx.
b)

3

x
5

xdx.
c)

(1 2x)
2001
dx.
d)

x

x
2
+ 4dx.
e)


5 4 ln x
x
dx
Bi 11.7: Tm h nguyn hm ca cc hm s:
a) f(x) = (1 2x
2
)
3
.
Ngi son : Th.s Minh Tun Trang 240 Khoa T nhin - Trng CSP Nam nh
T
h
.
s

M
i
n
h
T
u

n
11.6. Bi tp Chng 11. Tch phn
b) f(x) =
2

x x
3
.e
x
3x
2
x
3
.
c) f(x) =
(2 +

x)
3

x
dx.
d) f(x) =
1

3x + 5

3x 1
.
e) f(x) =
x + 1
x
2
6x + 5
.
f) f(x) =
4x
2
+ 6x + 1
2x + 1
.
g) f(x) =
4x
3
+ 4x
2
1
2x + 1
.
h) f(x) =
4x
3
+ 9x + 1
9 4x
2
.
i) f(x) = (sin x + cos x)
2
.
j) f(x) = cos

2x

3

. cos

2x +

.
k) f(x) = cos
3
x + cos
4
x + sin
4
x.
l) f(x) = sin
6
2x + cos
6
2x.
Bi 11.8: Tm h nguyn hm ca cc hm s sau:
a) f(x) = x
2
(x 1)
9
.
b) f(x) =
x
4
x
10
4
.
c) f(x) =
x
2
x
(x 2)
3
.
d) f(x) =
x
2
1
x
4
+ 1
.
e) f(x) =
2x
x +

x
2
1
.
f) f(x) =
1

(x
2
+a
2
)
3
(a > 0).
g) f(x) =
1
3

x
2

x
Bi 11.9: Tm h nguyn hm ca cc hm s:
Ngi son : Th.s Minh Tun Trang 241 Khoa T nhin - Trng CSP Nam nh
T
h
.
s

M
i
n
h
T
u

n
11.6. Bi tp Chng 11. Tch phn
a) f(x) =
cos
5
x
3

sin x
+
1
cos x
b) f(x) =
cos
3
x
sin x
+
1
sin
4
x
.
c) f(x) =
sin x + cos x
3

sin x cos x
.
Bi 11.10: Tm h nguyn hm ca cc hm s:
a) f(x) =
1

1 +e
2x
dx.
b) f(x) =
x + 1
x(1 +x.e
x
)
.
c) f(x) =
2
x
.3
x
9
x
4
x
.
d) f(x) =
1
xln xln(ln x)
.
Bi 11.11: Tm h nguyn hm ca cc hm s sau:
a) f(x) = ln x
b) f(x) = (x
2
+ 1)e
2x
.
c) f(x) = x
2
. sin x.
d) f(x) = e
3x
. sin 2x.
e) f(x) = x. cos

x.
f) f(x) = e
x
.(1 + tan x + tan
2
x).
Bi 11.12: Tm h nguyn hm ca hm s:
a) f(x) = e

x
.
b) f(x) =

lnx
x

2
.
c) f(x) = (x + 1)
2
cos
2
x.
d) f(x) = e
2x
. cos 3x.
e) f(x) = sin(ln x).
f) f(x) =

x
2
+a vi a = 0.
Bi 11.13: Tm h nguyn hm ca hm s:
a) f(x) = x
3
. ln x
Ngi son : Th.s Minh Tun Trang 242 Khoa T nhin - Trng CSP Nam nh
T
h
.
s

M
i
n
h
T
u

n
11.6. Bi tp Chng 11. Tch phn
b) f(x) = (x
2
+x 1). sin 2x
c) f(x) = x. sin

x
Bi 11.14: Tnh cc tch phn sau:
a)
dx
4x
2
+ 8x + 3
.
b)
dx
x
2
7x + 10
.
c)
dx
3x
2
2x 1
.
d)
2x 7
x
2
3x + 2
dx.
e)
5x 7
x
2
3x + 2
dx.
f)
2x + 7
x
2
+ 5x + 6
dx.
g)
2x + 5
9x
2
6x + 1
dx.
Bi 11.15: Tnh cc tch phn sau:
a)
xdx
(x + 1)(2x + 1)
.
b)
2x
2
+ 41x 91
(x 1)(x
2
x 12)
dx.
c)
dx
6x
3
7x
2
3x
.
d)
x
3
1
4x
3
x
dx.
e)
(x
3
3x + 2)dx
x.(x
2
+ 2x + 1)
.
f)
(x + 2)
2
dx
x(x
2
2x + 1)
.
Bi 11.16: Tnh cc tch phn:
a)
xdx
x
4
3x
2
+ 2
.
b)
x
7
dx
(x
4
+ 1)
2
.
Ngi son : Th.s Minh Tun Trang 243 Khoa T nhin - Trng CSP Nam nh
T
h
.
s

M
i
n
h
T
u

n
11.6. Bi tp Chng 11. Tch phn
c)
xdx
x
4
2x
2
1
.
d)
x
5
dx
x
6
x
3
2
.
e)
2dx
x(x
2
+ 1)
.
f)
dx
x(x
1
0 + 1)
2
.
g)
x
2
1
x
4
+ 1
dx.
h)
x
3
dx
(x
2
+ 1)
2
.
i)
x
2
dx
(1 x)
10
Bi 11.17: Cho hm s: f(x) =
2x
2
+ 2x + 5
x
3
3x + 2
.
a) Tm m, n, p f(x) =
m
(x 1)
2
+
n
x 1
+
p
x + 2
.
b) Tm h nguyn hm ca f(x).
Bi 11.18: Tm h cc nguyn hm ca hm s:
a) f(x) =
x
4
2
x
3
x
.
b) f(x) =
1
x
3
x
.
Bi 11.19: Cho hm s f(x) =
3x
2
+ 3x + 3
x
3
3x + 2
.
a) Xc nh a, b, c sao cho f(x) =
a
(x 1)
2
+
b
x 1
+
c
x + 2
.
b) Tm h nguyn hm ca f(x).
Bi 11.20: Tm h nguyn hm ca hm s.
a) f(x) =
x
2001
(1 +x
2
)
1002
.
b) f(x) =
1
x(x
1999
+ 2000)
.
Ngi son : Th.s Minh Tun Trang 244 Khoa T nhin - Trng CSP Nam nh
T
h
.
s

M
i
n
h
T
u

n
11.6. Bi tp Chng 11. Tch phn
c) f(x) =
x
2
1
(x
2
+ 5x + 1)(x
2
3x + 1)
.
Bi 11.21: Tm h nguyn hm ca hm s:
a) f(x) =
1
cos x. cos

x +

b) f(x) =
1

2 + sin x cos x
.
c) f(x) =
cos
2
x
sin x +

3 cos x
.
d) f(x) =
sin x
1 + sin 2x
.
e) f(x) = sin xsin 2xcos 5x.
f) f(x) = (sin 4x + cos 4x) . (sin 6x + cos 6x).
g) f(x) = sin

x

4

.(2 + sin 2x).


Bi 11.22: Tm h cc nguyn hm sau:
a) f(x) =
sin
3
x
3 sin 4x sin 6x 3 sin 2x
.
b) f(x) = cos 5xtan x + cos 3xtan x.
c) f(x) =
1
sin 2x 2 sin x
.
d) f(x) =
x
sin
2
x
.
e) f(x) =
cot x
1 + sin x
.
f) f(x) = tan

x +

. cot

x +

.
g) f(x) = (x
2
x 3) sin 3x.
Bi 11.23: Tm h nguyn hm ca cc hm s:
a)
x + 1
3

3x + 1
.
b)
x

2x + 1 + 1
.
Ngi son : Th.s Minh Tun Trang 245 Khoa T nhin - Trng CSP Nam nh
T
h
.
s

M
i
n
h
T
u

n
11.6. Bi tp Chng 11. Tch phn
c)
x
3

x + 2
.
d)
x
3
1 +
3

x
4
+ 1
.
e)
1
3

x +

x
.
f)
1
3

(2x + 1)
2

2x + 1
g)
x
10

x + 1
.
h) tanx +
1

2x + 1 +

2x 1
Bi 11.24: Tm h nguyn hm ca hm s:
a)
x

9x
2
6x
.
b)
1

x
2
+ 2x + 3
.
c)
1

x
2
+ 6x + 8
.
d)
1

x
2
x 1
.
e)
4x + 5

x
2
+ 6x + 1
.
f)
2x
x +

x
2
1
.
g)
x
2
+ 1
[x[

x
4
+ 1
.
h)
x

1 +x
2
+

(1 +x
2
)
3
.
Bi 11.25: a) Bit rng:
dx

x
2
+ 3
= ln(x +

x
2
+ 3) +C.
Tm nguyn hm ca hm F(x) =

x
2
+ 3.
b) Tnh

x
2
4x + 8dx
Ngi son : Th.s Minh Tun Trang 246 Khoa T nhin - Trng CSP Nam nh
T
h
.
s

M
i
n
h
T
u

n
11.6. Bi tp Chng 11. Tch phn
Bi 11.26: Tm h nguyn hm ca hm s:
a)
1

(x
2
+ 16)
3
.
b)
1

(1 x
2
)
3
.
c)
1
(x 1)

1 x
2
.
d)
x 1
(x + 1)

x
2
+ 1
.
e)
1
(x 1)

x
2
+ 2x + 3
.
f)
1
x +

x
2
+x + 1
.
g)
x
2

x
2
+x + 1
.
h)
1
1 +

x +

1 +x
.
Bi 11.27: Tm h nguyn hm ca hm s:
a) 2
x
.e
x
.
b)
1
1 +e
x
.
c)
1 +x
x.(1 +x.e
x
)
.
d)

ln x
x
.
e) e
x
. sin(e
x
).
f)
e
2x
e
2x
+ 2
.
g)
1
xln x
.
h) x.e
x
2
.
Bi 11.28: Tm h nguyn hm ca cc hm s sau:
a)
e
2x
1
e
x
.
Ngi son : Th.s Minh Tun Trang 247 Khoa T nhin - Trng CSP Nam nh
T
h
.
s

M
i
n
h
T
u

n
11.6. Bi tp Chng 11. Tch phn
b) (1 +e
3x
)
2
.e
3x
.
c)
e
2x
4

e
x
+ 1
.
d)
1

1 +e
x
.
e)

e
2x
4

e
x
+ 1
.
f)
e

x
.
g)
sin x
e
cos x
.
h)
1
e
x
(3 +e
x
)
.
Bi 11.29: Tm h nguyn hm ca cc hm s sau:
a) x
2
.e
3x
.
b) e
2x
. cos 3x.
c) e
x
. sin x.
d)

ln x
x

3
.
e) x
n
. ln x. (n = 1).
Bi 11.30: Tm h nguyn hm ca cc hm s:
a)
x
2
e
x
(x + 2)
2
.
b)
(1 + sin x)e
x
1 + cos x
.
c)

e
x
+e
x
+ 2.
d)
1
1 x
2
ln
1 +x
1 x
.
e) ln

x +

x
2
1

.
f)
ln x
x.

ln x + 1
.
g)
xln(x +

1 +x
2
)

x
2
+ 1
.
Ngi son : Th.s Minh Tun Trang 248 Khoa T nhin - Trng CSP Nam nh
T
h
.
s

M
i
n
h
T
u

n
11.6. Bi tp Chng 11. Tch phn
Tnh tch phn xc nh
Bi 11.31: Tnh cc tch phn
a)
4

1
dx

x
.
b)
1

0
x

1 xdx.
c)
1

0
x
2
2x 3
2 x
dx.
d)
2

1
dx

x + 1 +

x 1
.
Bi 11.32: Tnh cc tch phn:
a)

0
4 sin
3
xdx
1 + cos x
.
b)

0
tan
2
2x(1 + tan
2
2x)dx .
c)
ln2

0
e
x
dx
(e
x
+ 1)
2
.
d)
e
3

1
dx
x

1 + ln x
.
Bi 11.33: Tm cc gi tr ca a c ng thc:
2

1
[a
2
+ (4 4a)x + 4x
3
]dx = 12
Bi 11.34: Cho 2 hm s f(x) = 4 cos x + 3 sin x v g(x) = cos x + 2 sinx.
a) Tm cc s A, B sao cho g(x) = Af(x) +Bf

(x).
b) Tnh I =

0
g(x)dx
f(x)
.
Bi 11.35: Tm cc hng s A, B hm s f(x) = Asin x +B tha mn ng thi cc iu
kin:
f

(1) = 2,
2

0
f(x)dx = 4.
Bi 11.36: Tnh cc tch phn sau:
a)

0
e
x
sin 3xdx.
Ngi son : Th.s Minh Tun Trang 249 Khoa T nhin - Trng CSP Nam nh
T
h
.
s

M
i
n
h
T
u

n
11.6. Bi tp Chng 11. Tch phn
b)
1

0
(x + 1)
2
e
x
dx.
c)
e

1
(x. ln x)
2
dx.
d)
1

0
xln(1 +x
2
)dx.
e)

0
cos x. ln(1 + cos x)dx.
f)
e

1
e
ln xdx
(1 +x)
2
.
Bi 11.37: Tnh cc tch phn:
a)
5

3
([x + 2[ [x 2[) dx.
b)
1

1
([2x 1[ [x[)
2
dx.
c)
4

x
2
6x + 9dx.
d)
1

4 [x[dx.
e)
1

[x[ xdx.
f)
3

0
[2
x
4[dx.
g)
3

x
3
2x
2
+xdx.
Bi 11.38: Tnh cc tch phn:
a)

2
[ sin x[dx
b)

1 + sin xdx.
c)

1 sin 2xdx.
d)
2

1 + sin x.
Ngi son : Th.s Minh Tun Trang 250 Khoa T nhin - Trng CSP Nam nh
T
h
.
s

M
i
n
h
T
u

n
11.6. Bi tp Chng 11. Tch phn
Bi 11.39: Tnh cc tch phn:
a)
2

0
max(x, x
2
)dx.
b)
2

1
max(1, x
2
)dx
c)
2

0
max(x, x
3
)dx.
d)

0
max(sin x, cos x)dx
Bi 11.40: Tnh cc tch phn:
a)
1

1 x
2
dx
1 + 2
x
.
b)

2
x + cos x
4 sin
2
x
dx.
c)

0
x. sin
3
xdx.
d)

pi
sin
2
xdx
3
x
+ 1
.
e)

2
x
2
[ sin
2
x[dx
1 + 2
x
.
f)
1

1
x
4
+ sin x
1 +x
2
dx.
g)

0
cos
7
xdx
sin
7
x + cos
7
x
Bi 11.41: Tnh cc tch phn:
a)
3

0
x
4
1
x
2
+ 9
dx.
b)
1

1
xdx
(x + 2)
3
.
c)
5

1
(2x
2
+ 18)dx
(x
2
6x + 13)
2
.
d)
5

0
x
9
dx
(x
5
+ 1)
3
.
Ngi son : Th.s Minh Tun Trang 251 Khoa T nhin - Trng CSP Nam nh
T
h
.
s

M
i
n
h
T
u

n
11.6. Bi tp Chng 11. Tch phn
e)
4

0
x
1
5dx
4

(x
8
+ 1)
2
.
f)
1

0
(1 +x)
n
dx.
g)
1

0
x(1 x
2
)
n
dx.
Bi 11.42: Tnh cc tch phn sau:
a)
2

1
x
3
dx
x
8
+ 1
b)
1
2

0
x
3
dx
x
2
3x + 2
.
c)
2

0
dx
x(x
4
+ 1)
.
d)
tan a

1
e
xdx
1 + x
2
+
cot a

1
e
dx
x(1 +x
2
)
vi tan a > 0.
e)
b

0
(a x
2
)dx
(a +x
2
)
2
vi a, b > 0.
f)

2+

6
2

1
x
2
+ 1
x
4
+ 1
dx.
g)
1+

5
2

1
1 +x
2
x
4
x
2
+ 1
dx.
Bi 11.43: Tnh cc tch phn sau:
a)

0
cos 2xdx
sin 2x + cos 2x
.
b)

0
4 sin
3
xdx
1 + cos
4
x
.
c)

0
dx
sin
2
x + 2 sin xcos x 8 cos
2
x
.
d)

0
sin xdx
sin
6
x + cos
6
x
e)

4
sin
6
x + cos
6
x
6
x
+ 1
dx.
Ngi son : Th.s Minh Tun Trang 252 Khoa T nhin - Trng CSP Nam nh
T
h
.
s

M
i
n
h
T
u

n
11.6. Bi tp Chng 11. Tch phn
f)

0
cos 2xdx
(sin x + cos x + 2)
3
.
g)

0
sin x + 7 cos x + 6
4 sin x + 3 cos x + 5
dx.
h)

0
sin xcos xdx

a
2
cos
2
x +b
2
sin
2
x
vi a, b = 0.
Bi 11.44: Tnh cc tch phn sau:
a)

6
cos
3
xdx

sin x
.
b)

0
cos x sin x

2 + sin 2x
dx.
c)

3
cot x
3

sin
3
x sin xdx
sin
3
x
.
d)

0
x. sin x. cos
3
xdx.
e)

3
xsin xdx
cos
2
x
.
f)

0
(x cos
4
x) sin
3
xdx.
Bi 11.45: Tm 2 s A, B hm s f(x) =
sin 2x
(2 + sin x)
2
c th biu din di dng: f(x) =
A
cos x
(2 + sin x)
2
+B.
cos x
2 + sin x
.
T tnh:
0

2
f(x)dx.
Bi 11.46: Tnh cc tch phn sau:
a)

0
x
2
. cos xdx.
b)

2
4
cos
2
(

x)dx.
c)

4
xdx
sin
2
x
.
Ngi son : Th.s Minh Tun Trang 253 Khoa T nhin - Trng CSP Nam nh
T
h
.
s

M
i
n
h
T
u

n
11.6. Bi tp Chng 11. Tch phn
d)

0
x. tan
2
xdx.
e)

3
8

0
sin
3

xdx.
f)
2

0
x
2
. sin
x
2
dx.
Bi 11.47: Tnh cc tch phn:
a)

cos x

sin x

dx.
b)

0
cos
n
xdx
cos
n
x + sin
n
x
.
c)

0
5 cos x 4 sinx
(cos x + sin x)
3
dx.
d)

0
3 sin x + 4 cos x
3 sin
2
x + 4 cos
2
x
dx.
Bi 11.48: t I =

0
sin
2
xdx
sin x +

3 cos x
. v J =

0
cos
2
xdx
sin x +

3 cos x
a) Tnh I 3J, I +J.
b) T cc kt qu trn hy tnh cc gi tr ca I, J v K =
5
3

3
2
cos 2xdx
cos x

3 sin x
Bi 11.49: a) Chng minh rng

0
cos
6
x. cos 6xdx =

0
cos
5
x. sin x. sin 6xdx.
b) Tnh J =

0
cos
5
x. cos 7xdx
Bi 11.50: Tnh cc tch phn sau:
a)
3

2
3

x 1
x + 1
dx
(x 1)
2
b)
6

x 4
x + 2
dx
x + 2
.
c)
1

x
4 x
(x 2)dx.
Ngi son : Th.s Minh Tun Trang 254 Khoa T nhin - Trng CSP Nam nh
T
h
.
s

M
i
n
h
T
u

n
11.6. Bi tp Chng 11. Tch phn
d)
1

1 +x
1 x
dx.
e)
1

0
dx
(1 +x
m
)
m

1 +x
m
. (m N

)
Bi 11.51: Tnh cc tch phn sau:
a)
4

2
dx
x

16 x
2
.
b)
6

3
dx
x

x
2
9
.
c)
1

0
x
3
.

1 +x
2
dx.
d)

1
x
2
.

4 x
2
dx.
e)
2

0
x

(x
2
+ 4)
3
dx.
f)

3
2

0
x
2
.

(3 x
2
)
3
.
Bi 11.52: Tnh cc tch phn sau:
a)
4

4 x
2
x
dx.
b)
1

1 x
2
dx
x
2
.
c)
1
2

1
4
dx

x x
2
.
d)
1

0
x
2
dx

2x x
2
.
e)
a

0
x
2
.

a
2
x
2
dx.
f)
2a

0
x

2ax x
2
dx.
Ngi son : Th.s Minh Tun Trang 255 Khoa T nhin - Trng CSP Nam nh
T
h
.
s

M
i
n
h
T
u

n
11.6. Bi tp Chng 11. Tch phn
g)
n

a
2

0
x
n1
dx

a
2
x
2n
.
Bi 11.53: Tnh cc tch phn sau:
a)

0
dx

x + 3 +

x + 1
b)
1

1
dx
1 +x +

1 +x
2
.
c)
2

1
dx
x
2
.(

x
2
+ 1 +x)
.
d)
8

4
(2x + 1)dx

x
2
4x +x + 2
.
Bi 11.54: Tnh cc tch phn sau:
a)
ln2

1 e
2x
dx.
b)
ln5

0
e
x
.

e
x
1dx
e
x
+ 3
.
c)

1
dx
x.

1 ln
2
x
.
d)
e

1
dx
x(1 + ln
2
x)
.
e)
e

1 + ln
2
xdx
x
.
f)
e

1
ln x.
3

1 + ln
2
xdx
x
.
Bi 11.55: Tnh cc tch phn sau:
a)
2

1
ln xdx
x
2
.
b)
e
2

1
ln
2
x

1
ln x

dx.
c)
e
3

e
2
ln(lnx)dx
x
.
d)
1

0
ln(x + 1)dx

x + 1
.
Ngi son : Th.s Minh Tun Trang 256 Khoa T nhin - Trng CSP Nam nh
T
h
.
s

M
i
n
h
T
u

n
11.6. Bi tp Chng 11. Tch phn
e)
e

1
ln xdx
(x + 1)
2
.
f)

6
ln(sin x)dx
cos
2
x
.
Bi 11.56: Tnh cc tch phn sau:
a)

0
log
2
(1 + tan x)dx.
b)

0
ln(1 + tan x)dx.
c)

0
ln
(1 + sin x)
1+cos x
dx
1 + cos x
.
d)
1

0
x.e
x
.dx
(1 +e
x
)
3
.
ng dng ca tch phn
Bi 11.57: Cho Parabol (P) : y = x
2
4x + 3 v ng thng (d) : y = x 1. Tnh din tch
gii hn bi:
a) (P) v trc Ox.
b) (P), trc Ox v trc Oy.
c) (P), trc Ox, x = 2, x = 4.
d) (P) v (d).
e) (P), (d), x = 0, x = 2.
Bi 11.58: Tnh din tch gii hn bi cc ng:
a) (C) : y = x +
1
2x
2
.
b) y = x(x + 1)
5
, trc Ox, trc Oy v x = 1.
c) 2(y 1)
2
= x v (y 1)
2
= x 1.
d) y = x
2
2x + 2, y = x
2
+ 4x + 5, y = x
2
4x + 3 v y = 1.
e) y =
x
2
8
, y =
1
x
, y =
8
x
vi x > 0 .
Bi 11.59: Tnh din tch gii hn bi:
a) (C) : y = x
2
2x v tip tuyn vi (C) ti O(0, 0) v A(3, 3) trn (C).
Ngi son : Th.s Minh Tun Trang 257 Khoa T nhin - Trng CSP Nam nh
T
h
.
s

M
i
n
h
T
u

n
11.6. Bi tp Chng 11. Tch phn
b) (C) : y = x
3
2x
2
+ 4x 3, y = 0 v tip tuyn vi (C) ti im c honh x = 2.
Bi 11.60: Cho Parabol (P) : y
2
= x v ng trn x
2
+y
2
4x +
9
4
= 0.
a) Chng t (P) v (C) tip xc nhau ti A v B.
b) Tnh din tch hnh phng gii hn bi (P) v cc tip tuyn chung ti A, B.
Bi 11.61: ng thng (d) : x 3y + 5 = 0 chia ng trn (C) : x
2
+ y
2
= 5 thnh 2 phn.
Tnh din tch mi phn.
Bi 11.62: Tnh din tch hnh phng gii hn bi cc ng:
a) y = x
2
, y =

x
b) x y
3
+ 1 = 0, x +y 1 = 0.
c) x
2
+y
2
= 8, y
2
= 2x.
d) y = 2 x
2
, y
3
= x
2
.
e) y =
1

1 x
4
, x = 0, x =
1

2
.
Bi 11.63: Tnh din tch hnh phng gii hn bi cc ng:
a) y = x.e
x
, y = 0, x = 1, x = 2.
b) y = x. ln
2
x, y = 0, x = 1, x = e.
c) y = e
x
, y = e
x
, x = 1.
d) y = 5
x2
, y = 0, x = 0, y = 3 x.
e) y = (x + 1)
5
, y = e
x
, x = 1.
Bi 11.64: Tnh din tch hnh phng gii hn bi cc ng:
a) y =

x
2
2
+ 2x

v y = x + 4.
b) y = x
2
+ 2[x[ + 3 v 3x + 5y 9 = 0.
c) y =
x
[x[ + 1
v y = 0, x = 1, x = 2.
d) y = [ lnx[, y = 0, x =
1
e
, x = e.
Bi 11.65: Tnh din tch hnh phng gii hn bi cc ng:
a) y = sin x + cos
2
x vi cc trc ta v x = .
b) y = sin
2
x + sin x + 1, cc trc ta v x =

2
Ngi son : Th.s Minh Tun Trang 258 Khoa T nhin - Trng CSP Nam nh
T
h
.
s

M
i
n
h
T
u

n
11.6. Bi tp Chng 11. Tch phn
c) y = x + sin x, y = x, x = 0, x = 2.
d) y = x + sin
2
x, y = , x = 0, x = .
Bi 11.66: Din tch hnh phng gii hn bi cc ng thng x = 1, x = 2, y = 0 v Parabol
(P) l 15. Tm phng trnh ca (P), bit (P) c nh l I(1, 2).
Bi 11.67: Cho (H) : y =
2x
x 1
.
a) Chng minh rng hnh phng c gii hn bi (H), tim cn ngang v cc ng thng
x = a + 1, x = 2a + 1 c din tch khng ph thuc vo tham s a.
b) Lp phng trnh tip tuyn (d) ca (H) ti gc ta . Tnh din tch hnh phng gii hn
bi (H), (d) v ng thng x = 2.
Bi 11.68: Cho Parabol (P) : y = x
2
. Hai im A, B di ng trn (P) sao cho AB = 2.
a) Tm tp hp trung im I ca AB.
b) Xc nh v tr ca A, B sao cho din tch ca phn mt phng gii hn bi (P) v ct tuyn
AB t gi tr ln nht.
Bi 11.69: ng thng (d) i qua im M

1
2
; 1

v cc bn trc dng Ox, Oy lp thnh


mt tam gic. Xc nh (d) din tch tam gic nh nht v tnh gi tr .
Bi 11.70: Cho Parabol (P) : y = x
2
. Vit phng trnh ng thng d qua I(1, 3) sao cho din
tch hnh phng gii hn bi d v (P) t gi tr nh nht.
Bi 11.71: Trn Parabol (P) : y = x
2
ly 2 im A(1; 1) v B(3; 3). Tm im M trn cung

AB ca (P) sao cho tam gic MAB c din tch ln nht.


Bi 11.72: Xt hnh (H) gii hn bi Parabol (P) : y = x
2
+ 1 v y = 0, x = 0, x = 1. Tip
tuyn no ca (P) s ct t hnh (H) ra mt hnh thang c din tch ln nht.
Bi 11.73: Tnh th tch vt th trn xoay sinh ra bi php quay xung quanh trc Ox ca min
(D) gii hn bi cc ng:
a) y = ln x, y = 0, x = 2.
b) x
2
+y 5 = 0, x +y 3 = 0.
c) y = x
2
, y =

x.
d) y = x
2
4x + 6, y = x
2
2x + 6.
e) y = x(x 1)
2
.
f) y = x.e
x
, x = 1, y = 0, (0 x 1).
g) y = e
x
, y = x + 2, x = 0, x = 2.
h) y = x

ln(1 +x
3
), x = 1.
Ngi son : Th.s Minh Tun Trang 259 Khoa T nhin - Trng CSP Nam nh
T
h
.
s

M
i
n
h
T
u

n
11.6. Bi tp Chng 11. Tch phn
i) (P) : y = x
2
(x > 0), y = 3x + 10, y = 1 (min (D) ngoi (P)).
j) y =

cos
4
x + sin
4
x, y = 0, x =

2
, x = .
Bi 11.74: Tnh th tch khi trn xoay c to thnh do xoay quanh trc Oy hnh phng
gii hn bi cc ng:
a) y = x
2
, y = 1, y = 2.
b) y = x
2
, x = y
2
.
c) ng trn tm I(3, 0) bn knh R = 2.
Bi 11.75: Cho min (D) gii hn bi ng trn (C) : x
2
+y
2
= 8 v Parabol (P) : y
2
= 2x.
a) Tnh din tch (S) ca (D).
b) Tnh th tch V sinh bi (D) khi quay quanh Ox.
Bi 11.76: Tnh th tch vt th gii hn bi cc mt to nn khi quay cc ng:
a) y = b.

x
a

2/3
. (0 x a) quay quanh trc Ox.
b) y = sin x, y = 0, 0 x trong 2 trng hp. 1. Quanh Ox 2. Quanh Oy.
c) y = b

x
a

2
, y = b

x
a

. Khi quay quanh trc Ox v Oy.


d) y = e
x
, y = 0 (0 x < +) quay quanh trc Ox, Oy
Bi tp tng hp
Bi 11.77: Tnh cc tch phn:
a)
2

2 +[x[dx.
b)
1

0
x
2
dx

4 x
2
.
c)
2

x
2
1dx
x
.
d)
1

0
dx

(1 +x
2
)
3
.
e)
1

0
x
2
dx
(x
2
+ 1)
2
.
f)

0
xdx
cos
2
x
.
Ngi son : Th.s Minh Tun Trang 260 Khoa T nhin - Trng CSP Nam nh
T
h
.
s

M
i
n
h
T
u

n
11.6. Bi tp Chng 11. Tch phn
g)

0
e
x
. cos xdx.
h)

4
sin
4
x + cos
4
x
3
x
+ 1
dx.
i)

0
cos 2xdx
sin x + cos x + 2
.
j)
5
12

12
dx
sin 2x + 2

3 cos
2
x + 2

3
.
Bi 11.78: Bit f(x) =

2x + 1 Nu x 0
m(1 x
2
) Nu x > 0
. Tm m
1

1
f(x)dx = 1.
Bi 11.79: a) Cho hm s f(x) =
e
x
2

e
x
t lntdt. Tm honh im cc i.
b) Tm x

0,
3
2

sao cho hm s f(x) =


2x

x
sin t
t
dt t cc i.
Bi 11.80: Cho hm s f(x) =
x

0
2t + 1
t
2
2t + 2
dt, 1 x 1.
Tm gi tr ln nht, nh nht ca hm s f(x).
Bi 11.81: ng thng (d) : x 3y + 5 = 0 chia ng trn (C) : x
2
+ y
2
= 5 thnh 2 phn.
Tnh din tch mt phn.
Bi 11.82: Xt hnh phng (H) gii hn bi ng cong (C) : y =
1
x
, y = 0, x = 1, x = 2. Tm
ta im M trn (C) m tip tuyn ti M s ct t (H) ra mt hnh thang c din tch ln
nht.
Bi 11.83: Cho im A thuc (P) : y = x
2
, (A = O). Gi () l php tuyn ti A ca (P).
Xc nh v tr ca A din tch gii hn bi () v (P) l nh nht.
Bi 11.84: Cho hnh (H) gii hn bi
x
2
16

y
2
4
= 1 v x = 4

2. Tnh th tch sinh ra khi (H)


quay quanh Oy.
Bi 11.85 (A-2002): Tnh din tch hnh phng gii hn bi: y = [x
2
4x + 3[, y = x + 3.
Bi 11.86 (B-2002): Tnh din tch hnh phng gii hn bi cc ng: y =

4
x
2
4
v
y =
x
2
4

2
.
Bi 11.87 (D-2002): Tnh din tch hnh phng gii hn bi ng cong
(C) : y =
3x 1
x 1
v 2 trc ta
Ngi son : Th.s Minh Tun Trang 261 Khoa T nhin - Trng CSP Nam nh
T
h
.
s

M
i
n
h
T
u

n
11.6. Bi tp Chng 11. Tch phn
Bi 11.88 (A-2003): Tnh tch phn I =
2

5
dx
x

x
2
+ 4
Bi 11.89 (B-2003): Tnh tch phn I =

0
1 2 sin
2
x
1 + sin 2x
dx.
Bi 11.90 (D-2003): Tnh tch phn I =
2

0
[x
2
x[dx.
Bi 11.91 (A-2004): Tnh tch phn I =
2

1
xdx
1 +

x + 1
.
Bi 11.92 (B-2004): Tnh tch phn I =
e

1 + 3 ln x. ln xdx
x
Bi 11.93 (D-2004): Tnh tch phn I =
3

2
ln(x
2
x)dx
Ngi son : Th.s Minh Tun Trang 262 Khoa T nhin - Trng CSP Nam nh
T
h
.
s

M
i
n
h
T
u

n
Chng 12. S phc
Chng 12
S phc
12.1 Kin thc c bn
12.1.1 Cc kin thc chung
Gi i l s sao cho i
2
= 1.
C = a +b.i [ a, b R gi l tp s phc.
z = a + b.i gi l s phc, a gi l phn thc ca z k hiu Re z v b gi l phn o ca z k
hiu Imz
z = a b.i gi l s phc lin hp ca z.
Nu Re z = 0 th z gi l s thun o. Nu Imz = 0 th z l mt s thc. V z l s thc khi v
ch khi z = z.
r = [z[ =

a
2
+b
2
gi l m un ca s phc z.
Gc = Arg(z) l gc sao cho cos =
a

a
2
+b
2
v sin =
b

a
2
+b
2
.
z = r(cos +i. sin ) gi l dng lng gic ca s phc.
12.1.2 Cc php ton trn s phc
+) Php cng, php nhn, php chia:
z
1
= a +ib, z
2
= c +id.
th z
1
z
2
= (a c) +i(b d).
z
1
.z
2
= ac bd +i(ad +bc).
z
1
z
2
=
z
2
.z
1
[z
2
[
2
.
c bit nu z
1
= r
1
(cos
1
+i sin
1
) v z
2
= r
2
(cos
2
+i sin
2
) th
z
1
.z
2
= r
1
.r
2
(cos(
1
+
2
) +i sin(
1
+
2
)) v
z
1
z
2
=
r
1
r
2
(cos(
1

2
) +i sin(
1

2
))
Ch rng i
2
= 1 nn i
3
= i, i
4
= 1, ...
Tng qut i
n
=

1 Nu n = 4k
i Nu n = 4k + 1
1 Nu n = 4k + 2
i Nu n = 4k + 3
+) Php khai cn: gi l cn bc n ca z nu
n
= z.
Ch rng nu z = 0 th z c n cn bc n.
Ngi son : Th.s Minh Tun Trang 263 Khoa T nhin - Trng CSP Nam nh
T
h
.
s

M
i
n
h
T
u

n
12.2. Cc dng bi tp Chng 12. S phc
Nu z = r(cos +i. sin ) th
k
=
n

r.

cos
+k2
n
+ sin
+k2
n

vi k = 0, 1, 2, ..., n 1 l n cn bc n ca z.
c bit gi l cn bc 2 ca z nu
2
= z.
C 2 cch tm cn bc 2 ca z:
-) Cch 1: Dng nh ngha, ng nht h s.
-) Cch 2: Chuyn z v dng lng gic v khai cn theo cng thc:

1,2
=

cos

2
+ sin

r = [z[ v = Arg z.
Ngoi ra ta c th s dng my tnh in t (570 - ES) tm cn bc 2 ca z vi cch lm nh
cch 2 (S dng CMPLX v cc tnh nng tm m un, argument ca mt s phc cng nh
cch chuyn mt s phc t dng lng gic sang dng bnh thng v ngc li).
+) Cng thc Moivre:
(cos +i sin )
n
= cos n +i sin n
12.2 Cc dng bi tp
12.2.1 Thc hin cc php ton
V d 12.2.1: Thc hin cc php ton sau:
a) (1 + 2i).(2 i) +
1 3i
3 + 4i
.
b) (1 2i)
3
.
c) (1 i

3)
2010
.
Gii: a) (1 + 2i).(2 i) +
1 3i
3 + 4i
= 4 + 3i +
(1 3i)(3 4i)
3
2
+ 4
2
= 4 + 3i +
9 13i
25
=
91
25
+
62
25
i
b) (1 2i)
3
= 1 3.(2i) + 3.(2i)
2
(2i)
3
= 1 6i 12 + 8i = 11 + 2i.
c) z = 1 i

3 = 2.

cos

3
+i sin

Do z
2010
= 2
2010
.

cos
2010
3
+i sin
2010
3

= 2
2010
12.2.2 Khai cn bc 2
V d 12.2.2: Khai cn s phc
a) z = 1 i.
b) z = 5 + 12i
Ngi son : Th.s Minh Tun Trang 264 Khoa T nhin - Trng CSP Nam nh
T
h
.
s

M
i
n
h
T
u

n
12.2. Cc dng bi tp Chng 12. S phc
Gii: a) z = 1 i.
Cch 1: Gi = x +y.i l cn bc 2 ca z. Khi
(x +yi)
2
= 1 i x
2
y
2
+ 2xyi = 1 i

x
2
y
2
= 1
2xy = 1

x
2
+ (y
2
) = 1
x
2
.(y
2
) =
1
4
v xy < 0
Do x
2
, y
2
l 2 nghim ca phng trnh:
X
2
X
1
4
= 0 X =
1

2
2
.
Do

x
2
=
1 +

2
2
y
2
=
1

2
2

x =

2 + 2

2
2
y =

2 2
2
v ta c
1,2
=

2 + 2

2
2
i

2 2
2

Cch 2: Vit s phc di dng lng gic z =

2.

cos

4
+i sin

1,2
=
4

2.

cos

8
+i sin

2 + 2

2
2
i

2 2
2

b) z = 5 + 12i.
Cch 1: Gi = x +yi l cn bc 2 ca s phc.
Khi ta c h

x
2
y
2
= 5
2xy = 12

x
2
+ (y
2
) = 5
x
2
.(y
2
) = 36
v xy > 0
Do x
2
, y
2
l 2 nghim ca phng trnh:
X
2
+ 5X 36 = 0 X = 9 hoc X = 4.
Do ta c:

x
2
= 4
y
2
= 9

x = 2
y = 3
.
Vy cn bc 2 ca s phc z = 5 + 12i l
1,2
= (2 + 3i).
Cch 2: S dng my tnh 570 - ES ta tm c
= 2 + 3i.
Khi trnh by ta vit ngc li theo cc bc bin i sau:
(2 + 3i)
2
= 4 + 12i + 9i
2
= 4 + 12i 9 = 5 + 12i = z
do cn bc 2 ca z l (2 + 3i).
Cch 3: S dng dng lng gic.
Ngi son : Th.s Minh Tun Trang 265 Khoa T nhin - Trng CSP Nam nh
T
h
.
s

M
i
n
h
T
u

n
12.2. Cc dng bi tp Chng 12. S phc
z = 13.

5
13
+i.
12
13

v l gc sao cho cos =


5
13
v sin =
12
13
do nm gc
phn t th II v

2
nm gc phn t th I. V v th
cos
2

2
=
1 + cos
2
=
4
13
cos

2
=
2

13
. sin

2
=
sin
2 cos

2
=
3

13
v
1,2
=

13

13
+i
3

13

= (2 + 3i)
12.2.3 Gii phng trnh i s v cc vn lin quan
V d 12.2.3 (A-2009): Gii phng trnh sau: z
2
2z + 10 = 0. Gi z
1
, z
2
l 2 nghim ca
phng trnh trn. Tnh A = [z
1
[
2
+[z
2
[
2
.
Gii:

= 1 10 = 9 = (3i)
2
. Do
z
1
= 1 3i v z
2
= 1 + 3i.
A = [z
1
[
2
+[z
2
[
2
= (1
2
+ (3)
2
) + (1
2
+ 3
2
) = 20.
V d 12.2.4: Gii cc phng trnh sau:
a) z
2
(3 i)z + 8 +i = 0.
b) z
3
+z
2
3z 6 = 0.
c) 2z
4
8z
3
21z
2
17z 6 = 0.
Gii: a) = (3 i)
2
4(8 +i) = 9 6i +i
2
32 4i = 24 10i = 1 2.1.(5i) +(5i)
2
=
(1 5i)
2
Do z
1
=
3 i 1 + 5i
2
= 1 + 2i v z =
3 i + 1 5i
2
= 2 3i.
b) Phng trnh tng ng (z 2)(z
2
+ 3z + 3) = 0 z = 2 hoc z
2
+ 3z + 3 = 0.
Gii z
2
+ 3z + 3 = 0. = 9 3.4 = 3 = (i

3)
2
do z =
3 i

3
2
.
c) Phng trnh tng ng vi (z + 1)(z 6)(2z
2
+ 2z + 1) = 0

z = 1
z = 6
2z
2
+ 2z + 1 = 0

z = 1
z = 6
z =
1 i
2
V d 12.2.5 (B-2009): Tm s phc z tha mn iu kin: [z (2 +i)[ =

10 v z.z = 25.
Gii: Gi s phc z = x +yi th ta c

(x 2)
2
+ (y 1)
2
= 10
x
2
+y
2
= 25

x
2
+y
2
4x 2y 5 = 0 (1)
x
2
+y
2
= 25 (2)
Th (2) vo (1) ta c 25 4x 2y 5 = 0 2x +y = 10 y = 10 2x
Ngi son : Th.s Minh Tun Trang 266 Khoa T nhin - Trng CSP Nam nh
T
h
.
s

M
i
n
h
T
u

n
12.3. Bi tp Chng 12. S phc
Th vo (2) ta c : x
2
+ (10 2x)
2
= 25 5x
2
40x + 75 = 0

x = 3 y = 4
x = 5 y = 0
Kt lun: c 2 s phc tha mn l z
1
= 3 + 4i v z
2
= 5
12.2.4 Biu din s phc trn mt phng
V d 12.2.6 (D - 2009): Trn mt phng ta Oxy tm tp hp im biu din s phc z
tha mn: [z (3 4i)[ = 2
Gii: Gi s phc l z = x +yi ta c [(x 3) +i(y + 4)[ = 2

(x 3)
2
+ (y + 4)
2
= 2 (x 3)
2
+ (y + 4)
2
= 4
Do tp hp im l ng trn tm I(3; 4) v bn knh r = 2.
12.2.5 Chng minh ng thc t hp
V d 12.2.7: Biu din dng lng gic ca s phc z = 1 i

3. T tnh:
C
0
2010
3C
2
2010
+ 9C
4
2010
+ 3
1004
C
2008
2010
3
1005
C
2010
2010
Gii: z = 2

cos

3
+i sin

v z
2010
= 2
2010
. (cos(670) +i sin(670)) = 2
2010
.
Mt khc: z
2010
= C
0
2010
C
1
2010
(i

3) + C
2
2010
(i

3)
2
+ C
2008
2010
(i

3)
2008
C
2009
2010
(i

3)
2009
+
C
2010
2010
(i

3)
2010
= (C
0
2010
3C
2
2010
+ 9C
4
2010
+ 3
1004
C
2008
2010
3
1005
C
2010
2010
) i

3.(C
1
2010
+
3
1004
C
2009
2010
)
Do C
0
2010
3C
2
2010
+ 9C
4
2010
+ 3
1004
C
2008
2010
3
1005
C
2010
2010
= 2
2010
12.3 Bi tp
Bi 12.1: Thc hin cc php ton sau:
a)
2 +i
3 2i
+
1 +i

2
2 +i

3
b) 2i(3 +i)(2 + 4i) +
(1 +i)
2
(2i)
3
2 +i
.
c) (1 +i)
2010
Bi 12.2: Tm s phc lin hp, phn thc, phn o ca s phc sau:
z = (1 + 3i)
2
.(2 i)
3
Bi 12.3: Tm s z tha mn iu kin.
a) (3 2i)z + (4 + 5i) = 7 + 3i
b) (1 + 3i)z (2 + 5i) = (2 +i)z
c) z(5 i) + (2 + 3i)z = 10 +i
Ngi son : Th.s Minh Tun Trang 267 Khoa T nhin - Trng CSP Nam nh
T
h
.
s

M
i
n
h
T
u

n
12.3. Bi tp Chng 12. S phc
Bi 12.4: Xc nh tp hp im trong mt phng phc biu din cc s phc tha mn tng
iu kin sau:
a) [z i + 2[ = 3
b)

z i + 1
z + 2i 3

= 1.
c) [z i[ = [z 2 5i[.
Bi 12.5: Khai cn bc 2 s phc i, 4i, 4, 1 + 4

3i
Bi 12.6: Gii phng trnh :
a) z
2
= 2z 5
b) 2z
3
3z
2
+z 6 = 0
c) 2z
2
+ (3 +i)z + 1 + 7i = 0.
d) z
2
+ (1 6i)z 11 3i = 0
Bi 12.7: Vit cc s phc sau di dng lng gic:
a) 1 i

3, 1 +i, (1 i

3).(1 +i),
1 i

3
1 +i
.
b) 2i(

3 i).
c)
1
2 + 2i
d) 1 +i tan vi =

2
+k
e) sin +i cos .
f) 1 + cos i sin .
g) tan i.
Bi 12.8: Vit dng lng gic ca s phc z tha mn:
a) [z[ = 2 v Arg(iz) =
2
3
.
b) [z[ =

3 v Arg

3 i

4
Bi 12.9: Gii cc phng trnh:
a) (z 2 +i)
2
8(z +i 2) + 20 = 0.
b)

iz + 1
z 3i

2
4
iz + 1
z 3i
5 = 0.
c) (z
2
+ 3)
2
+ (z + 1)
2
= 0
Bi 12.10: Tnh gi tr biu thc:
C
0
2010
C
2
2010
+ +C
2008
2010
C
2010
2010
Ngi son : Th.s Minh Tun Trang 268 Khoa T nhin - Trng CSP Nam nh
T
h
.
s

M
i
n
h
T
u

n
Chng 13. Hnh hc khng gian
Chng 13
Hnh hc khng gian
13.1 M u v hnh hc khng gian
13.1.1 i tng ca hnh hc khng gian
im: Biu din bi mt chm trong khng gian. K hiu bi: A, B, C, ...
ng thng: Biu din bi mt ng k. K hiu bi: a, b, c, ....; , ...
Mt phng: Biu din bi mt hnh bnh hnh, tn ca mt phng c ghi gc. K
hiu: (), (), ...., (P), (Q), ....
13.1.2 Quan h
Thuc: Ta ni im A thuc ng thng a. K hiu A a.
Cha: ta ni ng thng a nm trn mt phng () hay () cha ng thng a nu n
cha mi im ca ng thng a. K hiu: a ().
13.1.3 Hnh biu din trong hnh hc khng gian
Nguyn tc chung
Chiu mt hnh trong khng gian bt k ln mt mt phng (biu din trn trang giy). Nhng
nt nhn thy c biu th bi nt lin. Nhng nt khng thy c biu th bi nt t.
Cc yu t bo ton
Ba im thng hng 3 im thng hng v gi nguyn th t cng nh t l chia ca
chng.
c bit: Trung im ca on thng thnh trung im ca on thng. Trng tm tam
gic thnh trng tm tam gic.
Hai ng thng cng phng (song song) 2 ng thng cng phng.
Hai on thng cng phng (song song hoc cng nm trn mt ng thng) 2 on
thng cng phng v gi nguyn t s di ca chng.
Ngi son : Th.s Minh Tun Trang 269 Khoa T nhin - Trng CSP Nam nh
T
h
.
s

M
i
n
h
T
u

n
13.1. M u v hnh hc khng gian Chng 13. Hnh hc khng gian
Cc yu t khng bo ton
Gc

= .
Hai on thng khng song song thnh 2 on thng khng song song v t s di c
th b thay i.
Mt s php bin i thng dng
Hnh vung, hnh ch nht, hnh bnh hnh hnh bnh hnh.
Hnh thang, hnh thang vung hnh thang (bo ton y ln, y nh).
Tam gic vung, tam gic cn, tam gic u, tam gic thng tam gic thng.
Gc vung gc thng (nhng vn k hiu gc vung).
Trung im thnh trung im, trung tuyn thnh trung tuyn, trng tm thnh trng tm.
ng cao, ng phn gic on bt k ni mt nh ca tam gic vi mt im trn
cnh i din.
ng trung trc ca mt tam gic thng ng thng bt k i qua trung im cnh
i (khng i qua nh i din)
13.1.4 Mt s hnh thng dng
Hnh chp tam gic, t din.
A
C
B D
Hnh chp t gic (y c th l hnh bnh hnh, hnh vung, hnh ch nht)
A
C
B
D
S
A
C B
D
S
Ngi son : Th.s Minh Tun Trang 270 Khoa T nhin - Trng CSP Nam nh
T
h
.
s

M
i
n
h
T
u

n
13.1. M u v hnh hc khng gian Chng 13. Hnh hc khng gian
Lng tr t gic
A
B C
D
A
B C
D
Lng tr tam gic
A
B
C
A
B
C
Hnh hp
A
B C
D
A
B C
D
Hnh hp ch nht, hnh lp phng.
A
B C
D
A
B C
D
Hnh khi: Hnh nn, hnh tr, hnh cu.
Ngi son : Th.s Minh Tun Trang 271 Khoa T nhin - Trng CSP Nam nh
T
h
.
s

M
i
n
h
T
u

n
13.2. V tr tng i Chng 13. Hnh hc khng gian
13.1.5 Cc tin hnh hc khng gian
13.1.6 Cc tin
Tin 1 (Xc nh ng thng). Qua 2 im phn bit c mt v ch mt ng thng.
Tin 2 (Xc nh mt phng). Qua 3 im khng thng hng tn ti duy nht mt mt
phng.
p dng: Vng nh king 3 chn
Tin 3 (Mi quan h gia ng thng v mt phng). Nu mt ng thng cha 2 im
thuc mt mt phng th mi im ca n u thuc mt phng .
Tin 4 (S im chung ca 2 mt phng). Nu 2 mt phng c mt im chung th chng
c mt im chung khc na
Tin 5 (Hnh hc khng gian v hnh phng). Cc nh l, mnh ng trong hnh hc
phng th cng ng trn mt mt phng bt k trong khng gian.
13.1.7 nh l v giao tuyn
nh l 17 (S tn ti ca giao tuyn). Nu 2 mt phng phn bit c mt im chung th n
s c ng thng chung duy nht.
nh l 18 (nh l 3 giao tuyn v.1). Nu 3 mt phng i mt ct nhau theo 3 giao tuyn
m c t nht 2 trong s ct nhau th 3 giao tuyn ng quy.
13.2 V tr tng i
13.2.1 V tr tng i ca mt phng v mt phng
(P) song song vi (Q) (P) (Q) = .
(P) (Q).
(P) (Q) = d l mt ng thng. d gi l giao tuyn ca 2 mt phng.
13.2.2 V tr tng i ca ng thng v mt phng
ng thng a gi l song song vi mt phng () nu a () = .
ng thng a gi l ct mt phng () ti im M nu a () = M.
ng thng a () nu mi im M a M ().
13.2.3 V tr tng i ca ng thng v ng thng
a song song vi b a, b cng nm trn mt mt phng v khng c im chung.
a b
a ct b nu a b = M.
a, b cho nhau nu chng khng cng nm trn mt mt phng.
Ngi son : Th.s Minh Tun Trang 272 Khoa T nhin - Trng CSP Nam nh
T
h
.
s

M
i
n
h
T
u

n
13.3. Cc dng ton Chng 13. Hnh hc khng gian
13.2.4 Cc phng php xc nh mt phng
C 4 phng php xc nh:
Qua 3 im khng thng hng A, B, C v ta c mt phng (ABC)
Qua 1 im A v cha mt ng thng d vi A / d. K hiu mp(A, d)
Cha 2 ng thng ct nhau d
1
, d
2
. K hiu mp(d
1
, d
2
).
Cha 2 ng thng song song d
1
, d
2
. K hiu mp(d
1
, d
2
).
13.3 Cc dng ton
13.3.1 S dng tin , v tr tng i
V d 13.3.1: Cho mt phng () v 2 tia Ax song song vi By nm trn (). Ly M, N ty
thuc Ax, By khc A, B. im O c nh / ().
a) M thuc cc mt phng no?
b) Chng minh rng: OA, MN cho nhau.
c) M, N di ng. Chng minh rng: OI mt phng c nh. (Vi I l trung im MN)
d) M, N di ng nhng AM + BN = const. Chng minh rng (OMN) lun cha mt ng
thng c nh.
Gii: Ta c hnh v:

A
B
x
y
M
N
I J
O
a) M (OMN), mp(0, Ax), ), (OJM), (OBM)
b) Gi s OA, MN () 0, A, M, N ()
m () = (AMN) () () (Tin 2)
Do O () (Mu thun)
c) I, J l trung im MN, AB nn IJ Ax By (nh l ng trung bnh ca hnh thang) Do
IJ l ng thng qua im J c nh v song song vi ng thng Ax c nh nn n
l mt ng thng c nh (Ch rng im I vn di ng)
Do (P) = mp(O, IJ) c nh. V OI lun nm trong mt mt phng c nh l (P).
Ngi son : Th.s Minh Tun Trang 273 Khoa T nhin - Trng CSP Nam nh
T
h
.
s

M
i
n
h
T
u

n
13.3. Cc dng ton Chng 13. Hnh hc khng gian
d) IJ =
AM +BN
2
khng i. Nn im I cng c nh v OI c nh.
M OI (OMN). Do (OMN) lun cha mt ng thng c nh.
V d 13.3.2: Cho 2 ng thng a, b cho nhau. Trn a ly 2 im A, B, trn b ly 2 im
C, D.
a) Chng minh: AC, BD cho nhau.
b) LyM AC, N BD th MN c th song song vi AB hoc CD.
c) Ly O MN. Chng minh AO ct CN, BO ct DM.
Gii: Ta c hnh v:
a
b
A
B
C
D
M
N
O
a) Gi s AC, BD ng phng. t () = mp(AC, BD),
khi A, B, C, D () AB, CD () (Tin 3)
Mu thun vi gi thit AB, CD cho nhau.
b) Gi s MN song song vi AB. Do MN, AB ng phng 4 im M, N, A, B ng phng
AM, BN ng phng AC, BD ng phng. Mu thun vi cu a).
c) AO, CN (ACN) v v th AO, CN ct nhau.
BO, DM (BDM) v v th BO, DM ct nhau.
V d 13.3.3: Trong mt phng () cho xOy, A / (), M, N di ng trn Ox, Oy.
a) Gi s lun c OM = ON. Chng minh: trung tuyn AP ca tam gic AMN lun nm
trong mt mt phng c nh.
b) Gi d l ng thng c nh qua A ct () ti mt im khng thuc Ox, Oy. ng thng
MN di ng nhng lun ct d.
Chng minh rng: MN lun i qua 1 im c nh.
Gi B l im c nh trn d, B = A, B / (). Gi Q = AM BN, chng minh Q thuc 2
mt phng c nh, t chng minh Q thuc ng thng c nh.
Gii: a) Ta c hnh v:
Ngi son : Th.s Minh Tun Trang 274 Khoa T nhin - Trng CSP Nam nh
T
h
.
s

M
i
n
h
T
u

n
13.3. Cc dng ton Chng 13. Hnh hc khng gian

O
M
N
P
A
x
y
z
Do AP l ng trung tuyn ca tam gic AMN nn P l trung im MN.
Mt khc OM = ON nn tam gic OMN cn ti O. Do trung tuyn OP ca n ng
thi l ng phn gic ca MON xOy, m xOy c nh nn ng phn gic Oz
ca n cng c nh.
V AP mp(A, Oz) c nh. pcm.
b) Ta c hnh v:

O
M
N
C
A
Q
B
x
y
d
Theo gi thit d ct MN ti C nn C = d MN d () = C

Do C C

v v
th C c nh v MN lun i qua mt im c nh l C.
Q AM mp(A, Ox) v Q BN mp(B, Oy) v ta c mp(A, Ox) v mp(B, Oy) l 2
mt phng c nh cn tm.Vy Q mp(A, Ox) mp(B, Oy) = a l ng thng c nh
cn tm.
V d 13.3.4: Nu 3 ng thng khng ng phng ct nhau i mt th chng ng quy.
Gii: Gi s A = b c, B = c a, C = a b. Ta c hnh v:
c
b
a
A
B
C
Ngi son : Th.s Minh Tun Trang 275 Khoa T nhin - Trng CSP Nam nh
T
h
.
s

M
i
n
h
T
u

n
13.3. Cc dng ton Chng 13. Hnh hc khng gian
+) Nu C / c, A = B th khi A, B, C khng thng hng v tn ti mt mt phng cha
A, B, C v n s cha a, b, c theo tin 3. iu ny mu thun vi gi thit.
+) Nu C / c, A B th khi b = CA CB = a. V a, c ct nhau nn a, c ng phng nn
a, b, c ng phng. iu ny mu thun vi gi thit.
+) Nu C c v A, B, C phn bit th a b c. Mu thun vi tnh khng ng phng ca
a, b, c.
+) Nu C c v C A = B th c = BA BC = a. V a, b ct nhau nn a, b ng phng v
a, b, c ng phng. Mu thun.
+) Nu C c v C B = A: Tng t nh TH trn.
+) Nu C c v A B = C: Tng t nh TH trn.
Vy A B C hay a, b, c ng quy.
13.3.2 Tm giao tuyn gia 2 mt phng (Cch 1)
~ Thut ton: Tm giao tuyn ca 2 mt phng (), () (Cch 1).
Tm 2 im chung ca 2 mt phng. ng thng i qua 2 im chung chnh l giao
tuyn ca 2 mt phng.
Tm im chung bng cch s dng nh l 3 giao tuyn. C th:
+) Chn mt mt phng () no (d tm giao tuyn vi 2 mt phng () v ())
+) Tm a = () (), b = () ()
+) im M = a b l im chung ca 2 mt phng.
V d 13.3.5: Cho hnh chp S.ABCD y l t gic ABCD. Gi ABCD = E, ACBD = F.
a) Tm giao tuyn ca (SAB), (SCD).
b) Tm giao tuyn ca (SEF) vi cc mt (SAD), (SBC).
Gii: Ta c hnh v.
A
B
C
D
S
E
F
M
N
a) S (SAB) (SCD).
(SAB) (SCD)
E = AB CD
(ABCD)
))
R
R
R
R
R
R
R
R
R
R
AB uul
l
l
l
l
l
l
l
l
l
CD
//
Ngi son : Th.s Minh Tun Trang 276 Khoa T nhin - Trng CSP Nam nh
T
h
.
s

M
i
n
h
T
u

n
13.3. Cc dng ton Chng 13. Hnh hc khng gian
Do (SAB) (SCD) = SE.
b) +) S (SEF) (SAD).
(SEF) (SAD)
M = EF AD
(ABCD)
))
R
R
R
R
R
R
R
R
R
R
R
EF uul
l
l
l
l
l
l
l
l
l
l
AD
//
Do (SEF) (SAD) = SM.
+) S (SEF) (SBC)
(SEF) (SBC)
N = EF BC
(ABCD)
))
R
R
R
R
R
R
R
R
R
R
EF uul
l
l
l
l
l
l
l
l
l
BC
//
Do (SEF) (SBC) = SN
V d 13.3.6: Cho hnh chp S.ABCD c y l hnh bnh hnh tm O. Gi M, N, P ln lt
l trung im cc cnh BC, CD, SO. Tm giao tuyn ca (MNP) vi cc mt phng (SAB),
(SAD), (SBC), (SCD).
A
B C
D
S
O
M
N
P
E
F
G
H
I
K
Gii: +) (MNP) (SAC):
P SO (SAC) P (SAC) (MNP) .
(MNP) (SAC)
E = MN AC
(ABCD)
))
R
R
R
R
R
R
R
R
R
R
MN uul
l
l
l
l
l
l
l
l
l
l
AC
//
Vy (MNP) (SAC) = PE.
+) (MNP) (SAB):
(MNP) (SAB)
F = PE SA
(SAC)
))
R
R
R
R
R
R
R
R
R
R
PE uul
l
l
l
l
l
l
l
l
l
l
SA
//
Ngi son : Th.s Minh Tun Trang 277 Khoa T nhin - Trng CSP Nam nh
T
h
.
s

M
i
n
h
T
u

n
13.3. Cc dng ton Chng 13. Hnh hc khng gian
(MNP) (SAB)
H = MN AB
(ABCD)
))
R
R
R
R
R
R
R
R
R
R
MN uul
l
l
l
l
l
l
l
l
l
l
AB
//
Vy (MNP) (SAB) = FH
+) (MNP) (SBC):
M BC (SBC)
(MNP) (SBC)
K = FH SB
(SAB)
))
R
R
R
R
R
R
R
R
R
R
R
FH uul
l
l
l
l
l
l
l
l
l
l
SB
//
Vy (MNP) (SBC) = MK.
+) (MNP) (SAD):
(MNP) (SAD)
F = FH SA
(SAB)
))
R
R
R
R
R
R
R
R
R
R
R
FH uul
l
l
l
l
l
l
l
l
l
l
SA
//
(MNP) (SAD)
G = MN AD
(ABCD)
))
R
R
R
R
R
R
R
R
R
R
MN uul
l
l
l
l
l
l
l
l
l
l
AD
//
(MNP) (SAD) = FG.
+) (MNP) (SCD):
N CD (SCD)
(MNP) (SCD)
I = FG SD
(SAD)
))
R
R
R
R
R
R
R
R
R
R
FG uul
l
l
l
l
l
l
l
l
l
l
SD
//
(MNP) (SCD) = NI
V d 13.3.7: Cho t din ABCD. Gi I, J ln lt l trung im ca AC, BC. Ly K l mt
im trn BD sao cho KD < KB. Tm giao tuyn ca (IJK) vi (ACD) v (ABD).
A
B
C
D
I
J
K
M
N
Gii: +) (IJK) (ACD):
I AC (ACD) I (IJK) (ACD).
(IJK) (ACD)
M = JK CD
(BCD)
))
R
R
R
R
R
R
R
R
R
R
R
R
JK
uul
l
l
l
l
l
l
l
l
l
l
CD
//
Ngi son : Th.s Minh Tun Trang 278 Khoa T nhin - Trng CSP Nam nh
T
h
.
s

M
i
n
h
T
u

n
13.3. Cc dng ton Chng 13. Hnh hc khng gian
(IJK) (ACD) = IM
+) (IJK) (ABD):
K BD (ABD) K (IJK) (ABD).
(IJK) (ABD)
N = IM AD
(ACD)
))
R
R
R
R
R
R
R
R
R
R
R
R
IM
uul
l
l
l
l
l
l
l
l
l
l
AD
//
(IJK) (ABD) = KN.
V d 13.3.8: Cho t din ABCD. Gi I, J l trung im AD, BC.
a) Tm giao tuyn ca 2 mt phng (IBC) v (JAD).
b) Ly M AB, N AC. Tm giao tuyn ca 2 mt phng (IBC), (DMN).
A
B
C
D
I
J
M
N
P
Q
Gii: a) I AD (JAD) I (IBC) (JAD).
J BC (IBC) J (IBC) (JAD).
Vy (IBC) (JAD) = IJ.
b) (IBC) (DMN):
(IBC) (DMN) Q = IB DM
(ABD)
))
R
R
R
R
R
R
R
R
R
R
R
R
IB
uul
l
l
l
l
l
l
l
l
l
l
DM
//
(IBC) (DMN)
P = IC DN
(ACD)
))
R
R
R
R
R
R
R
R
R
R
R
R
IC
uul
l
l
l
l
l
l
l
l
l
l
DN
//
Vy (IBC) (DMN) = PQ.
V d 13.3.9: Cho t din ABCD, M l im bn trong tam gic ABD, N l im bn trong
tam gic ACD. Tm giao tuyn ca 2 mt phng:
a) Mt phng (AMN) v (BCD).
b) Mt phng (DMN) v (ABC).
Ngi son : Th.s Minh Tun Trang 279 Khoa T nhin - Trng CSP Nam nh
T
h
.
s

M
i
n
h
T
u

n
13.3. Cc dng ton Chng 13. Hnh hc khng gian
A
B
C
D
M
N
E
F
G
H
Gii: a) Mt phng (AMN) v (BCD).
+)
(AMN) (BCD)
E = AM BD
(ABD)
))
R
R
R
R
R
R
R
R
R
R
R
AM uul
l
l
l
l
l
l
l
l
l
l
BD
//
+)
(AMN) (BCD)
F = AN CD
(ACD)
))
R
R
R
R
R
R
R
R
R
R
R
AN uul
l
l
l
l
l
l
l
l
l
l
CD
//
(AMN) (BCD) = EF.
b) Mt phng (DMN) v (ABC).
+)
(DMN) (ABC)
G = DM AB
(ABD)
))
R
R
R
R
R
R
R
R
R
R
R
DM uul
l
l
l
l
l
l
l
l
l
l
AB
//
+)
(DMN) (ABC)
H = DN AC
(ACD)
))
R
R
R
R
R
R
R
R
R
R
R
DN uul
l
l
l
l
l
l
l
l
l
l
AC
//
(DMN) (ABC) = GH.
V d 13.3.10: Cho t din ABCD, O l im bn trong tam gic BCD. M l im trn AO.
a) Tm giao tuyn ca (MCD) vi (ABC) v (ABD).
b) Ly I, J BC, BD sao cho IJ khng song song vi CD. Tm giao tuyn ca (IJM) v
(ACD).
Ngi son : Th.s Minh Tun Trang 280 Khoa T nhin - Trng CSP Nam nh
T
h
.
s

M
i
n
h
T
u

n
13.3. Cc dng ton Chng 13. Hnh hc khng gian
A
B
C
D
O
M
E
F
I
J
G
H
K
Gii: a) Trc tin ta tm giao tuyn ca (MCD) v (ABO):
M AO (ABO) M (MCD) (ABO).
(MCD) (ABO)
E = CD BO
(BCD)
))
R
R
R
R
R
R
R
R
R
R
R
CD uul
l
l
l
l
l
l
l
l
l
l
BO
//
Do (MCD) (ABO) = ME.
+) Mt phng (MCD) v (ABC):
C (MCD) (ABC).
(MCD) (ABC)
F = ME AB
(ABO)
))
R
R
R
R
R
R
R
R
R
R
R
ME uul
l
l
l
l
l
l
l
l
l
l
AB
//
(MCD) (ABC) = CF.
+) Mt phng (MCD) v (ABD).
D (MCD) (ABD).
(MCD) (ABD)
F = CF AB
(ABC)
))
R
R
R
R
R
R
R
R
R
R
R
CF uul
l
l
l
l
l
l
l
l
l
l
AB
//
(MCD) (ABD) = DF
b) Trc tin tm giao tuyn ca (IJM) v (ABO).
M AO (ABE) M (IJM) (ABE).
(IJM) (ABE)
G = IJ BO
(BCD)
))
R
R
R
R
R
R
R
R
R
R
R
IJ uul
l
l
l
l
l
l
l
l
l
BO
//
Ngi son : Th.s Minh Tun Trang 281 Khoa T nhin - Trng CSP Nam nh
T
h
.
s

M
i
n
h
T
u

n
13.3. Cc dng ton Chng 13. Hnh hc khng gian
(IJM) (ABE) = MG.
+) Mt phng (IJM) v (ACD).
(IJM) (ACD)
H = MG AE
(ABE)
))
R
R
R
R
R
R
R
R
R
R
R
R
MG
uul
l
l
l
l
l
l
l
l
l
l
AE
//
(IJM) (ACD)
K = IJ CD
(BCD)
))
R
R
R
R
R
R
R
R
R
R
R
IJ uul
l
l
l
l
l
l
l
l
l
l
CD
//
(IJM) (ACD) = HK.
V d 13.3.11: Cho hnh chp S.ABCD y ABCD l hnh bnh hnh. M, N l trung im
SB, SD. im P SC sao cho SP > SC. Tm giao tuyn ca mt phng (MNP) vi cc mt
phng (SAC), (SAB), (SAD) v (ABCD).
A
B
C
D
S
M
N
P
O
E
F
G
H
Gii: +) Trc tin ta tm giao tuyn ca (SAC) v (SBD).
S (SAC) (SBD).
(SAC) (SBD)
O = AC BD
(ABCD)
))
R
R
R
R
R
R
R
R
R
R
AC uul
l
l
l
l
l
l
l
l
l
BD
//
(SAC) (SBD) = SO.
+) Mt phng (MNP) v (SAC):
P SC (SAC) P (MNP) (SAC).
(MNP) (SAC)
E = MN SO
(SBD)
))
R
R
R
R
R
R
R
R
R
R
R
MN uul
l
l
l
l
l
l
l
l
l
l
SO
//
(MNP) (SAC) = PE.
Ngi son : Th.s Minh Tun Trang 282 Khoa T nhin - Trng CSP Nam nh
T
h
.
s

M
i
n
h
T
u

n
13.3. Cc dng ton Chng 13. Hnh hc khng gian
+) Mt phng (MNP) v (SAB):
M SA (SAB) M (MNP) (SAB).
(MNP) (SAB)
F = PE SA
(SAC)
))
R
R
R
R
R
R
R
R
R
R
PE uul
l
l
l
l
l
l
l
l
l
l
SA
//
(MNP) (SAB) = MF.
+) Tng t (MNP) (SAD) = NF.
+) Mt phng (MNP) v (ABCD):
(MNP) (ABCD)
G = MP BC
(SBC)
))
R
R
R
R
R
R
R
R
R
R
R
R
MP
uul
l
l
l
l
l
l
l
l
l
l
l
BC
//
(MNP) (ABCD)
H = NP CD
(SCD)
))
R
R
R
R
R
R
R
R
R
R
R
R
NP
uul
l
l
l
l
l
l
l
l
l
l
l
CD
//
(MNP) (ABCD) = GH.
13.3.3 Vit phng trnh ng thng i qua mt im v ct c 2
ng thng
~ Thut ton: Tm ng thng d qua im M ct c 2 ng thng a, b.
Tm cc mt phng () = mp(d, a) = mp(M, a), () = mp(d, b) = mp(M, b).
Khi d = () ().
V d 13.3.12: Cho t din ABCD, O l im bn trong tam gic BCD. im M AB.
a) Dng ng thng qua M ct AO v CD.
b) N l im trn BC v ON khng song song vi BD. Dng ng thng qua N ct AO v
DM.
A
B
C
D
O
M
N
E
F
G
Ngi son : Th.s Minh Tun Trang 283 Khoa T nhin - Trng CSP Nam nh
T
h
.
s

M
i
n
h
T
u

n
13.3. Cc dng ton Chng 13. Hnh hc khng gian
Gii: a) mp(M, AO) = mp(MAO) = mp(ABO). mp(M, CD) = mp(MCD).
M (MCD) (ABO).
(ABO) (MCD)
E = BO CD
(BCD)
))
R
R
R
R
R
R
R
R
R
R
R
BO
uul
l
l
l
l
l
l
l
l
l
l
CD
//
Vy (MCD) (ABO) = ME. Do ME l ng thng cn tm.
b) mp(N, AO) = mp(AON). mp(N, DM) = mp(DMN).
+) Trc tin ta tm giao tuyn ca (AON) v (ABD)
A (AON) (ABD)
(AON) (ABD)
F = ON BD
(BCD)
))
R
R
R
R
R
R
R
R
R
R
R
ON uul
l
l
l
l
l
l
l
l
l
l
BD
//
(AON) (ABD) = AF.
+) M N (AON) (DMN)
(AON) (DMN)
G = AF DM
(ABD)
))
R
R
R
R
R
R
R
R
R
R
R
R
AF
uul
l
l
l
l
l
l
l
l
l
l
DM
//
(AON) (DMN) = NG v NG l ng thng i qua G v ct c AO, DM.
V d 13.3.13: Trong mt phng () cho 2 ng thng d
1
, d
2
ct nhau ti O. l ng
thng ct () ti 1 im I = O.
a) Xc nh giao tuyn ca mp(O, ) v ().
b) im M di ng trn . Tm giao tuyn ca 2 mt phng mp(M, d
1
) v mp(M, d
2
).

d
2
d
1

O
I
M
Gii: a) O d
1
() O mp(O, ) ().
I = () mp(O, ) ()
Vy mp(O, ) () = OI.
Ngi son : Th.s Minh Tun Trang 284 Khoa T nhin - Trng CSP Nam nh
T
h
.
s

M
i
n
h
T
u

n
13.3. Cc dng ton Chng 13. Hnh hc khng gian
b) M mp(M, d
1
) mp(M, d
2
).
O = d
1
d
2
mp(M, d
1
) mp(M, d
2
)
Vy mp(M, d
1
) mp(M, d
2
) = MO.
V d 13.3.14: C v s ng thng ct c 3 ng thng cho trc i mt cho nhau.
Gii: Ly M d
1
bt k. R rng M / d
2
, d
3
t () = mp(M, d
2
) v () = mp(M, d
3
).
Do M l im chung ca 2 mt phng ny nh vy ch c 2 kh nng xy ra:
+) TH1: () (). Khi d
2
, d
3
(). iu ny mu thun vi tnh cho nhau ca ().
+) TH2: ()() = d. Khi d chnh l ng thng qua M ct d
2
, d
3
do n ct c d
1
, d
2
, d
3
.
13.3.4 Tm giao im ca ng thng v mt phng
~ Thut ton: Tm giao im ca ng thng d v mt phng () trong khng gian.
Chn mt phng () cha d.
Tm = () () v d () = d .
V d 13.3.15: Cho t din ABCD, trn AC, AD ly im M, N sao cho MN khng song
song CD. Gi O l im thuc BCD.
a) Tm giao tuyn ca (OMN) v (BCD).
b) Tm giao im ca BC v BD vi mt phng (OMN).
A
B
C
D
O
M N
P
Q
R
Gii: a) O (BCD) O (BCD) (OMN).
(OMN) (BCD)
P = MN CD
(ACD)
))
R
R
R
R
R
R
R
R
R
R
R
MN uul
l
l
l
l
l
l
l
l
l
l
l
CD
//
(BCD) (OMN) = OP.
b) BC (BCD). Theo cu trn th (BCD)(OMN) = OP BC(OMN) = BCOP = Q.
BD (BCD) v BD (OMN) = BD OP = R.
V d 13.3.16: Cho hnh chp S.ABCD, v M l im trn SC.
a) Tm giao im ca AM v (SBD).
Ngi son : Th.s Minh Tun Trang 285 Khoa T nhin - Trng CSP Nam nh
T
h
.
s

M
i
n
h
T
u

n
13.3. Cc dng ton Chng 13. Hnh hc khng gian
b) Gi N l im trn BC. Tm SD (AMN).
A
B
C
D
S
M
N
E
F
G
H
Gii: a) AM (SAC).
Trc tin ta tm (SAC) (SBD).
S (SAC) (SBD).
(SAC) (SBD)
E = AC BD
(ABCD)
))
R
R
R
R
R
R
R
R
R
R
AC uul
l
l
l
l
l
l
l
l
l
BD
//
(SAC) (SBD) = SE.
AM (SBD) = AM SE = F.
b) SD (SCD).
M SC M (AMN) (SCD).
(SCD) (AMN)
G = CD AN
(ABCD)
))
R
R
R
R
R
R
R
R
R
R
R
CD uul
l
l
l
l
l
l
l
l
l
AN
//
(SCD) (AMN) = MG.
SD (AMN) = SD MG = H.
V d 13.3.17: Cho t din ABCD. Gi M, N l trung im ca AC, BC. Ly K BD khng
l trung im ca BD. Tm giao im CD, AD vi mp(MNK).
A
B
C
D
M
N
K
P
Q
Ngi son : Th.s Minh Tun Trang 286 Khoa T nhin - Trng CSP Nam nh
T
h
.
s

M
i
n
h
T
u

n
13.3. Cc dng ton Chng 13. Hnh hc khng gian
Gii: + CD (MNK)
CD (ACD). Ta tm (ACD) (MNK):
M AC (ACD) M (ACD) (MNK)
(ACD) (MNK)
P = CD MK
(BCD)
))
R
R
R
R
R
R
R
R
R
R
R
R
CD
uul
l
l
l
l
l
l
l
l
l
l
MK
//
(ACD) (MNK) = MP.
CD (MNK) = CD MP = P.
AD (MNK) = AD MP = Q.
V d 13.3.18: Cho t din ABCD v M, N l 2 im trn AC, AD. Ly O l im bn trong
tam gic BCD. Tm giao im ca:
a) MN v (ABO).
b) AO v (BMN).
A
B
C
D
M
N
O
P
Q
R
Gii: a) MN (ACD).
Tm giao tuyn ca (ABO) v ACD.
A (ABO) (ACD).
(ABO) (ACD)
P = BO CD
(BCD)
))
R
R
R
R
R
R
R
R
R
R
R
BO uul
l
l
l
l
l
l
l
l
l
l
CD
//
(ABO) (ACD) = AP.
Do MN (ABO) = MN AP = Q.
b) AO (ABO)
Tm giao tuyn ca (ABO) v (BMN)
B (ABO) (BMN).
Ngi son : Th.s Minh Tun Trang 287 Khoa T nhin - Trng CSP Nam nh
T
h
.
s

M
i
n
h
T
u

n
13.3. Cc dng ton Chng 13. Hnh hc khng gian
(ABO) (BMN) Q = AP MN
(ACD)
))
R
R
R
R
R
R
R
R
R
R
R
R
AP
uul
l
l
l
l
l
l
l
l
l
l
MN
//
Vy (ABO) (BMN) = BQ.
Do AO (BMN) = AO BQ = R
13.3.5 Chng minh 3 im thng hng, 3 ng ng quy
~ Thut ton:
Ba im thng hng: A, B, C (), () phn bit th khi A, B, C thng hng (Cng
thuc giao tuyn ca 2 mt phng)
Ba ng ng quy:
() ()
()
c
?
?
?
?
?
?
b






a
Khi a, b, c ng quy.
Cch xy dng cc mt phng khi cho trc a, b, c:
() = mp(b, c), () = mp(a, c), () = mp(a, b).
V d 13.3.19: Cho hnh chp S.ABCD. Gi I, J l 2 im c nh vi SI > IA, SJ < JC.
Mt mt phng () quay quanh IJ ct SB ti M, ct SD ti N.
a) Chng minh rng: IJ, MN, SO ng quy. O = AC BD. T suy ra cch dng im
N khi bit im M.
b) AD ct BC ti E, IN ct MJ ti F. Chng minh rng S, E, F thng hng.
c) IN ct AD ti P, MJ ct BC ti Q. Chng minh rng PQ lun i qua mt im c nh
khi () di ng.
A
B
C
D
S
I
J
O
M
N
E
F
P
Q
K
Ngi son : Th.s Minh Tun Trang 288 Khoa T nhin - Trng CSP Nam nh
T
h
.
s

M
i
n
h
T
u

n
13.3. Cc dng ton Chng 13. Hnh hc khng gian
Gii: a) Bc 1: Xy dng cc mt phng: mp(IJ, MN) = mp(), mp(IJ, SO) = mp(SAC),
mp(MN, SO) = mp(SBD). (Lm nhp)
Bc 2: Gii bi ton:
() (SAC)
(SBD)
IJ
J
J
J
J
J
J
J
J
MN
t
t
t
t
t
t
t
SO
V MN ct SO nn MN, SO, IJ ng quy.
b) Xt 2 mt phng (SAD) v (SBC).
+) S (SAD) (SBC).
+) E = AD BC (SAD) (SBC).
+) F = IN MJ (SAD) (SBC).
Vy S, E, F (SAD) (SBC) do S, E, F thng hng.
c) Xt 2 mt phng (ABCD) v ().
Theo gi thit IJ ct AC. Gi K = IJ AC c nh.
K AC K (ABCD).
K IJ K ().
K (ABCD) ().
+) Tng t P = IN AD () (ABCD).
Q = MJ BC () (ABCD).
Do P, Q, K thng hng. (Cng thuc giao tuyn 2 mt phng (), (ABCD)).
Vy PQ lun i qua 1 im c nh l K.
V d 13.3.20: Cho mt phng (), A, B, C khng thng hng khng thuc (). Gi s cc
ng thng BC, CA, AB ln lt ct () ti D, E, F. Chng minh rng: D, E, F thng hng.

A
F
D
E
B
C
Ngi son : Th.s Minh Tun Trang 289 Khoa T nhin - Trng CSP Nam nh
T
h
.
s

M
i
n
h
T
u

n
13.3. Cc dng ton Chng 13. Hnh hc khng gian
Gii: Ta c D = BC () (ABC) ()
E = CA () (ABC) ()
F = AB () (ABC) ()
Do D, E, F thng hng. (Cng thuc giao tuyn ca 2 mt phng)
V d 13.3.21: Cho t din ABCD. Gi E, F, G ln lt l 3 im thuc 3 cnh AB, AC, BD
sao cho EF ct BC ti I, EG ct AD ti H. Chng minh rng: CD, IG, HF ng quy.
A
B
C
D
E
F
G
I
H
Gii: mp(CD, IG) = (BCD), mp(CD, HF) = ACD, mp(IG, HF) = (EFG)
(BCD) (ACD)
(EFG)
CD
J
J
J
J
J
J
J
J
IG
t
t
t
t
t
t
t
t
HF
V d 13.3.22: Cho 2 im c nh A, B ngoi mt phng () tha mn AB khng song song
vi (). im M di ng trong khng gian sao cho MA, MB ct mt phng () ti A

, B

M
A
C
B
A
B
Gii: AB khng song song vi () nn C = AB () (ABC) () c nh.
A

= MA () (MAB) ()
B

= MB () (MAB) ()
Do A

, B

, C thng hng. Do A

lun i qua im c nh C.
V d 13.3.23: Cho t din S.ABC. Qua C dng mt phng () ct AB, SB ti B
1
, B

. Qua
B dng () ct AC, SC ti C
1
, C

; BC

BC

= O

, B
1
C BC
1
= O
1
. Gi s O

O
1
ko di ct
SA ti I.
Ngi son : Th.s Minh Tun Trang 290 Khoa T nhin - Trng CSP Nam nh
T
h
.
s

M
i
n
h
T
u

n
13.3. Cc dng ton Chng 13. Hnh hc khng gian
a) Chng minh rng: AO
1
, SO

, BC ng quy.
b) Chng minh rng: I, B

, B
1
v I, C

, C
1
thng hng.
S
A
B
C
B
1
B
I
C
C
1
O
1
O
Gii: a) Ta c SA, O

O
1
ct nhau do S, A, O
1
, O

ng phng.
Do mp(AO
1
, SO

) = (SAO
1
)
mp(SO

, BC) = (SBC)
mp(AO
1
, BC) = (ABC)
(SAO
1
) (SBC)
(ABC)
SO

J
J
J
J
J
J
J
J
AO
1
t
t
t
t
t
t
t
t
BC
b) Xt 2 mt phng (CB
1
B

) v (SAB)
I = O

O
1
SA (CB
1
B

) (SAB)
B

, B
1
(CB
1
B

) (SAB)
Do I, B

, B
1
thng hng. Tng t vi I, C

, C
1
V d 13.3.24: Cho hnh chp S.ABCD. Gi I, J l 2 im thuc cnh AD, SB.
a) Tm cc giao im K, L ca IJ, DJ vi mt phng (SAC).
b) AD BC = O, OJ SC = M. Chng minh rng: A, K, L, M thng hng.
A
B
C
D
S
I
J
E
F
K
L
O
M
Ngi son : Th.s Minh Tun Trang 291 Khoa T nhin - Trng CSP Nam nh
T
h
.
s

M
i
n
h
T
u

n
13.3. Cc dng ton Chng 13. Hnh hc khng gian
Gii: a) +) IJ (SAC): Ta c IJ (SBI)
Ta tm (SBI) (SAC):
S (SBI) (SAC).
(SBI) (SAC)
E = BI AC
(ABCD)
))
R
R
R
R
R
R
R
R
R
R
BI uul
l
l
l
l
l
l
l
l
l
AC
//
Do (SBI) (SAC) = SE.
IJ (SAC) = IJ SE = K.
+) DJ (SAC): Ta c DJ (SBD).
Ta tm (SAC) (SBD).
S (SAC) (SBD).
(SAC) (SBD)
F = AC BD
(ABCD)
))
R
R
R
R
R
R
R
R
R
R
AC uul
l
l
l
l
l
l
l
l
l
BD
//
Do (SAC) (SBD) = SF DJ (SAC) = DJ SF = L.
b) Xt 2 mt phng (SAC) v (DIJ):
A DI (DIJ) A (DIJ) (SAC) (1)
O = AD BC O AD = DI (DIJ) M OJ (DIJ)
M SC (SAC) M (DIJ) (SAC) (2)
K = IJ (SAC) (DIJ) (SAC) (3)
L = DJ (SAC) (DIJ) (SAC) (4)
T (1),(2),(3),(4) ta c A, K, L, M thng hng.
13.3.6 Thit din
nh ngha 13.1. a gic gii hn bi giao tuyn ca mt phng () vi cc mt ca hnh
chp gi l thit din ca () vi hnh chp.
~ Thut ton: Xc nh thit din.
T mt im chung c sn, xc nh giao tuyn u tin ca () vi mt mt ca hnh
chp. (C th dng mt phng trung gian).
Cho giao tuyn ny ct cc cnh ca mt ca hnh chp ta s tm c im chung mi
ca () vi cc mt khc. T xc nh c cc giao tuyn mi vi cc mt ny.
Tip tc nh trn cho ti khi cc giao tuyn khp kn ta c thit din.
V d 13.3.25: Cho t din ABCD. Gi H, K ln lt l trung im ca cc cnh AB, BC.
Trn ng thng CD ly im M sao cho KM khng song song vi BD. Tm thit din ca
t din ABCD vi mt phng (HKM). Phn bit trng hp M gia C v D ngoi on
CD.
Ngi son : Th.s Minh Tun Trang 292 Khoa T nhin - Trng CSP Nam nh
T
h
.
s

M
i
n
h
T
u

n
13.3. Cc dng ton Chng 13. Hnh hc khng gian
Gii: a) Trng hp 1: M [C, D].
A
B
C
D
H
K
M
L
N
H AB H (HKM) (ABD).
(HKM) (ABD)
L = KM BD
(BCD)
))
R
R
R
R
R
R
R
R
R
R
R
KM uul
l
l
l
l
l
l
l
l
l
l
BD
//
(HKM) (ABD) = HL
HL (AD) = N.
Vy thit din l t gic HKMN.
b) Trng hp 2: M / [C, D].
A
B
C
D
H
K
M
L
Tng t nh trng hp trn v ta c thit din l tam gic HKL.
V d 13.3.26: Cho hnh chp S.ABCD c y l hnh bnh hnh tm O. Gi M, N, I l 3
im ly trn AD, CD, SO. Tm thit din ca hnh chp vi mt phng (MNI).
A
B C
D
S
M
N
O
I
E
F
G
H
P
Q
Ngi son : Th.s Minh Tun Trang 293 Khoa T nhin - Trng CSP Nam nh
T
h
.
s

M
i
n
h
T
u

n
13.3. Cc dng ton Chng 13. Hnh hc khng gian
Gii: +) (MNI) v (SBD).
I SO (SBD)
Do I (SBD) (MNI).
(MNI) (SBD)
E = MN BD
(ABCD)
))
R
R
R
R
R
R
R
R
R
R
R
MN uul
l
l
l
l
l
l
l
l
l
l
BD
//
Do (MNI) (SBD) = IE
+) (MNI) v (SBC)
(MNI) (SBC)
G = MN BC
(ABCD)
))
R
R
R
R
R
R
R
R
R
R
MN uul
l
l
l
l
l
l
l
l
l
l
BC
//
(MNI) (SBC)
F = IE SB
(SBD)
))
R
R
R
R
R
R
R
R
R
R
IE uul
l
l
l
l
l
l
l
l
l
l
SB
//
Do (MNI) (SBC) = FG
+) (MNI) v (SCD):
N CD (SCD) N (MNI) (SCD).
(MNI) (SCD)
H = FG SC
(SBC)
))
R
R
R
R
R
R
R
R
R
R
R
FG uul
l
l
l
l
l
l
l
l
l
l
SC
//
+) (MNI) v (SAB)
F SB, F (MNI) F (MNI) (SAB)
(MNI) (SAB)
P = MN AB
(ABCD)
))
R
R
R
R
R
R
R
R
R
R
MN uul
l
l
l
l
l
l
l
l
l
l
AB
//
Do (MNI) (SAB) = PF.
+) (MNI) v (SAD):
M AD M (SAD) (MNI)
(MNI) (SAD) Q = PF SA
(SAB)
))
R
R
R
R
R
R
R
R
R
R
R
PF uul
l
l
l
l
l
l
l
l
l
l
SA
//
Do (MNI) (SAB) = MQ.
Ta c thit din l ng gic MNHFQ
V d 13.3.27: Cho t din u ABCD cnh a. Ko di BC mt on CE = a. Ko di BD
mt on DF = a. Gi M l trung im ca AB.
a) Tm thit din ca t din vi mt phng (MEF).
b) Tnh din tch thit din.
Ngi son : Th.s Minh Tun Trang 294 Khoa T nhin - Trng CSP Nam nh
T
h
.
s

M
i
n
h
T
u

n
13.3. Cc dng ton Chng 13. Hnh hc khng gian
A
B
C
D
E
F
M
H
G
Gii: a) G = MF AD, H = ME AC.
Thit din ca (MEF) vi t din ABCD l tam gic MHG.
b) Tnh din tch tam gic MHG.
+) Xt tam gic ABF c FM, AD l cc ng trung tuyn do G l trng tm tam gic
ABF. Do AG =
2
3
AD =
2a
3
.
M l trung im SA nn MA =
a
2
.
Theo nh l hm s cosin ta c:
MG
2
= MA
2
+AG
2
2MA.AG. cos 60
0
=
a
2
4

4a
2
9
2.
a
2
.
2a
3
.
1
2
=
13a
2
36
MG =
a

13
6
Tng t ta cng c AH =
2
3
AC =
2a
3
v MH =
a

13
6
Do
AG
AD
=
AH
AC
=
2
3
, v th HG | CD
v ta c HG =
2
3
CD =
2a
3
(nh l Thales)
Xt tam gic MHG:
cos M =
MH
2
+MG
2
HG
2
2MH.MG
=
5
13
sin M =
12
13
Do S
MHG
=
MH.MG. sin M
2
=
a

13
6
.
a

13
6
.
12
13
2
=
a
2
6
V d 13.3.28: Cho hnh chp S.ABCD, M l 1 im trn cnh SC, N v P ln lt l trung
im AB, AD. Tm thit din ca hnh chp vi mt phng (MNP).
Ngi son : Th.s Minh Tun Trang 295 Khoa T nhin - Trng CSP Nam nh
T
h
.
s

M
i
n
h
T
u

n
13.3. Cc dng ton Chng 13. Hnh hc khng gian
A
B
C
D
S
M
N
P
E
F
Q
G
Gii: +) (MNP) v (SBC):
M SC (SBC) M (MNP) (SBC)
(MNP) (SBC)
E = NP BC
(ABCD)
))
R
R
R
R
R
R
R
R
R
R
NP uul
l
l
l
l
l
l
l
l
l
BC
//
Do (MNP) (SBC) = ME
+) (MNP) v (SAB):
N AB (SAB) N (MNP) (SAB).
(MNP) (SAB) Q = ME SB
(SBC)
))
R
R
R
R
R
R
R
R
R
R
R
ME uul
l
l
l
l
l
l
l
l
l
l
SB
//
Vy (MNP) (SAB) = NQ.
+) (MNP) v (SCD):
M SC (SCD) M (SCD) (MNP).
(MNP) (SCD)
F = NP CD
(ABCD)
))
R
R
R
R
R
R
R
R
R
R
NP uul
l
l
l
l
l
l
l
l
l
CD
//
(MNP) (SCD) = MF.
+) (MNP) v (SAD):
P AD (SAD) P (SAD) (MNP).
(MNP) (SAD)
G = MF SD
(SCD)
))
R
R
R
R
R
R
R
R
R
R
R
MF uul
l
l
l
l
l
l
l
l
l
l
SD
//
Do (MNP) (SAD) = PG.
Vy thit din l ng gic MQNPG.
V d 13.3.29: Cho hnh chp S.ABCD. Trong tam gic SBC ly im M, trong tam gic
SCD ly im N.
a) Tm giao im ca MN v (SAC).
b) Tm SC (AMN).
c) Thit din ca hnh chp vi mt phng (AMN).
Ngi son : Th.s Minh Tun Trang 296 Khoa T nhin - Trng CSP Nam nh
T
h
.
s

M
i
n
h
T
u

n
13.3. Cc dng ton Chng 13. Hnh hc khng gian
A
B
C
D
S
M
N
E
F
G
H
K
I
J
Gii: a) MN (SMN) = (SEF) vi E = SM BC, F = SN CD.
+) Mt phng (SEF) v (SAC):
S (SAC) (SEF).
(SEF) (SAC)
G = EF AC
(ABCD)
))
R
R
R
R
R
R
R
R
R
R
EF uul
l
l
l
l
l
l
l
l
l
AC
//
(SAC) (SEF) = SG.
Do MN (SAC) = MN SG = H
b) SC (AMN)
SC (SAC).
+) Mt phng (SAC) v (AMN).
H = MN (SAC) (AMN) (SAC).
A (SAC) (AMN) (SAC) (AMN) = AH.
Do SC (AMN) = SC AH = K
c) Thit din ca hnh chp vi mt phng (AMN).
+) (SBC) (AMN).
M SF (SBC) M (AMN) (SBC).
K = SC (AMN) (SBC) (AMN).
Do (AMN) (SBC) = MK.
+) (SCD) (AMN).
N SE (SCD) N (SCD) (AMN).
K = SC (AMN) (SCD) (AMN).
Do NK = (SCD) (AMN).
+) (SAB) (AMN).
A (SAB) (AMN).
Ngi son : Th.s Minh Tun Trang 297 Khoa T nhin - Trng CSP Nam nh
T
h
.
s

M
i
n
h
T
u

n
13.3. Cc dng ton Chng 13. Hnh hc khng gian
(SAB) (AMN)
I = SB MK
(SBC)
))
R
R
R
R
R
R
R
R
R
R
R
R
SB
uul
l
l
l
l
l
l
l
l
l
l
MK
//
Do (SAB) (AMN) = AI.
+) (SAD) (AMN).
A (SAD) (AMN).
(SAD) (AMN)
J = SD NK
(SCD)
))
R
R
R
R
R
R
R
R
R
R
R
R
SD
uul
l
l
l
l
l
l
l
l
l
l
NK
//
Vy (SAD) (AMN) = AJ
thit din ca mt phng (AMN) vi hnh chp l t gic AIKJ.
V d 13.3.30: Hnh chp S.ABCD c y ABCD l hnh bnh hnh tm O. Gi M, N, P ln
lt l trung im ca SB, SD, OC.
a) Tm giao tuyn ca (MNP) vi (SAC) v giao im (MNP) vi SA.
b) Xc nh thit din ca hnh chp (MNP) v t s m (MNP) chia cc cnh SA, BC, CD.
A
B C
D
S
O
M
N
P
E
F
Q
G
H
Gii: a) P AC P (SAC) (MNP).
(SAC) (MNP)
E = SO MN
(SBD)
))
R
R
R
R
R
R
R
R
R
R
R
R
SO
uul
l
l
l
l
l
l
l
l
l
l
MN
//
PE = (SAC) (MNP).
SA (SAC) SA (MNP) = SA PE = F.
Ngi son : Th.s Minh Tun Trang 298 Khoa T nhin - Trng CSP Nam nh
T
h
.
s

M
i
n
h
T
u

n
13.3. Cc dng ton Chng 13. Hnh hc khng gian
b) +(MNP) (SAB):
M SB (SAB) M (MNP) (SAB).
F = SA (MNP) (SAB) (MNP)
MF = (SAB) (MNP).
Tng t (SAD) (MNP) = FN.
+ FM AB = Q
+ PQ BC = G, PQ CD = H
Thit din l ng gic MFNHG.
+) Do MN l ng trung bnh ca tam gic SBD nn E l trung im ca SO.
p dng nh l Melenauyt trong tam gic SAO vi ng ct tuyn PEF:
PO
PA
.
ES
EO
.
FA
FS
= 1
1
3
.(1).
FA
FS
= 1
FA
FS
= 3
p dng nh l Melenauyt trong tam gic SAB vi ng ct tuyn QMF:
QB
QA
.
MS
MB
.
FA
FS
= 1
QB
QA
.(1).(3) = 1
QB
QA
=
1
3
M
PO
PA
=
1
3
BO|PQ
P l trung im OC do G, H l trung im ca BC, CD.
V d 13.3.31: Cho hnh chp S.ABCD y l hnh bnh hnh. Gi M l trung im ca SB,
G l trng tm tam gic SAD.
a) Tm I = GM (ABCD). Chng minh rng: (CGM) cha CD.
b) Chng minh rng (CGM) qua trung im ca SA. Tm thit din ca hnh chp ct bi
(CGM).
c) Tm thit din hnh chp ct bi (AGM).
A
B C
D
S
M
G
E
F
I
O
H
K
P
Ngi son : Th.s Minh Tun Trang 299 Khoa T nhin - Trng CSP Nam nh
T
h
.
s

M
i
n
h
T
u

n
13.3. Cc dng ton Chng 13. Hnh hc khng gian
Gii: a) Gi E, H, F l trung im ca AD, SD, SA.
Do G SE, DF, AH.
MG (SBE), (SBE) (ABCD) = BE
Do MG (ABCD) = MG BE = I.
p dng nh l Melenauyt trong tam gic SBI vi ct tuyn IGM:
IE
IB
.
GS
GE
.
MB
MS
= 1
IE
IB
.(2).(1) = 1
IE
IB
=
1
2
E l trung im ca BI, m E l trung im ca AD. Do t gic ABDI l hnh bnh
hnh do

AB =

ID =

DC D l trung im ca CI.
Do D CI (CGM) CD (CGM).
b) F DG (CGM) l trung im ca SA.
Vy thit din ca hnh chp vi mt phng (CGM) l t gic FMCD.
c) (AGM) = (AHM)
+) (AHM) (SAC)
A (AHM) (SAC).
(AHM) (SAC)
K = HM SO
(SBD)
))
R
R
R
R
R
R
R
R
R
R
R
HM uul
l
l
l
l
l
l
l
l
l
l
l
SO
//
Do (AHM) (SAC) = AK
AK SC = P
V ta c thit din l t gic AMPH.
13.3.7 Bi tp
Bi 13.1: Cho t din ABCD. Gi G
1
, G
2
ln lt l trng tm cc tam gic BCD, ACD.
a) Chng minh rng: AG
1
v BG
2
ct nhau. Gi giao im ny l I.
Tnh
IA
IG
1
,
IB
IG
2
.
b) Chng minh rng: I l trung im ca on thng ni trung im ca AB, CD.
c) Chng minh rng ng thng i qua nh ca t din v trng tm ca mt i in ng
quy ti mt im. im ny gi l trng tm ca t din.
Bi 13.2: Cho t din u ABCD cnh a. Gi I, J ln lt l trung im ca AB, CD.
a) Tnh IJ.
b) M l im di ng trn on BC. Tm tp hp giao im N ca AM v (ICD).
Ngi son : Th.s Minh Tun Trang 300 Khoa T nhin - Trng CSP Nam nh
T
h
.
s

M
i
n
h
T
u

n
13.3. Cc dng ton Chng 13. Hnh hc khng gian
c) Gi G l trng tm ca tam gic ACD. Xc nh thit din ca t din vi mt phng
(IGM) khi M l trung im ca BC. Tnh t s m (IGM) chia cc cnh CD, AD. Thit
din l hnh g v tnh din tch ca n.
Bi 13.3: Cho hnh chp S.ABCD, M l trung im BC, N l mt im trn cnh SD.
a) Tm giao im I ca BN v (SAC), giao im J ca MN v (SAC).
b) DM ct AC ti K. Chng minh rng S, K, J thng hng.
c) Xc nh thit din ca hnh chp S.ABCD vi mt phng (BCN).
Bi 13.4: Cho hnh chp S.ABCD, y l hnh thang ABCD, AB|CD, AB > CD. Gi I l
trung im SC. Mt phng () quay quanh AI ct cc cnh SB, SD ln lt ti M, N.
a) Chng minh rng: MN lun i qua 1 im c nh.
b) IM ko di ct BC ti P, IN ko di ct CD ti Q. Chng minh rng PQ lun i qua mt
im c nh.
Ngi son : Th.s Minh Tun Trang 301 Khoa T nhin - Trng CSP Nam nh

You might also like